16.08.2024

Как Π½Π°ΠΉΡ‚ΠΈ эдс зная силу Ρ‚ΠΎΠΊΠ° ΠΈ сопротивлСниС: Как Π½Π°ΠΉΡ‚ΠΈ Π­Π”Π‘ Ссли извСстно сопротивлСниС 32 Ома ΠΈ сила Ρ‚ΠΎΠΊΠ° 0,5 Π°

Π‘ΠΎΠ΄Π΅Ρ€ΠΆΠ°Π½ΠΈΠ΅

Π“Π»Π°Π²Π° 21. ЭлСктричСский Ρ‚ΠΎΠΊ. Π—Π°ΠΊΠΎΠ½Ρ‹ Ома ΠΈ ДТоуля-Π›Π΅Π½Ρ†Π°

Для Ρ€Π΅ΡˆΠ΅Π½ΠΈΡ Π·Π°Π΄Π°Ρ‡ Π•Π“Π­ Π½Π° постоянный Ρ‚ΠΎΠΊ Π½Π°Π΄ΠΎ Π·Π½Π°Ρ‚ΡŒ опрСдСлСния Ρ‚ΠΎΠΊΠ°, напряТСния, сопротивлСния, Π·Π°ΠΊΠΎΠ½ Ома для участка Ρ†Π΅ΠΏΠΈ ΠΈ Π·Π°ΠΌΠΊΠ½ΡƒΡ‚ΠΎΠΉ Ρ†Π΅ΠΏΠΈ, Π·Π°ΠΊΠΎΠ½ ДТоуля-Π›Π΅Π½Ρ†Π°, Π° Ρ‚Π°ΠΊΠΆΠ΅ ΡƒΠΌΠ΅Ρ‚ΡŒ Π½Π°Ρ…ΠΎΠ΄ΠΈΡ‚ΡŒ эквивалСнтныС сопротивлСния ΠΏΡ€ΠΎΡΡ‚Π΅ΠΉΡˆΠΈΡ… элСктричСски Ρ†Π΅ΠΏΠ΅ΠΉ. Рассмотрим эти вопросы.

ЭлСктричСским Ρ‚ΠΎΠΊΠΎΠΌ Π½Π°Π·Ρ‹Π²Π°ΡŽΡ‚ упорядочСнноС Π΄Π²ΠΈΠΆΠ΅Π½ΠΈΠ΅ заряТСнных частиц. Π‘ΠΈΠ»ΠΎΠΉ Ρ‚ΠΎΠΊΠ° Π² Π½Π΅ΠΊΠΎΡ‚ΠΎΡ€ΠΎΠΌ сСчСнии ΠΏΡ€ΠΎΠ²ΠΎΠ΄Π½ΠΈΠΊΠ° называСтся ΠΎΡ‚Π½ΠΎΡˆΠ΅Π½ΠΈΠ΅ заряда , ΠΏΡ€ΠΎΡ‚Π΅ΠΊΡˆΠ΅Π³ΠΎ Ρ‡Π΅Ρ€Π΅Π· это сСчСниС Π·Π° ΠΈΠ½Ρ‚Π΅Ρ€Π²Π°Π» Π²Ρ€Π΅ΠΌΠ΅Π½ΠΈ , ΠΊ этому ΠΈΠ½Ρ‚Π΅Ρ€Π²Π°Π»Ρƒ Π²Ρ€Π΅ΠΌΠ΅Π½ΠΈ


(21.1)

Π§Ρ‚ΠΎΠ±Ρ‹ Π² ΠΏΡ€ΠΎΠ²ΠΎΠ΄Π½ΠΈΠΊΠ΅ Ρ‚Π΅ΠΊ элСктричСский Ρ‚ΠΎΠΊ, Π² ΠΏΡ€ΠΎΠ²ΠΎΠ΄Π½ΠΈΠΊΠ΅ Π΄ΠΎΠ»ΠΆΠ½ΠΎ Π±Ρ‹Ρ‚ΡŒ элСктричСскоС ΠΏΠΎΠ»Π΅, ΠΈΠ»ΠΈ, Π΄Ρ€ΡƒΠ³ΠΈΠΌΠΈ словами, ΠΏΠΎΡ‚Π΅Π½Ρ†ΠΈΠ°Π»Ρ‹ Ρ€Π°Π·Π»ΠΈΡ‡Π½Ρ‹Ρ… Ρ‚ΠΎΡ‡Π΅ΠΊ ΠΏΡ€ΠΎΠ²ΠΎΠ΄Π½ΠΈΠΊΠ° Π΄ΠΎΠ»ΠΆΠ½Ρ‹ Π±Ρ‹Ρ‚ΡŒ Ρ€Π°Π·Π½Ρ‹ΠΌΠΈ. Но ΠΏΡ€ΠΈ Π΄Π²ΠΈΠΆΠ΅Π½ΠΈΠΈ элСктричСских зарядов ΠΏΠΎ ΠΏΡ€ΠΎΠ²ΠΎΠ΄Π½ΠΈΠΊΡƒ ΠΏΠΎΡ‚Π΅Π½Ρ†ΠΈΠ°Π»Ρ‹ Ρ€Π°Π·Π»ΠΈΡ‡Π½Ρ‹Ρ… Ρ‚ΠΎΡ‡Π΅ΠΊ ΠΏΡ€ΠΎΠ²ΠΎΠ΄Π½ΠΈΠΊΠ° Π±ΡƒΠ΄ΡƒΡ‚ Π²Ρ‹Ρ€Π°Π²Π½ΠΈΠ²Π°Ρ‚ΡŒΡΡ (см. Π³Π». 19). ΠŸΠΎΡΡ‚ΠΎΠΌΡƒ для протСкания Ρ‚ΠΎΠΊΠ° Π² Ρ‚Π΅Ρ‡Π΅Π½ΠΈΠ΅ Π΄Π»ΠΈΡ‚Π΅Π»ΡŒΠ½ΠΎΠ³ΠΎ Π²Ρ€Π΅ΠΌΠ΅Π½ΠΈ Π½Π° ΠΊΠ°ΠΊΠΈΡ…-Ρ‚ΠΎ участках Ρ†Π΅ΠΏΠΈ Π½Π΅ΠΎΠ±Ρ…ΠΎΠ΄ΠΈΠΌΠΎ ΠΎΠ±Π΅ΡΠΏΠ΅Ρ‡ΠΈΡ‚ΡŒ Π΄Π²ΠΈΠΆΠ΅Π½ΠΈΠ΅ зарядов Π² Π½Π°ΠΏΡ€Π°Π²Π»Π΅Π½ΠΈΠΈ ΠΏΡ€ΠΎΡ‚ΠΈΠ²ΠΎΠΏΠΎΠ»ΠΎΠΆΠ½ΠΎΠΌ полю. Π’Π°ΠΊΠΎΠ΅ Π΄Π²ΠΈΠΆΠ΅Π½ΠΈΠ΅ ΠΌΠΎΠΆΠ΅Ρ‚ Π±Ρ‹Ρ‚ΡŒ обСспСчСно Ρ‚ΠΎΠ»ΡŒΠΊΠΎ силами нСэлСктричСской ΠΏΡ€ΠΈΡ€ΠΎΠ΄Ρ‹, ΠΊΠΎΡ‚ΠΎΡ€Ρ‹Π΅ Π² этом контСкстС принято Π½Π°Π·Ρ‹Π²Π°Ρ‚ΡŒ сторонними. Π’ Π³Π°Π»ΡŒΠ²Π°Π½ΠΈΡ‡Π΅ΡΠΊΠΈΡ… элСмСнтах (Β«Π±Π°Ρ‚Π°Ρ€Π΅ΠΉΠΊΠ°Ρ…Β») сторонниС силы Π²ΠΎΠ·Π½ΠΈΠΊΠ°ΡŽΡ‚ Π² Ρ€Π΅Π·ΡƒΠ»ΡŒΡ‚Π°Ρ‚Π΅ элСктрохимичСских ΠΏΡ€Π΅Π²Ρ€Π°Ρ‰Π΅Π½ΠΈΠΉ Π½Π° Π³Ρ€Π°Π½ΠΈΡ†Π°Ρ… элСктродов ΠΈ элСктролита. Π­Ρ‚ΠΈ прСвращСния ΠΎΠ±Π΅ΡΠΏΠ΅Ρ‡ΠΈΠ²Π°ΡŽΡ‚ ΠΏΠ΅Ρ€Π΅ΠΌΠ΅Ρ‰Π΅Π½ΠΈΠ΅ заряда ΠΏΡ€ΠΎΡ‚ΠΈΠ²ΠΎΠΏΠΎΠ»ΠΎΠΆΠ½ΠΎ Π½Π°ΠΏΡ€Π°Π²Π»Π΅Π½ΠΈΡŽ поля, поддСрТивая Π΄Π²ΠΈΠΆΠ΅Π½ΠΈΠ΅ зарядов ΠΏΠΎ Π·Π°ΠΌΠΊΠ½ΡƒΡ‚ΠΎΠΌΡƒ ΠΏΡƒΡ‚ΠΈ.

Π‘ΠΈΠ»Π° Ρ‚ΠΎΠΊΠ° Π² ΠΎΠ΄Π½ΠΎΡ€ΠΎΠ΄Π½ΠΎΠΌ участкС ΠΏΡ€ΠΎΠ²ΠΎΠ΄Π½ΠΈΠΊΠ° ΠΏΡ€ΠΎΠΏΠΎΡ€Ρ†ΠΈΠΎΠ½Π°Π»ΡŒΠ½Π° напряТСнности элСктричСского поля Π²Π½ΡƒΡ‚Ρ€ΠΈ ΠΏΡ€ΠΎΠ²ΠΎΠ΄Π½ΠΈΠΊΠ°. А ΠΏΠΎΡΠΊΠΎΠ»ΡŒΠΊΡƒ Π½Π°ΠΏΡ€ΡΠΆΠ΅Π½Π½ΠΎΡΡ‚ΡŒ поля Π²Π½ΡƒΡ‚Ρ€ΠΈ ΠΏΡ€ΠΎΠ²ΠΎΠ΄Π½ΠΈΠΊΠ° связана с Ρ€Π°Π·Π½ΠΎΡΡ‚ΡŒΡŽ ΠΏΠΎΡ‚Π΅Π½Ρ†ΠΈΠ°Π»ΠΎΠ² Π΅Π³ΠΎ ΠΊΠΎΠ½Ρ†ΠΎΠ² (ΠΈΠ»ΠΈ элСктричСским напряТСниСм Π½Π° ΠΏΡ€ΠΎΠ²ΠΎΠ΄Π½ΠΈΠΊΠ΅ ), Ρ‚ΠΎ


(21.2)

ΠšΠΎΡΡ„Ρ„ΠΈΡ†ΠΈΠ΅Π½Ρ‚ ΠΏΡ€ΠΎΠΏΠΎΡ€Ρ†ΠΈΠΎΠ½Π°Π»ΡŒΠ½ΠΎΡΡ‚ΠΈ , ΠΊΠΎΡ‚ΠΎΡ€Ρ‹ΠΉ принято Π·Π°ΠΏΠΈΡΡ‹Π²Π°Ρ‚ΡŒ Π² Π·Π½Π°ΠΌΠ΅Π½Π°Ρ‚Π΅Π»ΡŒ Ρ„ΠΎΡ€ΠΌΡƒΠ»Ρ‹ (21.2), являСтся характСристикой ΠΏΡ€ΠΎΠ²ΠΎΠ΄Π½ΠΈΠΊΠ° ΠΈ называСтся Π΅Π³ΠΎ сопротивлСниСм. Π’ Ρ€Π΅Π·ΡƒΠ»ΡŒΡ‚Π°Ρ‚Π΅ Ρ„ΠΎΡ€ΠΌΡƒΠ»Π° (21.2) ΠΏΡ€ΠΈΠ½ΠΈΠΌΠ°Π΅Ρ‚ Π²ΠΈΠ΄


(21. 3)

Π€ΠΎΡ€ΠΌΡƒΠ»Π° (21.3) называСтся Π·Π°ΠΊΠΎΠ½ΠΎΠΌ Ома для ΠΎΠ΄Π½ΠΎΡ€ΠΎΠ΄Π½ΠΎΠ³ΠΎ участка Ρ†Π΅ΠΏΠΈ, Π° сам участок Ρ†Π΅ΠΏΠΈ часто Π½Π°Π·Ρ‹Π²Π°ΡŽΡ‚ рСзистором (ΠΎΡ‚ английского слова resistance β€” сопротивлСниС).

Если ΠΏΡ€ΠΎΠ²ΠΎΠ΄Π½ΠΈΠΊ являСтся ΠΎΠ΄Π½ΠΎΡ€ΠΎΠ΄Π½Ρ‹ΠΌ ΠΈ ΠΈΠΌΠ΅Π΅Ρ‚ Ρ†ΠΈΠ»ΠΈΠ½Π΄Ρ€ΠΈΡ‡Π΅ΡΠΊΡƒΡŽ Ρ„ΠΎΡ€ΠΌΡƒ (ΠΏΡ€ΠΎΠ²ΠΎΠ΄), Ρ‚ΠΎ Π΅Π³ΠΎ сопротивлСниС ΠΏΡ€ΠΎΠΏΠΎΡ€Ρ†ΠΈΠΎΠ½Π°Π»ΡŒΠ½ΠΎ Π΄Π»ΠΈΠ½Π΅ ΠΈ ΠΎΠ±Ρ€Π°Ρ‚Π½ΠΎ ΠΏΡ€ΠΎΠΏΠΎΡ€Ρ†ΠΈΠΎΠ½Π°Π»ΡŒΠ½ΠΎ ΠΏΠ»ΠΎΡ‰Π°Π΄ΠΈ сСчСния


(21.4)

Π³Π΄Π΅ коэффициСнт ΠΏΡ€ΠΎΠΏΠΎΡ€Ρ†ΠΈΠΎΠ½Π°Π»ΡŒΠ½ΠΎΡΡ‚ΠΈ зависит Ρ‚ΠΎΠ»ΡŒΠΊΠΎ ΠΎΡ‚ ΠΌΠ°Ρ‚Π΅Ρ€ΠΈΠ°Π»Π° ΠΏΡ€ΠΎΠ²ΠΎΠ΄Π½ΠΈΠΊΠ° ΠΈ называСтся Π΅Π³ΠΎ ΡƒΠ΄Π΅Π»ΡŒΠ½Ρ‹ΠΌ сопротивлСниСм.


Если участок Ρ†Π΅ΠΏΠΈ прСдставляСт собой нСсколько ΠΏΠΎΡΠ»Π΅Π΄ΠΎΠ²Π°Ρ‚Π΅Π»ΡŒΠ½ΠΎ соСдинСнных ΠΎΠ΄Π½ΠΎΡ€ΠΎΠ΄Π½Ρ‹Ρ… ΠΏΡ€ΠΎΠ²ΠΎΠ΄Π½ΠΈΠΊΠΎΠ² с сопротивлСниями (см. рисунок), Ρ‚ΠΎ сила Ρ‚ΠΎΠΊΠ° Ρ‡Π΅Ρ€Π΅Π· ΠΊΠ°ΠΆΠ΄Ρ‹ΠΉ ΠΏΡ€ΠΎΠ²ΠΎΠ΄Π½ΠΈΠΊ Π±ΡƒΠ΄Π΅Ρ‚ ΠΎΠ΄ΠΈΠ½Π°ΠΊΠΎΠ²ΠΎΠΉ , элСктричСскоС напряТСниС Π½Π° всСм участкС Ρ†Π΅ΠΏΠΈ Ρ€Π°Π²Π½ΠΎ суммС напряТСний Π½Π° ΠΊΠ°ΠΆΠ΄ΠΎΠΌ ΠΏΡ€ΠΎΠ²ΠΎΠ΄Π½ΠΈΠΊΠ΅ , Π° эквивалСнтноС сопротивлСниС всСго участка Ρ€Π°Π²Π½ΠΎ суммС сопротивлСний ΠΎΡ‚Π΄Π΅Π»ΡŒΠ½Ρ‹Ρ… ΠΏΡ€ΠΎΠ²ΠΎΠ΄Π½ΠΈΠΊΠΎΠ²


(21. 4)

Если участок Ρ†Π΅ΠΏΠΈ прСдставляСт собой нСсколько ΠΎΠ΄Π½ΠΎΡ€ΠΎΠ΄Π½Ρ‹Ρ… ΠΏΡ€ΠΎΠ²ΠΎΠ΄Π½ΠΈΠΊΠΎΠ² с сопротивлСниями , соСдинСнных ΠΏΠ°Ρ€Π°Π»Π»Π΅Π»ΡŒΠ½ΠΎ (см. рисунок), Ρ‚ΠΎ элСктричСскоС напряТСниС Π½Π° ΠΊΠ°ΠΆΠ΄ΠΎΠΌ ΠΏΡ€ΠΎΠ²ΠΎΠ΄Π½ΠΈΠΊΠ΅ Π±ΡƒΠ΄Π΅Ρ‚ ΠΎΠ΄ΠΈΠ½Π°ΠΊΠΎΠ²Ρ‹ΠΌ , Ρ‚ΠΎΠΊ Ρ‡Π΅Ρ€Π΅Π· участок Π±ΡƒΠ΄Π΅Ρ‚ Ρ€Π°Π²Π΅Π½ суммС Ρ‚ΠΎΠΊΠΎΠ², Ρ‚Π΅ΠΊΡƒΡ‰ΠΈΡ… Ρ‡Π΅Ρ€Π΅Π· ΠΊΠ°ΠΆΠ΄Ρ‹ΠΉ ΠΏΡ€ΠΎΠ²ΠΎΠ΄Π½ΠΈΠΊ , Π° Π²Π΅Π»ΠΈΡ‡ΠΈΠ½Π°, обратная эквивалСнтному ΡΠΎΠΏΡ€ΠΎΡ‚ΠΈΠ²Π»Π΅Π½ΠΈΡŽ всСго участка, Ρ€Π°Π²Π½ΠΎ суммС ΠΎΠ±Ρ€Π°Ρ‚Π½Ρ‹Ρ… сопротивлСний ΠΎΡ‚Π΄Π΅Π»ΡŒΠ½Ρ‹Ρ… ΠΏΡ€ΠΎΠ²ΠΎΠ΄Π½ΠΈΠΊΠΎΠ²


(21.5)

Рассмотрим Ρ‚Π΅ΠΏΠ΅Ρ€ΡŒ Π·Π°ΠΊΠΎΠ½ Ома для Π·Π°ΠΌΠΊΠ½ΡƒΡ‚ΠΎΠΉ элСктричСской Ρ†Π΅ΠΏΠΈ. ΠŸΡƒΡΡ‚ΡŒ имССтся замкнутая элСктричСская Ρ†Π΅ΠΏΡŒ, состоящая ΠΈΠ· источника сторонних сил с Π²Π½ΡƒΡ‚Ρ€Π΅Π½Π½ΠΈΠΌ сопротивлСниСм ΠΈ внСшнСго сопротивлСния . ΠŸΡƒΡΡ‚ΡŒ ΠΏΡ€ΠΈ ΠΏΡ€ΠΎΡ…ΠΎΠΆΠ΄Π΅Π½ΠΈΠΈ заряда Ρ‡Π΅Ρ€Π΅Π· источник сторонниС силы ΡΠΎΠ²Π΅Ρ€ΡˆΠ°ΡŽΡ‚ Ρ€Π°Π±ΠΎΡ‚Ρƒ . Π­Π»Π΅ΠΊΡ‚Ρ€ΠΎΠ΄Π²ΠΈΠΆΡƒΡ‰Π΅ΠΉ силой источника (часто ΠΈΡΠΏΠΎΠ»ΡŒΠ·ΡƒΠ΅Ρ‚ΡΡ Π°Π±Π±Ρ€Π΅Π²ΠΈΠ°Ρ‚ΡƒΡ€Π° Π­Π”Π‘) называСтся ΠΎΡ‚Π½ΠΎΡˆΠ΅Π½ΠΈΠ΅ Ρ€Π°Π±ΠΎΡ‚Ρ‹ сторонних сил ΠΊ заряду


(21. 6)

Π’ этом случаС сила Ρ‚ΠΎΠΊΠ° Π² Ρ†Π΅ΠΏΠΈ Ρ€Π°Π²Π½Π°


(21.7)

Π€ΠΎΡ€ΠΌΡƒΠ»Π° (21.7) называСтся Π·Π°ΠΊΠΎΠ½ΠΎΠΌ Ома для Π·Π°ΠΌΠΊΠ½ΡƒΡ‚ΠΎΠΉ элСктричСской Ρ†Π΅ΠΏΠΈ.

ΠŸΡ€ΠΈ ΠΏΡ€ΠΎΡ…ΠΎΠΆΠ΄Π΅Π½ΠΈΠΈ элСктричСского Ρ‚ΠΎΠΊΠ° Ρ‡Π΅Ρ€Π΅Π· участок Ρ†Π΅ΠΏΠΈ элСктричСскоС ΠΏΠΎΠ»Π΅ ΡΠΎΠ²Π΅Ρ€ΡˆΠ°Π΅Ρ‚ Ρ€Π°Π±ΠΎΡ‚Ρƒ (часто эту Ρ€Π°Π±ΠΎΡ‚Ρƒ Π½Π°Π·Ρ‹Π²Π°ΡŽΡ‚ Ρ€Π°Π±ΠΎΡ‚ΠΎΠΉ Ρ‚ΠΎΠΊΠ°, хотя Ρ‚Π΅Ρ€ΠΌΠΈΠ½ этот Π½Π΅ ΠΎΡ‡Π΅Π½ΡŒ Ρ‚ΠΎΡ‡Π½Ρ‹ΠΉ). ΠžΡ‡Π΅Π²ΠΈΠ΄Π½ΠΎ, вся эта Ρ€Π°Π±ΠΎΡ‚Π° прСвращаСтся Π² Ρ‚Π΅ΠΏΠ»ΠΎ. ΠŸΠΎΡΡ‚ΠΎΠΌΡƒ Ссли Ρ‡Π΅Ρ€Π΅Π· участок Ρ†Π΅ΠΏΠΈ ΠΏΡ€ΠΎΡˆΠ΅Π» заряд , Π³Π΄Π΅ β€” сила Ρ‚ΠΎΠΊΠ° Π² Ρ†Π΅ΠΏΠΈ, β€” врСмя, Ρ‚ΠΎ количСство Π²Ρ‹Π΄Π΅Π»ΠΈΠ²ΡˆΠ΅ΠΉΡΡ Ρ‚Π΅ΠΏΠ»ΠΎΡ‚Ρ‹ Ρ€Π°Π²Π½ΠΎ


(21.8)

(для получСния послСднСго ΠΈ прСдпослСднСго равСнств использован Π·Π°ΠΊΠΎΠ½ Ома для участка Ρ†Π΅ΠΏΠΈ). Π€ΠΎΡ€ΠΌΡƒΠ»Ρ‹ (21.8) Π½Π°Π·Ρ‹Π²Π°ΡŽΡ‚ΡΡ Π·Π°ΠΊΠΎΠ½ΠΎΠΌ ДТоуля-Π›Π΅Π½Ρ†Π°. Из Ρ„ΠΎΡ€ΠΌΡƒΠ»Ρ‹ (21.8) слСдуСт, Ρ‡Ρ‚ΠΎ количСство Π²Ρ‹Π΄Π΅Π»ΠΈΠ²ΡˆΠ΅ΠΉΡΡ ΠΏΡ€ΠΈ ΠΏΡ€ΠΎΡ‚Π΅ΠΊΠ°Π½ΠΈΠΈ элСктричСского Ρ‚ΠΎΠΊΠ° Ρ‚Π΅ΠΏΠ»ΠΎΡ‚Ρ‹ Π»ΠΈΠ½Π΅ΠΉΠ½ΠΎ зависит ΠΎΡ‚ Π²Ρ€Π΅ΠΌΠ΅Π½ΠΈ наблюдСния. ΠŸΠΎΡΡ‚ΠΎΠΌΡƒ ΠΎΡ‚Π½ΠΎΡˆΠ΅Π½ΠΈΠ΅


(21.9)

ΠΊΠΎΡ‚ΠΎΡ€ΠΎΠ΅ называСтся ΠΌΠΎΡ‰Π½ΠΎΡΡ‚ΡŒΡŽ Ρ‚ΠΎΠΊΠ°, Π½Π΅ зависит ΠΎΡ‚ Π²Ρ€Π΅ΠΌΠ΅Π½ΠΈ наблюдСния. Π€ΠΎΡ€ΠΌΡƒΠ»Ρƒ (21.9) Ρ‚Π°ΠΊΠΆΠ΅ Π½Π°Π·Ρ‹Π²Π°ΡŽΡ‚ Π·Π°ΠΊΠΎΠ½ΠΎΠΌ ДТоуля-Π›Π΅Π½Ρ†Π°.

Рассмотрим Ρ‚Π΅ΠΏΠ΅Ρ€ΡŒ Π·Π°Π΄Π°Ρ‡ΠΈ.

Π‘Ρ‚Ρ€ΡƒΠΊΡ‚ΡƒΡ€Π° ΠΌΠ΅Ρ‚Π°Π»Π»Π° ΠΊΡ€Π°Ρ‚ΠΊΠΎ ΠΎΠ±ΡΡƒΠΆΠ΄Π°Π»Π°ΡΡŒ Π² Π³Π». 16: ΠΏΠΎΠ»ΠΎΠΆΠΈΡ‚Π΅Π»ΡŒΠ½ΠΎ заряТСнныС ΠΈΠΎΠ½Ρ‹ располоТСны Π² ΡƒΠ·Π»Π°Ρ… кристалличСской Ρ€Π΅ΡˆΠ΅Ρ‚ΠΊΠΈ, ΠΎΠ±Ρ€Π°Π·ΠΎΠ²Π°Π²ΡˆΠΈΠ΅ΡΡ Π² Ρ€Π΅Π·ΡƒΠ»ΡŒΡ‚Π°Ρ‚Π΅ диссоциации Π²Π°Π»Π΅Π½Ρ‚Π½Ρ‹Π΅ элСктроны ΠΌΠΎΠ³ΡƒΡ‚ свободно ΠΏΠ΅Ρ€Π΅ΠΌΠ΅Ρ‰Π°Ρ‚ΡŒΡΡ ΠΏΠΎ ΠΏΡ€ΠΎΠ²ΠΎΠ΄Π½ΠΈΠΊΡƒ (свободныС элСктроны). Они ΠΈ ΠΎΡΡƒΡ‰Π΅ΡΡ‚Π²Π»ΡΡŽΡ‚ ΠΏΡ€ΠΎΠ²ΠΎΠ΄ΠΈΠΌΠΎΡΡ‚ΡŒ ΠΌΠ΅Ρ‚Π°Π»Π»Π° (Π·Π°Π΄Π°Ρ‡Π° 21.1.1 β€” ΠΎΡ‚Π²Π΅Ρ‚ 2).

Богласно ΠΎΠΏΡ€Π΅Π΄Π΅Π»Π΅Π½ΠΈΡŽ (21.1) Π½Π°Ρ…ΠΎΠ΄ΠΈΠΌ ΡΡ€Π΅Π΄Π½ΡŽΡŽ силу Ρ‚ΠΎΠΊΠ° Π² ΠΊΠ°Π½Π°Π»Π΅ ΠΌΠΎΠ»Π½ΠΈΠΈ (Π·Π°Π΄Π°Ρ‡Π° 21.1.2)


(ΠΎΡ‚Π²Π΅Ρ‚ 2).

Если Π·Π° 1 ΠΌΠΈΠ½ Ρ‡Π΅Ρ€Π΅Π· сСчСниС ΠΏΡ€ΠΎΠ²ΠΎΠ΄Π½ΠΈΠΊΠ° ΠΏΡ€ΠΎΡ‚Π΅ΠΊΠ°Π΅Ρ‚ заряд 60 Кл (Π·Π°Π΄Π°Ρ‡Π° 21.1.3), Ρ‚ΠΎ сила Ρ‚ΠΎΠΊΠ° Π² этом ΠΏΡ€ΠΎΠ²ΠΎΠ΄Π½ΠΈΠΊΠ΅ Ρ€Π°Π²Π½Π° А. ΠŸΡ€ΠΈΠΌΠ΅Π½ΡΡ Π΄Π°Π»Π΅Π΅ ΠΊ этому ΠΏΡ€ΠΎΠ²ΠΎΠ΄Π½ΠΈΠΊΡƒ Π·Π°ΠΊΠΎΠ½ Ома для участка Ρ†Π΅ΠΏΠΈ, ΠΏΠΎΠ»ΡƒΡ‡Π°Π΅ΠΌ Π’ (ΠΎΡ‚Π²Π΅Ρ‚ 2).

По Π·Π°ΠΊΠΎΠ½Ρƒ Ома для участка Ρ†Π΅ΠΏΠΈ ΠΈΠΌΠ΅Π΅ΠΌ для силы Ρ‚ΠΎΠΊΠ° Ρ‡Π΅Ρ€Π΅Π· участок Ρ†Π΅ΠΏΠΈ послС измСнСния Π΅Π³ΠΎ сопротивлСния ΠΈ элСктричСского напряТСния Π½Π° Π½Π΅ΠΌ (Π·Π°Π΄Π°Ρ‡Π° 21.1.4)


Π’Π°ΠΊΠΈΠΌ ΠΎΠ±Ρ€Π°Π·ΠΎΠΌ, сила Ρ‚ΠΎΠΊΠ° ΡƒΠΌΠ΅Π½ΡŒΡˆΠΈΠ»Π°ΡΡŒ Π² 4 Ρ€Π°Π·Π° (ΠΎΡ‚Π²Π΅Ρ‚ 3).

Богласно Π·Π°ΠΊΠΎΠ½Ρƒ Ома для участка Ρ†Π΅ΠΏΠΈ сопротивлСниС β€” это коэффициСнт ΠΏΡ€ΠΎΠΏΠΎΡ€Ρ†ΠΈΠΎΠ½Π°Π»ΡŒΠ½ΠΎΡΡ‚ΠΈ ΠΌΠ΅ΠΆΠ΄Ρƒ напряТСниСм Π½Π° этом участкС ΠΈ силой Ρ‚ΠΎΠΊΠ° Π² Π½Π΅ΠΌ. ΠŸΠΎΡΡ‚ΠΎΠΌΡƒ Π² Π·Π°Π΄Π°Ρ‡Π΅ 21.1.5 ΠΈΠΌΠ΅Π΅ΠΌ, Π½Π°ΠΏΡ€ΠΈΠΌΠ΅Ρ€, ΠΈΡΠΏΠΎΠ»ΡŒΠ·ΡƒΡ ΠΊΡ€Π°ΠΉΠ½ΡŽΡŽ Ρ‚ΠΎΡ‡ΠΊΡƒ Π³Ρ€Π°Ρ„ΠΈΠΊΠ°


(ΠΎΡ‚Π²Π΅Ρ‚ 2). Из-Π·Π° Π»ΠΈΠ½Π΅ΠΉΠ½ΠΎΠΉ зависимости Ρ‚ΠΎΠΊΠ° ΠΎΡ‚ напряТСния вычислСния ΠΌΠΎΠΆΠ½ΠΎ Π±Ρ‹Π»ΠΎ Π²Ρ‹ΠΏΠΎΠ»Π½ΠΈΡ‚ΡŒ ΠΈ ΠΏΠΎ Π΄Ρ€ΡƒΠ³ΠΈΠΌ Ρ‚ΠΎΡ‡ΠΊΠ°ΠΌ Π³Ρ€Π°Ρ„ΠΈΠΊΠ°, ΠΎΡ‚Π²Π΅Ρ‚ Π±Ρ‹Π» Π±Ρ‹ Ρ‚Π°ΠΊΠΈΠΌ ΠΆΠ΅.

Богласно Ρ„ΠΎΡ€ΠΌΡƒΠ»Π΅ (21.4) ΠΈΠΌΠ΅Π΅ΠΌ для ΠΏΠ΅Ρ€Π²ΠΎΠΉ ΠΏΡ€ΠΎΠ²ΠΎΠ»ΠΎΠΊΠΈ Π² Π·Π°Π΄Π°Ρ‡Π΅ 21.1.6


Π³Π΄Π΅ β€” ΡƒΠ΄Π΅Π»ΡŒΠ½ΠΎΠ΅ сопротивлСниС ΠΌΠ΅Π΄ΠΈ, β€” Π΄Π»ΠΈΠ½Π° ΠΏΡ€ΠΎΠ²ΠΎΠ΄Π½ΠΈΠΊΠ°, β€” Π΅Π³ΠΎ радиус. Для ΠΌΠ΅Π΄Π½ΠΎΠΉ ΠΏΡ€ΠΎΠ²ΠΎΠ»ΠΎΠΊΠΈ с Π²Π΄Π²ΠΎΠ΅ большСй Π΄Π»ΠΈΠ½ΠΎΠΉ ΠΈ Π²Ρ‚Ρ€ΠΎΠ΅ бóльшим радиусом сСчСния ΠΈΠΌΠ΅Π΅ΠΌ


(ΠΎΡ‚Π²Π΅Ρ‚ 3).

Как слСдуСт ΠΈΠ· Ρ„ΠΎΡ€ΠΌΡƒΠ»Ρ‹ (21.4) ΠΏΡ€ΠΈ Π΄Π²ΡƒΠΊΡ€Π°Ρ‚Π½ΠΎΠΌ ΡƒΠΌΠ΅Π½ΡŒΡˆΠ΅Π½ΠΈΠΈ Π΄Π»ΠΈΠ½Ρ‹ ΠΏΡ€ΠΎΠ²ΠΎΠ΄Π½ΠΈΠΊΠ° Π²Π΄Π²ΠΎΠ΅ ΡƒΠΌΠ΅Π½ΡŒΡˆΠ°Π΅Ρ‚ΡΡ Π΅Π³ΠΎ сопротивлСниС. ΠŸΠΎΡΡ‚ΠΎΠΌΡƒ ΠΈΠ· Π·Π°ΠΊΠΎΠ½Π° Ома для участка Ρ†Π΅ΠΏΠΈ (21.3) Π·Π°ΠΊΠ»ΡŽΡ‡Π°Π΅ΠΌ, Ρ‡Ρ‚ΠΎ ΠΏΡ€ΠΈ Π΄Π²ΡƒΠΊΡ€Π°Ρ‚Π½ΠΎΠΌ ΡƒΠΌΠ΅Π½ΡŒΡˆΠ΅Π½ΠΈΠΈ напряТСния Π½Π° ΠΏΡ€ΠΎΠ²ΠΎΠ΄Π½ΠΈΠΊΠ΅ ΠΈ Π΄Π²ΡƒΠΊΡ€Π°Ρ‚Π½ΠΎΠΌ ΡƒΠΌΠ΅Π½ΡŒΡˆΠ΅Π½ΠΈΠΈ Π΅Π³ΠΎ Π΄Π»ΠΈΠ½Ρ‹ (Π·Π°Π΄Π°Ρ‡Π° 21.1.7) сила Ρ‚ΠΎΠΊΠ° Π² ΠΏΡ€ΠΎΠ²ΠΎΠ΄Π½ΠΈΠΊΠ΅ Π½Π΅ измСнится (ΠΎΡ‚Π²Π΅Ρ‚ 4).

Π’ Π·Π°Π΄Π°Ρ‡Π΅ 21.1.8 слСдуСт ΠΈΡΠΏΠΎΠ»ΡŒΠ·ΠΎΠ²Π°Ρ‚ΡŒ Π·Π°ΠΊΠΎΠ½ Ома для Π·Π°ΠΌΠΊΠ½ΡƒΡ‚ΠΎΠΉ элСктричСской Ρ†Π΅ΠΏΠΈ (21.7). ИмССм


Π³Π΄Π΅ β€” Π­Π”Π‘ источника, β€” сопротивлСнии Π΅ внСшнСй Ρ†Π΅ΠΏΠΈ, β€” сопротивлСниС источника (ΠΎΡ‚Π²Π΅Ρ‚ 1).

Π’ Π·Π°Π΄Π°Ρ‡Π΅ 21.1.9 слСдуСт ΠΏΡ€ΠΈΠΌΠ΅Π½ΠΈΡ‚ΡŒ Π·Π°ΠΊΠΎΠ½ Ома для Π·Π°ΠΌΠΊΠ½ΡƒΡ‚ΠΎΠΉ элСктричСской Ρ†Π΅ΠΏΠΈ (21.7) ΠΊ ΠΊΠ°ΠΊΠΎΠΌΡƒ-Π½ΠΈΠ±ΡƒΠ΄ΡŒ Π·Π½Π°Ρ‡Π΅Π½ΠΈΡŽ внСшнСго сопротивлСния, ΠΏΠΎ Π³Ρ€Π°Ρ„ΠΈΠΊΡƒ Π½Π°ΠΉΡ‚ΠΈ силу Ρ‚ΠΎΠΊΠ° Π² Ρ†Π΅ΠΏΠΈ, Π° Π·Π°Ρ‚Π΅ΠΌ ΠΈ Π­Π”Π‘ источника. ΠŸΡ€ΠΎΡ‰Π΅ всСго ΠΏΡ€ΠΈΠΌΠ΅Π½ΠΈΡ‚ΡŒ Π·Π°ΠΊΠΎΠ½ Ома ΠΊ ΡΠ»ΡƒΡ‡Π°ΡŽ . Из Π³Ρ€Π°Ρ„ΠΈΠΊΠ° Π½Π°Ρ…ΠΎΠ΄ΠΈΠΌ силу Ρ‚ΠΎΠΊΠ° . ΠŸΠΎΡΡ‚ΠΎΠΌΡƒ


Π³Π΄Π΅ β€” Π²Π½ΡƒΡ‚Ρ€Π΅Π½Π½Π΅Π΅ сопротивлСниС источника (ΠΎΡ‚Π²Π΅Ρ‚ 3).

Из Ρ„ΠΎΡ€ΠΌΡƒΠ»Ρ‹ (21.9) слСдуСт, Ρ‡Ρ‚ΠΎ ΠΏΡ€ΠΈ фиксированном сопротивлСнии участка Ρ†Π΅ΠΏΠΈ ΡƒΠ²Π΅Π»ΠΈΡ‡Π΅Π½ΠΈΠ΅ элСктричСского напряТСния Π² 2 Ρ€Π°Π·Π° (Π·Π°Π΄Π°Ρ‡Π° 21.1.10) ΠΏΡ€ΠΈΠ²Π΅Π΄Π΅Ρ‚ ΠΊ ΡƒΠ²Π΅Π»ΠΈΡ‡Π΅Π½ΠΈΡŽ мощности Ρ‚ΠΎΠΊΠ° Π² 4 Ρ€Π°Π·Π° (ΠΎΡ‚Π²Π΅Ρ‚ 2).

Π’ Π·Π°Π΄Π°Ρ‡Π΅ 21.2.1 ΡƒΠ΄ΠΎΠ±Π½ΠΎ ΠΈΡΠΏΠΎΠ»ΡŒΠ·ΠΎΠ²Π°Ρ‚ΡŒ Π²Ρ‚ΠΎΡ€ΡƒΡŽ ΠΈΠ· Ρ„ΠΎΡ€ΠΌΡƒΠ» (21.9) . ИмССм Π’Ρ‚ (ΠΎΡ‚Π²Π΅Ρ‚ 3).

Часто школьники Π½Π΅ ΠΌΠΎΠ³ΡƒΡ‚ ΠΎΡ‚Π²Π΅Ρ‚ΠΈΡ‚ΡŒ Π½Π° Ρ‚Π°ΠΊΠΎΠΉ вопрос: ΠΈΠ· Ρ„ΠΎΡ€ΠΌΡƒΠ»Ρ‹ для мощности Ρ‚ΠΎΠΊΠ° слСдуСт, Ρ‡Ρ‚ΠΎ ΠΌΠΎΡ‰Π½ΠΎΡΡ‚ΡŒ Π»ΠΈΠ½Π΅ΠΉΠ½ΠΎ растСт с ростом сопротивлСния, Π° ΠΈΠ· Ρ„ΠΎΡ€ΠΌΡƒΠ»Ρ‹ β€” ΡƒΠ±Ρ‹Π²Π°Π΅Ρ‚ с ростом сопротивлСния. А ΠΊΠ°ΠΊ ΠΆΠ΅ Π² Π΄Π΅ΠΉΡΡ‚Π²ΠΈΡ‚Π΅Π»ΡŒΠ½ΠΎΡΡ‚ΠΈ ΠΌΠΎΡ‰Π½ΠΎΡΡ‚ΡŒ зависит ΠΎΡ‚ сопротивлСния? Π”Π°Π²Π°ΠΉΡ‚Π΅ разбСрСмся Π² этом вопросС Π½Π° ΠΏΡ€ΠΈΠΌΠ΅Ρ€Π΅ Π·Π°Π΄Π°Ρ‡ΠΈ 21.2.2. ΠšΠΎΠ½Π΅Ρ‡Π½ΠΎ, ΠΎΠ±Π° ΠΏΡ€Π΅Π΄Π»ΠΎΠΆΠ΅Π½Π½Ρ‹Ρ… Β«Ρ€Π΅ΡˆΠ΅Π½ΠΈΡΒ» Π½Π΅ΠΏΡ€Π°Π²ΠΈΠ»ΡŒΠ½Ρ‹: Π² Π½ΠΈΡ… ΠΌΠΎΠ»Ρ‡Π°Π»ΠΈΠ²ΠΎ ΠΏΡ€Π΅Π΄ΠΏΠΎΠ»Π°Π³Π°Π»ΠΎΡΡŒ, Ρ‡Ρ‚ΠΎ сила Ρ‚ΠΎΠΊΠ°, Ρ‚Π΅ΠΊΡƒΡ‰Π΅Π³ΠΎ Ρ‡Π΅Ρ€Π΅Π· это сопротивлСниС, ΠΈΠ»ΠΈ напряТСниС Π½Π° этом сопротивлСнии Π½Π΅ зависят ΠΎΡ‚ Π΅Π³ΠΎ Π²Π΅Π»ΠΈΡ‡ΠΈΠ½Ρ‹. А Π½Π° самом Π΄Π΅Π»Π΅ эти Π²Π΅Π»ΠΈΡ‡ΠΈΠ½Ρ‹ ΠΎΡ‚ сопротивлСния зависят, ΠΏΡ€ΠΈΡ‡Π΅ΠΌ эти зависимости ΠΌΠΎΠ³ΡƒΡ‚ Π±Ρ‹Ρ‚ΡŒ Ρ€Π°Π·Π½Ρ‹ΠΌΠΈ для Ρ€Π°Π·Π½Ρ‹Ρ… источников Ρ‚ΠΎΠΊΠ°. Π’Π½ΡƒΡ‚Ρ€Π΅Π½Π½Π΅Π΅ сопротивлСниС Π±Ρ‹Ρ‚ΠΎΠ²Ρ‹Ρ… элСктричСских сСтСй ΠΎΡ‡Π΅Π½ΡŒ ΠΌΠ°Π»ΠΎ. Π’ этом случаС ΠΈΠ· Π·Π°ΠΊΠΎΠ½ΠΎΠ² Ома для Π·Π°ΠΌΠΊΠ½ΡƒΡ‚ΠΎΠΉ Ρ†Π΅ΠΏΠΈ ΠΈ участка Ρ†Π΅ΠΏΠΈ (21.7), (21.3) слСдуСт, Ρ‡Ρ‚ΠΎ напряТСниС Π½Π° любом элСмСнтС, Π²ΠΊΠ»ΡŽΡ‡Π΅Π½Π½ΠΎΠΌ Π² Ρ‚Π°ΠΊΡƒΡŽ ΡΠ΅Ρ‚ΡŒ, Π½Π΅ зависит ΠΎΡ‚ сопротивлСния этого элСмСнта ΠΈ Ρ€Π°Π²Π½ΠΎ Π½ΠΎΠΌΠΈΠ½Π°Π»ΡŒΠ½ΠΎΠΌΡƒ Π½Π°ΠΏΡ€ΡΠΆΠ΅Π½ΠΈΡŽ сСти . ΠŸΠΎΡΡ‚ΠΎΠΌΡƒ ΠΈΠ· Ρ„ΠΎΡ€ΠΌΡƒΠ»Ρ‹ Π·Π°ΠΊΠ»ΡŽΡ‡Π°Π΅ΠΌ, Ρ‡Ρ‚ΠΎ ΠΌΠΎΡ‰Π½ΠΎΡΡ‚ΡŒ, которая выдСляСтся Π½Π° Ρ‚Π°ΠΊΠΎΠΌ элСмСнтС ΠΎΠ±Ρ€Π°Ρ‚Π½ΠΎ ΠΏΡ€ΠΎΠΏΠΎΡ€Ρ†ΠΈΠΎΠ½Π°Π»ΡŒΠ½Π° Π΅Π³ΠΎ ΡΠΎΠΏΡ€ΠΎΡ‚ΠΈΠ²Π»Π΅Π½ΠΈΡŽ (ΠΎΡ‚Π²Π΅Ρ‚ 3). ΠžΡ‚ΠΌΠ΅Ρ‚ΠΈΠΌ, Ρ‡Ρ‚ΠΎ ΠΈΠ· ΠΏΡ€ΠΎΠ²Π΅Π΄Π΅Π½Π½ΠΎΠ³ΠΎ рассуТдСния слСдуСт, Ρ‡Ρ‚ΠΎ выдСляСмая ΠΌΠΎΡ‰Π½ΠΎΡΡ‚ΡŒ Π±ΡƒΠ΄Π΅Ρ‚ ΠΎΡ‡Π΅Π½ΡŒ большой (опасная Π² Π±Ρ‹Ρ‚Ρƒ ситуация!) для ΠΌΠ°Π»ΠΎΠ³ΠΎ сопротивлСния внСшнСго участка Ρ†Π΅ΠΏΠΈ, Ρ‚.Π΅. Π² случаС ΠΊΠΎΡ€ΠΎΡ‚ΠΊΠΎΠ³ΠΎ замыкания, ΠΊΠΎΡ‚ΠΎΡ€ΠΎΠ³ΠΎ, Ρ‚Π°ΠΊΠΈΠΌ ΠΎΠ±Ρ€Π°Π·ΠΎΠΌ, Π½Π΅ΠΎΠ±Ρ…ΠΎΠ΄ΠΈΠΌΠΎ ΠΈΠ·Π±Π΅Π³Π°Ρ‚ΡŒ.

Если Π±Ρ‹ Π²Π½ΡƒΡ‚Ρ€Π΅Π½Π½Π΅Π΅ сопротивлСниС источника Π±Ρ‹Π»ΠΎ Π±Ρ‹ ΠΌΠ½ΠΎΠ³ΠΎ большС внСшнСго сопротивлСния, Ρ‚ΠΎΠΊ Π² Ρ†Π΅ΠΏΠΈ опрСдСлялся Π±Ρ‹, Π³Π»Π°Π²Π½Ρ‹ΠΌ ΠΎΠ±Ρ€Π°Π·ΠΎΠΌ, Π²Π½ΡƒΡ‚Ρ€Π΅Π½Π½ΠΈΠΌ сопротивлСниСм источника, Π° ΠΎΡ‚ внСшнСго сопротивлСния зависСл Π±Ρ‹ слабо. Π’ этом случаС ΠΌΠΎΡ‰Π½ΠΎΡΡ‚ΡŒ Ρ‚ΠΎΠΊΠ° Π±Ρ‹Π»Π° Π±Ρ‹ прямо ΠΏΡ€ΠΎΠΏΠΎΡ€Ρ†ΠΈΠΎΠ½Π°Π»ΡŒΠ½Π° ΡΠΎΠΏΡ€ΠΎΡ‚ΠΈΠ²Π»Π΅Π½ΠΈΡŽ участка Ρ†Π΅ΠΏΠΈ.

Как ΠΎΠ±ΡΡƒΠΆΠ΄Π°Π»ΠΎΡΡŒ Π² Ρ€Π΅ΡˆΠ΅Π½ΠΈΠΈ ΠΏΡ€Π΅Π΄Ρ‹Π΄ΡƒΡ‰Π΅ΠΉ Π·Π°Π΄Π°Ρ‡ΠΈ, сопротивлСниС элСмСнта, Ρ€Π°Π±ΠΎΡ‚Π°ΡŽΡ‰Π΅Π³ΠΎ Π² Π±Ρ‹Ρ‚ΠΎΠ²ΠΎΠΉ элСктросСти Ρ€Π°Π²Π½ΠΎ , Π³Π΄Π΅ β€” номинальная ΠΌΠΎΡ‰Π½ΠΎΡΡ‚ΡŒ Π΄Π°Π½Π½ΠΎΠ³ΠΎ элСмСнта, β€” напряТСниС Π² сСти. ΠŸΠΎΡΡ‚ΠΎΠΌΡƒ ΠΎΡ‚Π½ΠΎΡˆΠ΅Π½ΠΈΠ΅ сопротивлСний Π»Π°ΠΌΠΏ ΠΌΠΎΡ‰Π½ΠΎΡΡ‚ΡŒΡŽ Π’Ρ‚ ΠΈ Π’Ρ‚, рассчитанных Π½Π° Ρ€Π°Π±ΠΎΡ‚Ρƒ Π² ΠΎΠ΄Π½ΠΎΠΉ ΠΈ Ρ‚ΠΎΠΉ ΠΆΠ΅ Π±Ρ‹Ρ‚ΠΎΠ²ΠΎΠΉ элСктричСской сСти (Π·Π°Π΄Π°Ρ‡Π° 21.2.3) Ρ€Π°Π²Π½ΠΎ


(ΠΎΡ‚Π²Π΅Ρ‚ 2).

ΠŸΠΎΡΠΊΠΎΠ»ΡŒΠΊΡƒ рСзисторы Π² Π·Π°Π΄Π°Ρ‡Π΅ 21.2.4 соСдинСны ΠΏΠΎΡΠ»Π΅Π΄ΠΎΠ²Π°Ρ‚Π΅Π»ΡŒΠ½ΠΎ, Ρ‚ΠΎ сила Ρ‚ΠΎΠΊΠ° Π² Π½ΠΈΡ… ΠΎΠ΄ΠΈΠ½Π°ΠΊΠΎΠ²Π°. ΠŸΠΎΡΡ‚ΠΎΠΌΡƒ ΠΈΠ· Π·Π°ΠΊΠΎΠ½Π° Ома для участка Ρ†Π΅ΠΏΠΈ Π·Π°ΠΊΠ»ΡŽΡ‡Π°Π΅ΠΌ, Ρ‡Ρ‚ΠΎ


(ΠΎΡ‚Π²Π΅Ρ‚ 2).

ΠŸΡ€ΠΈ ΠΏΠ°Ρ€Π°Π»Π»Π΅Π»ΡŒΠ½ΠΎΠΌ соСдинСнии Π»Π°ΠΌΠΏ (Π·Π°Π΄Π°Ρ‡Π° 21.2.5) напряТСниС Π½Π° Π½ΠΈΡ… ΠΎΠ΄ΠΈΠ½Π°ΠΊΠΎΠ²ΠΎ (см. Π²Π²Π΅Π΄Π΅Π½ΠΈΠ΅ ΠΊ настоящСй Π³Π»Π°Π²Π΅). ΠŸΠΎΡΡ‚ΠΎΠΌΡƒ ΠΈΠ· Π·Π°ΠΊΠΎΠ½Π° Ома для участка Ρ†Π΅ΠΏΠΈ слСдуСт, Ρ‡Ρ‚ΠΎ


(ΠΎΡ‚Π²Π΅Ρ‚ 1).

РассматриваСмый Π² Π·Π°Π΄Π°Ρ‡Π΅ 21.2.6 участок прСдставляСт собой Π΄Π²Π° ΠΏΠΎΡΠ»Π΅Π΄ΠΎΠ²Π°Ρ‚Π΅Π»ΡŒΠ½Ρ‹Ρ… соСдинСнных элСмСнта, ΠΎΠ΄ΠΈΠ½ ΠΈΠ· ΠΊΠΎΡ‚ΠΎΡ€Ρ‹Ρ… Π΅ΡΡ‚ΡŒ рСзистор 6 Ом, Π²Ρ‚ΠΎΡ€ΠΎΠΉ β€” Π΄Π²Π° Ρ‚Π°ΠΊΠΈΡ… ΠΆΠ΅ рСзистора, соСдинСнных ΠΏΠ°Ρ€Π°Π»Π»Π΅Π»ΡŒΠ½ΠΎ. По ΠΏΡ€Π°Π²ΠΈΠ»Π°ΠΌ слоТСния сопротивлСний Π½Π°Ρ…ΠΎΠ΄ΠΈΠΌ эквивалСнтноС сопротивлСниС Π²Ρ‚ΠΎΡ€ΠΎΠ³ΠΎ участка


Π° Π·Π°Ρ‚Π΅ΠΌ ΠΈ эквивалСнтноС сопротивлСниС всСй Ρ†Π΅ΠΏΠΈ


(ΠΎΡ‚Π²Π΅Ρ‚ 3).

ΠŸΡ€ΠΈ Ρ€Π°Π·ΠΎΠΌΠΊΠ½ΡƒΡ‚ΠΎΠΌ ΠΊΠ»ΡŽΡ‡Π΅ сопротивлСниС участка Ρ†Π΅ΠΏΠΈ, Π΄Π°Π½Π½ΠΎΠ³ΠΎ Π² Π·Π°Π΄Π°Ρ‡Π΅ 21.2.7, ΠΌΠΎΠΆΠ½ΠΎ Π½Π°ΠΉΡ‚ΠΈ ΠΊΠ°ΠΊ Π² ΠΏΡ€Π΅Π΄Ρ‹Π΄ΡƒΡ‰Π΅ΠΉ Π·Π°Π΄Π°Ρ‡Π΅ , Π³Π΄Π΅ β€” сопротивлСниС ΠΊΠ°ΠΆΠ΄ΠΎΠ³ΠΎ рСзистора. Если ΠΊΠ»ΡŽΡ‡ Π·Π°ΠΌΠΊΠ½ΡƒΡ‚, Ρ‚ΠΎ Ρ†Π΅ΠΏΡŒ сводится ΠΊ ΠΎΠ΄Π½ΠΎΠΌΡƒ рСзистору (Ρ‚.ΠΊ. ΠΏΠ°Ρ€Π°Π»Π»Π΅Π»ΡŒΠ½ΠΎ Π΄Π²ΡƒΠΌ рСзисторам Π²ΠΊΠ»ΡŽΡ‡Π°Π΅Ρ‚ΡΡ ΠΏΡ€ΠΎΠ²ΠΎΠ΄Π½ΠΈΠΊ с ΠΏΡ€Π΅Π½Π΅Π±Ρ€Π΅ΠΆΠΈΠΌΠΎ ΠΌΠ°Π»Ρ‹ΠΌ сопротивлСниСм). ΠŸΠΎΡΡ‚ΠΎΠΌΡƒ Π² этом случаС сопротивлСниС Ρ†Π΅ΠΏΠΈ Ρ€Π°Π²Π½ΠΎ . Π’Π°ΠΊΠΈΠΌ ΠΎΠ±Ρ€Π°Π·ΠΎΠΌ, сопротивлСниС Π²Ρ‚ΠΎΡ€ΠΎΠΉ Ρ†Π΅ΠΏΠΈ составляСт Π΄Π²Π΅ Ρ‚Ρ€Π΅Ρ‚ΠΈ ΠΎΡ‚ сопротивлСния ΠΏΠ΅Ρ€Π²ΠΎΠΉ (ΠΎΡ‚Π²Π΅Ρ‚ 1).

Как ΠΎΠ±ΡΡƒΠΆΠ΄Π°Π»ΠΎΡΡŒ Π² Ρ€Π΅ΡˆΠ΅Π½ΠΈΠΈ Π·Π°Π΄Π°Ρ‡ΠΈ 21.2.2, сопротивлСниС элСмСнта номинальной мощности , Ρ€Π°Π±ΠΎΡ‚Π°ΡŽΡ‰Π΅Π³ΠΎ Π² Π±Ρ‹Ρ‚ΠΎΠ²ΠΎΠΉ элСктросСти Ρ€Π°Π²Π½Π°


Π³Π΄Π΅ Π’ β€” напряТСниС сСти. Из этой Ρ„ΠΎΡ€ΠΌΡƒΠ»Ρ‹ слСдуСт, Ρ‡Ρ‚ΠΎ Ρ‡Π΅ΠΌ большС номинальная ΠΌΠΎΡ‰Π½ΠΎΡΡ‚ΡŒ элСмСнта, Ρ‚Π΅ΠΌ мСньшС Π΄ΠΎΠ»ΠΆΠ½ΠΎ Π±Ρ‹Ρ‚ΡŒ Π΅Π³ΠΎ сопротивлСниС. Если Π΄Π²Π΅ Π»Π°ΠΌΠΏΡ‹ накаливания Π²ΠΊΠ»ΡŽΡ‡Π΅Π½Ρ‹ ΠΏΠΎΡΠ»Π΅Π΄ΠΎΠ²Π°Ρ‚Π΅Π»ΡŒΠ½ΠΎ (Π·Π°Π΄Π°Ρ‡Π° 21.2.8), Ρ‚ΠΎ сила Ρ‚ΠΎΠΊΠ° Π² Π½ΠΈΡ… ΠΎΠ΄ΠΈΠ½Π°ΠΊΠΎΠ²Π° ΠΈ ΠΎΡ‚Π½ΠΎΡˆΠ΅Π½ΠΈΠ΅ мощностСй Ρ‚ΠΎΠΊΠ° Π² этих Π»Π°ΠΌΠΏΠ°Ρ… Ρ€Π°Π²Π½ΠΎ ΠΎΡ‚Π½ΠΎΡˆΠ΅Π½ΠΈΡŽ ΠΈΡ… сопротивлСний. ΠžΡ‚ΡΡŽΠ΄Π° слСдуСт, Ρ‡Ρ‚ΠΎ ΠΎΡ‚Π½ΠΎΡˆΠ΅Π½ΠΈΠ΅ Ρ€Π΅Π°Π»ΡŒΠ½ΠΎ выдСляСмых Π² Π»Π°ΠΌΠΏΠ°Ρ… мощностСй ΠΈ ΠΎΠ±Ρ€Π°Ρ‚Π½ΠΎ ΠΎΡ‚Π½ΠΎΡˆΠ΅Π½ΠΈΡŽ Π½ΠΎΠΌΠΈΠ½Π°Π»ΡŒΠ½Ρ‹Ρ… мощностСй этих Π»Π°ΠΌΠΏ:


(ΠΎΡ‚Π²Π΅Ρ‚ 2).

Π Π°Π±ΠΎΡ‚Π°, ΡΠΎΠ²Π΅Ρ€ΡˆΠ°Π΅ΠΌΠ°Ρ элСктричСским ΠΏΠΎΠ»Π΅ΠΌ Π² ΠΏΡ€ΠΎΠ²ΠΎΠ΄Π½ΠΈΠΊΠ΅ ΠΏΡ€ΠΈ ΠΏΡ€ΠΎΡ‚Π΅ΠΊΠ°Π½ΠΈΠΈ ΠΏΠΎ Π½Π΅ΠΌΡƒ элСктричСского Ρ‚ΠΎΠΊΠ°, прСвращаСтся Π² ΡΠ½Π΅Ρ€Π³ΠΈΡŽ Ρ‚ΠΎΠΊΠ°, которая Π·Π°Ρ‚Π΅ΠΌ прСвращаСтся Π² Ρ‚Π΅ΠΏΠ»ΠΎΠ²ΡƒΡŽ ΡΠ½Π΅Ρ€Π³ΠΈΡŽ. ΠŸΠΎΡΡ‚ΠΎΠΌΡƒ Ρ€Π°Π±ΠΎΡ‚Ρƒ поля ΠΌΠΎΠΆΠ½ΠΎ Π½Π°ΠΉΡ‚ΠΈ ΠΈΠ· Π·Π°ΠΊΠΎΠ½Π° ДТоуля-Π›Π΅Π½Ρ†Π°. Для Ρ€Π°Π±ΠΎΡ‚Ρ‹ поля Π·Π° врСмя ΠΏΠΎΠ»ΡƒΡ‡Π°Π΅ΠΌ . Из этой Ρ„ΠΎΡ€ΠΌΡƒΠ»Ρ‹ Π½Π°Ρ…ΠΎΠ΄ΠΈΠΌ сопротивлСниС ΠΏΡ€ΠΎΠ²ΠΎΠ΄Π½ΠΈΠΊΠ° Π² Π·Π°Π΄Π°Ρ‡Π΅ 21.2.9 β€”


(ΠΎΡ‚Π²Π΅Ρ‚ 1).

ΠŸΠΎΡΠΊΠΎΠ»ΡŒΠΊΡƒ ΠΏΡ€ΠΈ ΠΏΠΎΡΠ»Π΅Π΄ΠΎΠ²Π°Ρ‚Π΅Π»ΡŒΠ½ΠΎΠΌ соСдинСнии рСзисторов Ρ‚ΠΎΠΊ Ρ‡Π΅Ρ€Π΅Π· ΠΊΠ°ΠΆΠ΄Ρ‹ΠΉ ΠΈΠ· Π½ΠΈΡ… ΠΎΠ΄ΠΈΠ½Π°ΠΊΠΎΠ², ΠΈΠ· Π·Π°ΠΊΠΎΠ½Π° ДТоуля-Π›Π΅Π½Ρ†Π° (22.8) Π·Π°ΠΊΠ»ΡŽΡ‡Π°Π΅ΠΌ, Ρ‡Ρ‚ΠΎ ΠΈΠ· Π΄Π²ΡƒΡ… сопротивлСний ΠΈ (Π·Π°Π΄Π°Ρ‡Π° 21.2.10; см. рисунок) наибольшСй Π±ΡƒΠ΄Π΅Ρ‚ ΠΌΠΎΡ‰Π½ΠΎΡΡ‚ΡŒ Ρ‚ΠΎΠΊΠ° Π½Π° сопротивлСнии , ΠΈΠ· Π΄Π²ΡƒΡ… сопротивлСний ΠΈ β€” Π½Π° сопротивлСнии . Π‘Ρ€Π°Π²Π½ΠΈΠΌ мощности Ρ‚ΠΎΠΊΠ° Π½Π° этих сопротивлСниях. Учитывая, Ρ‡Ρ‚ΠΎ ΠΏΡ€ΠΈ ΠΏΠ°Ρ€Π°Π»Π»Π΅Π»ΡŒΠ½ΠΎΠΌ соСдинСнии элСмСнтов элСктричСскоС напряТСниС Π½Π° ΠΊΠ°ΠΆΠ΄ΠΎΠΌ элСмСнтС ΠΎΠ΄ΠΈΠ½Π°ΠΊΠΎΠ²ΠΎΠ΅, Π° ΠΏΡ€ΠΈ ΠΏΠΎΡΠ»Π΅Π΄ΠΎΠ²Π°Ρ‚Π΅Π»ΡŒΠ½ΠΎΠΌ β€” ΡΠΊΠ»Π°Π΄Ρ‹Π²Π°ΡŽΡ‚ΡΡ значСния сопротивлСний, ΠΏΠΎΠ»ΡƒΡ‡ΠΈΠΌ ΠΈΠ· Π·Π°ΠΊΠΎΠ½ΠΎΠ² Ома для Π²Π΅Ρ€Ρ…Π½Π΅Π³ΠΎ ΠΈ Π½ΠΈΠΆΠ½Π΅Π³ΠΎ участков Ρ†Π΅ΠΏΠΈ ΠΈ Π·Π°ΠΊΠΎΠ½Π° ДТоуля-Π›Π΅Π½Ρ†Π°


Π³Π΄Π΅ β€” элСктричСскоС напряТСниС, ΠΏΡ€ΠΈΠ»ΠΎΠΆΠ΅Π½Π½ΠΎΠ΅ ΠΊΠΎ всСй Ρ†Π΅ΠΏΠΈ. ΠŸΠΎΡΠΊΠΎΠ»ΡŒΠΊΡƒ Ρ‚ΠΎ Π² прСдставлСнной схСмС наибольшая ΠΌΠΎΡ‰Π½ΠΎΡΡ‚ΡŒ Π±ΡƒΠ΄Π΅Ρ‚ Π²Ρ‹Π΄Π΅Π»ΡΡ‚ΡŒΡΡ Π½Π° сопротивлСнии (ΠΎΡ‚Π²Π΅Ρ‚ 2).

Π£Ρ€ΠΎΠΊ β„–6 Π’Π΅ΠΌΠ°: Β«Π—Π°ΠΊΠΎΠ½Ρ‹ постоянного Ρ‚ΠΎΠΊΠ°Β»

Π€ΠΈΠ·ΠΈΠΊΠ°. Π£Ρ€ΠΎΠΊ β„–6

Π’Π΅ΠΌΠ°: Β«Π—Π°ΠΊΠΎΠ½Ρ‹ постоянного
Ρ‚ΠΎΠΊΠ°Β».

Π—Π°Π΄Π°Ρ‡Π° 1.

РСзистор ΠΈ кондСнсатор соСдинСны
ΠΏΠΎΡΠ»Π΅Π΄ΠΎΠ²Π°Ρ‚Π΅Π»ΡŒΠ½ΠΎ с аккумулятором; ΠΏΡ€ΠΈ
этом заряд Π½Π° ΠΎΠ±ΠΊΠ»Π°Π΄ΠΊΠ°Ρ… кондСнсатора
q1
=

.
Если ΠΆΠ΅ рСзистор ΠΈ кондСнсатор ΠΏΠΎΠ΄ΠΊΠ»ΡŽΡ‡ΠΈΡ‚ΡŒ
ΠΊ аккумулятору ΠΏΠ°Ρ€Π°Π»Π»Π΅Π»ΡŒΠ½ΠΎ, Ρ‚ΠΎ заряд
Π½Π° ΠΎΠ±ΠΊΠ»Π°Π΄ΠΊΠ°Ρ… кондСнсатора q2
=

.
Найти Π²Π½ΡƒΡ‚Ρ€Π΅Π½Π½Π΅Π΅ сопротивлСниС
аккумулятора, Ссли сопротивлСниС
рСзистора R
= 45 Ом.

Π”Π°Π½ΠΎ: q1
=

,
q2
=

,
R
= 45 Ом.

Найти: r
— ?

РСшСниС:

Π’ ΠΏΠ΅Ρ€Π²ΠΎΠΌ случаС (рис.1) Ρ‚ΠΎΠΊΠ° Π² Ρ†Π΅ΠΏΠΈ
Π½Π΅Ρ‚, напряТСниС Π½Π° кондСнсаторС U1
Ρ€Π°Π²Π½ΠΎ Π­Π”Π‘ источника Ξ΅,
поэтому заряд Π½Π° кондСнсаторС Π΅ΠΌΠΊΠΎΡΡ‚ΡŒΡŽ
Π‘:

(1).

Рис. 1
Рис.
2

Π’ΠΎ Π²Ρ‚ΠΎΡ€ΠΎΠΌ случаС (рис.2) отсутствуСт
Ρ‚ΠΎΠΊ Π² Π²Π΅Ρ‚Π²ΠΈ АБВ, напряТСниС Π½Π° кондСнсаторС
Ρ‚Π°ΠΊΠΎΠ΅ ΠΆΠ΅ , ΠΊΠ°ΠΊ ΠΈ Π½Π° ΠΏΠΎΠ΄ΠΊΠ»ΡŽΡ‡Π΅Π½Π½ΠΎΠΌ ΠΊ Π½Π΅ΠΌΡƒ
ΠΏΠ°Ρ€Π°Π»Π»Π΅Π»ΡŒΠ½ΠΎ рСзисторС, Ρ‚.Π΅.
,
Π³Π΄Π΅ I
– сила Ρ‚ΠΎΠΊΠ°. Π•Π΅ ΠΌΡ‹ Π½Π°ΠΉΠ΄Π΅ΠΌ
ΠΏΠΎ Π·Π°ΠΊΠΎΠ½Ρƒ Ома для Π·Π°ΠΌΠΊΠ½ΡƒΡ‚ΠΎΠΉ Ρ†Π΅ΠΏΠΈ:

, Π³Π΄Π΅ r
– Π²Π½ΡƒΡ‚Ρ€Π΅Π½Π½Π΅Π΅ сопротивлСниС аккумулятора.
Π’ΠΎΠ³Π΄Π°:

, Π° заряд Π½Π° кондСнсаторС:

. (2)

На основании Π²Ρ‹Ρ€Π°ΠΆΠ΅Π½ΠΈΠΉ (1) ΠΈ (2)
ΠΏΠΎΠ»ΡƒΡ‡ΠΈΠΌ ΡƒΡ€Π°Π²Π½Π΅Π½ΠΈΠ΅:

, Ρ€Π΅ΡˆΠΈΠ² ΠΊΠΎΡ‚ΠΎΡ€ΠΎΠ΅ ΠΎΡ‚Π½ΠΎΡΠΈΡ‚Π΅Π»ΡŒΠ½ΠΎ r
, Π½Π°ΠΉΠ΄Π΅ΠΌ:

, r
= 23 Ом.

ΠžΡ‚Π²Π΅Ρ‚:

, r
= 23 Ом.

Рассмотрим Π±ΠΎΠ»Π΅Π΅ ΡΠ»ΠΎΠΆΠ½ΡƒΡŽ схСму, которая
ΠΏΡ€ΠΈΠ²Π΅Π΄Π΅Π½Π° Π½Π° рис. 3.

Π’ Ρ†Π΅ΠΏΠΈ сопротивлСния рСзисторов
R1,
R2,
R3
– соотвСтствСнно R1
= R2
= 2 Ом, R3
= 5Ом, Π­Π”Π‘ источника Ξ΅ =
34 Π’, Π΅Π³ΠΎ Π²Π½ΡƒΡ‚Ρ€Π΅Π½Π½Π΅Π΅ сопротивлСниС r
= 1 Ом, Π΅ΠΌΠΊΠΎΡΡ‚ΡŒ кондСнсатора Π‘
= 20 ΠΌΠΊΠ€. ΠžΠΏΡ€Π΅Π΄Π΅Π»ΠΈΡ‚ΡŒ, ΠΊΠ°ΠΊΠΎΠΉ
заряд q
ΠΏΡ€ΠΎΠΉΠ΄Π΅Ρ‚ Ρ‡Π΅Ρ€Π΅Π· ΠΊΠ»ΡŽΡ‡ К ΠΏΡ€ΠΈ
Π΅Π³ΠΎ Π·Π°ΠΌΡ‹ΠΊΠ°Π½ΠΈΠΈ.

Рис. 3

Π”Π°Π½ΠΎ: R1
= R2
= 2 Ом, R3
= 5Ом, Ξ΅ = 34 Π’,
r
= 1 Ом, Б = 20
ΠΌΠΊΠ€.

Найти: q
— ?

РСшСниС:

ΠŸΡ€ΠΈ Π·Π°ΠΌΠΊΠ½ΡƒΡ‚ΠΎΠΌ ΠΊΠ»ΡŽΡ‡Π΅ кондСнсатор
зарядится Π΄ΠΎ Π½Π΅ΠΊΠΎΡ‚ΠΎΡ€ΠΎΠ³ΠΎ напряТСния U,
послС Ρ‡Π΅Π³ΠΎ Ρ‚ΠΎΠΊ Ρ‡Π΅Ρ€Π΅Π· рСзистор R2
ΠΏΡ€ΠΎΡ…ΠΎΠ΄ΠΈΡ‚ΡŒ Π½Π΅ Π±ΡƒΠ΄Π΅Ρ‚. НапряТСниС U
Π½Π° кондСнсаторС Ρ€Π°Π²Π½ΠΎ Π½Π°ΠΏΡ€ΡΠΆΠ΅Π½ΠΈΡŽ ΠΌΠ΅ΠΆΠ΄Ρƒ
Ρ‚ΠΎΡ‡ΠΊΠ°ΠΌΠΈ А ΠΈ Π’. ΠΌΠ΅ΠΆΠ΄Ρƒ этими Ρ‚ΠΎΡ‡ΠΊΠ°ΠΌΠΈ
ΠΏΠ°Ρ€Π°Π»Π»Π΅Π»ΡŒΠ½ΠΎ Π²ΠΊΠ»ΡŽΡ‡Π΅Π½Ρ‹ рСзисторы R1
ΠΈ R3
(Ρ‚ΠΎΠΊ Ρ‡Π΅Ρ€Π΅Π· R2
Π½Π΅ ΠΈΠ΄Π΅Ρ‚). ΠŸΠΎΡΡ‚ΠΎΠΌΡƒ:

.

Заряд Π½Π° кондСнсаторС:

.

Π‘ΠΈΠ»Ρƒ Ρ‚ΠΎΠΊΠ° Π½Π°ΠΉΠ΄Π΅ΠΌ ΠΏΠΎ Π·Π°ΠΊΠΎΠ½Ρƒ Ома
для Π·Π°ΠΌΠΊΠ½ΡƒΡ‚ΠΎΠΉ Ρ†Π΅ΠΏΠΈ:

.

ΠŸΠΎΠ΄ΡΡ‚Π°Π²ΠΈΠ² это Π·Π½Π°Ρ‡Π΅Π½ΠΈΠ΅ Π² Π²Ρ‹Ρ€Π°ΠΆΠ΅Π½ΠΈΠ΅
для заряда, ΠΏΠΎΠ»ΡƒΡ‡ΠΈΠΌ:

, q
=
.

ΠžΡ‚Π²Π΅Ρ‚:

, q
=
.

Π—Π°Π΄Π°Ρ‡Π° 2.

Π”Π²Π° сопротивлСния R1=2
Ом и R2=8
Ом ΠΏΠΎΠ΄ΠΊΠ»ΡŽΡ‡Π΅Π½Ρ‹ Π² ΡΠ΅Ρ‚ΡŒ ΠΎΠ΄ΠΈΠ½ Ρ€Π°Π·
ΠΏΠΎΡΠ»Π΅Π΄ΠΎΠ²Π°Ρ‚Π΅Π»ΡŒΠ½ΠΎ, Π° Π²Ρ‚ΠΎΡ€ΠΎΠΉ Ρ€Π°Π· ΠΏΠ°Ρ€Π°Π»Π»Π΅Π»ΡŒΠ½ΠΎ.
Π’ΠΎ сколько Ρ€Π°Π· отличаСтся сила Ρ‚ΠΎΠΊΠ° Π²ΠΎ
Π²Ρ‚ΠΎΡ€ΠΎΠΉ Ρ†Π΅ΠΏΠΈ ΠΎΡ‚ силы Ρ‚ΠΎΠΊΠ° Π² ΠΏΠ΅Ρ€Π²ΠΎΠΉ Ρ†Π΅ΠΏΠΈ?
Π’ ΠΊΠ°ΠΊΠΎΠΉ Ρ†Π΅ΠΏΠΈ выдСлится большая ΠΌΠΎΡ‰Π½ΠΎΡΡ‚ΡŒ
ΠΈ Π²ΠΎ сколько Ρ€Π°Π·?

Π”Π°Π½ΠΎ: R1
= 2 Ом, R2
= 8 Ом

—————————————

Найти:

РСшСниС:

«По
ΡƒΠΌΠΎΠ»Ρ‡Π°Π½ΠΈΡŽΒ» ΠΌΠΎΠΆΠ½ΠΎ ΡΡ‡ΠΈΡ‚Π°Ρ‚ΡŒ, Ρ‡Ρ‚ΠΎ: 1) напряТСниС
Π² сСти постоянно, ΠΏΡƒΡΡ‚ΡŒ ΠΎΠ½ΠΎ Ρ€Π°Π²Π½ΠΎ U;
2) сопротивлСниСм ΡΠΎΠ΅Π΄ΠΈΠ½ΠΈΡ‚Π΅Π»ΡŒΠ½Ρ‹Ρ… ΠΏΡ€ΠΎΠ²ΠΎΠ΄ΠΎΠ²
ΠΌΠΎΠΆΠ½ΠΎ ΠΏΡ€Π΅Π½Π΅Π±Ρ€Π΅Ρ‡ΡŒ; 3) ΠΏΠΎΠ΄ силой Ρ‚ΠΎΠΊΠ° Π²ΠΎ
Π²Ρ‚ΠΎΡ€ΠΎΠΉ Ρ†Π΅ΠΏΠΈ подразумСваСтся сила Ρ‚ΠΎΠΊΠ°
Π² Π΅Π΅ Π½Π΅Ρ€Π°Π·Π²Π΅Ρ‚Π²Π»Π΅Π½Π½ΠΎΠΉ части

ΠŸΡ€ΠΈ ΠΏΠΎΡΠ»Π΅Π΄ΠΎΠ²Π°Ρ‚Π΅Π»ΡŒΠ½ΠΎΠΌ cΠΎΠ΅Π΄ΠΈΠ½Π΅Π½ΠΈΠΈ
ΠΎΠ±Ρ‰Π΅Π΅ сопротивлСниС Rn=R1+R2
Β­Β­Β­
ΠΈ сила Ρ‚ΠΎΠΊΠ°
.

ΠŸΡ€ΠΈ
ΠΏΠ°Ρ€Π°Π»Π»Π΅Π»ΡŒΠ½ΠΎΠΌ ΠΏΠΎΠ΄ΠΊΠ»ΡŽΡ‡Π΅Π½ΠΈΠΈ ΠΎΠ±Ρ‰Π΅Π΅
сопротивлСниС

ΠΈ Ρ‚ΠΎΠΊ Π² Π½Π΅Ρ€Π°Π·Π²Π΅Ρ‚Π²Π»Π΅Π½Π½ΠΎΠΉ части Ρ†Π΅ΠΏΠΈ
.

ΠžΡ‚Π½ΠΎΡˆΠ΅Π½ΠΈΠ΅
Ρ‚ΠΎΠΊΠΎΠ²
,
.

Π’Ρ‹Ρ€Π°ΠΆΠ°Π΅ΠΌ
мощности Ρ‡Π΅Ρ€Π΅Π· ΠΎΠ±Ρ‰Π΅Π΅ напряТСниС.
ΠœΠΎΡ‰Π½ΠΎΡΡ‚ΡŒ Π² ΠΏΠ΅Ρ€Π²ΠΎΠΉ Ρ†Π΅ΠΏΠΈ
,
ΠΌΠΎΡ‰Π½ΠΎΡΡ‚ΡŒ Π²ΠΎ Π²Ρ‚ΠΎΡ€ΠΎΠΉ Ρ†Π΅ΠΏΠΈ
.
Π’Π°ΠΊ ΠΊΠ°ΠΊ ΠΏΡ€ΠΈ ΠΏΠΎΡΠ»Π΅Π΄ΠΎΠ²Π°Ρ‚Π΅Π»ΡŒΠ½ΠΎΠΌ соСдинСнии
ΠΎΠ±Ρ‰Π΅Π΅ сопротивлСниС большС, Ρ‚ΠΎ Π 1

2
ΠΈ
,
.

ΠžΡ‚Π²Π΅Ρ‚:

,

.

Π—Π°Π΄Π°Ρ‡Π° 3.

Π˜ΠΌΠ΅Π΅Ρ‚ΡΡ источник Ρ‚ΠΎΠΊΠ° с Π­Π”Π‘,
Ρ€Π°Π²Π½ΠΎΠΉ Ξ΅, ΠΈ
Π²Π½ΡƒΡ‚Ρ€Π΅Π½Π½ΠΈΠΌ сопротивлСниСм r,
Π·Π°ΠΌΠΊΠ½ΡƒΡ‚Ρ‹ΠΉ Π½Π° рСостат (рис.4). Π’Ρ‹Ρ€Π°Π·ΠΈΡ‚ΡŒ
ΠΌΠΎΡ‰Π½ΠΎΡΡ‚ΡŒ P1,
Π²Ρ‹Π΄Π΅Π»ΡΠ΅ΠΌΡƒΡŽ Π²ΠΎ внСшнСй части Ρ†Π΅ΠΏΠΈ, ΠΊΠ°ΠΊ
Ρ„ΡƒΠ½ΠΊΡ†ΠΈΡŽ силы Ρ‚ΠΎΠΊΠ° I.
ΠŸΠΎΡΡ‚Ρ€ΠΎΠΈΡ‚ΡŒ Π³Ρ€Π°Ρ„ΠΈΠΊ этой Ρ„ΡƒΠ½ΠΊΡ†ΠΈΠΈ. ΠŸΡ€ΠΈ ΠΊΠ°ΠΊΠΎΠΉ
силС Ρ‚ΠΎΠΊΠ° эта ΠΌΠΎΡ‰Π½ΠΎΡΡ‚ΡŒ Π±ΡƒΠ΄Π΅Ρ‚ максимальной?

Рис. 4

Π”Π°Π½ΠΎ: Ξ΅, r.

Найти: P1
(I)
— ?
I
— ?

РСшСниС:

РазвиваСмая источником полная
ΠΌΠΎΡ‰Π½ΠΎΡΡ‚ΡŒ
.
Π§Π°ΡΡ‚ΡŒ этой мощности

выдСляСтся Π²Π½ΡƒΡ‚Ρ€ΠΈ источника, ΠΎΡΡ‚Π°Π»ΡŒΠ½Π°Ρ
– Π²ΠΎ внСшнСй Ρ†Π΅ΠΏΠΈ:

(1)

Π“Ρ€Π°Ρ„ΠΈΠΊΠΎΠΌ этой Ρ„ΡƒΠ½ΠΊΡ†ΠΈΠΈ (Ρ‚.Π΅. P1
(I))
являСтся ΠΏΠ°Ρ€Π°Π±ΠΎΠ»Π°, обращСнная вСтвями
Π²Π½ΠΈΠ·. Для построСния Π³Ρ€Π°Ρ„ΠΈΠΊΠ° ΠΏΡ€Π΅ΠΎΠ±Ρ€Π°Π·ΡƒΠ΅ΠΌ
Π²Ρ‹Ρ€Π°ΠΆΠ΅Π½ΠΈΠ΅ (1):

ΠžΡ‚ΡΡŽΠ΄Π° Π²ΠΈΠ΄Π½ΠΎ, Ρ‡Ρ‚ΠΎ ΠΊΠΎΠΎΡ€Π΄ΠΈΠ½Π°Ρ‚Ρ‹
Π²Π΅Ρ€ΡˆΠΈΠ½Ρ‹ ΠΏΠ°Ρ€Π°Π±ΠΎΠ»Ρ‹ ΡΠΎΠΎΡ‚Π²Π΅Ρ‚ΡΡ‚Π²ΡƒΡŽΡ‚ значСниям
,

(рис. 5).

Рис. 5

Π‘Π»Π΅Π΄ΠΎΠ²Π°Ρ‚Π΅Π»ΡŒΠ½ΠΎ, ΠΏΡ€ΠΈ Ρ‚ΠΎΠΊΠ΅ силой:

(2) ΠΌΠΎΡ‰Π½ΠΎΡΡ‚ΡŒ, выдСляСмая Π²ΠΎ
внСшнСй части Ρ†Π΅ΠΏΠΈ, Π±ΡƒΠ΄Π΅Ρ‚ ΠΈΠΌΠ΅Ρ‚ΡŒ
максимальноС Π·Π½Π°Ρ‡Π΅Π½ΠΈΠ΅:
.

ΠŸΡƒΡΡ‚ΡŒ внСшний участок Ρ†Π΅ΠΏΠΈ ΠΈΠΌΠ΅Π΅Ρ‚
Ρ‚Π°ΠΊΠΎΠ΅ сопротивлСниС R,
ΠΏΡ€ΠΈ ΠΊΠΎΡ‚ΠΎΡ€ΠΎΠΌ сила Ρ‚ΠΎΠΊΠ° Ρ€Π°Π²Π½Π°
I1
. Π’ΠΎΠ³Π΄Π° ΠΏΠΎ Π·Π°ΠΊΠΎΠ½Ρƒ Ома
для Π·Π°ΠΌΠΊΠ½ΡƒΡ‚ΠΎΠΉ Ρ†Π΅ΠΏΠΈ:
.

Бравнивая это Π²Ρ‹Ρ€Π°ΠΆΠ΅Π½ΠΈΠ΅ с Ρ„ΠΎΡ€ΠΌΡƒΠ»ΠΎΠΉ
(2), Π½Π°Ρ…ΠΎΠ΄ΠΈΠΌ, Ρ‡Ρ‚ΠΎ R
= r
.

Π’Π°ΠΊΠΈΠΌ ΠΎΠ±Ρ€Π°Π·ΠΎΠΌ, ΠΌΡ‹ ΠΏΡ€ΠΈΡ…ΠΎΠ΄ΠΈΠΌ ΠΊ
Π²Π°ΠΆΠ½ΠΎΠΌΡƒ Π²Ρ‹Π²ΠΎΠ΄Ρƒ: полСзная ΠΌΠΎΡ‰Π½ΠΎΡΡ‚ΡŒ
(ΠΌΠΎΡ‰Π½ΠΎΡΡ‚ΡŒ выдСляСмая Π½Π° внСшнСм участкС
Ρ†Π΅ΠΏΠΈ) максимальна Π² Ρ‚ΠΎΠΌ случаС, ΠΊΠΎΠ³Π΄Π°
Π²Π½ΡƒΡ‚Ρ€Π΅Π½Π½Π΅Π΅ сопротивлСниС источника
Ρ€Π°Π²Π½ΠΎ ΡΠΎΠΏΡ€ΠΎΡ‚ΠΈΠ²Π»Π΅Π½ΠΈΡŽ внСшнСго участка
Ρ†Π΅ΠΏΠΈ. ΠŸΡ€ΠΈ этом ΠšΠŸΠ” источника:

, ΠΈΠ»ΠΈ Ξ· =
50%.

Из Π³Ρ€Π°Ρ„ΠΈΠΊΠ°, ΠΏΡ€ΠΈΠ²Π΅Π΄Π΅Π½Π½ΠΎΠ³ΠΎ Π½Π°
рис.5, Ρ‚Π°ΠΊΠΆΠ΅ Π²ΠΈΠ΄Π½ΠΎ, Ρ‡Ρ‚ΠΎ ΠΊΠ°ΠΆΠ΄ΠΎΠΌΡƒ Π·Π½Π°Ρ‡Π΅Π½ΠΈΡŽ
ΠΏΠΎΠ»Π΅Π·Π½ΠΎΠΉ мощности, ΠΊΡ€ΠΎΠΌΠ΅ максимального,
ΡΠΎΠΎΡ‚Π²Π΅Ρ‚ΡΡ‚Π²ΡƒΡŽΡ‚ Π΄Π²Π° значСния сопротивлСния
внСшнСго участка Ρ†Π΅ΠΏΠΈ. ΠŸΡ€ΠΈ силС Ρ‚ΠΎΠΊΠ°
ΠΊΠΎΡ€ΠΎΡ‚ΠΊΠΎΠ³ΠΎ замыкания

полСзная ΠΌΠΎΡ‰Π½ΠΎΡΡ‚ΡŒ Ρ€Π°Π²Π½Π° Π½ΡƒΠ»ΡŽ.

ΠŸΠΎΠ»ΡƒΡ‡ΠΈΡ‚ΡŒ условиС,
Ρ‡Ρ‚ΠΎ ΠΌΠΎΡ‰Π½ΠΎΡΡ‚ΡŒ, выдСляСмая Π½Π° внСшнСм
участкС Ρ†Π΅ΠΏΠΈ максимальна Π² Ρ‚ΠΎΠΌ случаС,
ΠΊΠΎΠ³Π΄Π° Π²Π½ΡƒΡ‚Ρ€Π΅Π½Π½Π΅Π΅ сопротивлСниС источника
Ρ€Π°Π²Π½ΠΎ ΡΠΎΠΏΡ€ΠΎΡ‚ΠΈΠ²Π»Π΅Π½ΠΈΡŽ внСшнСго участка
Ρ†Π΅ΠΏΠΈ (Ρ‚.Π΅. r
= R),
ΠΌΠΎΠΆΠ½ΠΎ Π΄Ρ€ΡƒΠ³ΠΈΠΌ матСматичСским
способом.

Π’Ρ‹Ρ€Π°Π·ΠΈΠΌ ΠΌΠΎΡ‰Π½ΠΎΡΡ‚ΡŒ P1,
Π²Ρ‹Π΄Π΅Π»ΡΠ΅ΠΌΡƒΡŽ Π²ΠΎ внСшнСй Ρ†Π΅ΠΏΠΈ, ΠΊΠ°ΠΊ Ρ„ΡƒΠ½ΠΊΡ†ΠΈΡŽ
сопротивлСния R
этого участка, Ρ‚.Π΅.
.

ΠŸΡ€ΠΈ Π½Π΅ΠΊΠΎΡ‚ΠΎΡ€ΠΎΠΌ Π·Π½Π°Ρ‡Π΅Π½ΠΈΠΈ сопротивлСния
R,
Π·Π½Π°Ρ‡Π΅Π½ΠΈΠ΅ Ρ„ΡƒΠ½ΠΊΡ†ΠΈΠΈ максимально (экстрСмум
Ρ„ΡƒΠ½ΠΊΡ†ΠΈΠΈ)! ΠŸΠΎΡΡ‚ΠΎΠΌΡƒ пСрвая производная
этой Ρ„ΡƒΠ½ΠΊΡ†ΠΈΠΈ Π΄ΠΎΠ»ΠΆΠ½Π° Π±Ρ‹Ρ‚ΡŒ Ρ€Π°Π²Π½Π° Π½ΡƒΠ»ΡŽ,
Ρ‚.Π΅.
.

; =>

=>
.

ΠŸΡ€ΠΈ этом ΠΏΠΎΠ»ΡƒΡ‡Π΅Π½Π½ΠΎΠΌ условии
максимальная ΠΌΠΎΡ‰Π½ΠΎΡΡ‚ΡŒ
,
ΠΈΠ»ΠΈ
.

ΠžΡ‚Π²Π΅Ρ‚:
,
.

Π—Π°Π΄Π°Ρ‡Π° 4.

Π”Π²Π° Π²ΠΎΠ»ΡŒΡ‚ΠΌΠ΅Ρ‚Ρ€Π° с Π²Π½ΡƒΡ‚Ρ€Π΅Π½Π½ΠΈΠΌΠΈ
сопротивлСниями R1
= 6,0 кОм и R2
= 4,0 кОм соСдинСны, ΠΊΠ°ΠΊ ΠΏΠΎΠΊΠ°Π·Π°Π½ΠΎ Π½Π° рисункС
6, сопротивлСниС R3
= 10,0 кОм, напряТСниС U
= 180 Π’. ΠžΠΏΡ€Π΅Π΄Π΅Π»ΠΈΡ‚ΡŒ показания Π²ΠΎΠ»ΡŒΡ‚ΠΌΠ΅Ρ‚Ρ€ΠΎΠ²
U1
ΠΈ U2
ΠΏΡ€ΠΈ Ρ€Π°Π·ΠΎΠΌΠΊΠ½ΡƒΡ‚ΠΎΠΌ ΠΈ ΠΏΡ€ΠΈ Π·Π°ΠΌΠΊΠ½ΡƒΡ‚ΠΎΠΌ ΠΊΠ»ΡŽΡ‡Π΅
К ΠΈ установкС Π΄Π²ΠΈΠΆΠΊΠ° D
Π½Π° сСрСдинС сопротивлСния R3.
На ΠΊΠ°ΠΊΠΈΠ΅ части R4
ΠΈ R5
Π±ΡƒΠ΄Π΅Ρ‚ Π΄Π΅Π»ΠΈΡ‚ΡŒ Π΄Π²ΠΈΠΆΠΎΠΊ сопротивлСниС R3
ΠΏΡ€ΠΈ ΠΎΠ΄ΠΈΠ½Π°ΠΊΠΎΠ²Ρ‹Ρ… показаниях Π²ΠΎΠ»ΡŒΡ‚ΠΌΠ΅Ρ‚Ρ€ΠΎΠ²?

Π”Π°Π½ΠΎ: R1
= 6,0 кОм , R2
= 4,0 кОм , R3
= 10,0 кОм, U
= 180 Π’.

Найти: U1
-? U2
— ?

-?
-?
R4
— ? R5
— ?

РСшСниС:

ΠŸΡ€ΠΈ Ρ€Π°Π·ΠΎΠΌΠΊΠ½ΡƒΡ‚ΠΎΠΌ ΠΊΠ»ΡŽΡ‡Π΅ К схСма,
прСдставлСнная Π½Π° рис. 6 эквивалСнтна
схСмС, ΡƒΠΊΠ°Π·Π°Π½Π½ΠΎΠΉ Π½Π° рис. 7. Π’.ΠΊ. Π²ΠΎΠ»ΡŒΡ‚ΠΌΠ΅Ρ‚Ρ€Ρ‹
присоСдинСны ΠΊ сопротивлСния R3
ΠΈ ΠΊ сСти ΠΏΠ°Ρ€Π°Π»Π»Π΅Π»ΡŒΠ½ΠΎ, Ρ‚ΠΎ силу Ρ‚ΠΎΠΊΠ°,
ΠΏΡ€ΠΎΡ‚Π΅ΠΊΠ°ΡŽΡ‰Π΅Π³ΠΎ Ρ‡Π΅Ρ€Π΅Π· Π²ΠΎΠ»ΡŒΡ‚ΠΌΠ΅Ρ‚Ρ€Ρ‹, ΠΌΠΎΠΆΠ½ΠΎ
Π½Π°ΠΉΡ‚ΠΈ ΠΏΠΎ Ρ„ΠΎΡ€ΠΌΡƒΠ»Π΅
.

Рис. 6 Рис. 7
Рис. 8

Π‘Π»Π΅Π΄ΠΎΠ²Π°Ρ‚Π΅Π»ΡŒΠ½ΠΎ, напряТСниС Π½Π°
ΠΏΠ΅Ρ€Π²ΠΎΠΌ ΠΈ Π²Ρ‚ΠΎΡ€ΠΎΠΌ Π²ΠΎΠ»ΡŒΡ‚ΠΌΠ΅Ρ‚Ρ€Π°Ρ… соотвСтствСнно
Ρ€Π°Π²Π½Ρ‹:

,
.

ΠŸΡ€ΠΈ Π·Π°ΠΌΠΊΠ½ΡƒΡ‚ΠΎΠΌ ΠΊΠ»ΡŽΡ‡Π΅ К схСма,
указанная Π½Π° рис. 6 эквивалСнтна схСмС,
прСдставлСнной Π½Π° рис. 8:

,

,

,

.

ΠžΡ‚Π²Π΅Ρ‚: U1=108
Π’, U2=72Π’,

,
,
,

Π—Π°Π΄Π°Ρ‡Π° 5.

Π’ΠΎ сколько Ρ€Π°Π· измСнится тСпловая
ΠΌΠΎΡ‰Π½ΠΎΡΡ‚ΡŒ, выдСляСмая Π² Ρ†Π΅ΠΏΠΈ, ΠΏΡ€ΠΈ ΠΏΠ΅Ρ€Π΅ΠΌΠ΅Π½Π΅
полярности Π½Π° ΠΊΠ»Π΅ΠΌΠΌΠ°Ρ… 1 ΠΈ 2(рис.9)? Π’Π΅Π»ΠΈΡ‡ΠΈΠ½Ρƒ
напряТСния Π½Π° ΠΊΠ»Π΅ΠΌΠΌΠ°Ρ… ΡΡ‡ΠΈΡ‚Π°Ρ‚ΡŒ постоянной,
Π΄ΠΈΠΎΠ΄Ρ‹ ΠΈΠ΄Π΅Π°Π»ΡŒΠ½Ρ‹ΠΌΠΈ, сопротивлСния
рСзисторов R1 = 10 Ом
, R2 = R3
= 5 Ом.

Π”Π°Π½ΠΎ: R1
= 10 Ом , R2 = R3
= 5 Ом.

Найти: P1
/P2
— ?

Рис. 9
Рис. 10

РСшСниС:

На рисункС 10 (Π° ΠΈ Π±) ΠΏΡ€ΠΈΠ²Π΅Π΄Π΅Π½Ρ‹
эквивалСнтныС схСмы, ΡΠΎΠΎΡ‚Π²Π΅Ρ‚ΡΡ‚Π²ΡƒΡŽΡ‰ΠΈΠ΅
ΠΎΠ±ΠΎΠΈΠΌ случаям. Π’ ΠΏΠ΅Ρ€Π²ΠΎΠΌ случаС ΠΌΠΎΡ‰Π½ΠΎΡΡ‚ΡŒ:

, Π²ΠΎ Π²Ρ‚ΠΎΡ€ΠΎΠΌ –

, Π³Π΄Π΅ U – напряТСниС Π½Π°
ΠΊΠ»Π΅ΠΌΠΌΠ°Ρ….

Π’ΠΎΠ³Π΄Π°

,
Ρ‚.Π΅. ΠΌΠΎΡ‰Π½ΠΎΡΡ‚ΡŒ ΡƒΠΌΠ΅Π½ΡŒΡˆΠΈΡ‚ΡΡ Π² 10 Ρ€Π°Π·.

ΠžΡ‚Π²Π΅Ρ‚:
.

Π—Π°Π΄Π°Ρ‡Π° 6.

Π§Π΅Ρ€Π΅Π· Π΄Π²ΡƒΡ…ΡΠ»Π΅ΠΊΡ‚Ρ€ΠΎΠ΄Π½ΡƒΡŽ Π»Π°ΠΌΠΏΡƒ
(Π΄ΠΈΠΎΠ΄) с плоскими элСктродами (рис. 11)
ΠΈΠ΄Π΅Ρ‚ Ρ‚ΠΎΠΊ силой I = 10
мА. НапряТСниС Π½Π° Π»Π°ΠΌΠΏΠ΅ U
= 100 Π’. Π‘ ΠΊΠ°ΠΊΠΎΠΉ силой Π΄Π΅ΠΉΡΡ‚Π²ΡƒΡŽΡ‚ Π½Π° Π°Π½ΠΎΠ΄
Π»Π°ΠΌΠΏΡ‹ ΠΏΠ°Π΄Π°ΡŽΡ‰ΠΈΠ΅ Π½Π° Π½Π΅Π³ΠΎ элСктроны, Ссли
ΡΠΊΠΎΡ€ΠΎΡΡ‚ΡŒ ΠΈΡ… Π²Π±Π»ΠΈΠ·ΠΈ ΠΊΠ°Ρ‚ΠΎΠ΄Π° Ρ€Π°Π²Π½Π° Π½ΡƒΠ»ΡŽ?
ΠžΡ‚Π½ΠΎΡˆΠ΅Π½ΠΈΠ΅ заряда элСктрона ΠΊ Π΅Π³ΠΎ массС
e / me
=.

Рис. 11

Π”Π°Π½ΠΎ:
I = 10 мА , U
= 100 Π’, e / me
=.

Найти: F
?

РСшСниС:

ΠŸΡƒΡΡ‚ΡŒ

– ΡΠΊΠΎΡ€ΠΎΡΡ‚ΡŒ элСктрона Π² ΠΌΠΎΠΌΠ΅Π½Ρ‚ соударСния
Π΅Π³ΠΎ с Π°Π½ΠΎΠ΄ΠΎΠΌ. Π—Π° врСмя t
ΠΏΡ€ΠΈ силС Ρ‚ΠΎΠΊΠ° I число
соударСний:
,
(1)

Π³Π΄Π΅ q – заряд,
пСрСносимый N элСктронами;
e – заряд элСктрона.

По Π²Ρ‚ΠΎΡ€ΠΎΠΌΡƒ Π·Π°ΠΊΠΎΠ½Ρƒ ΠΡŒΡŽΡ‚ΠΎΠ½Π° ΠΈΠΌΠΏΡƒΠ»ΡŒΡ
силы, Π΄Π΅ΠΉΡΡ‚Π²ΡƒΡŽΡ‰ΠΈΠΉ со стороны Π°Π½ΠΎΠ΄Π° Π½Π°
элСктроны ΠΏΡ€ΠΈ соударСниях, Ρ€Π°Π²Π΅Π½
измСнСнию суммарного ΠΈΠΌΠΏΡƒΠ»ΡŒΡΠ° элСктронов:

,

Π³Π΄Π΅ me
– масса элСктрона;

– ΡΠΊΠΎΡ€ΠΎΡΡ‚ΡŒ элСктрона послС соударСния.
Π’ проСкциях Π½Π° ΠΊΠΎΠΎΡ€Π΄ΠΈΠ½Π°Ρ‚Π½ΡƒΡŽ ось,
Π½Π°ΠΏΡ€Π°Π²Π»Π΅Π½Π½ΡƒΡŽ ΠΎΡ‚ Π°Π½ΠΎΠ΄Π° ΠΊ ΠΊΠ°Ρ‚ΠΎΠ΄Ρƒ, это
ΡƒΡ€Π°Π²Π½Π΅Π½ΠΈΠ΅ Π±ΡƒΠ΄Π΅Ρ‚ ΠΈΠΌΠ΅Ρ‚ΡŒ Π²ΠΈΠ΄:

,
ΠΈΠ»ΠΈ с ΡƒΡ‡Π΅Ρ‚ΠΎΠΌ Ρ‚ΠΎΠ³ΠΎ, Ρ‡Ρ‚ΠΎ
=
0,

,
(2)

Π³Π΄Π΅ F – ΠΌΠΎΠ΄ΡƒΠ»ΡŒ
суммарной силы, с ΠΊΠΎΡ‚ΠΎΡ€ΠΎΠΉ Π°Π½ΠΎΠ΄ дСйствуСт
Π½Π° элСктроны. Богласно Ρ‚Ρ€Π΅Ρ‚ΡŒΠ΅ΠΌΡƒ Π·Π°ΠΊΠΎΠ½Ρƒ
ΠΡŒΡŽΡ‚ΠΎΠ½Π°, с Ρ‚Π°ΠΊΠΎΠΉ ΠΆΠ΅ ΠΏΠΎ ΠΌΠΎΠ΄ΡƒΠ»ΡŽ силой
Π΄Π΅ΠΉΡΡ‚Π²ΡƒΡŽΡ‚ элСктроны Π½Π° Π°Π½ΠΎΠ΄. Из ΡΠΎΠΎΡ‚Π½ΠΎΡˆΠ΅Π½ΠΈΠΉ
(1) ΠΈ (2) ΠΏΠΎΠ»ΡƒΡ‡ΠΈΠΌ:
.
(3)

НайдСм ΠΌΠΎΠ΄ΡƒΠ»ΡŒ скорости
,
исходя ΠΈΠ· Ρ‚ΠΎΠ³ΠΎ, Ρ‡Ρ‚ΠΎ ΠΈΠ·ΠΌΠ΅Π½Π΅Π½ΠΈΠ΅ кинСтичСской
энСргии элСктрона Π² ΠΏΡ€ΠΎΠΌΠ΅ΠΆΡƒΡ‚ΠΊΠ΅ ΠΌΠ΅ΠΆΠ΄Ρƒ
Π°Π½ΠΎΠ΄ΠΎΠΌ ΠΈ ΠΊΠ°Ρ‚ΠΎΠ΄ΠΎΠΌ Ρ€Π°Π²Π½ΠΎ Ρ€Π°Π±ΠΎΡ‚Π΅ элСктричСского
поля:

.

ΠžΡ‚ΡΡŽΠ΄Π°

. ΠŸΠΎΠ΄ΡΡ‚Π°Π²ΠΈΠ² это Π·Π½Π°Ρ‡Π΅Π½ΠΈΠ΅ скорости Π²
Ρ„ΠΎΡ€ΠΌΡƒΠ»Ρƒ (3), ΠΏΠΎΠ»ΡƒΡ‡ΠΈΠΌ:

,
Н
.

ΠžΡ‚Π²Π΅Ρ‚:
Н.

Π—Π°Π΄Π°Ρ‡ΠΈ для ΡΠ°ΠΌΠΎΡΡ‚ΠΎΡΡ‚Π΅Π»ΡŒΠ½ΠΎΠ³ΠΎ
Ρ€Π΅ΡˆΠ΅Π½ΠΈΡ

Π—Π°Π΄Π°Ρ‡Π° 1.

ЭлСктричСский Π½Π°Π³Ρ€Π΅Π²Π°Ρ‚Π΅Π»ΡŒ
Ρ€Π°Π±ΠΎΡ‚Π°Π΅Ρ‚ ΠΎΡ‚ сСти с напряТСниСм U
= 120 Π’ ΠΏΡ€ΠΈ силС Ρ‚ΠΎΠΊΠ° I = 5,0
А. Π—Π° врСмя Ο„ = 20 ΠΌΠΈΠ½ Π½Π°Π³Ρ€Π΅Π²Π°Π΅Ρ‚ массу m
= 1,5 ΠΊΠ³ Π²ΠΎΠ΄Ρ‹ ΠΎΡ‚ Π½Π°Ρ‡Π°Π»ΡŒΠ½ΠΎΠΉ Ρ‚Π΅ΠΌΠΏΠ΅Ρ€Π°Ρ‚ΡƒΡ€Ρ‹ 16
ΠΎΠ‘ Π΄ΠΎ Ρ‚Π΅ΠΌΠΏΠ΅Ρ€Π°Ρ‚ΡƒΡ€Ρ‹ кипСния t2
= 100 ΠΎΠ‘. УдСльная Ρ‚Π΅ΠΏΠ»ΠΎΠ΅ΠΌΠΊΠΎΡΡ‚ΡŒ Π²ΠΎΠ΄Ρ‹

.
ΠžΠΏΡ€Π΅Π΄Π΅Π»ΠΈΡ‚ΡŒ коэффициСнт ΠΏΠΎΠ»Π΅Π·Π½ΠΎΠ³ΠΎ
дСйствия нагрСватСля (Π² %), ΠΎΠΏΡ€Π΅Π΄Π΅Π»ΠΈΡ‚ΡŒ
ΠΏΠΎΡ‚Π΅Ρ€ΠΈ энСргии Π² процСссС нагрСвания
().

Π—Π°Π΄Π°Ρ‡Π° 2.

Рис. 12

Зная всС ΠΏΠ°Ρ€Π°ΠΌΠ΅Ρ‚Ρ€Ρ‹ ΡƒΠΊΠ°Π·Π°Π½Π½ΠΎΠΉ Π½Π°
рисункС 12 схСмы (значСния Π­Π”Π‘ ΠΈ Π²Π½ΡƒΡ‚Ρ€Π΅Π½Π½Π΅Π³ΠΎ
сопротивлСния источника, сопротивлСния
рСзисторов ΠΈ ΡΠ»Π΅ΠΊΡ‚Ρ€ΠΎΠ΅ΠΌΠΊΠΎΡΡ‚ΡŒ кондСнсатора),
ΠΎΠΏΡ€Π΅Π΄Π΅Π»ΠΈΡ‚ΡŒ силу Ρ‚ΠΎΠΊΠ°, ΠΏΡ€ΠΎΡ‚Π΅ΠΊΠ°ΡŽΡ‰Π΅Π³ΠΎ Π²
Π½Π΅Ρ€Π°Π·Π²Π΅Ρ‚Π²Π»Π΅Π½Π½ΠΎΠΌ участкС Ρ†Π΅ΠΏΠΈ:

  1. сразу
    послС замыкания ΠΊΠ»ΡŽΡ‡Π° К;

  2. спустя
    достаточно большой ΠΏΡ€ΠΎΠΌΠ΅ΠΆΡƒΡ‚ΠΎΠΊ Π²Ρ€Π΅ΠΌΠ΅Π½ΠΈ
    послС замыкания ΠΊΠ»ΡŽΡ‡Π° К.

Π—Π°Π΄Π°Ρ‡Π° 3.

ΠŸΡ€ΠΈ ΠΏΠΎΠ΄ΠΊΠ»ΡŽΡ‡Π΅Π½ΠΈΠΈ рСзистора
сопротивлСниСм R = 15 Ом ΠΊ
источнику Ρ‚ΠΎΠΊΠ° с Π­Π”Π‘ Ξ΅=10Π’, ΠΌΠΎΡ‰Π½ΠΎΡΡ‚ΡŒ,
выдСляСмая Π½Π° этом рСзисторС, составляСт
Ξ· = 75% ΠΏΠΎΠ»Π½ΠΎΠΉ мощности, ΠΊΠΎΡ‚ΠΎΡ€ΡƒΡŽ Ρ€Π°Π·Π²ΠΈΠ²Π°Π΅Ρ‚
источник. ΠšΠ°ΠΊΡƒΡŽ ΠΌΠ°ΠΊΡΠΈΠΌΠ°Π»ΡŒΠ½ΡƒΡŽ ΠΌΠΎΡ‰Π½ΠΎΡΡ‚ΡŒ
ΠΌΠΎΠΆΠ΅Ρ‚ Π²Ρ‹Π΄Π΅Π»ΠΈΡ‚ΡŒ Π΄Π°Π½Π½Ρ‹ΠΉ источник Π²ΠΎ
внСшнСй Ρ†Π΅ΠΏΠΈ?

Π—Π°Π΄Π°Ρ‡Π° 4.

Какой заряд ΠΏΡ€ΠΎΡ…ΠΎΠ΄ΠΈΡ‚ Ρ‡Π΅Ρ€Π΅Π·
раствор ΠΌΠ΅Π΄Π½ΠΎΠ³ΠΎ купороса Π·Π° врСмя t
= 10 с, Ссли сила Ρ‚ΠΎΠΊΠ° Π·Π° это врСмя Ρ€Π°Π²Π½ΠΎΠΌΠ΅Ρ€Π½ΠΎ
возрастаСт ΠΎΡ‚ I1 = 0
A Π΄ΠΎ I2
= 4,0 А? Бколько ΠΌΠ΅Π΄ΠΈ выдСляСтся ΠΏΡ€ΠΈ этом
Π½Π° ΠΊΠ°Ρ‚ΠΎΠ΄Π΅? ЭлСктрохимичСский эквивалСнт
ΠΌΠ΅Π΄ΠΈ k =
.

Π—Π°Π΄Π°Ρ‡Π° 5.

Рис. 13

Если Π²ΠΎΠ»ΡŒΡ‚ΠΌΠ΅Ρ‚Ρ€, ΠΈΠΌΠ΅ΡŽΡ‰ΠΈΠΉ ΠΊΠΎΠ½Π΅Ρ‡Π½ΠΎΠ΅
сопротивлСниС, ΠΏΠΎΠ΄ΠΊΠ»ΡŽΡ‡Π΅Π½ ΠΏΠ°Ρ€Π°Π»Π»Π΅Π»ΡŒΠ½ΠΎ
ΠΊ рСзистору с сопротивлСниСм R1
Π² схСмС, ΡƒΠΊΠ°Π·Π°Π½Π½ΠΎΠΉ Π½Π° рис.13, Ρ‚ΠΎ ΠΎΠ½
ΠΏΠΎΠΊΠ°Π·Ρ‹Π²Π°Π΅Ρ‚ напряТСниС U1
= 6Π’. Если этот Π²ΠΎΠ»ΡŒΡ‚ΠΌΠ΅Ρ‚Ρ€ ΠΏΠΎΠ΄ΠΊΠ»ΡŽΡ‡ΠΈΡ‚ΡŒ
ΠΏΠ°Ρ€Π°Π»Π»Π΅Π»ΡŒΠ½ΠΎ ΠΊ рСзистору с сопротивлСниСм
R2, Ρ‚ΠΎ Π΅Π³ΠΎ показания
ΡΠΎΠΎΡ‚Π²Π΅Ρ‚ΡΡ‚Π²ΡƒΡŽΡ‚ Π½Π°ΠΏΡ€ΡΠΆΠ΅Π½ΠΈΡŽ U2
= 4 Π’. Каково Π±ΡƒΠ΄Π΅Ρ‚ напряТСниС Π½Π° ΠΊΠ°ΠΆΠ΄ΠΎΠΌ
ΠΈΠ· этих рСзисторов, Ссли ΠΏΠΎΠ΄ΠΊΠ»ΡŽΡ‡Π°Ρ‚ΡŒ
ΠΈΠ΄Π΅Π°Π»ΡŒΠ½Ρ‹ΠΉ Π²ΠΎΠ»ΡŒΡ‚ΠΌΠ΅Ρ‚Ρ€? Π­Π”Π‘ источника Ξ΅ =
12 Π’, Π΅Π³ΠΎ Π²Π½ΡƒΡ‚Ρ€Π΅Π½Π½Π΅Π΅ сопротивлСниС
ΠΏΡ€Π΅Π½Π΅Π±Ρ€Π΅ΠΆΠΈΠΌΠΎ ΠΌΠ°Π»ΠΎ.

Π“Π»Π°Π²Π°
1. Π§Ρ‚ΠΎ Π½Π° ΠΏΠ»Π°Ρ‚Π΅ элСктронного устройства?

resistor

РСзистор

Как
ΠΈ собирался, я с ΠΏΠΎΠΌΠΎΡ‰ΡŒΡŽ ΠΎΡ‚Π²Π΅Ρ€Ρ‚ΠΊΠΈ,
ΠΎΡ‚ΠΊΡ€ΡƒΡ‚ΠΈΠ² Π²ΠΈΠ½Ρ‚Ρ‹, ΠΎΡ‚ΠΊΡ€Ρ‹Π» испорчСнный
стрСлочный ΠΏΡ€ΠΈΠ±ΠΎΡ€, ΠΊΠΎΡ‚ΠΎΡ€Ρ‹ΠΉ нашСл Π² своСй
Β«Ρ…Π»Π°ΠΌΠ΅ΠΆΠΊΠ΅Β». Из Π½Π΅Π³ΠΎ ΡƒΠ΄Π°Π»ΠΎΡΡŒ ΠΈΠ·Π²Π»Π΅Ρ‡ΡŒ
нСсколько
ΠΏΠ»Π°Ρ‚ ΠΈΠ· стСклотСкстолита с ΠΏΠ΅Ρ‡Π°Ρ‚Π½Ρ‹ΠΌ
ΠΌΠΎΠ½Ρ‚Π°ΠΆΠΎΠΌ; Π½Π° ΠΏΠ»Π°Ρ‚Π°Ρ… ΠΌΠ½ΠΎΠ³ΠΎ рСзисторов,
Π΅ΡΡ‚ΡŒ кондСнсаторы, транзисторы, микросхСмы
– ΠΏΠΎΡ‡Ρ‚ΠΈ всС, Ρ‡Ρ‚ΠΎ ΠΌΠ½Π΅ Π½ΡƒΠΆΠ½ΠΎ для рассказа.

На
самой большой ΠΏΠ»Π°Ρ‚Π΅ ΠΏΠ΅Ρ€Π΅Π΄ΠΎ ΠΌΠ½ΠΎΠΉ, слСдовало
Π±Ρ‹ ΠΏΠΎΡΡ‡ΠΈΡ‚Π°Ρ‚ΡŒ, Π½ΠΎ Π½Π΅ хочСтся, Π½Π° ΠΏΠ΅Ρ€Π²Ρ‹ΠΉ
взгляд большС всСго рСзисторов. РСзистор
ΠΈΠ»ΠΈ сопротивлСниС, Π½Π°Π²Π΅Ρ€Π½ΠΎΠ΅, ΠΏΡ€ΠΎΡΡ‚Π΅ΠΉΡˆΠΈΠΉ
для понимания ΠΊΠΎΠΌΠΏΠΎΠ½Π΅Π½Ρ‚ любого
элСктронного устройства…

Π’ΠΎΡ‚
Ρ‚Π°ΠΊ, Π½Π΅ успСв Π½Π°Ρ‡Π°Ρ‚ΡŒ рассказ, я ΡƒΠΆΠ΅ сдСлал
ΠΎΡˆΠΈΠ±ΠΊΡƒ. Π‘ΠΎΠ»ΡŒΡˆΠ΅ всСго Π½Π° ΠΏΠ»Π°Ρ‚Π΅ Π½Π΅
рСзисторов, Π° ΠΏΡ€ΠΎΠ²ΠΎΠ΄Π½ΠΈΠΊΠΎΠ²! И, ΠΏΠΎΠΆΠ°Π»ΡƒΠΉ,
ΠΈΠΌΠ΅Π½Π½ΠΎ ΠΏΡ€ΠΎΠ²ΠΎΠ΄Π½ΠΈΠΊΠΈ самыС простыС
ΠΊΠΎΠΌΠΏΠΎΠ½Π΅Π½Ρ‚Ρ‹. ΠŸΡ€ΠΎΠ²ΠΎΠ΄Π½ΠΈΠΊΠΈ ΡΠΎΠ΅Π΄ΠΈΠ½ΡΡŽΡ‚ всС
ΠΎΡΡ‚Π°Π»ΡŒΠ½Ρ‹Π΅ элСмСнты устройства Π² слоТныС
ΠΈΠ»ΠΈ простыС Ρ†Π΅ΠΏΠΈ, поэтому ΡΠ»Π΅ΠΊΡ‚Ρ€ΠΈΡ‡Π΅ΡΠΊΡƒΡŽ
схСму я Π±ΡƒΠ΄Ρƒ часто Π½Π°Π·Ρ‹Π²Π°Ρ‚ΡŒ элСктричСской
Ρ†Π΅ΠΏΡŒΡŽ. Как ΠΏΡ€Π°Π²ΠΈΠ»ΠΎ, ΠΏΡ€ΠΎΠ²ΠΎΠ΄Π½ΠΈΠΊΠΈ Π΄Π΅Π»Π°ΡŽΡ‚
ΠΈΠ· ΠΌΠ΅Ρ‚Π°Π»Π»Π°, вСщСства Ρ…ΠΎΡ€ΠΎΡˆΠΎ проводящСго
элСктричСский Ρ‚ΠΎΠΊ. Если ΠΏΠΎΠ΄ Ρ‚ΠΎΠΊΠΎΠΌ
ΠΏΠΎΠ½ΠΈΠΌΠ°Ρ‚ΡŒ любоС Π½Π°ΠΏΡ€Π°Π²Π»Π΅Π½Π½ΠΎΠ΅ Π΄Π²ΠΈΠΆΠ΅Π½ΠΈΠ΅
элСктричСских зарядов, Ρ‚ΠΎ ΠΏΡ€ΠΎΠ²ΠΎΠ΄Π½ΠΈΠΊΠΈ
ΠΌΠ°Π»ΠΎ ΡΠΎΠΏΡ€ΠΎΡ‚ΠΈΠ²Π»ΡΡŽΡ‚ΡΡ этому двиТСнию, Ρ‚ΠΎ
Π΅ΡΡ‚ΡŒ, ΠΈΡ… сопротивлСниС ΠΎΠ±Ρ‹Ρ‡Π½ΠΎ Π½Π΅Π²Π΅Π»ΠΈΠΊΠΎ.
Бвойства ΠΏΡ€ΠΎΠ²ΠΎΠ΄Π½ΠΈΠΊΠΎΠ² Ρ…ΠΎΡ€ΠΎΡˆΠΎ понятны,
Ссли Ρ€Π°ΡΡΠΌΠ°Ρ‚Ρ€ΠΈΠ²Π°Ρ‚ΡŒ Π°Ρ‚ΠΎΠΌΠ½ΠΎΠ΅ строСниС
вСщСства, Π΄ΠΎΠ³ΠΎΠ²ΠΎΡ€ΠΈΠ²ΡˆΠΈΡΡŒ, Ρ‡Ρ‚ΠΎ Π°Ρ‚ΠΎΠΌΡ‹
состоят ΠΈΠ· тяТСлого элСктричСски
заряТСнного ядра ΠΈ Π»Π΅Π³ΠΊΠΈΡ… элСктронов,
субатомных частиц ΠΏΡ€ΠΎΡ‚ΠΈΠ²ΠΎΠΏΠΎΠ»ΠΎΠΆΠ½ΠΎΠ³ΠΎ
Π·Π½Π°ΠΊΠ°, располоТСнных Π²ΠΎΠΊΡ€ΡƒΠ³ ядра. Π£
Ρ€Π°Π·Π½Ρ‹Ρ… вСщСств заряд ядра Ρ€Π°Π·Π½Ρ‹ΠΉ, Π½ΠΎ
количСство элСктронов Ρ‚Π°ΠΊΠΎΠ΅, Ρ‡Ρ‚ΠΎ Π² Ρ†Π΅Π»ΠΎΠΌ
Π°Ρ‚ΠΎΠΌ элСктричСски Π½Π΅ΠΉΡ‚Ρ€Π°Π»Π΅Π½. Π£ ΠΌΠ΅Ρ‚Π°Π»Π»ΠΎΠ²,
ΡƒΠΆ Ρ‚Π°ΠΊ ΠΎΠ½ΠΈ устроСны, элСктроны, Π΄Π°Π»Π΅ΠΊΠΎ
располоТСнныС ΠΎΡ‚ ядра, слабо связаны с
Π½ΠΈΠΌ ΠΈ ΠΌΠΎΠ³ΡƒΡ‚ Β«Π±Ρ€ΠΎΠ΄ΠΈΡ‚ΡŒΒ» ΠΏΠΎ ΠΌΠ΅Ρ‚Π°Π»Π»Ρƒ
ΠΎΡ‚ Π°Ρ‚ΠΎΠΌΠ° ΠΊ Π°Ρ‚ΠΎΠΌΡƒ (Π½ΠΎ Π½Π΅ ΠΌΠΎΠ³ΡƒΡ‚ ΡΠ°ΠΌΠΎΠΏΡ€ΠΎΠΈΠ·Π²ΠΎΠ»ΡŒΠ½ΠΎ
ΠΏΠΎΠΊΠΈΠ½ΡƒΡ‚ΡŒ ΠΌΠ΅Ρ‚Π°Π»Π»). ДвиТутся ΠΎΠ½ΠΈ, ΠΊΠΎΠ½Π΅Ρ‡Π½ΠΎ,
бСспорядочно, Π½ΠΎ ΠΏΠΎΠ΄ дСйствиСм внСшнСго
элСктричСского поля, ΠΊΠΎΡ‚ΠΎΡ€ΠΎΠ΅ ΠΌΠΎΠΆΠ½ΠΎ
ΡΠΎΠ·Π΄Π°Ρ‚ΡŒ с ΠΏΠΎΠΌΠΎΡ‰ΡŒΡŽ источника питания,
Π΅Π³ΠΎ Π΅Ρ‰Π΅ Π½Π°Π·Ρ‹Π²Π°ΡŽΡ‚ источником элСктродвиТущСй
силы (Π±Π°Ρ‚Π°Ρ€Π΅ΠΉΠΊΠ°, аккумулятор, Π±Π»ΠΎΠΊ
питания), ΠΈΡ… Π΄Π²ΠΈΠΆΠ΅Π½ΠΈΠ΅ упорядочиваСтся
ΠΈ ΠΌΠΎΠΆΠ½ΠΎ Π³ΠΎΠ²ΠΎΡ€ΠΈΡ‚ΡŒ ΠΎ ΠΏΡ€ΠΎΡ‚Π΅ΠΊΠ°Π½ΠΈΠΈ Ρ‚ΠΎΠΊΠ° ΠΎΡ‚
ΠΎΠ΄Π½ΠΎΠ³ΠΎ полюса источника питания ΠΊ
Π΄Ρ€ΡƒΠ³ΠΎΠΌΡƒ; благодаря Π±ΠΎΠ»ΡŒΡˆΠΎΠΌΡƒ количСству
носитСлСй зарядов Π² ΠΌΠ΅Ρ‚Π°Π»Π»Π°Ρ…
(элСктронов-бродяг) Ρ‚Π΅ ΠΎΠΊΠ°Π·Ρ‹Π²Π°ΡŽΡ‚ΡΡ
Ρ…ΠΎΡ€ΠΎΡˆΠΈΠΌΠΈ ΠΏΡ€ΠΎΠ²ΠΎΠ΄Π½ΠΈΠΊΠ°ΠΌΠΈ Ρ‚ΠΎΠΊΠ°. Π—Π° тСхничСскоС
Π½Π°ΠΏΡ€Π°Π²Π»Π΅Π½ΠΈΠ΅ Ρ‚ΠΎΠΊΠ° принято Π½Π°ΠΏΡ€Π°Π²Π»Π΅Π½ΠΈΠ΅
ΠΎΡ‚ плюса источника Π­Π”Π‘ (элСктродвиТущСй
силы) ΠΊ минусу, хотя Ρ€Π΅Π°Π»ΡŒΠ½ΠΎ Π² ΠΌΠ΅Ρ‚Π°Π»Π»Π΅
ΠΏΠΎΠ΄ дСйствиСм внСшнСго элСктричСского
поля Π΄Π²ΠΈΠ³Π°Ρ‚ΡŒΡΡ Π±ΡƒΠ΄ΡƒΡ‚ ΠΎΡ‚Ρ€ΠΈΡ†Π°Ρ‚Π΅Π»ΡŒΠ½ΠΎ
заряТСнныС элСктроны ΠΎΡ‚ минуса источника,
ΠΏΠΎΡΡ‚Π°Π²Π»ΡΡŽΡ‰Π΅Π³ΠΎ элСктроны Π² ΠΌΠ΅Ρ‚Π°Π»Π», ΠΊ Π΅Π³ΠΎ
ΠΏΠ»ΡŽΡΡƒ. Если ΠΌΠΎΠΆΠ½ΠΎ ΠΏΠΎΡΡ‡ΠΈΡ‚Π°Ρ‚ΡŒ количСство
зарядов, проходящих Ρ‡Π΅Ρ€Π΅Π· ΠΏΠΎΠΏΠ΅Ρ€Π΅Ρ‡Π½ΠΎΠ΅
сСчСниС ΠΏΡ€ΠΎΠ²ΠΎΠ΄Π½ΠΈΠΊΠ°, Ρ‚ΠΎ ΠΌΠΎΠΆΠ½ΠΎ ΠΎΡ†Π΅Π½ΠΈΡ‚ΡŒ
силу Ρ‚ΠΎΠΊΠ° – Ρ‡Π΅ΠΌ большС зарядов ΠΏΡ€ΠΎΡ…ΠΎΠ΄ΠΈΡ‚
Ρ‡Π΅Ρ€Π΅Π· это сСчСниС, Ρ‚Π΅ΠΌ большС Ρ‚ΠΎΠΊ.
ΠžΠΏΡ€Π΅Π΄Π΅Π»ΡΠ΅Ρ‚ΡΡ сила Ρ‚ΠΎΠΊΠ° ΠΎΡ‚Π½ΠΎΡˆΠ΅Π½ΠΈΠ΅ΠΌ
количСства зарядов, ΠΏΡ€ΠΎΡˆΠ΅Π΄ΡˆΠΈΡ… Π·Π°
ΠΎΠΏΡ€Π΅Π΄Π΅Π»Π΅Π½Π½ΠΎΠ΅ врСмя Ρ‡Π΅Ρ€Π΅Π· ΠΏΠΎΠΏΠ΅Ρ€Π΅Ρ‡Π½ΠΎΠ΅
сСчСниС, ΠΊ этому Π²Ρ€Π΅ΠΌΠ΅Π½ΠΈ. И Π΅Ρ‰Π΅ ΠΎ Ρ‚ΠΎΠΊΠ΅
ΠΌΠΎΠΆΠ½ΠΎ ΡΠΊΠ°Π·Π°Ρ‚ΡŒ, Ρ‡Ρ‚ΠΎ Ссли Π΅Π³ΠΎ Π²Π΅Π»ΠΈΡ‡ΠΈΠ½Π° ΠΈ
Π½Π°ΠΏΡ€Π°Π²Π»Π΅Π½ΠΈΠ΅ Π½Π΅ мСняСтся со Π²Ρ€Π΅ΠΌΠ΅Π½Π΅ΠΌ,
Ρ‚ΠΎ ΠΌΡ‹ ΠΈΠΌΠ΅Π΅ΠΌ Π΄Π΅Π»ΠΎ с постоянным Ρ‚ΠΎΠΊΠΎΠΌ,
ΠΈΠ½Π°Ρ‡Π΅ с ΠΏΠ΅Ρ€Π΅ΠΌΠ΅Π½Π½Ρ‹ΠΌ Ρ‚ΠΎΠΊΠΎΠΌ. Π‘Π°Ρ‚Π°Ρ€Π΅ΠΉΠΊΠ° –
источник постоянного Ρ‚ΠΎΠΊΠ°, Π° силовая
ΡΠ΅Ρ‚ΡŒ, ΠΊΡƒΠ΄Π° ΠΌΡ‹ ΠΏΠΎΠ΄ΠΊΠ»ΡŽΡ‡Π°Π΅ΠΌ пылСсос ΠΈΠ»ΠΈ
Ρ‚Π΅Π»Π΅Π²ΠΈΠ·ΠΎΡ€, источник ΠΏΠ΅Ρ€Π΅ΠΌΠ΅Π½Π½ΠΎΠ³ΠΎ Ρ‚ΠΎΠΊΠ°.

ВсС
вСщСства ΠΏΠΎ Ρ‚ΠΎΠΌΡƒ, ΠΊΠ°ΠΊ ΠΎΠ½ΠΈ проводят
элСктричСский Ρ‚ΠΎΠΊ, ΠΌΠΎΠΆΠ½ΠΎ Ρ€Π°Π·Π΄Π΅Π»ΠΈΡ‚ΡŒ Π½Π°
ΠΏΡ€ΠΎΠ²ΠΎΠ΄Π½ΠΈΠΊΠΈ, Ρ…ΠΎΡ€ΠΎΡˆΠΎ проводящиС Ρ‚ΠΎΠΊ,
изоляторы, вСщСства Π½Π΅ проводящиС Ρ‚ΠΎΠΊ,
ΠΈ ΠΏΠΎΠ»ΡƒΠΏΡ€ΠΎΠ²ΠΎΠ΄Π½ΠΈΠΊΠΈ – Β«Π½ΠΈ Ρ€Ρ‹Π±Π°, Π½ΠΈ мясо»,
проводят Ρ‚ΠΎΠΊ ΠΌΠ½ΠΎΠ³ΠΎ Ρ…ΡƒΠΆΠ΅ ΠΏΡ€ΠΎΠ²ΠΎΠ΄Π½ΠΈΠΊΠΎΠ²,
Π½ΠΎ Π»ΡƒΡ‡ΡˆΠ΅ изоляторов. Π˜Ρ… ΠΎΡ†Π΅Π½ΠΈΠ²Π°ΡŽΡ‚ ΠΏΠΎ
ΡΠΎΠΏΡ€ΠΎΡ‚ΠΈΠ²Π»Π΅Π½ΠΈΡŽ, ΠΌΠ°Π»Π΅Π½ΡŒΠΊΠΎΠΌΡƒ Ρƒ ΠΏΡ€ΠΎΠ²ΠΎΠ΄Π½ΠΈΠΊΠΎΠ²
ΠΈ ΠΎΠ³Ρ€ΠΎΠΌΠ½ΠΎΠΌΡƒ Ρƒ изоляторов. Π•Π΄ΠΈΠ½ΠΈΡ†Π°
сопротивлСния Π² элСктротСхникС ΠΎΠΌ.

Π’Π΅ΠΏΠ΅Ρ€ΡŒ
ΠΌΠΎΠΆΠ½ΠΎ ΠΏΠ΅Ρ€Π΅ΠΉΡ‚ΠΈ ΠΊ Ρ‚ΠΎΠΌΡƒ, с Ρ‡Π΅Π³ΠΎ я ΠΏΠΎ ошибкС
Π½Π°Ρ‡Π°Π», ΠΊ рСзисторам. ΠœΡ‹ ΡƒΠΆΠ΅ Π΄ΠΎΠ³ΠΎΠ²ΠΎΡ€ΠΈΠ»ΠΈΡΡŒ,
Ρ‡Ρ‚ΠΎ Ρ€Π°Π·Π½Ρ‹Π΅ вСщСства ΠΏΠΎ-Ρ€Π°Π·Π½ΠΎΠΌΡƒ проводят
элСктричСский Ρ‚ΠΎΠΊ. Π­Ρ‚ΠΎ касаСтся ΠΈ
ΠΌΠ΅Ρ‚Π°Π»Π»ΠΎΠ². Π₯отя ΠΎΠ½ΠΈ всС ΠΏΡ€ΠΎΠ²ΠΎΠ΄Π½ΠΈΠΊΠΈ, Π½ΠΎ
ΠΎΠ΄Π½ΠΈ ΠΌΠ΅Ρ‚Π°Π»Π»Ρ‹ Π»ΡƒΡ‡ΡˆΠ΅ проводят Ρ‚ΠΎΠΊ, Π΄Ρ€ΡƒΠ³ΠΈΠ΅
Ρ…ΡƒΠΆΠ΅. ΠžΡ‡Π΅Π½ΡŒ Ρ…ΠΎΡ€ΠΎΡˆΠΎ, Π½Π°ΠΏΡ€ΠΈΠΌΠ΅Ρ€, проводят
Ρ‚ΠΎΠΊ мСдь, сСрСбро, Π·ΠΎΠ»ΠΎΡ‚ΠΎ. Π₯ΡƒΠΆΠ΅ алюминий.
Π•Ρ‰Π΅ Ρ…ΡƒΠΆΠ΅ сплавы ΠΌΠ΅Ρ‚Π°Π»Π»ΠΎΠ², ΠΊΠ°ΠΊ Π½ΠΈΡ…Ρ€ΠΎΠΌ,
ΠΌΠ°Π½Π³Π°Π½ΠΈΠ½, константан. ΠžΡ‚Ρ‡Π΅Π³ΠΎ зависит
сопротивлСниС ΠΏΡ€ΠΎΠ²ΠΎΠ΄Π½ΠΈΠΊΠ°? ΠžΡ‚ вСщСства,
ΠΈΠ· ΠΊΠΎΡ‚ΠΎΡ€ΠΎΠ³ΠΎ ΠΎΠ½ сдСлан, ΠΎΡ‚ Ρ‚ΠΎΠ»Ρ‰ΠΈΠ½Ρ‹
ΠΏΡ€ΠΎΠ²ΠΎΠ΄Π½ΠΈΠΊΠ° ΠΈ ΠΎΡ‚ Π΄Π»ΠΈΠ½Ρ‹ ΠΏΡ€ΠΎΠ²ΠΎΠ΄Π½ΠΈΠΊΠ°.

Если
Ρƒ вас Π΅ΡΡ‚ΡŒ ΠΌΡƒΠ»ΡŒΡ‚ΠΈΠΌΠ΅Ρ‚Ρ€ ΠΈ ΠΌΠ΅Π΄Π½Ρ‹Π΅ ΠΏΡ€ΠΎΠ²ΠΎΠ΄Π°
ΠΎΠ΄ΠΈΠ½Π°ΠΊΠΎΠ²ΠΎΠΉ Π΄Π»ΠΈΠ½Ρ‹, Π½ΠΎ Ρ€Π°Π·Π½ΠΎΠ³ΠΎ Π΄ΠΈΠ°ΠΌΠ΅Ρ‚Ρ€Π°,
Π²Ρ‹ ΠΌΠΎΠΆΠ΅Ρ‚Π΅ ΠΈΠ·ΠΌΠ΅Ρ€ΠΈΡ‚ΡŒ ΠΈΡ… сопротивлСниС
(ΠΏΡ€ΠΎΠ²ΠΎΠ΄Π° Π»ΡƒΡ‡ΡˆΠ΅ Π²Π·ΡΡ‚ΡŒ достаточно Π΄Π»ΠΈΠ½Π½Ρ‹Π΅).
Если Ρƒ вас найдСтся ΠΏΡ€ΠΎΠ²ΠΎΠ΄Π½ΠΈΠΊ ΠΈΠ· Π½ΠΈΡ…Ρ€ΠΎΠΌΠ°
(ΠΎΡ‚ ΠΏΠ΅Ρ€Π΅Π³ΠΎΡ€Π΅Π²ΡˆΠ΅ΠΉ спирали старого
Π½Π°Π³Ρ€Π΅Π²Π°Ρ‚Π΅Π»ΡŒΠ½ΠΎΠ³ΠΎ ΠΏΡ€ΠΈΠ±ΠΎΡ€Π°) Ρ‚Π°ΠΊΠΎΠΉ ΠΆΠ΅ Π΄Π»ΠΈΠ½Ρ‹,
Π²Ρ‹ Π½Π΅ΠΏΡ€Π΅ΠΌΠ΅Π½Π½ΠΎ Π·Π°ΠΌΠ΅Ρ‚ΠΈΡ‚Π΅ Ρ€Π°Π·Π½ΠΈΡ†Ρƒ.

Π•ΡΡ‚ΡŒ
Π΅Ρ‰Π΅ ΠΎΠ΄Π½ΠΎ ΠΎΠ±ΡΡ‚ΠΎΡΡ‚Π΅Π»ΡŒΡΡ‚Π²ΠΎ, Π²Π»ΠΈΡΡŽΡ‰Π΅Π΅ Π½Π°
сопротивлСниС ΠΏΡ€ΠΎΠ²ΠΎΠ΄Π½ΠΈΠΊΠ°, это Ρ‚Π΅ΠΌΠΏΠ΅Ρ€Π°Ρ‚ΡƒΡ€Π°.
ΠŸΡ€ΠΈ Π½Π°Π³Ρ€Π΅Π²Π°Π½ΠΈΠΈ сопротивлСниС ΠΏΡ€ΠΎΠ²ΠΎΠ΄Π½ΠΈΠΊΠ°
увСличиваСтся, ΠΏΠΎΡ‚ΠΎΠΌΡƒ Ρ‡Ρ‚ΠΎ ΠΏΡ€ΠΈ Π½Π°Π³Ρ€Π΅Π²Π°Π½ΠΈΠΈ
элСктроны бродяги становятся Π΅Ρ‰Π΅
энСргичнСС Π² своСм хаотичСском Π΄Π²ΠΈΠΆΠ΅Π½ΠΈΠΈ
ΠΈ ΠΈΡ… Ρ‚Ρ€ΡƒΠ΄Π½Π΅Π΅ Π·Π°ΡΡ‚Π°Π²ΠΈΡ‚ΡŒ Π΄Π²ΠΈΠ³Π°Ρ‚ΡŒΡΡ
Π½Π°ΠΏΡ€Π°Π²Π»Π΅Π½Π½ΠΎ. Π£Π±Π΅Π΄ΠΈΡ‚ΡŒΡΡ Π² Ρ‚ΠΎΠΌ, Ρ‡Ρ‚ΠΎ ΠΏΡ€ΠΈ
Π½Π°Π³Ρ€Π΅Π²Π°Π½ΠΈΠΈ сопротивлСниС увСличиваСтся,
ΠΌΠΎΠΆΠ½ΠΎ ΠΏΠΎΠ΄ΠΊΠ»ΡŽΡ‡ΠΈΠ² ΠΊ ΠΌΡƒΠ»ΡŒΡ‚ΠΈΠΌΠ΅Ρ‚Ρ€Ρƒ Π² Ρ€Π΅ΠΆΠΈΠΌΠ΅
измСрСния сопротивлСния рСзистор, ΠΈ
Π½Π°Π³Ρ€Π΅Ρ‚ΡŒ Π²Ρ‹Π²ΠΎΠ΄ рСзистора паяльником.
Если интСрСсно, ΠΌΠΎΠΆΠ΅Ρ‚Π΅ ΠΏΠΎΠΏΡ€ΠΎΠ±ΠΎΠ²Π°Ρ‚ΡŒ,
Ρ‚ΠΎΠ»ΡŒΠΊΠΎ Π°ΠΊΠΊΡƒΡ€Π°Ρ‚Π½ΠΎ, Ρ‡Ρ‚ΠΎΠ±Ρ‹ Π½Π΅ ΠΈΡΠΏΠΎΡ€Ρ‚ΠΈΡ‚ΡŒ
свой ΠΏΡ€ΠΈΠ±ΠΎΡ€. И Π½Π΅ Π·Π°Π±Ρ‹Π²Π°ΠΉΡ‚Π΅ ΠΎΠ± этом,
ΠΊΠΎΠ³Π΄Π°, особСнно Π² ΠΈΠ·ΠΌΠ΅Ρ€ΠΈΡ‚Π΅Π»ΡŒΠ½Ρ‹Ρ… цСпях,
ΠΏΡ‹Ρ‚Π°Π΅Ρ‚Π΅ΡΡŒ ΠΏΠΎΠ»ΡƒΡ‡ΠΈΡ‚ΡŒ Π½ΡƒΠΆΠ½ΡƒΡŽ Π²Π΅Π»ΠΈΡ‡ΠΈΠ½Ρƒ
сопротивлСния, подпаивая ΠΊ ΠΎΠ΄Π½ΠΎΠΌΡƒ
рСзистору Π΄Ρ€ΡƒΠ³ΠΎΠΉ. ΠžΠ±ΡΠ·Π°Ρ‚Π΅Π»ΡŒΠ½ΠΎ Π΄Π°ΠΉΡ‚Π΅
ΠΎΡΡ‚Ρ‹Ρ‚ΡŒ рСзисторам ΠΏΡ€Π΅ΠΆΠ΄Π΅, Ρ‡Π΅ΠΌ ΠΎΡ†Π΅Π½ΠΈΠ²Π°Ρ‚ΡŒ
Ρ€Π΅Π·ΡƒΠ»ΡŒΡ‚Π°Ρ‚.

РСзисторы
для Π½ΡƒΠΆΠ΄ элСктроники ΠΈΠ·Π³ΠΎΡ‚Π°Π²Π»ΠΈΠ²Π°ΡŽΡ‚ ΠΏΠΎ
Ρ€Π°Π·Π½Ρ‹ΠΌ тСхнологиям ΠΈ ΠΈΠ· Ρ€Π°Π·Π½Ρ‹Ρ… ΠΌΠ°Ρ‚Π΅Ρ€ΠΈΠ°Π»ΠΎΠ²
Ρ‚Π°ΠΊ, Ρ‡Ρ‚ΠΎ Π²Π΅Π»ΠΈΡ‡ΠΈΠ½Π° ΠΈΡ… сопротивлСния
колСблСтся ΠΎΡ‚ Π΄ΠΎΠ»Π΅ΠΉ ΠΎΠΌΠ° Π΄ΠΎ дСсятков
ΠΌΠΈΠ»Π»ΠΈΠΎΠ½ΠΎΠ² ΠΎΠΌ (ΠΌΠ΅Π³Π°ΠΎΠΌ). Π‘ΠΎΠΏΡ€ΠΎΡ‚ΠΈΠ²Π»Π΅Π½ΠΈΠ΅ Π²
элСктричСской Ρ†Π΅ΠΏΠΈ ΠΌΠΎΠΆΠ΅Ρ‚ Π±Ρ‹Ρ‚ΡŒ Π²Ρ€Π΅Π΄Π½ΠΎ,
Ρ‚Π°ΠΊ получаСтся Π² силовых цСпях, Π½ΠΎ ΠΌΠΎΠΆΠ΅Ρ‚
Π±Ρ‹Ρ‚ΡŒ ΠΏΠΎΠ»Π΅Π·Π½ΠΎ ΠΏΡ€ΠΈ Ρ€Π°Π·Π½ΠΎΠ³ΠΎ Ρ€ΠΎΠ΄Π° манипуляциях
с элСктричСством. Π‘Π°ΠΌΠΎΠ΅ простоС ΠΏΠΎΠ»Π΅Π·Π½ΠΎΠ΅
дСйствиС элСктричСского Ρ‚ΠΎΠΊΠ° – Π½Π°Π³Ρ€Π΅Π².
ΠŸΡ€ΠΈ ΠΏΡ€ΠΎΡ‚Π΅ΠΊΠ°Π½ΠΈΠΈ элСктричСского Ρ‚ΠΎΠΊΠ° ΠΏΠΎ
ΠΏΡ€ΠΎΠ²ΠΎΠ΄Π½ΠΈΠΊΡƒ, ΠΎΠΊΠ°Π·Ρ‹Π²Π°ΡŽΡ‰Π΅ΠΌΡƒ сопротивлСниС,
ΠΏΡ€ΠΎΠ²ΠΎΠ΄Π½ΠΈΠΊ нагрСваСтся. Π’ ΠΎΠ±ΠΎΠ³Ρ€Π΅Π²Π°Ρ‚Π΅Π»Π΅,
Π³Π΄Π΅ ΠΈΡΠΏΠΎΠ»ΡŒΠ·ΡƒΠ΅Ρ‚ΡΡ сопротивлСниС,
ΠΈΠ·Π³ΠΎΡ‚ΠΎΠ²Π»Π΅Π½Π½ΠΎΠ΅ ΠΈΠ· Π½ΠΈΡ…Ρ€ΠΎΠΌΠ°, Ρ‚Π°ΠΊΠΎΠΉ рСзистор
нагрСваСтся Π΄ΠΎ красна. А Π² элСктричСской
Π»Π°ΠΌΠΏΠΎΡ‡ΠΊΠ΅ рСзистор (ΡΠΏΠΈΡ€Π°Π»ΡŒ Π»Π°ΠΌΠΏΠΎΡ‡ΠΊΠΈ
накаливания) раскаляСтся Π΄ΠΎ Π±Π΅Π»Π°. И Π²
Ρ‚ΠΎΠΌ, ΠΈ Π΄Ρ€ΡƒΠ³ΠΎΠΌ случаС сопротивлСниС ΠΌΡ‹
ΠΈΡΠΏΠΎΠ»ΡŒΠ·ΡƒΠ΅ΠΌ для прСвращСния элСктричСского
Ρ‚ΠΎΠΊΠ° Π² ΠΏΠΎΠ»Π΅Π·Π½Ρ‹Π΅ для нас Ρ‚Π΅ΠΏΠ»ΠΎ ΠΈ свСт.

РСзисторы
ΡˆΠΈΡ€ΠΎΠΊΠΎ ΠΈΡΠΏΠΎΠ»ΡŒΠ·ΡƒΡŽΡ‚ΡΡ Π² элСктроникС. Π•ΡΡ‚ΡŒ
ΠΏΡ€ΠΎΠ²ΠΎΠ»ΠΎΡ‡Π½Ρ‹Π΅ ΠΈ Π½Π΅ΠΏΡ€ΠΎΠ²ΠΎΠ»ΠΎΡ‡Π½Ρ‹Π΅ рСзисторы,
Π΅ΡΡ‚ΡŒ рСзисторы ΠΏΠ΅Ρ€Π΅ΠΌΠ΅Π½Π½ΠΎΠ³ΠΎ сопротивлСния
(ΠΏΠΎΡ‚Π΅Π½Ρ†ΠΈΠΎΠΌΠ΅Ρ‚Ρ€Ρ‹), Π΅ΡΡ‚ΡŒ тСрморСзисторы ΠΈ
фоторСзисторы. А Ρ‚Π°ΠΊΠΎΠ΅ свойство
рСзисторов, ΠΊΠ°ΠΊ ΠΈΠ·ΠΌΠ΅Π½Π΅Π½ΠΈΠ΅ сопротивлСния
ΠΏΡ€ΠΈ мСханичСском воздСйствии, Π½Π°Ρ…ΠΎΠ΄ΠΈΡ‚
ΠΏΡ€ΠΈΠΌΠ΅Π½Π΅Π½ΠΈΠ΅ Π² Ρ‚Π΅Π½Π·ΠΎΠ΄Π°Ρ‚Ρ‡ΠΈΠΊΠ°Ρ….

Π―
сСйчас ΠΎΡ‚ΠΏΠ°ΡΡŽ нСсколько рСзисторов с
ΠΏΠ»Π°Ρ‚Ρ‹ ΠΏΡ€ΠΈΠ±ΠΎΡ€Π° ΠΈ пСрСнСсу ΠΈΡ… Π½Π° ΠΌΠ°ΠΊΠ΅Ρ‚Π½ΡƒΡŽ
ΠΏΠ»Π°Ρ‚Ρƒ, Ρ‡Ρ‚ΠΎΠ±Ρ‹ Ρ€Π°ΡΡΠΊΠ°Π·Π°Ρ‚ΡŒ ΠΎ Π½Π΅ΡΠΊΠΎΠ»ΡŒΠΊΠΈΡ…
простых, Π½ΠΎ ΠΎΡ‡Π΅Π½ΡŒ ΠΏΠΎΠ»Π΅Π·Π½Ρ‹Ρ… ΠΏΡ€Π°Π²ΠΈΠ»Π°Ρ…,
ΠΊΠΎΡ‚ΠΎΡ€Ρ‹Π΅ Π½Π°Π·Ρ‹Π²Π°ΡŽΡ‚ Π·Π°ΠΊΠΎΠ½Π°ΠΌΠΈ для элСктричСских
Ρ†Π΅ΠΏΠ΅ΠΉ. ΠœΠ°ΠΊΠ΅Ρ‚Π½Π°Ρ ΠΏΠ»Π°Ρ‚Π° Ρƒ мСня покупная.
Π’Π°ΠΊΠΈΠ΅ ΠΏΠ»Π°Ρ‚Ρ‹ ΠΏΡ€ΠΈΠΌΠ΅Π½ΡΡŽΡ‚ ΠΏΡ€ΠΈ создании
ΠΏΡ€ΠΎΡ‚ΠΎΡ‚ΠΈΠΏΠΎΠ² элСктричСских устройств.
На ΠΌΠ°ΠΊΠ΅Ρ‚Π½ΠΎΠΉ ΠΏΠ»Π°Ρ‚Π΅ ΠΌΠΎΠΆΠ½ΠΎ ΡΠΏΠ°ΡΡ‚ΡŒ устройство,
ΠΏΡ€ΠΎΠ²Π΅Ρ€ΠΈΡ‚ΡŒ, Π½Π°Π»Π°Π΄ΠΈΡ‚ΡŒ, Π° ΠΊΠΎΠ³Π΄Π° ΠΎΠ½ΠΎ ΠΏΠΎΠ»Π½ΠΎΡΡ‚ΡŒΡŽ
Π³ΠΎΡ‚ΠΎΠ²ΠΎ, ΠΌΠΎΠΆΠ½ΠΎ ΠΏΠ΅Ρ€Π΅ΠΉΡ‚ΠΈ ΠΊ ΠΈΠ·Π³ΠΎΡ‚ΠΎΠ²Π»Π΅Π½ΠΈΡŽ
ΠΎΠ±Ρ€Π°Π·Ρ†Π°. ΠžΡ‡Π΅Π½ΡŒ часто макСтная ΠΏΠ»Π°Ρ‚Π° –
это Π½Π°Π±ΠΎΡ€ ΠΊΠΎΠ½Ρ‚Π°ΠΊΡ‚Π½Ρ‹Ρ… ΠΏΠ»ΠΎΡ‰Π°Π΄ΠΎΠΊ ΠΈΠ· ΠΌΠ΅Π΄ΠΈ
с отвСрстиями для Π²Ρ‹Π²ΠΎΠ΄ΠΎΠ² ΠΊΠΎΠΌΠΏΠΎΠ½Π΅Π½Ρ‚ΠΎΠ²
элСктричСской схСмы. ΠŸΠ»Π°Ρ‚Ρƒ ΠΌΠΎΠΆΠ½ΠΎ
ΠΈΠ·Π³ΠΎΡ‚ΠΎΠ²ΠΈΡ‚ΡŒ ΡΠ°ΠΌΠΎΡΡ‚ΠΎΡΡ‚Π΅Π»ΡŒΠ½ΠΎ ΠΈΠ·
Ρ„ΠΎΠ»ΡŒΠ³ΠΈΡ€ΠΎΠ²Π°Π½Π½ΠΎΠ³ΠΎ листового ΠΌΠ°Ρ‚Π΅Ρ€ΠΈΠ°Π»Π°,
Π° ΠΏΡ€ΠΈ Π΅Π³ΠΎ отсутствии ΠΈΠ· любого ТСсткого
листового изолятора, ΠΆΠ΅Π»Π°Ρ‚Π΅Π»ΡŒΠ½ΠΎ
тСрмостойкого, Ρ‡Ρ‚ΠΎΠ±Ρ‹ ΠΏΠ»Π°Ρ‚Π° Π½Π΅ плавилась
ΠΏΡ€ΠΈ ΠΏΠ°ΠΉΠΊΠ΅. ΠŸΡ€ΠΈ мСханичСской ΠΎΠ±Ρ€Π°Π±ΠΎΡ‚ΠΊΠ΅
стСклотСкстолита — свСрлСнии, распиливании,
ΠΎΠ±Ρ€Π°Π±ΠΎΡ‚ΠΊΠ΅ напильником – слСдуСт
ΡΠΎΠ±Π»ΡŽΠ΄Π°Ρ‚ΡŒ ΠΎΡΡ‚ΠΎΡ€ΠΎΠΆΠ½ΠΎΡΡ‚ΡŒ, ΠΏΠΎΡΠΊΠΎΠ»ΡŒΠΊΡƒ ΠΏΡ‹Π»ΡŒ
стСкловолокна Ρ‚Ρ€Π°Π²ΠΌΠΈΡ€ΡƒΠ΅Ρ‚ Π΄Ρ‹Ρ…Π°Ρ‚Π΅Π»ΡŒΠ½Ρ‹Π΅
ΠΏΡƒΡ‚ΠΈ. МоТно ΠΈΡΠΏΠΎΠ»ΡŒΠ·ΠΎΠ²Π°Ρ‚ΡŒ подходящий
кусок Ρ„Π°Π½Π΅Ρ€Ρ‹. ΠšΠΎΠ½Ρ‚Π°ΠΊΡ‚Π½Ρ‹Π΅ ΠΏΠ»ΠΎΡ‰Π°Π΄ΠΊΠΈ для
припаивания ΠΊΠΎΠΌΠΏΠΎΠ½Π΅Π½Ρ‚ΠΎΠ² ΠΌΠΎΠΆΠ½ΠΎ ΡΠ΄Π΅Π»Π°Ρ‚ΡŒ
ΠΈΠ· кусочков ΠΌΠ΅Π΄Π½ΠΎΠ³ΠΎ элСктричСского
ΠΏΡ€ΠΎΠ²ΠΎΠ΄Π° Π±Π΅Π· изоляции, ΠΏΡ€ΠΎΠ΄Π΅Π² ΠΈΠ· Π² Π΄Π²Π°
просвСрлСнных рядом отвСрстия ΠΈ Π·Π°Π³Π½ΡƒΠ²
с ΠΎΠ±Ρ€Π°Ρ‚Π½ΠΎΠΉ стороны. МоТно ΠΎΠ±ΠΎΠΉΡ‚ΠΈΡΡŒ ΠΈ
Π±Π΅Π· ΠΊΠΎΠ½Ρ‚Π°ΠΊΡ‚Π½Ρ‹Ρ… ΠΏΠ»ΠΎΡ‰Π°Π΄ΠΎΠΊ, просвСрлив
отвСрстия, Π² ΠΊΠΎΡ‚ΠΎΡ€Ρ‹Π΅ ΠΏΡ€ΠΎΠ΄Π΅Π²Π°ΡŽΡ‚ΡΡ Π²Ρ‹Π²ΠΎΠ΄Ρ‹
элСмСнтов схСмы, Π° ΠΊ Π²Ρ‹Π²ΠΎΠ΄Π°ΠΌ с ΠΎΠ±Ρ€Π°Ρ‚Π½ΠΎΠΉ
стороны ΠΏΡ€ΠΈΠΏΠ°ΠΈΠ²Π°ΡŽΡ‚ΡΡ ΠΏΡ€ΠΎΠ²ΠΎΠ΄Π½ΠΈΠΊΠΈ. Если
ΠΌΠ°ΠΊΠ΅Ρ‚Π½ΡƒΡŽ ΠΏΠ»Π°Ρ‚Ρƒ ΡΠ½Π°Π±Π΄ΠΈΡ‚ΡŒ стойками Π²
1-1.5 см, Ρ‚ΠΎ Ρ€Π°Π±ΠΎΡ‚Π°Ρ‚ΡŒ с Π½Π΅ΠΉ Π±ΡƒΠ΄Π΅Ρ‚ Π΅Ρ‰Π΅
ΡƒΠ΄ΠΎΠ±Π½Π΅Π΅.

Для
ΠΏΠ°ΠΉΠΊΠΈ ΠΈΡΠΏΠΎΠ»ΡŒΠ·ΡƒΠ΅Ρ‚ΡΡ паяльник (Π΅Ρ‰Π΅ ΠΎΠ΄ΠΈΠ½
ΠΏΡ€ΠΈΠΌΠ΅Ρ€ ΠΏΠΎΠ»Π΅Π·Π½ΠΎΠ³ΠΎ использования
сопротивлСния), Ρƒ мСня паяльник Π½Π° 25
Π²Π°Ρ‚Ρ‚ 220 Π²ΠΎΠ»ΡŒΡ‚, ΠΈΠ·Π³ΠΎΡ‚ΠΎΠ²Π»Π΅Π½Π½Ρ‹ΠΉ Π² ПодмосковьС.
БСгодня ΠΌΠΎΠΆΠ½ΠΎ ΠΊΡƒΠΏΠΈΡ‚ΡŒ Ρ…ΠΎΡ€ΠΎΡˆΡƒΡŽ ΠΏΠ°ΡΠ»ΡŒΠ½ΡƒΡŽ
ΡΡ‚Π°Π½Ρ†ΠΈΡŽ – паяльник с Ρ€Π΅Π³ΡƒΠ»ΠΈΡ€ΠΎΠ²ΠΊΠΎΠΉ
Ρ‚Π΅ΠΌΠΏΠ΅Ρ€Π°Ρ‚ΡƒΡ€Ρ‹ Π½Π°Π³Ρ€Π΅Π²Π°, с мноТСством
насадок для ΠΏΠ°ΠΉΠΊΠΈ ΠΈ ΡƒΠ΄ΠΎΠ±Π½ΠΎΠΉ подставкой.
Π ΡƒΡ‡ΠΊΠ° ΠΌΠΎΠ΅Π³ΠΎ паяльника сдСлана Ρ‚Π°ΠΊ, Ρ‡Ρ‚ΠΎ
Π΅Π³ΠΎ ΠΌΠΎΠΆΠ½ΠΎ ΠΏΠΎΠ»ΠΎΠΆΠΈΡ‚ΡŒ Π½Π° Ρ€ΠΎΠ²Π½ΡƒΡŽ ΠΏΠΎΠ²Π΅Ρ€Ρ…Π½ΠΎΡΡ‚ΡŒ
Π±Π΅Π· подставки, Π½ΠΎ я ΠΏΡ€ΠΈΠ²Ρ‹ΠΊ ΠΈΡΠΏΠΎΠ»ΡŒΠ·ΠΎΠ²Π°Ρ‚ΡŒ
подставку, ΠΊΠΎΡ‚ΠΎΡ€ΡƒΡŽ сСгодня, Π΄ΡƒΠΌΠ°ΡŽ, Ρ‚ΠΎΠΆΠ΅
ΠΌΠΎΠΆΠ½ΠΎ ΠΊΡƒΠΏΠΈΡ‚ΡŒ Π² ΠΌΠ°Π³Π°Π·ΠΈΠ½Π΅. ΠšΡ€ΠΎΠΌΠ΅ паяльника
для ΠΏΠ°ΠΉΠΊΠΈ Π½ΡƒΠΆΠ΅Π½ ΠΏΡ€ΠΈΠΏΠΎΠΉ, Π»ΡƒΡ‡ΡˆΠ΅ ПОБ-61 Π²
Π²ΠΈΠ΄Π΅ Ρ‚ΠΎΠ½ΠΊΠΎΠ³ΠΎ ΠΏΡ€ΡƒΡ‚ΠΊΠ°, ΠΈ ΠΏΠ°ΡΠ»ΡŒΠ½Ρ‹ΠΉ Ρ„Π»ΡŽΡ,
Π½Π°ΠΏΡ€ΠΈΠΌΠ΅Ρ€, ΠΊΠ°Π½ΠΈΡ„ΠΎΠ»ΡŒ, хотя я ΠΈΡΠΏΠΎΠ»ΡŒΠ·ΡƒΡŽ
ΠΆΠΈΠ΄ΠΊΠΈΠΉ Ρ„Π»ΡŽΡ Π›Π’Π˜-120, ΠΊΠΎΡ‚ΠΎΡ€Ρ‹ΠΉ Π΄Π΅Ρ€ΠΆΡƒ Π²
ΠΏΡƒΠ·Ρ‹Ρ€ΡŒΠΊΠ΅ ΠΈΠ·-ΠΏΠΎΠ΄ Π»Π°ΠΊΠ° для Π½ΠΎΠ³Ρ‚Π΅ΠΉ с
кисточкой, достаточно ΡƒΠ΄ΠΎΠ±Π½ΠΎ. Ѐлюс
растСкаСтся ΠΏΠΎ мСсту ΠΏΠ°ΠΉΠΊΠΈ, помогая
ΠΏΡ€ΠΈΠΏΠΎΡŽ Π»ΡƒΡ‡ΡˆΠ΅ ΡΠΎΠ΅Π΄ΠΈΠ½ΠΈΡ‚ΡŒ Π΄Π΅Ρ‚Π°Π»ΠΈ, вСдь
ΠΏΠ°ΠΉΠΊΠ° – это ΠΎΠ΄ΠΈΠ½ ΠΈΠ· способов соСдинСния
Π΄Π΅Ρ‚Π°Π»Π΅ΠΉ, кстати, Π½Π΅ СдинствСнный, хотя
Π² элСктронных издСлиях Π½Π°ΠΈΠ±ΠΎΠ»Π΅Π΅ ΡˆΠΈΡ€ΠΎΠΊΠΎ
примСняСмый. ΠšΡ€ΠΎΠΌΠ΅ ΠΏΠ°ΠΉΠΊΠΈ ΠΌΠΎΠΆΠ½ΠΎ ΠΈΡΠΏΠΎΠ»ΡŒΠ·ΠΎΠ²Π°Ρ‚ΡŒ
скрутку, ΠΎΠ΄Π½ΠΎ врСмя ΠΌΠΎΠ½Ρ‚Π°ΠΆ с ΠΏΠΎΠΌΠΎΡ‰ΡŒΡŽ
скрутки Π±Ρ‹Π» ΠΎΡ‡Π΅Π½ΡŒ популярСн. ΠŸΡ€ΠΈ Ρ€Π°Π±ΠΎΡ‚Π΅
с паяльником Ρ‚ΠΎΠΆΠ΅ слСдуСт ΡΠΎΠ±Π»ΡŽΠ΄Π°Ρ‚ΡŒ
ΠΎΡΡ‚ΠΎΡ€ΠΎΠΆΠ½ΠΎΡΡ‚ΡŒ ΠΈ Π½Π΅ Ρ‚ΠΎΠ»ΡŒΠΊΠΎ с Ρ‚Π΅ΠΌ, Ρ‡Ρ‚ΠΎΠ±Ρ‹
Π½Π΅ ΠΎΠ±ΠΆΠ΅Ρ‡ΡŒΡΡ. ΠŸΡ€ΠΈΠΏΠΎΠΉ, ΠΈΡΠΏΠ°Ρ€ΡΡΡΡŒ, Π½Π΅
принСсСт ΠΏΠΎΠ»ΡŒΠ·Ρ‹ ΠΏΡ€ΠΈ Π²Π΄Ρ‹Ρ…Π°Π½ΠΈΠΈ. НС слСдуСт
паяльник постоянно Π΄Π΅Ρ€ΠΆΠ°Ρ‚ΡŒ Π²ΠΊΠ»ΡŽΡ‡Π΅Π½Π½Ρ‹ΠΌ,
Π»ΡƒΡ‡ΡˆΠ΅ лишний Ρ€Π°Π· ΠΏΠΎΠ΄ΠΎΠΆΠ΄Π°Ρ‚ΡŒ, ΠΊΠΎΠ³Π΄Π° ΠΎΠ½
нагрССтся, ΠΈΠ»ΠΈ ΡΠΎΠ±Ρ€Π°Ρ‚ΡŒ ΠΎΠ΄Π½Ρƒ ΠΈΠ· простых
схСм, ΠΎ ΠΊΠΎΡ‚ΠΎΡ€Ρ‹Ρ… ΠΌΡ‹ ΠΏΠΎΠ³ΠΎΠ²ΠΎΡ€ΠΈΠΌ ΠΏΠΎΠ·ΠΆΠ΅.

Пока
я всС это рассказывал, я успСл Π²ΠΊΠ»ΡŽΡ‡ΠΈΡ‚ΡŒ
паяльник ΠΈ Π²Ρ‹ΠΏΠ°ΡΡ‚ΡŒ рСзисторы ΠΈΠ· ΠΏΠ»Π°Ρ‚Ρ‹.

Π”ΠΎ
Ρ‚ΠΎΠ³ΠΎ, ΠΊΠ°ΠΊ ΠΏΡ€ΠΎΠ΄ΠΎΠ»ΠΆΠΈΡ‚ΡŒ рассказ ΠΎΠ±
элСктричСских цСпях, я Ρ…ΠΎΡ‡Ρƒ Π·Π°ΠΌΠ΅Ρ‚ΠΈΡ‚ΡŒ,
Ρ‡Ρ‚ΠΎ ΠΎΠΏΠΈΡΠ°Ρ‚ΡŒ ΡΠ»Π΅ΠΊΡ‚Ρ€ΠΈΡ‡Π΅ΡΠΊΡƒΡŽ Ρ†Π΅ΠΏΡŒ ΠΈ всС,
Ρ‡Ρ‚ΠΎ с Π½Π΅ΠΉ связано, ΠΌΠΎΠΆΠ½ΠΎ ΠΎΠ΄Π½ΠΈΠΌΠΈ словами,
Π½Π΅ прибСгая Π½ΠΈ ΠΊ Ρ‡Π΅ΠΌΡƒ Π΄Ρ€ΡƒΠ³ΠΎΠΌΡƒ, Π½ΠΎ
получаСтся Π΄Π»ΠΈΠ½Π½ΠΎ, ΠΈ Π΄Π°Π»Π΅ΠΊΠΎ Π½Π΅ всСгда
понятно. ΠŸΠΎΡΡ‚ΠΎΠΌΡƒ для изобраТСния
элСктричСских схСм ΠΈΡΠΏΠΎΠ»ΡŒΠ·ΡƒΡŽΡ‚ графичСскоС
прСдставлСниС – Π»ΡƒΡ‡ΡˆΠ΅ ΠΎΠ΄ΠΈΠ½ Ρ€Π°Π· ΡƒΠ²ΠΈΠ΄Π΅Ρ‚ΡŒ,
Ρ‡Π΅ΠΌ сто Ρ€Π°Π· ΡƒΡΠ»Ρ‹ΡˆΠ°Ρ‚ΡŒ. ΠšΠ°ΠΆΠ΄Ρ‹ΠΉ ΠΊΠΎΠΌΠΏΠΎΠ½Π΅Π½Ρ‚
Ρ€ΠΈΡΡƒΡŽΡ‚ Π² Π²ΠΈΠ΄Π΅ нСбольшой простой ΠΊΠ°Ρ€Ρ‚ΠΈΠ½ΠΊΠΈ,
Π° ΠΏΡ€ΠΎΠ²ΠΎΠ΄Π°, ΡΠΎΠ΅Π΄ΠΈΠ½ΡΡŽΡ‰ΠΈΠ΅ элСмСнты схСмы,
ΠΈΠ·ΠΎΠ±Ρ€Π°ΠΆΠ°ΡŽΡ‚ Π² Π²ΠΈΠ΄Π΅ Π»ΠΈΠ½ΠΈΠΉ. БобствСнно,
Ρ‚Π°ΠΊΠΎΠ΅ графичСскоС прСдставлСниС ΠΈ
Π½Π°Π·Ρ‹Π²Π°ΡŽΡ‚ схСмой устройства. ΠŸΡ€ΠΎΡΡ‚Ρ‹Π΅
схСмы ΠΌΠΎΠΆΠ½ΠΎ Π½Π°Ρ€ΠΈΡΠΎΠ²Π°Ρ‚ΡŒ Ρ‚Π°ΠΊ, ΠΊΠ°ΠΊ ΠΎΠ½ΠΈ
Π±ΡƒΠ΄ΡƒΡ‚ нарисованы Π½ΠΈΠΆΠ΅, Π±ΠΎΠ»Π΅Π΅ слоТныС
схСмы Ρ€ΠΈΡΡƒΡŽΡ‚ΡΡ Π½Π° ΠΌΠ½ΠΎΠ³ΠΈΡ… листах Π±ΡƒΠΌΠ°Π³ΠΈ,
Π° для объяснСния ΠΈΡ… Ρ€Π°Π±ΠΎΡ‚Ρ‹ ΠΈΡΠΏΠΎΠ»ΡŒΠ·ΡƒΡŽΡ‚
Π΅Ρ‰Π΅ ΠΎΠ΄ΠΈΠ½ графичСский Π²ΠΈΠ΄ – Ρ„ΡƒΠ½ΠΊΡ†ΠΈΠΎΠ½Π°Π»ΡŒΠ½Ρ‹Π΅
схСмы: всС устройство ΠΌΠΎΠΆΠ½ΠΎ, ΠΈ Π΄ΠΎΠ»ΠΆΠ½ΠΎ,
Ρ€Π°Π·Π±ΠΈΡ‚ΡŒ Π½Π° части, Ρ„ΡƒΠ½ΠΊΡ†ΠΈΠΎΠ½Π°Π»ΡŒΠ½Ρ‹Π΅ ΡƒΠ·Π»Ρ‹,
ΠΊΠ°ΠΊ, скаТСм, Π²Ρ‹ΠΏΡ€ΡΠΌΠΈΡ‚Π΅Π»ΡŒ, ΡƒΡΠΈΠ»ΠΈΡ‚Π΅Π»ΡŒ,
ΠΏΡ€Π΅ΠΎΠ±Ρ€Π°Π·ΠΎΠ²Π°Ρ‚Π΅Π»ΡŒ ΠΈ Ρ‚. Π΄., ΠΊΠΎΡ‚ΠΎΡ€Ρ‹Π΅ ΠΈΠ·ΠΎΠ±Ρ€Π°ΠΆΠ°ΡŽΡ‚ΡΡ
Π² Π²ΠΈΠ΄Π΅ ΠΏΡ€ΡΠΌΠΎΡƒΠ³ΠΎΠ»ΡŒΠ½ΠΈΠΊΠΎΠ², связанных
линиями ΠΈΠ»ΠΈ стрСлками. К Ρ‚Π°ΠΊΠΈΠΌ слоТным
(ΠΎΡ‡Π΅Π½ΡŒ ΠΏΠΎΠ»Π΅Π·Π½ΠΎ, Ссли ΠΈ ΠΊ простым) схСмам
ΠΏΡ€ΠΈΠ»Π°Π³Π°ΡŽΡ‚ ΠΈΡ… описаниС, ΠΊΠΎΡ‚ΠΎΡ€ΠΎΠ΅ ΠΌΠΎΠΆΠ΅Ρ‚
Π·Π°Π½ΠΈΠΌΠ°Ρ‚ΡŒ нСсколько Ρ‚ΠΎΠΌΠΎΠ².

ГрафичСскоС
ΠΈΠ·ΠΎΠ±Ρ€Π°ΠΆΠ΅Π½ΠΈΠ΅ элСмСнтов элСктричСской
схСмы Π² Ρ€Π°Π·Π½Ρ‹Ρ… странах, Π² Ρ€Π°Π·Π½Ρ‹Π΅ Π³ΠΎΠ΄Ρ‹
выглядСло нСсколько ΠΏΠΎ-Ρ€Π°Π·Π½ΠΎΠΌΡƒ. Π’Π°ΠΊ для
изобраТСния Π±Π°Ρ‚Π°Ρ€Π΅ΠΉΠΊΠΈ ΠΈΡΠΏΠΎΠ»ΡŒΠ·ΡƒΡŽΡ‚
ΠΈΠ·ΠΎΠ±Ρ€Π°ΠΆΠ΅Π½ΠΈΠ΅ ΠΈΠ· Π΄Π²ΡƒΡ… Ρ‡Π΅Ρ€Ρ‚ΠΎΡ‡Π΅ΠΊ, ΠΎΠ΄Π½Π° ΠΈΠ·
ΠΊΠΎΡ‚ΠΎΡ€Ρ‹Ρ… Π΄Π»ΠΈΠ½Π½Π΅Π΅ Π΄Ρ€ΡƒΠ³ΠΎΠΉ, с пСрпСндикулярными
ΠΊ Π½ΠΈΠΌ Π²Ρ‹Π²ΠΎΠ΄Π°ΠΌΠΈ, аккумулятор ΠΈΠ·ΠΎΠ±Ρ€Π°ΠΆΠ°Π»ΠΈ
Π² Π²ΠΈΠ΄Π΅ Π½Π΅ΡΠΊΠΎΠ»ΡŒΠΊΠΈΡ… Ρ‚Π°ΠΊΠΈΡ… Π±Π°Ρ‚Π°Ρ€Π΅Π΅ΠΊ. Но,
ΠΏΠΎΡ€ΠΎΠΉ, Π² схСмС Π½Π΅ дСлаСтся различия
ΠΌΠ΅ΠΆΠ΄Ρƒ этими двумя источниками Π­Π”Π‘. Π’
послСднСС врСмя, особСнно Π² ΠΏΡ€ΠΎΠ³Ρ€Π°ΠΌΠΌΠ°Ρ…,
эти источники питания ΠΎΠ±ΡŠΠ΅Π΄ΠΈΠ½ΡΡŽΡ‚ с
Π΄Ρ€ΡƒΠ³ΠΈΠΌΠΈ источниками Ρ‚ΠΎΠΊΠ° Π² ΠΎΠ±Ρ‰ΠΈΠΉ класс
источников (source) ΠΈ ΠΈΠ·ΠΎΠ±Ρ€Π°ΠΆΠ°ΡŽΡ‚ Π² Π²ΠΈΠ΄Π΅
ΠΊΡ€ΡƒΠΆΠΊΠ° со Π·Π½Π°Ρ‡ΠΊΠ°ΠΌΠΈ плюс ΠΈ минус. ΠŸΠΎΠ΄ΠΎΠ±Π½Ρ‹Π΅
отличия Π΅ΡΡ‚ΡŒ Π² ΠΈΠ·ΠΎΠ±Ρ€Π°ΠΆΠ΅Π½ΠΈΠΈ рСзисторов
Π² Π²ΠΈΠ΄Π΅ ΠΏΡ€ΡΠΌΠΎΡƒΠ³ΠΎΠ»ΡŒΠ½ΠΈΠΊΠΎΠ², ΠΎΠ±ΠΎΠ·Π½Π°Ρ‡Π΅Π½Π½Ρ‹Ρ…
латинской Π±ΡƒΠΊΠ²ΠΎΠΉ R с порядковым Π½ΠΎΠΌΠ΅Ρ€ΠΎΠΌ
ΠΎΠ΄Π½ΠΎΡ‚ΠΈΠΏΠ½Ρ‹Ρ… элСмСнтов, ΠΈ Π² Π²ΠΈΠ΄Π΅ Π»ΠΎΠΌΠ°Π½ΠΎΠΉ
Π»ΠΈΠ½ΠΈΠΈ. Π•ΡΡ‚ΡŒ отличия Π² графичСском
ΠΈΠ·ΠΎΠ±Ρ€Π°ΠΆΠ΅Π½ΠΈΠΈ Π΄Ρ€ΡƒΠ³ΠΈΡ… элСмСнтов, ΠΎ ΠΊΠΎΡ‚ΠΎΡ€Ρ‹Ρ…
я Π±ΡƒΠ΄Ρƒ Ρ€Π°ΡΡΠΊΠ°Π·Ρ‹Π²Π°Ρ‚ΡŒ ΠΏΠΎ ΠΌΠ΅Ρ€Π΅ ΠΈΡ… появлСния
Π² ΠΊΠ½ΠΈΠ³Π΅. ΠžΠ±Ρ‹Ρ‡Π½ΠΎ это Π½Π΅ Π²Ρ‹Π·Ρ‹Π²Π°Π΅Ρ‚ Π±ΠΎΠ»ΡŒΡˆΠΈΡ…
Π·Π°Ρ‚Ρ€ΡƒΠ΄Π½Π΅Π½ΠΈΠΉ, Π½ΠΎ Ссли Π²Ρ‹ Π±ΡƒΠ΄Π΅Ρ‚Π΅ Ρ€ΠΈΡΠΎΠ²Π°Ρ‚ΡŒ
свои схСмы, Π»ΡƒΡ‡ΡˆΠ΅ Π²Ρ‹Π±Ρ€Π°Ρ‚ΡŒ ΠΎΠ΄ΠΈΠ½ ΡΡ‚ΠΈΠ»ΡŒ.

Для
чСрчСния схСм ΠΈ пояснСния ΠΈΡ… Ρ€Π°Π±ΠΎΡ‚Ρ‹ я
Π±ΡƒΠ΄Ρƒ ΠΏΠΎΠ»ΡŒΠ·ΠΎΠ²Π°Ρ‚ΡŒΡΡ ΠΊΠΎΠΌΠΏΡŒΡŽΡ‚Π΅Ρ€ΠΎΠΌ, Ρ‚ΠΎΡ‡Π½Π΅Π΅
дСмонстрационной вСрсиСй ΠΏΡ€ΠΎΠ³Ρ€Π°ΠΌΠΌΡ‹
PSIM, ΠΊΠΎΡ‚ΠΎΡ€ΡƒΡŽ скачал с сайта производитСля.
ΠŸΡ€ΠΎΠ³Ρ€Π°ΠΌΠΌΠ° ΠΏΡ€Π΅Π΄Π½Π°Π·Π½Π°Ρ‡Π΅Π½Π° для Ρ€Π°Π·Ρ€Π°Π±ΠΎΡ‚ΠΊΠΈ
схСм силовой элСктроники. МногиС
ΠΏΡ€ΠΎΠ³Ρ€Π°ΠΌΠΌΡ‹ ΠΈΠΌΠ΅ΡŽΡ‚ свою ΡΠΏΠ΅Ρ†ΠΈΠ°Π»ΠΈΠ·Π°Ρ†ΠΈΡŽ.
Если эта ΠΏΡ€ΠΎΠ³Ρ€Π°ΠΌΠΌΠ° пСрСстанСт ΠΌΠ½Π΅
ΠΏΠΎΠΌΠΎΠ³Π°Ρ‚ΡŒ Π² рассказС, я ΠΈΡΠΏΠΎΠ»ΡŒΠ·ΡƒΡŽ Π΄Ρ€ΡƒΠ³ΡƒΡŽ.
ΠŸΡ€ΠΎΠ³Ρ€Π°ΠΌΠΌΠ° PSIM ΠΏΡ€Π΅Π΄Π½Π°Π·Π½Π°Ρ‡Π΅Π½Π° для Ρ€Π°Π±ΠΎΡ‚Ρ‹
Π½Π° ΠΏΠ»Π°Ρ‚Ρ„ΠΎΡ€ΠΌΠ΅ Windows, Π½ΠΎ Ρ€Π°Π±ΠΎΡ‚Π°Π΅Ρ‚ Ρƒ мСня с
Linux, ΠΏΡ€ΠΈ этом я ΠΈΡΠΏΠΎΠ»ΡŒΠ·ΡƒΡŽ эмулятор Wine.
Как это всС дСлаСтся я расскаТу ΠΏΠΎΠ·ΠΆΠ΅,
Π° сСйчас Ρ…ΠΎΡ‡Ρƒ Π·Π°ΠΌΠ΅Ρ‚ΠΈΡ‚ΡŒ, Ρ‡Ρ‚ΠΎ ΠΈΠ·ΠΎΠ±Ρ€Π°ΠΆΠ΅Π½ΠΈΠ΅
рСзистора Π² ΠΏΡ€ΠΎΠ³Ρ€Π°ΠΌΠΌΠ΅ Π΅ΡΡ‚ΡŒ Ρ‚ΠΎΠ»ΡŒΠΊΠΎ Π²
Π²ΠΈΠ΄Π΅ Π»ΠΎΠΌΠ°Π½ΠΎΠΉ Π»ΠΈΠ½ΠΈΠΈ. Π’Ρ‚ΠΎΡ€ΠΎΠ΅ ΠΈΠ·ΠΎΠ±Ρ€Π°ΠΆΠ΅Π½ΠΈΠ΅
ΠΌΠ½Π΅ ΠΏΡ€ΠΈΡˆΠ»ΠΎΡΡŒ ΠΏΡ€ΠΈΡ€ΠΈΡΠΎΠ²Π°Ρ‚ΡŒ Π² графичСском
Ρ€Π΅Π΄Π°ΠΊΡ‚ΠΎΡ€Π΅.

Π‘Ρ…Π΅ΠΌΠ°
(схСмы) Π½Π° рисункС Π½ΠΈΠΆΠ΅ ΠΈΠΌΠ΅ΡŽΡ‚ ΠΎΠ΄ΠΈΠ½
графичСский элСмСнт, ΠΎ ΠΊΠΎΡ‚ΠΎΡ€ΠΎΠΌ я Π΅Ρ‰Π΅ Π½Π΅
Π³ΠΎΠ²ΠΎΡ€ΠΈΠ». Π­Ρ‚ΠΎ зСмля ΠΈΠ»ΠΈ ΠΎΠ±Ρ‰ΠΈΠΉ схСмный
ΠΏΡ€ΠΎΠ²ΠΎΠ΄. ΠŸΠΎΡΠΌΠΎΡ‚Ρ€ΠΈΡ‚Π΅
Π½Π° ΠΈΠ·ΠΎΠ±Ρ€Π°ΠΆΠ΅Π½ΠΈΠ΅ схСмы (Π΄Π²Π° изобраТСния),
Π° ΠΏΠΎΡ‚ΠΎΠΌ я ΠΏΠΎΡΡ‚Π°Ρ€Π°ΡŽΡΡŒ ΠΎΡ‚Π²Π΅Ρ‚ΠΈΡ‚ΡŒ Π½Π° вопрос
ΠΎΠ± ΠΎΠ±Ρ‰Π΅ΠΌ ΠΏΡ€ΠΎΠ²ΠΎΠ΄Π΅ схСмы.

Рис.
1.1. ГрафичСскоС ΠΈΠ·ΠΎΠ±Ρ€Π°ΠΆΠ΅Π½ΠΈΠ΅ ΠΏΡ€ΠΎΠ²ΠΎΠ΄Π½ΠΈΠΊΠ°,
сопротивлСния ΠΈ Π±Π°Ρ‚Π°Ρ€Π΅ΠΉΠΊΠΈ

Π—Π°Ρ‡Π΅ΠΌ
ΠΆΠ΅ Π½ΡƒΠΆΠ½ΠΎ Π²Π²ΠΎΠ΄ΠΈΡ‚ΡŒ ΠΎΠ±ΠΎΠ·Π½Π°Ρ‡Π΅Π½ΠΈΠ΅ для ΠΎΠ±Ρ‰Π΅Π³ΠΎ
ΠΏΡ€ΠΎΠ²ΠΎΠ΄Π° схСмы, ΠΊΠΎΡ‚ΠΎΡ€Ρ‹ΠΉ часто Π½Π°Π·Ρ‹Π²Π°ΡŽΡ‚
зСмляным ΠΏΡ€ΠΎΠ²ΠΎΠ΄ΠΎΠΌ, ΠΈΠ»ΠΈ, просто, Π·Π΅ΠΌΠ»Π΅ΠΉ?

ЗабСгая
Π²ΠΏΠ΅Ρ€Π΅Π΄, скаТу, Ρ‡Ρ‚ΠΎ Π² схСмС ΡƒΠ΄ΠΎΠ±Π½ΠΎ ΠΈΠ·ΠΌΠ΅Ρ€ΡΡ‚ΡŒ
напряТСния ΠΎΡ‚Π½ΠΎΡΠΈΡ‚Π΅Π»ΡŒΠ½ΠΎ ΠΎΠ΄Π½ΠΎΠΉ Ρ‚ΠΎΡ‡ΠΊΠΈ
схСмы, ΠΈΠ»ΠΈ ΠΎΠ΄Π½ΠΎΠ³ΠΎ ΠΎΠ±Ρ‰Π΅Π³ΠΎ ΠΏΡ€ΠΎΠ²ΠΎΠ΄Π½ΠΈΠΊΠ°,
ΡƒΠ΄ΠΎΠ±Π½ΠΎ ΡΠΌΠΎΡ‚Ρ€Π΅Ρ‚ΡŒ сигналы ΠΎΡ‚Π½ΠΎΡΠΈΡ‚Π΅Π»ΡŒΠ½ΠΎ
этого ΠΏΡ€ΠΎΠ²ΠΎΠ΄Π½ΠΈΠΊΠ°. ΠšΡ€ΠΎΠΌΠ΅ Ρ‚ΠΎΠ³ΠΎ, Ссли этот
ΠΎΠ±Ρ‰ΠΈΠΉ ΠΏΡ€ΠΎΠ²ΠΎΠ΄ схСмы ΡΠΎΠ΅Π΄ΠΈΠ½ΠΈΡ‚ΡŒ с Π·Π΅ΠΌΠ»Π΅ΠΉ
– ΡΠΏΠ΅Ρ†ΠΈΠ°Π»ΡŒΠ½ΠΎ устраиваСмым Π·Π°Π·Π΅ΠΌΠ»Π΅Π½ΠΈΠ΅ΠΌ,
ΠΈΠΌΠ΅ΡŽΡ‰ΠΈΠΌ Ρ…ΠΎΡ€ΠΎΡˆΠΈΠΉ ΠΊΠΎΠ½Ρ‚Π°ΠΊΡ‚ ΠΈΠΌΠ΅Π½Π½ΠΎ с Π·Π΅ΠΌΠ»Π΅ΠΉ
(Π³Ρ€ΡƒΠ½Ρ‚ΠΎΠΌ, ΠΏΠΎΡ‡Π²ΠΎΠΉ) – схСма мСньшС
ΠΏΠΎΠ΄Π²Π΅Ρ€ΠΆΠ΅Π½Π° Π²Ρ€Π΅Π΄Π½Ρ‹ΠΌ влияниям Π²Π½Π΅ΡˆΠ½ΠΈΡ…
элСктричСских ΠΏΠΎΠ»Π΅ΠΉ, поэтому ΠΎΠ±Ρ‰ΠΈΠΉ
ΠΏΡ€ΠΎΠ²ΠΎΠ΄ схСмы часто Π½Π°Π·Ρ‹Π²Π°ΡŽΡ‚ Β«Π·Π΅ΠΌΠ»Π΅ΠΉΒ».

ΠœΡ‹
ΠΏΠΎΠΊΠ° познакомились Ρ‚ΠΎΠ»ΡŒΠΊΠΎ с трСмя
ΠΊΠΎΠΌΠΏΠΎΠ½Π΅Π½Ρ‚Π°ΠΌΠΈ элСктронного устройства:
Π±Π°Ρ‚Π°Ρ€Π΅ΠΉΠΊΠ°, ΠΏΡ€ΠΎΠ²ΠΎΠ΄Π½ΠΈΠΊ, рСзистор. МоТно
Π»ΠΈ с ΠΈΡ… ΠΏΠΎΠΌΠΎΡ‰ΡŒΡŽ ΠΏΠΎΡΡ‚Ρ€ΠΎΠΈΡ‚ΡŒ Ρ‡Ρ‚ΠΎ-Π»ΠΈΠ±ΠΎ
интСрСсноС?

МоТно.
Π’ΠΎ-ΠΏΠ΅Ρ€Π²Ρ‹Ρ…, ΠΌΠΎΠΆΠ½ΠΎ провСсти нСсколько
экспСримСнтов для знакомства с трСмя
Π·Π°ΠΊΠΎΠ½Π°ΠΌΠΈ: Π·Π°ΠΊΠΎΠ½ Ома ΠΈ Π΄Π²Π° Π·Π°ΠΊΠΎΠ½Π° ΠšΠΈΡ€Ρ…Π³ΠΎΡ„Π°.
ВсС Π·Π°ΠΊΠΎΠ½Ρ‹ ΠΌΡ‹ рассмотрим Π² ΠΏΡ€ΠΎΡΡ‚Π΅ΠΉΡˆΠ΅ΠΌ
Π²ΠΈΠ΄Π΅, Π° Π±ΠΎΠ»Π΅Π΅ слоТный ΠΈΡ… Π²ΠΈΠ΄ ΠΏΡ€ΠΈ
нСобходимости ΠΌΠΎΠΆΠ½ΠΎ Π½Π°ΠΉΡ‚ΠΈ Π² ΡƒΡ‡Π΅Π±Π½ΠΎΠΉ
Π»ΠΈΡ‚Π΅Ρ€Π°Ρ‚ΡƒΡ€Π΅. Π­Ρ‚ΠΈΡ… Ρ‚Ρ€Π΅Ρ… Π·Π°ΠΊΠΎΠ½ΠΎΠ²
элСктротСхники, я надСюсь, ΠΌΠ½Π΅ Ρ…Π²Π°Ρ‚ΠΈΡ‚
Π½Π° протяТСнии всСй ΠΊΠ½ΠΈΠ³ΠΈ, ΠΈ Π½Π΅ появится
нСобходимости Π² Π΄Ρ€ΡƒΠ³ΠΈΡ…. Как схСмы ΡƒΠ΄ΠΎΠ±Π½Π΅Π΅
ΠΈΠ·ΠΎΠ±Ρ€Π°ΠΆΠ°Ρ‚ΡŒ Π² графичСском Π²ΠΈΠ΄Π΅, Ρ‚Π°ΠΊ
Π·Π°ΠΊΠΎΠ½Ρ‹ ΡƒΠ΄ΠΎΠ±Π½Π΅Π΅ Π·Π°ΠΏΠΈΡΡ‹Π²Π°Ρ‚ΡŒ Π² Π²ΠΈΠ΄Π΅
матСматичСских ΡΠΎΠΎΡ‚Π½ΠΎΡˆΠ΅Π½ΠΈΠΉ. Для этого
ΠΈΡΠΏΠΎΠ»ΡŒΠ·ΡƒΠ΅Ρ‚ΡΡ латинская Π±ΡƒΠΊΠ²Π° R для
сопротивлСния, латинская Π±ΡƒΠΊΠ²Π° I для
Ρ‚ΠΎΠΊΠ°, ΠΈ латинскиС U ΠΈΠ»ΠΈ V для напряТСния.

Π—Π°ΠΊΠΎΠ½
Ома (для участка Ρ†Π΅ΠΏΠΈ) Π·Π²ΡƒΡ‡ΠΈΡ‚, ΠΏΡ€ΠΈΠ±Π»ΠΈΠ·ΠΈΡ‚Π΅Π»ΡŒΠ½ΠΎ,
Ρ‚Π°ΠΊ:

ПадСниС
напряТСния (ΠΈΠ»ΠΈ напряТСниС) Π½Π° участкС
Ρ†Π΅ΠΏΠΈ Ρ€Π°Π²Π½ΠΎ ΠΏΡ€ΠΎΠΈΠ·Π²Π΅Π΄Π΅Π½ΠΈΡŽ Ρ‚ΠΎΠΊΠ° Π½Π°
сопротивлСниС: U=I*R.

НС
знаю, ΠΊΠ°ΠΊ вас, мСня удивляСт, Π²Ρ‹Π·Ρ‹Π²Π°Π΅Ρ‚
чувство уваТСния ΠΈ восторга ΠΏΡ€ΠΎΠ½ΠΈΡ†Π°Ρ‚Π΅Π»ΡŒΠ½ΠΎΡΡ‚ΡŒ
ΠΈ ΡƒΠΌ Ρ‡Π΅Π»ΠΎΠ²Π΅ΠΊΠ°, ΠΊΠΎΡ‚ΠΎΡ€Ρ‹ΠΉ смог ΠΏΠΎΠ΄ΠΌΠ΅Ρ‚ΠΈΡ‚ΡŒ
ΠΈ ΠΎΠΏΡ€Π΅Π΄Π΅Π»ΠΈΡ‚ΡŒ ΡΡ‚ΠΎΠ»ΡŒ ΠΏΡ€ΠΎΡΡ‚ΡƒΡŽ ΠΈ ΠΏΠΎΠ»Π΅Π·Π½ΡƒΡŽ
связь ΠΌΠ΅ΠΆΠ΄Ρƒ этими трСмя Π²Π΅Π»ΠΈΡ‡ΠΈΠ½Π°ΠΌΠΈ.
ИмСнно Π΅Π³ΠΎ ΠΈΠΌΠ΅Π½Π΅ΠΌ Π½Π°Π·Π²Π°Π½Π° Π΅Π΄ΠΈΠ½ΠΈΡ†Π°
сопротивлСния.

Π‘ΠΎΠΎΡ‚Π½ΠΎΡˆΠ΅Π½ΠΈΠ΅
ΠΌΠ΅ΠΆΠ΄Ρƒ этими трСмя Π²Π΅Π»ΠΈΡ‡ΠΈΠ½Π°ΠΌΠΈ с Ρ‚ΠΎΡ‡ΠΊΠΈ
зрСния ΠΌΠ°Ρ‚Π΅ΠΌΠ°Ρ‚ΠΈΠΊΠΈ ΠΌΠΎΠΆΠ½ΠΎ Π·Π°ΠΏΠΈΡΠ°Ρ‚ΡŒ Π² Ρ‚Ρ€Π΅Ρ…
Π²ΠΈΠ΄Π°Ρ…: U=R*I, I=U/R ΠΈ R=U/I. ВсС Ρ‚Ρ€ΠΈ Π²ΠΈΠ΄Π° записи
ΠΌΠΎΠΆΠ½ΠΎ ΠΏΡ€ΠΈΠΌΠ΅Π½ΡΡ‚ΡŒ Π½Π° ΠΏΡ€Π°ΠΊΡ‚ΠΈΠΊΠ΅, Π½ΠΎ Π½Π΅
слСдуСт Π·Π°Π±Ρ‹Π²Π°Ρ‚ΡŒ, Ρ‡Ρ‚ΠΎ, Π½Π°ΠΏΡ€ΠΈΠΌΠ΅Ρ€, послСднСС
ΡΠΎΠΎΡ‚Π½ΠΎΡˆΠ΅Π½ΠΈΠ΅ позволяСт Π²Ρ‹Ρ‡ΠΈΡΠ»ΠΈΡ‚ΡŒ
сопротивлСниС, ΠΎΠ΄Π½Π°ΠΊΠΎ сопротивлСниС
ΠΏΡ€ΠΎΠ²ΠΎΠ΄Π½ΠΈΠΊΠ° ΠΈΠ»ΠΈ рСзистора опрСдСляСтся
свойствами ΠΌΠ°Ρ‚Π΅Ρ€ΠΈΠ°Π»Π° ΠΈ Π³Π΅ΠΎΠΌΠ΅Ρ‚Ρ€ΠΈΠ΅ΠΉ
ΠΏΡ€ΠΎΠ²ΠΎΠ΄Π½ΠΈΠΊΠ°, Π° Ρ‚ΠΎΠΊ, вычисляСмый ΠΏΠΎ
Π½Π°ΠΏΡ€ΡΠΆΠ΅Π½ΠΈΡŽ ΠΈ ΡΠΎΠΏΡ€ΠΎΡ‚ΠΈΠ²Π»Π΅Π½ΠΈΡŽ, ΠΊΠ°ΠΊ
упорядочСнноС Π΄Π²ΠΈΠΆΠ΅Π½ΠΈΠ΅ носитСлСй
заряда, вызваСтся источником элСктродвиТущСй
силы. НапряТСниС Π² этом смыслС ΠΌΠΎΠΆΠ½ΠΎ
Ρ€Π°ΡΡΠΌΠ°Ρ‚Ρ€ΠΈΠ²Π°Ρ‚ΡŒ, ΠΊΠ°ΠΊ Π½Π΅ΠΊΠΎΡ‚ΠΎΡ€ΠΎΠ΅ напряТСниС
Π² ΠΎΡ‚Π½ΠΎΡˆΠ΅Π½ΠΈΡΡ… ΠΌΠ΅ΠΆΠ΄Ρƒ Ρ‚ΠΎΠΊΠΎΠΌ, ΠΏΡ€ΠΎΡ‚Π΅ΠΊΠ°ΡŽΡ‰ΠΈΠΌ
ΠΏΠΎ ΠΏΡ€ΠΎΠ²ΠΎΠ΄Π½ΠΈΠΊΡƒ, ΠΈ ΠΌΠ°Ρ‚Π΅Ρ€ΠΈΠ°Π»ΠΎΠΌ ΠΏΡ€ΠΎΠ²ΠΎΠ΄Π½ΠΈΠΊΠ°,
ΠΎΠΊΠ°Π·Ρ‹Π²Π°ΡŽΡ‰ΠΈΠΌ сопротивлСниС ΠΏΡ€ΠΎΡ‚Π΅ΠΊΠ°Π½ΠΈΡŽ
Ρ‚ΠΎΠΊΠ°.

Для
ΡΠΊΡΠΏΠ΅Ρ€ΠΈΠΌΠ΅Π½Ρ‚Π°Π»ΡŒΠ½ΠΎΠΉ ΠΏΡ€ΠΎΠ²Π΅Ρ€ΠΊΠΈ ΡΠΎΠΎΡ‚Π½ΠΎΡˆΠ΅Π½ΠΈΡ
ΠΌΠ΅ΠΆΠ΄Ρƒ напряТСниСм, Ρ‚ΠΎΠΊΠΎΠΌ ΠΈ сопротивлСниСм
ΠΌΠΎΠΆΠ½ΠΎ ΡΠΎΠ±Ρ€Π°Ρ‚ΡŒ схСму, Π°Π½Π°Π»ΠΎΠ³ΠΈΡ‡Π½ΡƒΡŽ
ΠΈΠ·ΠΎΠ±Ρ€Π°ΠΆΠ΅Π½Π½ΠΎΠΉ Π½Π° рис. 1.1, Π² ΠΊΠΎΡ‚ΠΎΡ€ΡƒΡŽ слСдуСт
Π΄ΠΎΠ±Π°Π²ΠΈΡ‚ΡŒ Π΄Π²Π° ΠΈΠ·ΠΌΠ΅Ρ€ΠΈΡ‚Π΅Π»ΡŒΠ½Ρ‹Ρ… ΠΏΡ€ΠΈΠ±ΠΎΡ€Π°:
Π²ΠΎΠ»ΡŒΡ‚ΠΌΠ΅Ρ‚Ρ€ ΠΈ Π°ΠΌΠΏΠ΅Ρ€ΠΌΠ΅Ρ‚Ρ€ (ΠΈΠ»ΠΈ ΠΈΡΠΏΠΎΠ»ΡŒΠ·ΠΎΠ²Π°Ρ‚ΡŒ
ΠΌΡƒΠ»ΡŒΡ‚ΠΈΠΌΠ΅Ρ‚Ρ€, произвСдя Π΄Π²Π° измСрСния).
Π’ΠΎΠ»ΡŒΡ‚ΠΌΠ΅Ρ‚Ρ€ – это ΠΏΡ€ΠΈΠ±ΠΎΡ€ ΠΈΡΠΏΠΎΠ»ΡŒΠ·ΡƒΠ΅ΠΌΡ‹ΠΉ
для измСрСния напряТСния. Π‘Ρ‹Π²Π°ΡŽΡ‚
Π²ΠΎΠ»ΡŒΡ‚ΠΌΠ΅Ρ‚Ρ€Ρ‹ для измСрСния постоянного
напряТСния, ΠΊΠΎΡ‚ΠΎΡ€Ρ‹ΠΌ Π²ΠΎΡΠΏΠΎΠ»ΡŒΠ·ΡƒΠ΅ΠΌΡΡ ΠΌΡ‹,
Π° Π±Ρ‹Π²Π°ΡŽΡ‚ Π²ΠΎΠ»ΡŒΡ‚ΠΌΠ΅Ρ‚Ρ€Ρ‹ ΠΏΠ΅Ρ€Π΅ΠΌΠ΅Π½Π½ΠΎΠ³ΠΎ
напряТСния, ΠΎ ΠΊΠΎΡ‚ΠΎΡ€Ρ‹Ρ… Ρ€Π΅Ρ‡ΡŒ ΠΏΠΎΠΉΠ΄Π΅Ρ‚ дальшС.
АмпСрмСтр – это ΠΏΡ€ΠΈΠ±ΠΎΡ€ для измСрСния
Ρ‚ΠΎΠΊΠ°, ΠΊΠΎΡ‚ΠΎΡ€Ρ‹ΠΉ Ρ‚Π°ΠΊΠΆΠ΅ Π±Ρ‹Π²Π°Π΅Ρ‚ для ΠΈΠ·ΠΌΠ΅Ρ€Π΅Π½ΠΈΠΉ
Π² цСпях постоянного ΠΈ ΠΏΠ΅Ρ€Π΅ΠΌΠ΅Π½Π½ΠΎΠ³ΠΎ Ρ‚ΠΎΠΊΠ°.

Π­Ρ‚ΠΈ
ΠΏΡ€ΠΈΠ±ΠΎΡ€Ρ‹ Π±Ρ‹Π²Π°ΡŽΡ‚ ΠΎΡ‡Π΅Π½ΡŒ Ρ€Π°Π·Π½ΠΎΠΉ конструкции.
Π― Π³ΠΎΠ²ΠΎΡ€ΠΈΠ» ΠΎ ΠΌΡƒΠ»ΡŒΡ‚ΠΈΠΌΠ΅Ρ‚Ρ€Π΅. Π’ Π΅Π³ΠΎ основС
Π»Π΅ΠΆΠΈΡ‚ Ρ€Π°Π±ΠΎΡ‚Π° спСциализированной ΠΈ
достаточно слоТной микросхСмы, Π½Π°Π·Ρ‹Π²Π°Π΅ΠΌΠΎΠΉ
Π°Π½Π°Π»ΠΎΠ³ΠΎ-Ρ†ΠΈΡ„Ρ€ΠΎΠ²ΠΎΠΉ ΠΏΡ€Π΅ΠΎΠ±Ρ€Π°Π·ΠΎΠ²Π°Ρ‚Π΅Π»ΡŒ (АЦП).
МногиС ΠΌΡƒΠ»ΡŒΡ‚ΠΈΠΌΠ΅Ρ‚Ρ€Ρ‹ ΠΌΠΎΠ³ΡƒΡ‚ ΠΊΡ€ΠΎΠΌΠ΅ этой
микросхСмы Π½Π΅ ΠΈΠΌΠ΅Ρ‚ΡŒ Π΄Ρ€ΡƒΠ³ΠΈΡ… микросхСм,
лишь Π²ΡΠΏΠΎΠΌΠΎΠ³Π°Ρ‚Π΅Π»ΡŒΠ½Ρ‹Π΅ рСзисторы,
ΠΏΠ΅Ρ€Π΅ΠΊΠ»ΡŽΡ‡Π°Ρ‚Π΅Π»ΡŒ ΠΈ дисплСй, ΠΎΡ‚ΠΎΠ±Ρ€Π°ΠΆΠ°ΡŽΡ‰ΠΈΠΉ
Ρ†ΠΈΡ„Ρ€Ρ‹. Π’ΠΎΠ·ΠΌΠΎΠΆΠ½ΠΎ, ΠΊ ΠΊΠΎΠ½Ρ†Ρƒ ΠΊΠ½ΠΈΠ³ΠΈ ΠΌΡ‹
рассмотрим Ρ€Π°Π±ΠΎΡ‚Ρƒ Ρ‚Π°ΠΊΠΎΠ³ΠΎ ΠΏΡ€ΠΈΠ±ΠΎΡ€Π°, Π°
сСйчас, всС-Ρ‚Π°ΠΊΠΈ ΠΎΠΏΡΡ‚ΡŒ забСгая Π²ΠΏΠ΅Ρ€Π΅Π΄,
я Π½Π΅ΠΌΠ½ΠΎΠ³ΠΎ расскаТу ΠΎ стрСлочном ΠΈΠ·ΠΌΠ΅Ρ€ΠΈΡ‚Π΅Π»Π΅
Ρ‚ΠΎΠΊΠ° ΠΈ стрСлочном ΠΈΠ·ΠΌΠ΅Ρ€ΠΈΡ‚Π΅Π»Π΅ напряТСния.
ΠŸΡ€Π°Π²Π΄Π° ΠΈ ΠΎΠ½ΠΈ Π±Ρ‹Π²Π°ΡŽΡ‚ ΠΎΡ‡Π΅Π½ΡŒ Ρ€Π°Π·Π½Ρ‹Ρ…
конструкций, ΠΈ ΠΈΡ… Ρ€Π°Π±ΠΎΡ‚Π° ΠΌΠΎΠΆΠ΅Ρ‚ Π±Ρ‹Ρ‚ΡŒ
основана Π½Π° Ρ€Π°Π·Π½Ρ‹Ρ… ΠΏΡ€ΠΈΠ½Ρ†ΠΈΠΏΠ°Ρ…. Однако
достаточно часто Π² стрСлочных ΠΈΠ·ΠΌΠ΅Ρ€ΠΈΡ‚Π΅Π»ΡŒΠ½Ρ‹Ρ…
ΠΏΡ€ΠΈΠ±ΠΎΡ€Π°Ρ… ΠΈΡΠΏΠΎΠ»ΡŒΠ·ΡƒΡŽΡ‚ ΡΠ»Π΅Π΄ΡƒΡŽΡ‰ΡƒΡŽ ΠΊΠΎΠ½ΡΡ‚Ρ€ΡƒΠΊΡ†ΠΈΡŽ:
Ρ€Π°ΠΌΠΊΡƒ с Π½Π°ΠΌΠΎΡ‚Π°Π½Π½Ρ‹ΠΌ Π½Π° Π½Π΅Π΅ ΠΏΡ€ΠΎΠ²ΠΎΠ΄ΠΎΠΌ
ΠΏΠΎΠΌΠ΅Ρ‰Π°ΡŽΡ‚ Π² ΠΌΠ°Π³Π½ΠΈΡ‚Π½ΠΎΠ΅ ΠΏΠΎΠ»Π΅, создаваСмоС
постоянными ΠΌΠ°Π³Π½ΠΈΡ‚Π°ΠΌΠΈ, ΠΈ крСпят с ΠΏΠΎΠΌΠΎΡ‰ΡŒΡŽ
ΡΠΏΠΈΡ€Π°Π»ΡŒΠ½Ρ‹Ρ… ΠΏΡ€ΡƒΠΆΠΈΠ½ΠΎΠΊ Π½Π° оси Ρ‚Π°ΠΊ, Ρ‡Ρ‚ΠΎΠ±Ρ‹
стрСлка, прикрСплСнная ΠΊ этой Ρ€Π°ΠΌΠΊΠ΅,
Π½Π°Ρ…ΠΎΠ΄ΠΈΠ»Π°ΡΡŒ Π² Π½ΡƒΠ»Π΅Π²ΠΎΠΌ ΠΏΠΎΠ»ΠΎΠΆΠ΅Π½ΠΈΠΈ, ΠΎΡ‚ΠΌΠ΅Ρ‡Π΅Π½Π½ΠΎΠΌ
Π½Π° шкалС. Когда ΠΏΠΎ Ρ€Π°ΠΌΠΊΠ΅ ΠΏΡ€ΠΎΡ‚Π΅ΠΊΠ°Π΅Ρ‚
постоянный Ρ‚ΠΎΠΊ, Ρ‚ΠΎ Π² ΠΌΠ°Π³Π½ΠΈΡ‚Π½ΠΎΠΌ ΠΏΠΎΠ»Π΅ Π½Π°
Ρ€Π°ΠΌΠΊΡƒ дСйствуСт сила, заставляя Π΅Π΅
Π²Ρ€Π°Ρ‰Π°Ρ‚ΡŒΡΡ. ΠŸΠΎΠ²ΠΎΡ€ΠΎΡ‚ происходит Π΄ΠΎ Ρ‚Π΅Ρ…
ΠΏΠΎΡ€, ΠΏΠΎΠΊΠ° сила, вызываСмая Ρ‚ΠΎΠΊΠΎΠΌ, Π½Π΅
уравновСсит Π²ΠΎΠ·Π²Ρ€Π°Ρ‰Π°ΡŽΡ‰ΡƒΡŽ силу ΠΏΡ€ΡƒΠΆΠΈΠ½ΠΎΠΊ.
Π‘Ρ‚Ρ€Π΅Π»ΠΊΠ° останавливаСтся Π² ΠΏΠΎΠ»ΠΎΠΆΠ΅Π½ΠΈΠΈ,
ΠΊΠΎΡ‚ΠΎΡ€ΠΎΠ΅ ΠΏΡ€ΠΎΠΏΠΎΡ€Ρ†ΠΈΠΎΠ½Π°Π»ΡŒΠ½ΠΎ ΠΏΡ€ΠΎΡ‚Π΅ΠΊΠ°ΡŽΡ‰Π΅ΠΌΡƒ
Ρ‚ΠΎΠΊΡƒ, Π·Π½Π°Ρ‡Π΅Π½ΠΈΠ΅ ΠΊΠΎΡ‚ΠΎΡ€ΠΎΠ³ΠΎ считываСтся со
ΡˆΠΊΠ°Π»Ρ‹ ΠΏΡ€ΠΈΠ±ΠΎΡ€Π°. Если Ρƒ вас Π΅ΡΡ‚ΡŒ тСстСр
со стрСлочной ΠΈΠ·ΠΌΠ΅Ρ€ΠΈΡ‚Π΅Π»ΡŒΠ½ΠΎΠΉ Π³ΠΎΠ»ΠΎΠ²ΠΊΠΎΠΉ,
Ρ‚ΠΎ ΠΎΠ½Π° ΠΌΠΎΠΆΠ΅Ρ‚ Π±Ρ‹Ρ‚ΡŒ устроСна ΠΈΠΌΠ΅Π½Π½ΠΎ Ρ‚Π°ΠΊ
ΠΈΠ»ΠΈ ΠΏΠΎΡ…ΠΎΠΆΠΈΠΌ ΠΎΠ±Ρ€Π°Π·ΠΎΠΌ: часто оси вращСния
ΠΈ ΠΏΡ€ΡƒΠΆΠΈΠ½ΠΊΠΈ Π²ΠΎΠ·Π²Ρ€Π°Ρ‚Π° стрСлки Π² ноль
Π·Π°ΠΌΠ΅Π½ΡΡŽΡ‚ «растяТками». Π’ΠΎ Π΅ΡΡ‚ΡŒ, ΠΊ
Ρ€Π°ΠΌΠΊΠ΅ с ΠΏΡ€ΠΎΠ²ΠΎΠ΄ΠΎΠΌ ΠΏΡ€ΠΈΠΊΡ€Π΅ΠΏΠ»ΡΡŽΡ‚ ΠΎΡ‡Π΅Π½ΡŒ
Ρ‚ΠΎΠ½ΠΊΡƒΡŽ полоску ΠΈΠ· пруТинящСго ΠΌΠ°Ρ‚Π΅Ρ€ΠΈΠ°Π»Π°.
Растянутая Π½Π° Π΄Π²ΡƒΡ… Ρ‚Π°ΠΊΠΈΡ… полосках Ρ€Π°ΠΌΠΊΠ°
ΠΏΠΎΠ΄ ΠΈΡ… дСйствиСм Π·Π°Π½ΠΈΠΌΠ°Π΅Ρ‚ Β«Π½ΡƒΠ»Π΅Π²ΠΎΠ΅Β»
ΠΏΠΎΠ»ΠΎΠΆΠ΅Π½ΠΈΠ΅, Π° ΠΏΡ€ΠΈ ΠΏΡ€ΠΎΡ‚Π΅ΠΊΠ°Π½ΠΈΠΈ Ρ‚ΠΎΠΊΠ°
поворачиваСтся, ΠΎΡ‚Ρ‡Π΅Π³ΠΎ полоски ΡƒΠΏΡ€ΡƒΠ³ΠΎ
ΡΠΊΡ€ΡƒΡ‡ΠΈΠ²Π°ΡŽΡ‚ΡΡ, образуя силу, Π²ΠΎΠ·Π²Ρ€Π°Ρ‰Π°ΡŽΡ‰ΡƒΡŽ
Ρ€Π°ΠΌΠΊΡƒ со стрСлкой Π² ΠΏΠ΅Ρ€Π²ΠΎΠ½Π°Ρ‡Π°Π»ΡŒΠ½ΠΎΠ΅
ΠΏΠΎΠ»ΠΎΠΆΠ΅Π½ΠΈΠ΅, ΠΊΠΎΠ³Π΄Π° Ρ‚ΠΎΠΊ пСрСстаСт ΠΏΡ€ΠΎΡ‚Π΅ΠΊΠ°Ρ‚ΡŒ
ΠΏΠΎ Ρ€Π°ΠΌΠΊΠ΅. Но ΠΎ ΠΌΠ΅Ρ…Π°Π½ΠΈΠ·ΠΌΠ΅ взаимодСйствия
ΠΏΡ€ΠΎΠ²ΠΎΠ΄Π° с Ρ‚ΠΎΠΊΠΎΠΌ ΠΈ ΠΌΠ°Π³Π½ΠΈΡ‚Π½ΠΎΠ³ΠΎ поля я
Π½ΠΈΡ‡Π΅Π³ΠΎ Π²Π°ΠΌ Π½Π΅ Π³ΠΎΠ²ΠΎΡ€ΠΈΠ», поэтому Π±ΡƒΠ΄Π΅ΠΌ
ΡΡ‡ΠΈΡ‚Π°Ρ‚ΡŒ, Ρ‡Ρ‚ΠΎ Π½ΠΈΡ‡Π΅Π³ΠΎ Π½Π΅ Π³ΠΎΠ²ΠΎΡ€ΠΈΠ» ΠΈ ΠΎ
стрСлочных ΠΏΡ€ΠΈΠ±ΠΎΡ€Π°Ρ…. Π₯отя, ΠΏΠΎΠΆΠ°Π»ΡƒΠΉ,
добавлю ΠΊ Ρ‚ΠΎΠΌΡƒ, ΠΎ Ρ‡Π΅ΠΌ Π½Π΅ Π³ΠΎΠ²ΠΎΡ€ΠΈΠ», Ρ‡Ρ‚ΠΎ
ΠΈΠ·ΠΌΠ΅Ρ€ΠΈΡ‚Π΅Π»ΡŒ Ρ‚ΠΎΠΊΠ° Π»Π΅Π³ΠΊΠΎ ΠΏΡ€Π΅Π²Ρ€Π°Ρ‚ΠΈΡ‚ΡŒ Π²
ΠΈΠ·ΠΌΠ΅Ρ€ΠΈΡ‚Π΅Π»ΡŒ напряТСния, Π΄ΠΎΠ±Π°Π²ΠΈΠ² ΠΊ Π½Π΅ΠΌΡƒ
рСзистор.

Π˜Ρ‚Π°ΠΊ,
посмотрим, Π½Π° Π·Π°ΠΊΠΎΠ½ Ома, ΠΊΠΎΡ‚ΠΎΡ€Ρ‹ΠΉ я
ΠΏΠΎΡΡ‚Π°Ρ€Π°ΡŽΡΡŒ ΠΏΡ€ΠΎΠΈΠ»Π»ΡŽΡΡ‚Ρ€ΠΈΡ€ΠΎΠ²Π°Ρ‚ΡŒ Π² ΠΏΡ€ΠΎΠ³Ρ€Π°ΠΌΠΌΠ΅
PSIM. ΠšΡ€ΠΎΠΌΠ΅ Ρ‚ΠΎΠ³ΠΎ, я спаяю схСму Π½Π° ΠΌΠ°ΠΊΠ΅Ρ‚Π½ΠΎΠΉ
ΠΏΠ»Π°Ρ‚Π΅, ΠΈΡΠΏΠΎΠ»ΡŒΠ·ΡƒΡ ΠΎΠ΄ΠΈΠ½ ΠΈΠ· рСзисторов,
ΠΊΠΎΡ‚ΠΎΡ€Ρ‹Π΅ выпаял ΠΈΠ· ΠΏΠ»Π°Ρ‚Ρ‹, Π²Π΄Ρ€ΡƒΠ³ я Ρ‡Ρ‚ΠΎ-Ρ‚ΠΎ
Π½Π°ΠΏΡƒΡ‚Π°Π» ΠΈ Π²Π²Π΅Π» вас Π² Π·Π°Π±Π»ΡƒΠΆΠ΄Π΅Π½ΠΈΠ΅. ДовСряй,
Π½ΠΎ провСряй.

Рис.
1.2. ЭкспСримСнт, ΠΈΠ»Π»ΡŽΡΡ‚Ρ€ΠΈΡ€ΡƒΡŽΡ‰ΠΈΠΉ Π·Π°ΠΊΠΎΠ½
Ома Π² ΠΏΡ€ΠΎΠ³Ρ€Π°ΠΌΠΌΠ΅ PSIM

Как
Π²ΠΈΠ΄Π½ΠΎ ΠΈΠ· рисунка, ΠΊ источнику питания
10V (10 Π’ΠΎΠ»ΡŒΡ‚, Π½Π° схСмС ΠΎΠ±ΠΎΠ·Π½Π°Ρ‡Π΅Π½ VDC1)
ΠΏΠΎΠ΄ΠΊΠ»ΡŽΡ‡Π΅Π½ рСзистор 1 кОм (1000 Ом), ΠΏΠ°Ρ€Π°Π»Π»Π΅Π»ΡŒΠ½ΠΎ
ΠΊΠΎΡ‚ΠΎΡ€ΠΎΠΌΡƒ Π²ΠΊΠ»ΡŽΡ‡Π΅Π½ Π²ΠΎΠ»ΡŒΡ‚ΠΌΠ΅Ρ‚Ρ€ VP1, ΠΈ
ΠΏΠΎΡΠ»Π΅Π΄ΠΎΠ²Π°Ρ‚Π΅Π»ΡŒΠ½ΠΎ с ΠΊΠΎΡ‚ΠΎΡ€Ρ‹ΠΌ Π²ΠΊΠ»ΡŽΡ‡Π΅Π½
Π°ΠΌΠΏΠ΅Ρ€ΠΌΠ΅Ρ‚Ρ€ I1. Π“Ρ€Π°Ρ„ΠΈΠΊ измСрСния Ρ‚ΠΎΠΊΠ°
ΠΏΠΎΠΊΠ°Π·Ρ‹Π²Π°Π΅Ρ‚, Ρ‡Ρ‚ΠΎ Ρ‚ΠΎΠΊ Ρ€Π°Π²Π΅Π½ 10.00m (10 ΠΌΠΈΠ»Π»ΠΈΠ°ΠΌΠΏΠ΅Ρ€)
ΠΈΠ»ΠΈ 0.01 А. Если ΡƒΠΌΠ½ΠΎΠΆΠΈΡ‚ΡŒ сопротивлСниС
Π½Π° Ρ‚ΠΎΠΊ, Ρ‚ΠΎ Π΅ΡΡ‚ΡŒ, 1000 Ом ΡƒΠΌΠ½ΠΎΠΆΠΈΡ‚ΡŒ Π½Π° 0.01 А,
Ρ‚ΠΎ получится ΠΏΠ°Π΄Π΅Π½ΠΈΠ΅ напряТСния Π² 10
Π²ΠΎΠ»ΡŒΡ‚, ΠΊΠΎΡ‚ΠΎΡ€Ρ‹Π΅ ΠΈ ΠΏΠΎΠΊΠ°Π·Ρ‹Π²Π°Π΅Ρ‚ Π²Ρ‚ΠΎΡ€ΠΎΠΉ
Π³Ρ€Π°Ρ„ΠΈΠΊ, ΠΎΡ‚ΠΎΠ±Ρ€Π°ΠΆΠ°ΡŽΡ‰ΠΈΠΉ показания Π²ΠΎΠ»ΡŒΡ‚ΠΌΠ΅Ρ‚Ρ€Π°.
Для получСния ΠΏΡ€Π°Π²ΠΈΠ»ΡŒΠ½Ρ‹Ρ… Π·Π½Π°Ρ‡Π΅Π½ΠΈΠΉ ΠΏΡ€ΠΈ
расчСтах слСдуСт ΠΏΠΎΠ»ΡŒΠ·ΠΎΠ²Π°Ρ‚ΡŒΡΡ основными
Π΅Π΄ΠΈΠ½ΠΈΡ†Π°ΠΌΠΈ, Π² Π΄Π°Π½Π½ΠΎΠΌ случаС Π°ΠΌΠΏΠ΅Ρ€, ΠΎΠΌ ΠΈ
Π²ΠΎΠ»ΡŒΡ‚.

А
Π²ΠΎΡ‚, Ρ‡Ρ‚ΠΎ ΠΏΠΎΠ»ΡƒΡ‡ΠΈΠ»ΠΎΡΡŒ с ΠΌΠ°ΠΊΠ΅Ρ‚ΠΎΠΌ. На
Π±Π°Ρ‚Π°Ρ€Π΅ΠΉΠΊΠ΅, ΠΊΠΎΡ‚ΠΎΡ€ΡƒΡŽ я ΠΈΡΠΏΠΎΠ»ΡŒΠ·ΡƒΡŽ, написано
9V, Π½Π° рСзисторС 1ΠΊ, Ρ‚ΠΎΠΊ Π΄ΠΎΠ»ΠΆΠ΅Π½ ΠΏΠΎΠ»ΡƒΡ‡ΠΈΡ‚ΡŒΡΡ
9мА (ΠΌΠΈΠ»Π»ΠΈΠ°ΠΌΠΏΠ΅Ρ€, 0.009А). Π˜Π·ΠΌΠ΅Ρ€Π΅Π½Π½Ρ‹ΠΉ Ρ‚ΠΎΠΊ
8мА.

Π’
Ρ‡Π΅ΠΌ ошибка? Π’ΠΎ-ΠΏΠ΅Ρ€Π²Ρ‹Ρ…, я Π½Π΅ ΠΈΠ·ΠΌΠ΅Ρ€ΠΈΠ» Π­Π”Π‘
(напряТСниС Π½Π° Π±Π°Ρ‚Π°Ρ€Π΅ΠΉΠΊΠ΅), Π²ΠΎ-Π²Ρ‚ΠΎΡ€Ρ‹Ρ…, Π½Π΅
ΠΈΠ·ΠΌΠ΅Ρ€ΠΈΠ» сопротивлСниС. РСальноС
сопротивлСниС рСзистора, Ссли Π΅Π³ΠΎ
ΠΈΠ·ΠΌΠ΅Ρ€ΠΈΡ‚ΡŒ, Π½Π΅ 1 кОм (ΠΊΠΈΠ»ΠΎΠΎΠΌ, 1000 Ом), Π° 910 Ом.
А Π­Π”Π‘ Π±Π°Ρ‚Π°Ρ€Π΅ΠΉΠΊΠΈ послС ΠΏΠΎΠ΄ΠΊΠ»ΡŽΡ‡Π΅Π½ΠΈΡ
рСзистора оказываСтся Ρ€Π°Π²Π½ΠΎΠΉ 7.31 Π²ΠΎΠ»ΡŒΡ‚.
ΠžΡ‚ΡΡŽΠ΄Π° ΠΈ расхоТдСниС, скорСС Π½Π΅ Π² Ρ‚Π΅ΠΎΡ€ΠΈΠΈ
ΠΈ ΠΏΡ€Π°ΠΊΡ‚ΠΈΠΊΠ΅, Π° Π² ΠΌΠΎΠ΅ΠΌ прСдставлСнии ΠΎ
Ρ‚ΠΎΠΌ, Ρ‡Ρ‚ΠΎ я дСлаю, ΠΈ Ρ‚Π΅ΠΌ, Ρ‡Ρ‚ΠΎ дСлаю Π²
Π΄Π΅ΠΉΡΡ‚Π²ΠΈΡ‚Π΅Π»ΡŒΠ½ΠΎΡΡ‚ΠΈ. ДовСряй, Π½ΠΎ провСряй!

ΠžΡ‚
Π·Π°ΠΊΠΎΠ½Π° Ома ΠΌΠΎΠΆΠ½ΠΎ ΠΏΠ»Π°Π²Π½ΠΎ ΠΏΠ΅Ρ€Π΅ΠΉΡ‚ΠΈ ΠΊΠΎ
Π²Ρ‚ΠΎΡ€ΠΎΠΌΡƒ Π·Π°ΠΊΠΎΠ½Ρƒ ΠšΠΈΡ€Ρ…Π³ΠΎΡ„Π°. ΠŸΠΎΡ‡Π΅ΠΌΡƒ ΠΊΠΎ
Π²Ρ‚ΠΎΡ€ΠΎΠΌΡƒ, Π° Π½Π΅ ΠΊ ΠΏΠ΅Ρ€Π²ΠΎΠΌΡƒ? МнС Ρ‚Π°ΠΊ ΡƒΠ΄ΠΎΠ±Π½Π΅Π΅.

Π’
ΡƒΠΏΡ€ΠΎΡ‰Π΅Π½Π½ΠΎΠΌ Π²ΠΈΠ΄Π΅ Π·Π°ΠΊΠΎΠ½ ΠšΠΈΡ€Ρ…Π³ΠΎΡ„Π° Π·Π²ΡƒΡ‡ΠΈΡ‚
Ρ‚Π°ΠΊ:

Π­Π”Π‘
Π² Π·Π°ΠΌΠΊΠ½ΡƒΡ‚ΠΎΠΌ ΠΊΠΎΠ½Ρ‚ΡƒΡ€Π΅ Ρ€Π°Π²Π½Π° суммС ΠΏΠ°Π΄Π΅Π½ΠΈΠΉ
напряТСний.

Π”Π΅ΠΉΡΡ‚Π²ΠΈΡ‚Π΅Π»ΡŒΠ½ΠΎ,
Π½Π° рисункС Π²Ρ‹ΡˆΠ΅ Π­Π”Π‘ (источника питания
VDC1) 10 Π’, Π° напряТСниС Π½Π° рСзисторС Ρ‚ΠΎΠΆΠ΅
10 Π’. МоТно ΠΈΠ·ΠΌΠ΅Π½ΠΈΡ‚ΡŒ схСму, Π²ΠΊΠ»ΡŽΡ‡ΠΈΠ²
ΠΏΠΎΡΠ»Π΅Π΄ΠΎΠ²Π°Ρ‚Π΅Π»ΡŒΠ½ΠΎ Π΄Π²Π° рСзистора, Π½Π°ΠΏΡ€ΠΈΠΌΠ΅Ρ€,
ΠΏΠΎ 500 Ом, ΠΈΠ·ΠΌΠ΅Ρ€ΠΈΡ‚ΡŒ Π½Π° Π½ΠΈΡ… напряТСния ΠΈ
ΡƒΠ±Π΅Π΄ΠΈΡ‚ΡŒΡΡ, Ρ‡Ρ‚ΠΎ Π½Π° ΠΊΠ°ΠΆΠ΄ΠΎΠΌ ΠΈΠ· Π½ΠΈΡ… Π±ΡƒΠ΄Π΅Ρ‚
ΠΏΠ°Π΄Π΅Π½ΠΈΠ΅ напряТСния 5 Π’, Π° сумма этих
напряТСний получится 10 Π’.

Π—Π΄Π΅ΡΡŒ
умСстно Π΄ΠΎΠ±Π°Π²ΠΈΡ‚ΡŒ, Ρ‡Ρ‚ΠΎ Π±ΠΎΠ»Π΅Π΅ Π²Π΅Ρ€Π½ΠΎ этот
Π·Π°ΠΊΠΎΠ½ Π·Π²ΡƒΡ‡Π°Π» Π±Ρ‹ Ρ‚Π°ΠΊ: алгСбраичСская
сумма всСх Π­Π”Π‘ Π² Π·Π°ΠΌΠΊΠ½ΡƒΡ‚ΠΎΠΌ ΠΊΠΎΠ½Ρ‚ΡƒΡ€Π΅ Ρ€Π°Π²Π½Π°
алгСбраичСской суммС ΠΏΠ°Π΄Π΅Π½ΠΈΠΉ напряТСний.
Но ΠΎΠ± этом Π»ΡƒΡ‡ΡˆΠ΅ ΠΏΠΎΡ‡ΠΈΡ‚Π°Ρ‚ΡŒ Π² ΡƒΡ‡Π΅Π±Π½ΠΈΠΊΠ΅.

Π₯отя
для Ρ€Π°Π·Π³ΠΎΠ²ΠΎΡ€Π° ΠΎ ΠΏΠ΅Ρ€Π²ΠΎΠΌ Π·Π°ΠΊΠΎΠ½Π΅ ΠšΠΈΡ€Ρ…Π³ΠΎΡ„Π°
слСдовало Π±Ρ‹ Π½Π°Ρ€ΠΈΡΠΎΠ²Π°Ρ‚ΡŒ Π΄Ρ€ΡƒΠ³ΡƒΡŽ схСму
(Π΄Ρ€ΡƒΠ³ΡƒΡŽ Ρ†Π΅ΠΏΡŒ), я ΠΈΡΠΏΠΎΠ»ΡŒΠ·ΡƒΡŽ схСму рис.
1.2. Π’Ρ‹ΡˆΠ΅ я Π³ΠΎΠ²ΠΎΡ€ΠΈΠ» ΠΎ Ρ‚ΠΎΠΌ, Ρ‡Ρ‚ΠΎ часто
Π²ΠΎΠ»ΡŒΡ‚ΠΌΠ΅Ρ‚Ρ€ – это Ρ‚ΠΎΠΆΠ΅ ΠΈΠ·ΠΌΠ΅Ρ€ΠΈΡ‚Π΅Π»ΡŒ Ρ‚ΠΎΠΊΠ°,
Ρ‚ΠΎ Π΅ΡΡ‚ΡŒ, Ρ‚ΠΎΠΊ ΠΏΡ€ΠΎΡ‚Π΅ΠΊΠ°Π΅Ρ‚ Π½Π΅ Ρ‚ΠΎΠ»ΡŒΠΊΠΎ Ρ‡Π΅Ρ€Π΅Π·
рСзистор R1, Π½ΠΎ ΠΈ Ρ‡Π΅Ρ€Π΅Π· Π²ΠΎΠ»ΡŒΡ‚ΠΌΠ΅Ρ‚Ρ€. Π’Π°ΠΊΠΈΠΌ
ΠΎΠ±Ρ€Π°Π·ΠΎΠΌ, Π² Ρ‚ΠΎΡ‡ΠΊΠ΅ соСдинСния Π²ΠΎΠ»ΡŒΡ‚ΠΌΠ΅Ρ‚Ρ€Π°
ΠΈ рСзистора Ρ‚ΠΎΠΊ развСтвляСтся, ΠΎΠ΄Π½Π° Π΅Π³ΠΎ
Ρ‡Π°ΡΡ‚ΡŒ ΠΏΡ€ΠΎΡ‚Π΅ΠΊΠ°Π΅Ρ‚ ΠΏΠΎ рСзистору, другая
ΠΏΠΎ Π²ΠΎΠ»ΡŒΡ‚ΠΌΠ΅Ρ‚Ρ€Ρƒ, Π° Π·Π°Ρ‚Π΅ΠΌ ΠΎΠ±Π΅ эти части
ΡΠΎΠ΅Π΄ΠΈΠ½ΡΡŽΡ‚ΡΡ ΠΈ ΠΏΡ€ΠΎΡ‚Π΅ΠΊΠ°ΡŽΡ‚ Ρ‡Π΅Ρ€Π΅Π· Π°ΠΌΠΏΠ΅Ρ€ΠΌΠ΅Ρ‚Ρ€.

Π‘ΡƒΠΌΠΌΠ°
Ρ‚ΠΎΠΊΠΎΠ², Π²Ρ‹Ρ‚Π΅ΠΊΠ°ΡŽΡ‰ΠΈΡ… ΠΈΠ· ΡƒΠ·Π»Π° элСктричСской
Ρ†Π΅ΠΏΠΈ ΠΏΡ€ΠΈ Π΅Π΅ Π²Π΅Ρ‚Π²Π»Π΅Π½ΠΈΠΈ, Ρ€Π°Π²Π½Π° Ρ‚ΠΎΠΊΡƒ,
Π²Ρ‚Π΅ΠΊΠ°ΡŽΡ‰Π΅ΠΌΡƒ Π² этот ΡƒΠ·Π΅Π».

Π’Π°ΠΊ
Π² ΡƒΠΏΡ€ΠΎΡ‰Π΅Π½Π½ΠΎΠΌ Π²ΠΈΠ΄Π΅ Π·Π²ΡƒΡ‡ΠΈΡ‚ ΠΏΠ΅Ρ€Π²Ρ‹ΠΉ Π·Π°ΠΊΠΎΠ½
ΠšΠΈΡ€Ρ…Π³ΠΎΡ„Π°. Π’ Ρ€Π΅Π°Π»ΡŒΠ½ΠΎΠΉ схСмС это ΠΌΠΎΠΆΠ½ΠΎ
Π±Ρ‹Π»ΠΎ Π±Ρ‹ ΠΏΡ€ΠΎΠ²Π΅Ρ€ΠΈΡ‚ΡŒ, Π²ΠΊΠ»ΡŽΡ‡ΠΈΠ² Π΅Ρ‰Π΅ Π΄Π²Π°
Π°ΠΌΠΏΠ΅Ρ€ΠΌΠ΅Ρ‚Ρ€Π° Π² вСтвях схСмы, ΠΎΠ΄Π½Π° ΠΈΠ·
ΠΊΠΎΡ‚ΠΎΡ€Ρ‹Ρ… относится ΠΊ рСзистору, Π° вторая
ΠΊ Π²ΠΎΠ»ΡŒΡ‚ΠΌΠ΅Ρ‚Ρ€Ρƒ.

На
ΠΏΡ€Π°ΠΊΡ‚ΠΈΠΊΠ΅, проводя измСрСния, всСгда
слСдуСт ΠΏΠΎΠΌΠ½ΠΈΡ‚ΡŒ, Ρ‡Ρ‚ΠΎ Π²ΠΎΠ»ΡŒΡ‚ΠΌΠ΅Ρ‚Ρ€ ΠΈΠΌΠ΅Π΅Ρ‚
Π½Π΅ΠΊΠΎΡ‚ΠΎΡ€ΠΎΠ΅ Π²Π½ΡƒΡ‚Ρ€Π΅Π½Π½Π΅Π΅ сопротивлСниС,
ΠΊΠΎΡ‚ΠΎΡ€ΠΎΠ΅ ΠΌΠΎΠΆΠ΅Ρ‚ ΠΏΠΎΠ²Π»ΠΈΡΡ‚ΡŒ Π½Π° Ρ€Π΅Π·ΡƒΠ»ΡŒΡ‚Π°Ρ‚Ρ‹
измСрСния. Π’Π°ΠΊ достаточно Ρ…ΠΎΡ€ΠΎΡˆΠΈΠΉ
Π²ΠΎΠ»ΡŒΡ‚ΠΌΠ΅Ρ‚Ρ€ ΠΌΠΎΠΆΠ΅Ρ‚ ΠΈΠΌΠ΅Ρ‚ΡŒ Π²Π½ΡƒΡ‚Ρ€Π΅Π½Π½Π΅Π΅
сопротивлСниС Π² 100 кОм. Много это ΠΈΠ»ΠΈ
ΠΌΠ°Π»ΠΎ? Π­Ρ‚ΠΎ Π½Π΅ ΠΌΠ½ΠΎΠ³ΠΎ ΠΈ Π½Π΅ ΠΌΠ°Π»ΠΎ, Π½ΠΎ Ρ€ΠΎΠ²Π½ΠΎ
ΡΡ‚ΠΎΠ»ΡŒΠΊΠΎ, сколько Π΅ΡΡ‚ΡŒ. Как это ΠΌΠΎΠΆΠ΅Ρ‚
ΠΏΠΎΠ²Π»ΠΈΡΡ‚ΡŒ Π½Π° вашС ΠΏΠΎΠ½ΠΈΠΌΠ°Π½ΠΈΠ΅ происходящСго?
ПолоТим, Ρƒ вас Π΅ΡΡ‚ΡŒ Π²ΠΎΠ»ΡŒΡ‚ΠΌΠ΅Ρ‚Ρ€ ΠΈ Π°ΠΌΠΏΠ΅Ρ€ΠΌΠ΅Ρ‚Ρ€,
ΠΈ Π²Ρ‹ Ρ…ΠΎΡ‚ΠΈΡ‚Π΅ провСсти измСрСния. Π’ΠΎΠ»ΡŒΡ‚ΠΌΠ΅Ρ‚Ρ€
ΠΈΠΌΠ΅Π΅Ρ‚ Π²Π½ΡƒΡ‚Ρ€Π΅Π½Π½Π΅Π΅ сопротивлСниС Ρ€Π°Π²Π½ΠΎΠ΅
100 кОм. Π’Ρ‹ Ρ…ΠΎΡ‚ΠΈΡ‚Π΅ ΠΎΠΏΡ€Π΅Π΄Π΅Π»ΠΈΡ‚ΡŒ, ΠΈΡΠΏΠΎΠ»ΡŒΠ·ΡƒΡ
Π·Π°ΠΊΠΎΠ½ Ома, Π²Π΅Π»ΠΈΡ‡ΠΈΠ½Ρƒ сопротивлСния,
ΠΌΠ°Ρ€ΠΊΠΈΡ€ΠΎΠ²ΠΊΠ° ΠΊΠΎΡ‚ΠΎΡ€ΠΎΠ³ΠΎ ΡΡ‚Π΅Ρ€Π»Π°ΡΡŒ ΠΎΡ‚ Π²Ρ€Π΅ΠΌΠ΅Π½ΠΈ
(Π½ΠΎ это Π±Ρ‹Π» рСзистор Π² 100 кОм, ΠΎ Ρ‡Π΅ΠΌ Π½ΠΈ
Π²Ρ‹, Π½ΠΈ я Π½Π΅ Π·Π½Π°Π΅ΠΌ). Π’ схСмС рис. 1.2 я замСню
Π²ΠΎΠ»ΡŒΡ‚ΠΌΠ΅Ρ‚Ρ€ (напряТСниС Π½Π° рСзисторС R1
ΠΏΡ€ΠΈ ΠΈΠ·ΠΌΠ΅Ρ€Π΅Π½ΠΈΠΈ Π±ΡƒΠ΄Π΅Ρ‚ Ρ€Π°Π²Π½ΠΎ 10 Π’) сопротивлСниСм
Π² 100 кОм, ΠΈ Ρ‚Π°ΠΊΠΎΠ΅ ΠΆΠ΅ сопротивлСниС Π±ΡƒΠ΄Π΅Ρ‚
ΠΈΠΌΠ΅Ρ‚ΡŒ рСзистор R1. По Ρ€Π΅Π·ΡƒΠ»ΡŒΡ‚Π°Ρ‚Ρƒ измСрСния
Ρ‚ΠΎΠΊΠ° Π² Ρ†Π΅ΠΏΠΈ ΠΎΠΏΡ€Π΅Π΄Π΅Π»ΠΈΠΌ Π²Π΅Π»ΠΈΡ‡ΠΈΠ½Ρƒ
сопротивлСния.

Рис.
1.3. Π’ΠΎΠΊ Π² Ρ†Π΅ΠΏΠΈ ΠΏΡ€ΠΈ ΠΎΠΏΡ€Π΅Π΄Π΅Π»Π΅Π½ΠΈΠΈ Π²Π΅Π»ΠΈΡ‡ΠΈΠ½Ρ‹
сопротивлСния ΠΏΠΎ Π·Π°ΠΊΠΎΠ½Ρƒ Ома

Как
Π²ΠΈΠ΄Π½ΠΎ ΠΈΠ· рисунка, Ρ‚ΠΎΠΊ Ρ€Π°Π²Π΅Π½ 200 мкА. ΠŸΡ€ΠΈ
ΠΏΠ°Π΄Π΅Π½ΠΈΠΈ напряТСния Π² 10 Π’, Π²Π΅Π»ΠΈΡ‡ΠΈΠ½Π°
сопротивлСния опрСдСлится Π΄Π΅Π»Π΅Π½ΠΈΠ΅ΠΌ
этого напряТСния Π½Π° Ρ‚ΠΎΠΊ ΠΈ Π±ΡƒΠ΄Π΅Ρ‚ Ρ€Π°Π²Π½Π°
50 кОм. Π’Π°ΠΊ ΠΌΡ‹ ΠΈΠ·ΠΌΠ΅Ρ€ΠΈΠ»ΠΈ нСизвСстноС
сопротивлСниС Π±Π΅Π· ΡƒΡ‡Π΅Ρ‚Π° сопротивлСния
Π²ΠΎΠ»ΡŒΡ‚ΠΌΠ΅Ρ‚Ρ€Π°. И ошиблись Π² Π΄Π²Π° Ρ€Π°Π·Π°. А это
ΡƒΠΆΠ΅ Π½Π΅ ΠΌΠ°Π»ΠΎ. Π’ Π΄Π°Π½Π½ΠΎΠΌ ΠΊΠΎΠ½ΠΊΡ€Π΅Ρ‚Π½ΠΎΠΌ случаС
Π½Π°ΠΌ ΠΏΠΎΠΌΠΎΠ³Π»ΠΎ Π±Ρ‹ ΠΏΡ€ΠΎΠ²Π΅Π΄Π΅Π½ΠΈΠ΅ Π΄Π²ΡƒΡ… ΠΈΠ·ΠΌΠ΅Ρ€Π΅Π½ΠΈΠΉ
с ΠΏΠΎΠΌΠΎΡ‰ΡŒΡŽ тСстСра ΠΈΠ»ΠΈ ΠΌΡƒΠ»ΡŒΡ‚ΠΈΠΌΠ΅Ρ‚Ρ€Π°.
ΠŸΠ΅Ρ€Π²Ρ‹ΠΉ Ρ€Π°Π· ΠΌΡ‹ ΠΌΠΎΠ³Π»ΠΈ Π±Ρ‹ ΠΈΠ·ΠΌΠ΅Ρ€ΠΈΡ‚ΡŒ напряТСниС,
Π° Π²Ρ‚ΠΎΡ€ΠΎΠΉ Ρ€Π°Π· Ρ‚ΠΎΠΊ. Π”ΡƒΠΌΠ°ΡŽ, ΠΌΡ‹ ΠΏΠΎΠ»ΡƒΡ‡ΠΈΠ»ΠΈ Π±Ρ‹
ΠΏΡ€Π°Π²ΠΈΠ»ΡŒΠ½Ρ‹ΠΉ Ρ€Π΅Π·ΡƒΠ»ΡŒΡ‚Π°Ρ‚. Но… Π½ΠΎ Ρ‚ΠΎΠ»ΡŒΠΊΠΎ Π²
этом случаС. Если Π½Π΅ΠΌΠ½ΠΎΠ³ΠΎ ΡƒΡΠ»ΠΎΠΆΠ½ΠΈΡ‚ΡŒ
Ρ†Π΅ΠΏΡŒ, скаТСм, Π²ΠΊΠ»ΡŽΡ‡ΠΈΠ² ΠΏΠΎΡΠ»Π΅Π΄ΠΎΠ²Π°Ρ‚Π΅Π»ΡŒΠ½ΠΎ
с измСряСмым сопротивлСниСм Π΅Ρ‰Π΅ ΠΎΠ΄Π½ΠΎ
Ρ‚Π°ΠΊΠΎΠΉ ΠΆΠ΅ Π²Π΅Π»ΠΈΡ‡ΠΈΠ½Ρ‹, Ρ‚ΠΎ ΠΈ ΠΌΠ΅Ρ‚ΠΎΠ΄ΠΈΠΊΠ° Π΄Π²ΡƒΡ…
ΠΈΠ·ΠΌΠ΅Ρ€Π΅Π½ΠΈΠΉ ΠΌΠΎΠΆΠ΅Ρ‚ Π΄Π°Ρ‚ΡŒ Π½Π΅Π²Π΅Ρ€Π½Ρ‹ΠΉ Ρ€Π΅Π·ΡƒΠ»ΡŒΡ‚Π°Ρ‚.

Рис.
1.4. Π˜Π·ΠΌΠ΅Ρ€Π΅Π½ΠΈΠ΅ Π²Π΅Π»ΠΈΡ‡ΠΈΠ½Ρ‹ сопротивлСния
Π±Π΅Π· ΡƒΡ‡Π΅Ρ‚Π° сопротивлСния Π²ΠΎΠ»ΡŒΡ‚ΠΌΠ΅Ρ‚Ρ€Π°

По
Π³Ρ€Π°Ρ„ΠΈΠΊΠ°ΠΌ Ρ‚Ρ€ΡƒΠ΄Π½ΠΎ ΠΎΠΏΡ€Π΅Π΄Π΅Π»ΠΈΡ‚ΡŒ Ρ‚ΠΎΡ‡Π½Ρ‹Π΅
значСния, Π½ΠΎ ΠΏΡ€ΠΈΠ±Π»ΠΈΠ·ΠΈΡ‚Π΅Π»ΡŒΠ½ΠΎ это Π±ΡƒΠ΄Π΅Ρ‚
напряТСниС 3.35 Π’ ΠΈ Ρ‚ΠΎΠΊ 66 мкА. Π’ Ρ€Π΅Π·ΡƒΠ»ΡŒΡ‚Π°Ρ‚Π΅
дСлСния получаСтся Π·Π½Π°Ρ‡Π΅Π½ΠΈΠ΅ ΠΎΠΊΠΎΠ»ΠΎ 51
кОм. Π’Π°ΠΊΠΎΠ΅ ΠΈΠ·ΠΌΠ΅Ρ€Π΅Π½ΠΈΠ΅ Π² ΠΎΠ΄Π½ΠΈΡ… случаях
ΠΌΠΎΠΆΠ΅Ρ‚ Ρ‚ΠΎΠ»ΡŒΠΊΠΎ Π·Π°Ρ‚Ρ€ΡƒΠ΄Π½ΠΈΡ‚ΡŒ ΠΏΠΎΠ½ΠΈΠΌΠ°Π½ΠΈΠ΅
Ρ€Π΅Π°Π»ΡŒΠ½ΠΎΠ³ΠΎ полоТСния Π΄Π΅Π», Π½ΠΎ Π² Π΄Ρ€ΡƒΠ³ΠΈΡ…
ΠΌΠΎΠΆΠ΅Ρ‚ привСсти ΠΊ нСприятностям, ΠΊΠΎΡ‚ΠΎΡ€Ρ‹Ρ…
Π»ΡƒΡ‡ΡˆΠ΅ ΠΈΠ·Π±Π΅ΠΆΠ°Ρ‚ΡŒ, Ссли ΠΏΠΎΠΌΠ½ΠΈΡ‚ΡŒ, Ρ‡Ρ‚ΠΎ ΠΊΠ°ΠΆΠ΄Ρ‹ΠΉ
ΠΏΡ€ΠΈΠ±ΠΎΡ€ ΠΈΠΌΠ΅Π΅Ρ‚ ΠΎΠΏΡ€Π΅Π΄Π΅Π»Π΅Π½Π½Ρ‹Π΅ ΠΏΠ°Ρ€Π°ΠΌΠ΅Ρ‚Ρ€Ρ‹,
ΠΈ ΠΈΡ… слСдуСт ΡƒΡ‡ΠΈΡ‚Ρ‹Π²Π°Ρ‚ΡŒ ΠΏΡ€ΠΈ пользовании
ΠΏΡ€ΠΈΠ±ΠΎΡ€ΠΎΠΌ.

МоТно
ΠΎΠ±ΡŠΠ΅Π΄ΠΈΠ½ΠΈΡ‚ΡŒ всС Ρ‚Ρ€ΠΈ Π·Π°ΠΊΠΎΠ½Π° Π² ΠΎΠ΄Π½ΠΎ ΠΏΡ€Π°Π²ΠΈΠ»ΠΎ,
Ссли ΠΊΠΎΠΌΡƒ-Ρ‚ΠΎ ΡƒΠ΄ΠΎΠ±Π½ΠΎ Π·Π°ΠΏΠΎΠΌΠ½ΠΈΡ‚ΡŒ Π΅Π³ΠΎ.

Π‘ΡƒΠΌΠΌΠ°
напряТСний Π² элСктричСской Ρ†Π΅ΠΏΠΈ, Ρ€Π°Π²Π½Ρ‹Ρ…
ΠΏΡ€ΠΎΠΈΠ·Π²Π΅Π΄Π΅Π½ΠΈΡŽ сопротивлСний Π½Π° Ρ‚ΠΎΠΊ,
ΠΏΡ€ΠΎΡ‚Π΅ΠΊΠ°ΡŽΡ‰ΠΈΠΉ Ρ‡Π΅Ρ€Π΅Π· Π½ΠΈΡ…, Ρ€Π°Π²Π½Π° суммС Π­Π”Π‘
Π² этой Ρ†Π΅ΠΏΠΈ, Π° сумма Ρ‚ΠΎΠΊΠΎΠ², Π²Ρ‹Ρ‚Π΅ΠΊΠ°ΡŽΡ‰ΠΈΡ…
ΠΈΠ· ΡƒΠ·Π»Π° ΠΊΠΎ ΠΏΠΎ всСм вСтвям Ρ†Π΅ΠΏΠΈ, Ρ€Π°Π²Π½Π°
Π²Ρ‚Π΅ΠΊΠ°ΡŽΡ‰Π΅ΠΌΡƒ Π² ΡƒΠ·Π΅Π» Ρ‚ΠΎΠΊΡƒ.

И
слСдуСт ΠΏΠΎΠΌΠ½ΠΈΡ‚ΡŒ, Ρ‡Ρ‚ΠΎ Π½Π° всСх ΠΊΠΎΠΌΠΏΠΎΠ½Π΅Π½Ρ‚Π°Ρ…
элСктронных устройств ΠΏΡ€ΠΎΡΡ‚Π°Π²Π»ΡΡŽΡ‚
номинальноС Π·Π½Π°Ρ‡Π΅Π½ΠΈΠ΅, ΠΊΠΎΡ‚ΠΎΡ€ΠΎΠ΅, Π²
зависимости ΠΎΡ‚ изготовлСния, ΠΌΠΎΠΆΠ΅Ρ‚
ΠΎΡ‚Π»ΠΈΡ‡Π°Ρ‚ΡŒΡΡ ΠΎΡ‚ Ρ€Π΅Π°Π»ΡŒΠ½ΠΎΠ³ΠΎ Π½Π° 5-10%, хотя
Π΅ΡΡ‚ΡŒ ΠΈ ΠΊΠΎΠΌΠΏΠΎΠ½Π΅Π½Ρ‚Ρ‹, ΠΈΠ·Π³ΠΎΡ‚ΠΎΠ²Π»Π΅Π½Π½Ρ‹Π΅ с
большСй Ρ‚ΠΎΡ‡Π½ΠΎΡΡ‚ΡŒΡŽ, 0. 1-1%, Π½ΠΎ ΠΎΠ½ΠΈ ΠΏΡ€ΠΈΠΌΠ΅Π½ΡΡŽΡ‚ΡΡ
Ρ€Π΅ΠΆΠ΅ ΠΈ стоят Π΄ΠΎΡ€ΠΎΠΆΠ΅.

Напомню,
Ρ‡Ρ‚ΠΎ Π·Π° тСхничСскоС Π½Π°ΠΏΡ€Π°Π²Π»Π΅Π½ΠΈΠ΅ протСкания
постоянного Ρ‚ΠΎΠΊΠ° принято Π½Π°ΠΏΡ€Π°Π²Π»Π΅Π½ΠΈΠ΅
ΠΎΡ‚ плюса ΠΊ минусу. Π¦ΠΈΡ„Ρ€ΠΎΠ²Ρ‹Π΅ ΠΌΡƒΠ»ΡŒΡ‚ΠΈΠΌΠ΅Ρ‚Ρ€Ρ‹,
ΠΊΠ°ΠΊ ΠΏΡ€Π°Π²ΠΈΠ»ΠΎ, ΠΏΠΎΠΊΠ°Π·Ρ‹Π²Π°ΡŽΡ‚ Π·Π½Π°ΠΊ измСряСмого
напряТСния ΠΈ Ρ‚ΠΎΠΊΠ°, ΠΈ ΠΎΠ½ΠΈ Π½Π΅ Ρ‚Π°ΠΊ Ρ‡ΡƒΠ²ΡΡ‚Π²ΠΈΡ‚Π΅Π»ΡŒΠ½Ρ‹
ΠΊ ΠΏΠΎΠ΄ΠΊΠ»ΡŽΡ‡Π΅Π½ΠΈΡŽ с Π½Π΅Π²Π΅Ρ€Π½ΠΎΠΉ ΠΏΠΎΠ»ΡΡ€Π½ΠΎΡΡ‚ΡŒΡŽ,
ΠΊΠ°ΠΊ стрСлочныС ΠΏΡ€ΠΈΠ±ΠΎΡ€Ρ‹. ПослСдниС
Π·Π°ΡˆΠΊΠ°Π»ΠΈΠ²Π°ΡŽΡ‚ Β«Π² ΠΎΠ±Ρ€Π°Ρ‚Π½ΡƒΡŽ сторону»
ΠΈ ΠΌΠΎΠ³ΡƒΡ‚ ΠΎΡ‚ этого ΠΏΠΎΡΡ‚Ρ€Π°Π΄Π°Ρ‚ΡŒ. ВсСгда
слСдуСт ΠΏΡ€ΠΎΠ²Π΅Ρ€ΡΡ‚ΡŒ ΠΏΠΎΠ»ΡΡ€Π½ΠΎΡΡ‚ΡŒ ΠΏΠΎΠ΄ΠΊΠ»ΡŽΡ‡Π΅Π½ΠΈΡ
стрСлочных ΠΏΡ€ΠΈΠ±ΠΎΡ€ΠΎΠ². ΠœΠΈΠ½ΡƒΡ Ρƒ тСстСров
ΠΏΡ€ΠΈ ΠΈΠ·ΠΌΠ΅Ρ€Π΅Π½ΠΈΠΈ напряТСния ΠΈ Ρ‚ΠΎΠΊΠ° Π·Π°Ρ‡Π°ΡΡ‚ΡƒΡŽ
ΠΏΠΎΠΌΠ΅Ρ‡Π°ΡŽΡ‚ Π·Π½Π°Ρ‡ΠΊΠΎΠΌ Π² Π²ΠΈΠ΄Π΅ Π·Π²Π΅Π·Π΄ΠΎΡ‡ΠΊΠΈ. Π’
простых цСпях, ΠΈΠ·ΠΎΠ±Ρ€Π°ΠΆΠ΅Π½Π½Ρ‹Ρ… Π½Π° рисунках,
всС достаточно ΠΎΡ‡Π΅Π²ΠΈΠ΄Π½ΠΎ, Π½ΠΎ Π² слоТных
цСпях, ΠΊΠΎΠ³Π΄Π° Π΅ΡΡ‚ΡŒ ΠΌΠ½ΠΎΠ³ΠΎ сопротивлСний,
Π²ΠΊΠ»ΡŽΡ‡Π΅Π½Π½Ρ‹Ρ… слоТным ΠΎΠ±Ρ€Π°Π·ΠΎΠΌ, ΠΊΠΎΠ³Π΄Π° Π΅ΡΡ‚ΡŒ
нСсколько источников питания, Ρ‚ΠΎΠ³Π΄Π°
ΠΎΠΏΡ€Π΅Π΄Π΅Π»Π΅Π½ΠΈΠ΅ напряТСний ΠΈ Ρ‚ΠΎΠΊΠΎΠ²
услоТняСтся. ΠšΠΎΠ½Π΅Ρ‡Π½ΠΎ, ΡΡƒΡ‰Π΅ΡΡ‚Π²ΡƒΡŽΡ‚
матСматичСскиС ΠΌΠ΅Ρ‚ΠΎΠ΄Ρ‹ расчСта ΠΏΠΎΠ΄ΠΎΠ±Π½Ρ‹Ρ…
Ρ†Π΅ΠΏΠ΅ΠΉ, описанныС Π² ΡƒΡ‡Π΅Π±Π½ΠΈΠΊΠ°Ρ…, ΠΎΠ΄Π½Π°ΠΊΠΎ я
сомнСваюсь Π² ΠΈΡ… популярности Π² Π»ΡŽΠ±ΠΈΡ‚Π΅Π»ΡŒΡΠΊΠΈΡ…
ΠΊΡ€ΡƒΠ³Π°Ρ…, ΠΏΠΎΡΠΊΠΎΠ»ΡŒΠΊΡƒ ΡƒΠ΄ΠΎΠ±Π½Π΅Π΅ ΠΈΠ·ΠΌΠ΅Ρ€ΠΈΡ‚ΡŒ
ΠΈΠ½Ρ‚Π΅Ρ€Π΅ΡΡƒΡŽΡ‰ΡƒΡŽ Π²Π΅Π»ΠΈΡ‡ΠΈΠ½Ρƒ, Ρ‡Π΅ΠΌ Ρ€Π°ΡΡΡ‡ΠΈΡ‚Π°Ρ‚ΡŒ
Π΅Π΅, Π½ΠΎ ΠΎΡ‡Π΅Π½ΡŒ Π²Π°ΠΆΠ½ΠΎ ΠΈΠΌΠ΅Ρ‚ΡŒ ясноС прСдставлСниС
ΠΎΠ± основных процСссах Π² элСктричСской
Ρ†Π΅ΠΏΠΈ. ΠŸΠΎΠ΄ΠΊΠ»ΡŽΡ‡Π°Ρ ΠΏΡ€ΠΈΠ±ΠΎΡ€, слСдуСт Π½Π°Ρ‡ΠΈΠ½Π°Ρ‚ΡŒ
ΠΈΠ·ΠΌΠ΅Ρ€Π΅Π½ΠΈΠ΅ с бСзопасного для ΠΏΡ€ΠΈΠ±ΠΎΡ€Π°
ΠΏΡ€Π΅Π΄Π΅Π»Π° измСрСния – самого большого
напряТСния ΠΈΠ»ΠΈ Ρ‚ΠΎΠΊΠ°, ΠΏΠΎΠ·ΠΆΠ΅ Π΅Π³ΠΎ ΠΌΠΎΠΆΠ½ΠΎ
ΠΈΠ·ΠΌΠ΅Π½ΠΈΡ‚ΡŒ.

Π›ΡŽΠ±ΠΎΠΉ
справочник, любая Ρ€Π΅Π°Π»ΡŒΠ½Π°Ρ схСма содСрТит
ΠΎΡ‡Π΅Π½ΡŒ большоС количСство элСмСнтов
ΠΏΠΎΠΌΠΈΠΌΠΎ Ρ‚Π΅Ρ…, ΠΎ ΠΊΠΎΡ‚ΠΎΡ€Ρ‹Ρ… шла Ρ€Π΅Ρ‡ΡŒ Π²Ρ‹ΡˆΠ΅, ΠΈ
ΠΏΠΎΠ·ΠΆΠ΅ ΠΌΡ‹ ΠΏΠΎΠ³ΠΎΠ²ΠΎΡ€ΠΈΠΌ ΠΎΠ± этом, ΠΈ я ΠΏΠΎΡΡ‚Π°Ρ€Π°ΡŽΡΡŒ
ΠΏΠΎΠΊΠ°Π·Π°Ρ‚ΡŒ, Ρ‡Ρ‚ΠΎ достаточно слоТныС с
тСорСтичСской Ρ‚ΠΎΡ‡ΠΊΠΈ зрСния процСссы
ΠΌΠΎΠΆΠ½ΠΎ Π² Π»ΡŽΠ±ΠΈΡ‚Π΅Π»ΡŒΡΠΊΠΎΠΉ ΠΏΡ€Π°ΠΊΡ‚ΠΈΠΊΠ΅ свСсти
ΠΊ Ρ‚Π°ΠΊΠΈΠΌ понятиям, ΠΊΠ°ΠΊ сопротивлСниС,
напряТСниС ΠΈ Ρ‚ΠΎΠΊ.

ΠŸΡ€ΠΈ
Ρ€Π°Π±ΠΎΡ‚Π΅ с элСктричСскими цСпями Π²Π°ΠΆΠ½ΠΎ
ΡƒΡ‡ΠΈΡ‚Ρ‹Π²Π°Ρ‚ΡŒ ΠΌΠΎΡ‰Π½ΠΎΡΡ‚ΡŒ, ΠΎΠΏΡ€Π΅Π΄Π΅Π»ΡΠ΅ΠΌΡƒΡŽ
ΠΏΡ€ΠΎΠΈΠ·Π²Π΅Π΄Π΅Π½ΠΈΠ΅ΠΌ напряТСния Π½Π° Ρ‚ΠΎΠΊ. Если
ΠΌΡ‹ измСряСм Ρ‚ΠΎΠΊ, уходящий ΠΎΡ‚ источника
питания, ΠΈ ΡƒΠΌΠ½ΠΎΠΆΠ°Π΅ΠΌ Π΅Π³ΠΎ Π½Π° Π­Π”Π‘ (напряТСниС)
источника питания, Ρ‚ΠΎ ΠΌΡ‹ ΠΌΠΎΠΆΠ΅ΠΌ Π³ΠΎΠ²ΠΎΡ€ΠΈΡ‚ΡŒ
ΠΎ мощности, потрСбляСмой схСмой. Если
ΠΌΡ‹ измСряСм напряТСниС Π½Π° сопротивлСнии
ΠΈ Ρ‚ΠΎΠΊ, ΠΏΡ€ΠΎΡ‚Π΅ΠΊΠ°ΡŽΡ‰ΠΈΠΉ Ρ‡Π΅Ρ€Π΅Π· Π½Π΅Π³ΠΎ, Ρ‚ΠΎ ΠΌΠΎΠΆΠ΅ΠΌ
Π³ΠΎΠ²ΠΎΡ€ΠΈΡ‚ΡŒ ΠΎ мощности, потрСбляСмой этим
сопротивлСниСм ΠΈ ΠΎΠ±Ρ‹Ρ‡Π½ΠΎ Π²Ρ‹Π΄Π΅Π»ΡΡŽΡ‰Π΅ΠΉΡΡ
Π½Π° Π½Π΅ΠΌ Π² Π²ΠΈΠ΄Π΅ Ρ‚Π΅ΠΏΠ»Π°. ЕстСствСнно, Ρ‡Ρ‚ΠΎ
рСзистор ΠΏΡ€ΠΈ этом нагрСваСтся, ΠΈ Ссли
Π½Π΅ΠΏΡ€Π°Π²ΠΈΠ»ΡŒΠ½ΠΎ Π²Ρ‹Π±Ρ€Π°Ρ‚ΡŒ Π΅Π³ΠΎ ΠΏΠ°Ρ€Π°ΠΌΠ΅Ρ‚Ρ€,
Π΄ΠΎΠΏΡƒΡΡ‚ΠΈΠΌΡƒΡŽ ΠΌΠΎΡ‰Π½ΠΎΡΡ‚ΡŒ рассСяния, Ρ‚ΠΎ
сопротивлСниС пСрСгрССтся ΠΈ ΠΌΠΎΠΆΠ΅Ρ‚
ΡΠ³ΠΎΡ€Π΅Ρ‚ΡŒ. ΠžΠ±Ρ‹Ρ‡Π½ΠΎ Π½Π° схСмС указываСтся
ΠΌΠΎΡ‰Π½ΠΎΡΡ‚ΡŒ любого сопротивлСния, ΠΈΠ»ΠΈ ΠΎΠ½Π°
указываСтся Π² спСцификации – ΠΏΠ΅Ρ€Π΅Ρ‡Π½Π΅
всСх элСмСнтов схСмы с ΠΈΡ… ΠΏΠ°Ρ€Π°ΠΌΠ΅Ρ‚Ρ€Π°ΠΌΠΈ.
По ΠΏΠ°Ρ€Π°ΠΌΠ΅Ρ‚Ρ€Ρƒ допустимой мощности
сопротивлСния дСлятся Π½Π° ряд Π·Π½Π°Ρ‡Π΅Π½ΠΈΠΉ,
ΠΈΠ· ΠΊΠΎΡ‚ΠΎΡ€Ρ‹Ρ… Π½Π°ΠΈΠ±ΠΎΠ»Π΅Π΅ часто употрСбляСмыС
Π² схСмах – это рСзисторы Π² Ρ‡Π΅Ρ‚Π²Π΅Ρ€Ρ‚ΡŒ ΠΈ
ΠΏΠΎΠ»ΠΎΠ²ΠΈΠ½Ρƒ Π²Π°Ρ‚Ρ‚Π°. ΠœΠΎΡ‰Π½ΠΎΡΡ‚ΡŒ, рассСиваСмая
рСзистором, Π½Π΅ Π΄ΠΎΠ»ΠΆΠ½Π° ΠΏΡ€Π΅Π²Ρ‹ΡˆΠ°Ρ‚ΡŒ этой
Π²Π΅Π»ΠΈΡ‡ΠΈΠ½Ρ‹. ВмСстС с Ρ‚Π΅ΠΌ, слСдуСт ΠΏΠΎΠΌΠ½ΠΈΡ‚ΡŒ,
Ρ‡Ρ‚ΠΎ Π½Π°Π³Ρ€Π΅Π² сопротивлСния ΠΏΡ€ΠΈΠ²ΠΎΠ΄ΠΈΡ‚ ΠΊ
измСнСнию Π΅Π³ΠΎ Π²Π΅Π»ΠΈΡ‡ΠΈΠ½Ρ‹. Π§Π΅ΠΌ Π±Π»ΠΈΠΆΠ΅
допустимая ΠΌΠΎΡ‰Π½ΠΎΡΡ‚ΡŒ рассСивания
рСзистора ΠΊ мощности, выдСляСмой Π½Π°
Π½Π΅ΠΌ, Ρ‚Π΅ΠΌ сильнСС ΠΎΠ½ Π±ΡƒΠ΄Π΅Ρ‚ Ρ€Π°Π·ΠΎΠ³Ρ€Π΅Π²Π°Ρ‚ΡŒΡΡ.
Если Π²Π°ΠΌ Π²Π°ΠΆΠ½ΠΎ ΡΠΎΡ…Ρ€Π°Π½ΠΈΡ‚ΡŒ Π²Π΅Π»ΠΈΡ‡ΠΈΠ½Ρƒ
сопротивлСния, Ρ‚ΠΎ слСдуСт Π²Ρ‹Π±Ρ€Π°Ρ‚ΡŒ Π±ΠΎΠ»Π΅Π΅
ΠΌΠΎΡ‰Π½ΠΎΠ΅ сопротивлСниС. Π’ ΠΈΠ·ΠΌΠ΅Ρ€ΠΈΡ‚Π΅Π»ΡŒΠ½Ρ‹Ρ…
ΠΏΡ€ΠΈΠ±ΠΎΡ€Π°Ρ…, Ρ€Π°Π²Π½ΠΎ ΠΊΠ°ΠΊ Π»ΡŽΠ±Ρ‹Ρ… цСпях,
относящихся ΠΊ ΠΈΠ·ΠΌΠ΅Ρ€Π΅Π½ΠΈΡŽ, Ρ‚Π°ΠΌ Π³Π΄Π΅ Π·Π½Π°Ρ‡Π΅Π½ΠΈΠ΅
сопротивлСния ΠΎΡ‡Π΅Π½ΡŒ Π²Π°ΠΆΠ½ΠΎ, ΠΊΡ€ΠΎΠΌΠ΅
сопротивлСний с Π±ΠΎΠ»Π΅Π΅ высокой допустимой
ΠΌΠΎΡ‰Π½ΠΎΡΡ‚ΡŒΡŽ рассСяния ΠΏΡ€ΠΈΠΌΠ΅Π½ΡΡŽΡ‚ ΡΠΏΠ΅Ρ†ΠΈΠ°Π»ΡŒΠ½Ρ‹Π΅
сопротивлСния, ΠΌΠ°Π»ΠΎ ΠΌΠ΅Π½ΡΡŽΡ‰ΠΈΠ΅ своС
Π·Π½Π°Ρ‡Π΅Π½ΠΈΠ΅ ΠΏΡ€ΠΈ Π½Π°Π³Ρ€Π΅Π²Π΅.

Π•ΡΡ‚ΡŒ
Π΅Ρ‰Π΅ нСсколько интСрСсных, ΠΈ ΠΊΠ°ΠΊ ΠΌΠ½Π΅
каТСтся, Π²Π°ΠΆΠ½Ρ‹Ρ… ΠΌΠΎΠΌΠ΅Π½Ρ‚ΠΎΠ², относящихся
ΠΊ ΡΠΎΠΏΡ€ΠΎΡ‚ΠΈΠ²Π»Π΅Π½ΠΈΡŽ. МногиС нСисправности
Π² элСктронных устройствах связаны с
ΠΏΡ€ΠΎΠ±Π»Π΅ΠΌΠ°ΠΌΠΈ источников питания. Π‘Π°Ρ‚Π°Ρ€Π΅ΠΉΠΊΠΈ,
Π½Π°ΠΏΡ€ΠΈΠΌΠ΅Ρ€, со Π²Ρ€Π΅ΠΌΠ΅Π½Π΅ΠΌ «садятся».
ΠŸΡ€ΠΎΡΡ‚Π΅ΠΉΡˆΠΈΠΉ способ ΠΏΡ€ΠΎΠ²Π΅Ρ€ΠΈΡ‚ΡŒ Π±Π°Ρ‚Π°Ρ€Π΅ΠΉΠΊΡƒ
– ΠΈΠ·ΠΌΠ΅Ρ€ΠΈΡ‚ΡŒ Ρ‚ΠΎΠΊ, ΠΊΠΎΡ‚ΠΎΡ€Ρ‹ΠΉ ΠΎΠ½Π° ΠΌΠΎΠΆΠ΅Ρ‚
ΠΎΡ‚Π΄Π°Π²Π°Ρ‚ΡŒ. Для этого достаточно Π²ΠΊΠ»ΡŽΡ‡ΠΈΡ‚ΡŒ
ΠΌΡƒΠ»ΡŒΡ‚ΠΈΠΌΠ΅Ρ‚Ρ€ (ΠΈΠ»ΠΈ тСстСр) Π² Ρ€Π΅ΠΆΠΈΠΌ измСрСния
максимального для ΠΊΠΎΠ½ΠΊΡ€Π΅Ρ‚Π½ΠΎΠ³ΠΎ ΠΏΡ€ΠΈΠ±ΠΎΡ€Π°
постоянного Ρ‚ΠΎΠΊΠ°, ΠΌΠΎΠΉ ΠΌΡƒΠ»ΡŒΡ‚ΠΈΠΌΠ΅Ρ‚Ρ€ измСряСт
Ρ‚ΠΎΠΊΠΈ Π΄ΠΎ 10 А, ΠΈ ΠΏΠΎΠ΄ΠΊΠ»ΡŽΡ‡ΠΈΡ‚ΡŒ Π°ΠΌΠΏΠ΅Ρ€ΠΌΠ΅Ρ‚Ρ€ ΠΊ
Π±Π°Ρ‚Π°Ρ€Π΅ΠΉΠΊΠ΅. БвСТая Π±Π°Ρ‚Π°Ρ€Π΅ΠΉΠΊΠ°, Π² зависимости
ΠΎΡ‚ Ρ‚ΠΈΠΏΠ°, ΠΏΠΎΠΊΠ°ΠΆΠ΅Ρ‚ Ρ‚ΠΎΠΊ Π² нСсколько Π°ΠΌΠΏΠ΅Ρ€,
Ρ‚ΠΎΠ³Π΄Π° ΠΊΠ°ΠΊ разряТСнная смоТСт Π΄Π°Ρ‚ΡŒ Ρ‚ΠΎΠ»ΡŒΠΊΠΎ
дСсятки ΠΌΠΈΠ»Π»ΠΈΠ°ΠΌΠΏΠ΅Ρ€. Зная Π·Π°ΠΊΠΎΠ½ Ома, ΠΌΡ‹
ΠΌΠΎΠΆΠ΅ΠΌ ΠΎΠΏΡ€Π΅Π΄Π΅Π»ΠΈΡ‚ΡŒ ΠΎΠΆΠΈΠ΄Π°Π΅ΠΌΡ‹ΠΉ Ρ‚ΠΎΠΊ Π΄ΠΎ
провСдСния измСрСния. ПолоТим Π±Π°Ρ‚Π°Ρ€Π΅ΠΉΠΊΠ°
ΠΈΠΌΠ΅Π΅Ρ‚ напряТСниС 1.5 Π²ΠΎΠ»ΡŒΡ‚Π°. АмпСрмСтр
ΠΈΠΌΠ΅Π΅Ρ‚ сопротивлСниС 0.1 Ом. Π’ΠΎΠ³Π΄Π° ΠΌΡ‹
Π΄ΠΎΠ»ΠΆΠ½Ρ‹ ΠΏΠΎΠ»ΡƒΡ‡ΠΈΡ‚ΡŒ Ρ‚ΠΎΠΊ 15 А. ΠŸΡ€ΠΎΠ²Π΅Ρ€ΠΈΠΌ это
ΡƒΡ‚Π²Π΅Ρ€ΠΆΠ΄Π΅Π½ΠΈΠ΅ сначала Π² ΠΏΡ€ΠΎΠ³Ρ€Π°ΠΌΠΌΠ΅ PSIM.

Рис.
1.5. ΠŸΡ€ΠΎΠ²Π΅Ρ€ΠΊΠ° Π±Π°Ρ‚Π°Ρ€Π΅ΠΉΠΊΠΈ Π½Π° ΠΏΡ€ΠΈΠ³ΠΎΠ΄Π½ΠΎΡΡ‚ΡŒ

Π’
схСмС Π½Π° рисункС сопротивлСниС R1 – это
сопротивлСниС Ρ€Π΅Π°Π»ΡŒΠ½ΠΎΠ³ΠΎ ΠΈΠ·ΠΌΠ΅Ρ€ΠΈΡ‚Π΅Π»ΡŒΠ½ΠΎΠ³ΠΎ
ΠΏΡ€ΠΈΠ±ΠΎΡ€Π°. Как ΠΌΡ‹ ΠΈ рассчитали, ΠΏΡ€ΠΈ ΠΈΠ·ΠΌΠ΅Ρ€Π΅Π½ΠΈΠΈ
ΠΏΡ€ΠΎΡ‚Π΅ΠΊΠ°Π΅Ρ‚ Ρ‚ΠΎΠΊ Π² 15 А. Но Π΅Π΄Π²Π° Π»ΠΈ Π²Ρ‹ ΡƒΠ²ΠΈΠ΄ΠΈΡ‚Π΅
Ρ‚Π°ΠΊΠΎΠΉ Ρ‚ΠΎΠΊ, Ссли ΠΏΡ€ΠΎΠ²Π΅Π΄Π΅Ρ‚Π΅ Ρ€Π΅Π°Π»ΡŒΠ½ΠΎΠ΅
ΠΈΠ·ΠΌΠ΅Ρ€Π΅Π½ΠΈΠ΅. И Π΄Π΅Π»ΠΎ Π½Π΅ Π² ΠΏΠ»ΠΎΡ…ΠΎΠΉ Π±Π°Ρ‚Π°Ρ€Π΅ΠΉΠΊΠ΅.
Π”Π΅Π»ΠΎ Π² Ρ‚ΠΎΠΌ, Ρ‡Ρ‚ΠΎ Ρ€Π΅Π°Π»ΡŒΠ½Π°Ρ Π±Π°Ρ‚Π°Ρ€Π΅ΠΉΠΊΠ°, ΠΊΠ°ΠΊ
любой источник питания, это Π½Π΅ ΠΈΠ΄Π΅Π°Π»ΡŒΠ½Ρ‹ΠΉ
источник Π­Π”Π‘. Π›ΡŽΠ±ΠΎΠΉ Ρ€Π΅Π°Π»ΡŒΠ½Ρ‹ΠΉ источник
питания ΠΈΠΌΠ΅Π΅Ρ‚, ΠΊΠ°ΠΊ ΠΈ Ρ€Π΅Π°Π»ΡŒΠ½Ρ‹ΠΉ ΠΈΠ·ΠΌΠ΅Ρ€ΠΈΡ‚Π΅Π»ΡŒΠ½Ρ‹ΠΉ
ΠΏΡ€ΠΈΠ±ΠΎΡ€, Π²Π½ΡƒΡ‚Ρ€Π΅Π½Π½Π΅Π΅ сопротивлСниС.
НСбольшоС, зависящСС ΠΎΡ‚ Ρ‚ΠΈΠΏΠ° источника,
Π½ΠΎ это Π²Π½ΡƒΡ‚Ρ€Π΅Π½Π½Π΅Π΅ сопротивлСниС Π΅ΡΡ‚ΡŒ.
Оно-Ρ‚ΠΎ ΠΈ ΡƒΠΌΠ΅Π½ΡŒΡˆΠ°Π΅Ρ‚ Ρ‚ΠΎΠΊ Ρ‡Π΅Ρ€Π΅Π· Π±Π°Ρ‚Π°Ρ€Π΅ΠΉΠΊΡƒ.
Π’ΠΎ Π΅ΡΡ‚ΡŒ, Π² схСмС Π½Π° рис. 1.5 слСдовало Π±Ρ‹
Π½Π°Ρ€ΠΈΡΠΎΠ²Π°Ρ‚ΡŒ ΠΏΠΎΡΠ»Π΅Π΄ΠΎΠ²Π°Ρ‚Π΅Π»ΡŒΠ½ΠΎ с источником
питания Π΅Ρ‰Π΅ ΠΎΠ΄Π½ΠΎ сопротивлСниС, Π° ΠΏΡ€ΠΈ
расчСтС ΠΏΠΎ Π·Π°ΠΊΠΎΠ½Ρƒ Ома ΠΈΡΠΏΠΎΠ»ΡŒΠ·ΠΎΠ²Π°Ρ‚ΡŒ
сумму Π΄Π²ΡƒΡ… рСзисторов. Однако Π²Π½ΡƒΡ‚Ρ€Π΅Π½Π½Π΅Π΅
сопротивлСниС Π±Π°Ρ‚Π°Ρ€Π΅ΠΉΠΊΠΈ – это ΠΏΠ°Ρ€Π°ΠΌΠ΅Ρ‚Ρ€,
ΠΊΠΎΡ‚ΠΎΡ€Ρ‹ΠΉ Π½Π΅ ΡƒΠ²ΠΈΠ΄ΠΈΡˆΡŒ Π½Π° этикСткС. ΠŸΠ»ΠΎΡ…ΠΎ
Π»ΠΈ, Ρ‡Ρ‚ΠΎ Π±Π°Ρ‚Π°Ρ€Π΅ΠΉΠΊΠ° ΠΈΠΌΠ΅Π΅Ρ‚ Π²Π½ΡƒΡ‚Ρ€Π΅Π½Π½Π΅Π΅
сопротивлСниС, Π΄Π° Π΅Ρ‰Π΅ ΠΈ ΠΌΠ΅Π½ΡΡŽΡ‰Π΅Π΅ΡΡ со
Π²Ρ€Π΅ΠΌΠ΅Π½Π΅ΠΌ? Π‘ ΠΎΠ΄Π½ΠΎΠΉ стороны ΠΏΠ»ΠΎΡ…ΠΎ. А с
Π΄Ρ€ΡƒΠ³ΠΎΠΉ… соврСмСнныС ΠΌΠΎΠ±ΠΈΠ»ΡŒΠ½Ρ‹Π΅ Ρ‚Π΅Π»Π΅Ρ„ΠΎΠ½Ρ‹
ΠΈΠΌΠ΅ΡŽΡ‚ аккумуляторы с ΠΎΡ‡Π΅Π½ΡŒ Π½ΠΈΠ·ΠΊΠΈΠΌ
Π²Π½ΡƒΡ‚Ρ€Π΅Π½Π½ΠΈΠΌ сопротивлСниСм. Они ΠΌΠΎΠ³ΡƒΡ‚
ΠΎΡ‚Π΄Π°Π²Π°Ρ‚ΡŒ большой Ρ‚ΠΎΠΊ. По этой ΠΏΡ€ΠΈΡ‡ΠΈΠ½Π΅
ΠΌΠ½ΠΎΠ³ΠΈΠ΅ ΠΈΠ· Π½ΠΈΡ… приходится ΡΠ½Π°Π±ΠΆΠ°Ρ‚ΡŒ
ΡΠΏΠ΅Ρ†ΠΈΠ°Π»ΡŒΠ½Ρ‹ΠΌ устройством, ΠΎΠ³Ρ€Π°Π½ΠΈΡ‡ΠΈΠ²Π°ΡŽΡ‰ΠΈΠΌ
этот Ρ‚ΠΎΠΊ. И ΠΏΡ€ΠΎΠ±Π»Π΅ΠΌΠ° Π½Π΅ Π² Ρ‚ΠΎΠΌ, Ρ‡Ρ‚ΠΎ Ссли
этого Π½Π΅ ΡΠ΄Π΅Π»Π°Ρ‚ΡŒ, ΠΈ Π²Ρ‹ ΠΊΠΎΡΠ½Π΅Ρ‚Π΅ΡΡŒ Π²Ρ‹Π²ΠΎΠ΄ΠΎΠ²
аккумулятора, Ρ‚ΠΎ вас «тряхнСт»
Ρ‚ΠΎΠΊΠΎΠΌ. НСт. ΠŸΡ€ΠΎΠ±Π»Π΅ΠΌΠ° Π² Ρ‚ΠΎΠΌ, Ρ‡Ρ‚ΠΎ Ссли этого
Π½Π΅ ΡΠ΄Π΅Π»Π°Ρ‚ΡŒ, ΠΈ Π²Ρ‹ Ρ€Π΅ΡˆΠΈΡ‚Π΅ ΠΏΠΎΡ‡ΠΈΡΡ‚ΠΈΡ‚ΡŒ Π²Ρ‹Π²ΠΎΠ΄Ρ‹
аккумулятора бСзопасной Π±Ρ€ΠΈΡ‚Π²ΠΎΠΉ, Π±Ρ‹Π²Π°Π΅Ρ‚
Ρ‚Π°ΠΊΠΎΠ΅, Ρ‚ΠΎ ΠΏΡ€ΠΈ Π·Π°ΠΌΡ‹ΠΊΠ°Π½ΠΈΠΈ Π²Ρ‹Π²ΠΎΠ΄ΠΎΠ² Π±Ρ€ΠΈΡ‚Π²ΠΎΠΉ
большой Ρ‚ΠΎΠΊ Ρ‡Π΅Ρ€Π΅Π· Π½Π΅Π΅ ΠΌΠΎΠΆΠ΅Ρ‚ Ρ€Π°ΡΠΏΠ»Π°Π²ΠΈΡ‚ΡŒ
Π±Ρ€ΠΈΡ‚Π²Ρƒ (вспомнитС ΠΏΡ€ΠΎ Π½Π°Π³Ρ€Π΅Π²Π°Π½ΠΈΠ΅
рСзистора), Π° расплавлСнный ΠΌΠ΅Ρ‚Π°Π»Π»
Π²Ρ‹Π·Π²Π°Ρ‚ΡŒ ΡΠ΅Ρ€ΡŒΠ΅Π·Π½Ρ‹Π΅ ΠΎΠΆΠΎΠ³ΠΈ. ΠŸΡ€ΠΈ этом процСсс
происходит Π½Π°ΡΡ‚ΠΎΠ»ΡŒΠΊΠΎ быстро, Ρ‡Ρ‚ΠΎ
расплавлСнный ΠΌΠ΅Ρ‚Π°Π»Π» разбрызгиваСтся
Π² Ρ€Π°Π·Π½Ρ‹Π΅ стороны. Π§Ρ‚ΠΎ Ρ…ΠΎΡ€ΠΎΡˆΠΎ Π² ΠΎΠ΄Π½ΠΈΡ…
случаях, ΠΏΠ»ΠΎΡ…ΠΎ Π² Π΄Ρ€ΡƒΠ³ΠΈΡ…. На ΠΏΡ€Π°ΠΊΡ‚ΠΈΠΊΠ΅,
Ссли Π²Π½ΡƒΡ‚Ρ€Π΅Π½Π½Π΅Π΅ сопротивлСниС источника
Π½Π° порядок мСньшС сопротивлСния Ρ†Π΅ΠΏΠΈ,
Ρ‚ΠΎ ΠΈΠΌ ΠΌΠΎΠΆΠ½ΠΎ ΠΏΡ€Π΅Π½Π΅Π±Ρ€Π΅Ρ‡ΡŒ.

Коль
скоро я упомянул Π²Π½ΡƒΡ‚Ρ€Π΅Π½Π½Π΅Π΅ сопротивлСниС
Π±Π°Ρ‚Π°Ρ€Π΅Π΅ΠΊ, Ρ…ΠΎΡ‡Ρƒ Π½Π΅ΠΌΠ½ΠΎΠ³ΠΎ Ρ€Π°ΡΡΠΊΠ°Π·Π°Ρ‚ΡŒ ΠΎ
граблях, Π½Π° ΠΊΠΎΡ‚ΠΎΡ€Ρ‹Π΅ сам Π²Ρ€Π΅ΠΌΠ΅Π½Π°ΠΌΠΈ
Π½Π°ΡΡ‚ΡƒΠΏΠ°ΡŽ. Π‘Ρ…Π΅ΠΌΡƒ Π½Π° рисункС Π½ΠΈΠΆΠ΅ я Π²Ρ‹Π±Π΅Ρ€Ρƒ
ΡΠ°ΠΌΡƒΡŽ ΠΏΡ€ΠΎΡΡ‚ΡƒΡŽ.

Π’Ρ‹
всС Π·Π½Π°Π΅Ρ‚Π΅, Ρ‡Ρ‚ΠΎ часто Π² ΠΏΡƒΠ»ΡŒΡ‚Π°Ρ…, ΠΏΠ»Π΅Π΅Ρ€Π°Ρ…
ΠΈ Ρ‚.ΠΏ. ставят Π΄Π²Π΅ Π±Π°Ρ‚Π°Ρ€Π΅ΠΉΠΊΠΈ, Π²ΠΊΠ»ΡŽΡ‡Π°Ρ ΠΈΡ…
ΠΏΠΎΡΠ»Π΅Π΄ΠΎΠ²Π°Ρ‚Π΅Π»ΡŒΠ½ΠΎ. Иногда Π΄Π²Π΅ Π±Π°Ρ‚Π°Ρ€Π΅ΠΉΠΊΠΈ
Π²ΠΊΠ»ΡŽΡ‡Π°ΡŽΡ‚ ΠΏΠ°Ρ€Π°Π»Π»Π΅Π»ΡŒΠ½ΠΎ, соСдиняя ΠΈΡ…
ΠΏΠΎΠ»ΠΎΠΆΠΈΡ‚Π΅Π»ΡŒΠ½Ρ‹Π΅ ΠΈ ΠΎΡ‚Ρ€ΠΈΡ†Π°Ρ‚Π΅Π»ΡŒΠ½Ρ‹Π΅ Π²Ρ‹Π²ΠΎΠ΄Ρ‹.
Π’ этом случаС ΠΎΠ½ΠΈ ΠΌΠΎΠ³ΡƒΡ‚ Π΄Π°Ρ‚ΡŒ больший
Ρ‚ΠΎΠΊ, ΠΈΠ»ΠΈ слуТат Π²Π΄Π²ΠΎΠ΅ дольшС. Π§Ρ‚ΠΎ Π±ΡƒΠ΄Π΅Ρ‚,
Ссли ΠΏΡ€ΠΈ ΠΏΠ°Ρ€Π°Π»Π»Π΅Π»ΡŒΠ½ΠΎΠΌ Π²ΠΊΠ»ΡŽΡ‡Π΅Π½ΠΈΠΈ ΡΠΎΠ΅Π΄ΠΈΠ½ΠΈΡ‚ΡŒ
ΠΈΡ… разнополярно, ΠΏΠΎΠ»ΠΎΠΆΠΈΡ‚Π΅Π»ΡŒΠ½Ρ‹ΠΉ Π²Ρ‹Π²ΠΎΠ΄
ΠΎΠ΄Π½ΠΎΠΉ Π±Π°Ρ‚Π°Ρ€Π΅ΠΉΠΊΠΈ ΡΠΎΠ΅Π΄ΠΈΠ½ΠΈΡ‚ΡŒ с ΠΎΡ‚Ρ€ΠΈΡ†Π°Ρ‚Π΅Π»ΡŒΠ½Ρ‹ΠΌ
Π΄Ρ€ΡƒΠ³ΠΎΠΉ, Π° ΠΎΡ‚Ρ€ΠΈΡ†Π°Ρ‚Π΅Π»ΡŒΠ½Ρ‹ΠΉ с ΠΏΠΎΠ»ΠΎΠΆΠΈΡ‚Π΅Π»ΡŒΠ½Ρ‹ΠΌ?
Богласно Π·Π°ΠΊΠΎΠ½Ρƒ ΠšΠΈΡ€Ρ…Π³ΠΎΡ„Π° сумма Π­Π”Π‘
Π΄ΠΎΠ»ΠΆΠ½Π° Π±Ρ‹Ρ‚ΡŒ Ρ€Π°Π²Π½Π° Π½Π°ΠΏΡ€ΡΠΆΠ΅Π½ΠΈΡŽ. Но сумма
Π­Π”Π‘ (алгСбраичСская, Ρ‚ΠΎ Π΅ΡΡ‚ΡŒ, с ΡƒΡ‡Π΅Ρ‚ΠΎΠΌ
Π·Π½Π°ΠΊΠ°) Ρ€Π°Π²Π½Π° Π½ΡƒΠ»ΡŽ. Π—Π½Π°Ρ‡ΠΈΡ‚ напряТСниС
Ρ‚ΠΎΠΆΠ΅ Π±ΡƒΠ΄Π΅Ρ‚ Ρ€Π°Π²Π½ΠΎ Π½ΡƒΠ»ΡŽ. А Ρ‚ΠΎΠΊ?

Если
я ΠΎΡ‚Π²Π΅Ρ‡Π°ΡŽ Π½Π΅ ΠΏΠΎΠ΄ΡƒΠΌΠ°Π², Ρ‚ΠΎ Π³ΠΎΡ‚ΠΎΠ² ΡΠΊΠ°Π·Π°Ρ‚ΡŒ
– Π½Π΅Ρ‚ напряТСния, Π½Π΅Ρ‚ ΠΈ Ρ‚ΠΎΠΊΠ°. Но это Π½Π΅
Ρ‚Π°ΠΊ. НарисуСм эту схСму Π² ΠΏΡ€ΠΎΠ³Ρ€Π°ΠΌΠΌΠ΅
PSIM, Π΄ΠΎΠ±Π°Π²ΠΈΠ² Π²Π½ΡƒΡ‚Ρ€Π΅Π½Π½ΠΈΠ΅ сопротивлСния
ΠΈ ΠΏΡ€ΠΈΠ±ΠΎΡ€Ρ‹.

Рис.
1.6. ВстрСчноС Π²ΠΊΠ»ΡŽΡ‡Π΅Π½ΠΈΠ΅ Π±Π°Ρ‚Π°Ρ€Π΅Π΅ΠΊ

Как
Π²ΠΈΠ΄Π½ΠΎ Π½Π° рисункС напряТСниС (VP1) Π±Π»ΠΈΠ·ΠΊΠΎ
ΠΊ Π½ΡƒΠ»ΡŽ ΠΈ Ρ€Π°Π²Π½ΠΎ нСскольким ΠΌΠΈΠΊΡ€ΠΎΠ²ΠΎΠ»ΡŒΡ‚Π°ΠΌ,
Ρ‚ΠΎΠ³Π΄Π° ΠΊΠ°ΠΊ Ρ‚ΠΎΠΊ (I1) Ρ€Π°Π²Π΅Π½ 15 А. Π”Π΅ΠΉΡΡ‚Π²ΠΈΡ‚Π΅Π»ΡŒΠ½ΠΎ,
Ссли ΡƒΠ±Ρ€Π°Ρ‚ΡŒ ΠΎΠ±Ρ‰ΠΈΠΉ ΠΏΡ€ΠΎΠ²ΠΎΠ΄ (зСмлю) ΠΈ
ΠΏΠΎΠ²Π΅Ρ€Π½ΡƒΡ‚ΡŒ схСму, Ρ‚ΠΎ ΠΌΠΎΠΆΠ½ΠΎ Ρ€Π°ΡΡΠΌΠ°Ρ‚Ρ€ΠΈΠ²Π°Ρ‚ΡŒ
Π΄Π²Π° источника питания, ΠΊΠ°ΠΊ Π²ΠΊΠ»ΡŽΡ‡Π΅Π½Π½Ρ‹Π΅
ΠΏΠΎΡΠ»Π΅Π΄ΠΎΠ²Π°Ρ‚Π΅Π»ΡŒΠ½ΠΎ ΠΈ Π½Π°Π³Ρ€ΡƒΠΆΠ΅Π½Π½Ρ‹Π΅ Π½Π° Ρ†Π΅ΠΏΡŒ,
ΡΠΎΡΡ‚ΠΎΡΡ‰ΡƒΡŽ ΠΈΠ· рСзисторов (Π²Π½ΡƒΡ‚Ρ€Π΅Π½Π½ΠΈΠ΅
сопротивлСния Π±Π°Ρ‚Π°Ρ€Π΅Π΅ΠΊ) R1 ΠΈ R2. Π’ΠΎΠ³Π΄Π° Π­Π”Π‘
схСмы Π±ΡƒΠ΄Π΅Ρ‚ 3 Π’, сопротивлСниС Ρ†Π΅ΠΏΠΈ 0.2
Ома, Ρ‚ΠΎΠΊ ΠΏΠΎ Π·Π°ΠΊΠΎΠ½Ρƒ Ома 15 А, Π° сумма ΠΏΠ°Π΄Π΅Π½ΠΈΠΉ
напряТСний U1=15A*0.1 Ом плюс Ρ‚Π°ΠΊΠΎΠ΅ ΠΆΠ΅ U2.
ПадСниС напряТСния происходит Π½Π°
Π²Π½ΡƒΡ‚Ρ€Π΅Π½Π½ΠΈΡ… сопротивлСниях источников
питания. Если Π±Ρ‹ Π²Π½ΡƒΡ‚Ρ€Π΅Π½Π½Π΅Π΅ сопротивлСниС
Π½Π΅ Π±Ρ‹Π»ΠΎ скрыто ΠΎΡ‚ Π³Π»Π°Π·, Π±Ρ‹Π»ΠΎ Π±Ρ‹ ΠΏΡ€ΠΎΡ‰Π΅
ΠΎΡ‚Π²Π΅Ρ‚ΠΈΡ‚ΡŒ ΠΏΡ€Π°Π²ΠΈΠ»ΡŒΠ½ΠΎ, Π½ΠΎ, Ρ‡Ρ‚ΠΎ Π³Π»Π°Π· Π½Π΅
Π²ΠΈΠ΄ΠΈΡ‚, Ρ‚ΠΎΠ³ΠΎ, Π²Ρ€ΠΎΠ΄Π΅ Π±Ρ‹, ΠΈ Π½Π΅Ρ‚.

Π”Π²Π΅
Π±Π°Ρ‚Π°Ρ€Π΅ΠΉΠΊΠΈ ΠΏΠΎ 1.5 Π²ΠΎΠ»ΡŒΡ‚Π° Π² Ρ€Π΅Π°Π»ΡŒΠ½ΠΎΠΌ
экспСримСнтС ΠΏΠΎΠΊΠ°Π·Π°Π»ΠΈ 0.2Π’. ΠžΡ‚Ρ‡Π΅Π³ΠΎ Ρ‚Π°ΠΊ
ΠΌΠ½ΠΎΠ³ΠΎ? ΠŸΠΎΠΏΡ€ΠΎΠ±ΡƒΠΉΡ‚Π΅ ΠΌΠ΅Π½ΡΡ‚ΡŒ Π²Π½ΡƒΡ‚Ρ€Π΅Π½Π½Π΅Π΅
сопротивлСниС R1 Π² схСмС рис. 1.6.

Π•ΡΡ‚ΡŒ
Π΅Ρ‰Π΅ ΠΎΠ΄ΠΈΠ½ интСрСсный экспСримСнт, ΠΊΠΎΡ‚ΠΎΡ€Ρ‹ΠΉ
Π»Π΅Π³ΠΊΠΎ провСсти, Ρ‡Ρ‚ΠΎΠ±Ρ‹ ΠΏΠΎΠ½ΡΡ‚ΡŒ, Ρ‡Ρ‚ΠΎ Π²
элСктротСхникС Π½Π°Π·Ρ‹Π²Π°ΡŽΡ‚ источником
Ρ‚ΠΎΠΊΠ° Π² ΠΎΡ‚Π»ΠΈΡ‡ΠΈΠ΅ ΠΎΡ‚ источника напряТСния.
ΠŸΡ€Π΅Π΄ΡΡ‚Π°Π²ΠΈΠΌ, Ρ‡Ρ‚ΠΎ Π²Π½ΡƒΡ‚Ρ€Π΅Π½Π½Π΅Π΅ сопротивлСниС
Π±Π°Ρ‚Π°Ρ€Π΅ΠΉΠΊΠΈ ΠΎΡ‡Π΅Π½ΡŒ Π²Π΅Π»ΠΈΠΊΠΎ. Π‘ΠΊΠ°ΠΆΠ΅ΠΌ, 100 кОм.
НапряТСниС Π±Π°Ρ‚Π°Ρ€Π΅ΠΉΠΊΠΈ для опрСдСлСнности
ΠΏΡƒΡΡ‚ΡŒ Π±ΡƒΠ΄Π΅Ρ‚ 10 Π’. Π’ΠΎΠ³Π΄Π° ΠΌΠ°ΠΊΡΠΈΠΌΠ°Π»ΡŒΠ½Ρ‹ΠΉ Ρ‚ΠΎΠΊ,
ΠΊΠΎΡ‚ΠΎΡ€Ρ‹ΠΉ Π±Π°Ρ‚Π°Ρ€Π΅ΠΉΠΊΠ° ΠΌΠΎΠΆΠ΅Ρ‚ ΠΎΡ‚Π΄Π°Π²Π°Ρ‚ΡŒ Π²ΠΎ
внСшнюю Ρ†Π΅ΠΏΡŒ Π±ΡƒΠ΄Π΅Ρ‚ Π½Π΅ Π±ΠΎΠ»Π΅Π΅, Ρ‡Π΅ΠΌ 10Π’/100кОм
= 0.0001А (ΠΈΠ»ΠΈ 100 мкА). Если ΠΌΡ‹ ΠΊ Ρ‚Π°ΠΊΠΎΠΉ Π±Π°Ρ‚Π°Ρ€Π΅ΠΉΠΊΠ΅
ΠΏΠΎΠ΄ΠΊΠ»ΡŽΡ‡ΠΈΠΌ сопротивлСниС Π² 1 кОм, Ρ‚ΠΎ Ρ‚ΠΎΠΊ,
практичСски, Π½Π΅ измСнится. Π’ΠΎ Π΅ΡΡ‚ΡŒ, Π²
достаточно ΡˆΠΈΡ€ΠΎΠΊΠΎΠΌ Π΄ΠΈΠ°ΠΏΠ°Π·ΠΎΠ½Π΅ ΠΈΠ·ΠΌΠ΅Π½Π΅Π½ΠΈΠΉ
сопротивлСния внСшнСй Ρ†Π΅ΠΏΠΈ Ρ‚ΠΎΠΊ,
ΠΏΡ€ΠΎΡ‚Π΅ΠΊΠ°ΡŽΡ‰ΠΈΠΉ ΠΏΠΎ этой Ρ†Π΅ΠΏΠΈ, Π½Π΅ Π±ΡƒΠ΄Π΅Ρ‚
Π·Π°Π²ΠΈΡΠ΅Ρ‚ΡŒ ΠΎΡ‚ сопротивлСния Ρ†Π΅ΠΏΠΈ. ΠšΠΎΠ½Π΅Ρ‡Π½ΠΎ,
напряТСниС Ρ‚Π°ΠΊΠΎΠΉ Π±Π°Ρ‚Π°Ρ€Π΅ΠΉΠΊΠΈ Π±ΡƒΠ΄Π΅Ρ‚
ΠΌΠ΅Π½ΡΡ‚ΡŒΡΡ ΠΎΡ‡Π΅Π½ΡŒ сильно, Π½ΠΎ Ρ‚ΠΎΠΊ Π½Π΅Ρ‚, Ρ‡Ρ‚ΠΎ
ΠΈ Π½Π°Ρ…ΠΎΠ΄ΠΈΡ‚ ΠΏΡ€ΠΈΠΌΠ΅Π½Π΅Π½ΠΈΠ΅ Π½Π° ΠΏΡ€Π°ΠΊΡ‚ΠΈΠΊΠ΅.

Π˜ΡΡ‚ΠΎΡ‡Π½ΠΈΠΊ
питания, напряТСниС ΠΊΠΎΡ‚ΠΎΡ€ΠΎΠ³ΠΎ ΠΌΠ°Π»ΠΎ
зависит ΠΎΡ‚ сопротивлСния внСшнСй Ρ†Π΅ΠΏΠΈ,
ΠΌΡ‹ Π±ΡƒΠ΄Π΅ΠΌ Π½Π°Π·Ρ‹Π²Π°Ρ‚ΡŒ источником напряТСния,
Π° Ρ‚ΠΎΡ‚, Ρ‚ΠΎΠΊ ΠΊΠΎΡ‚ΠΎΡ€ΠΎΠ³ΠΎ ΠΌΠ°Π»ΠΎ зависит ΠΎΡ‚
сопротивлСния Π½Π°Π³Ρ€ΡƒΠ·ΠΊΠΈ, источником
Ρ‚ΠΎΠΊΠ°.

Π˜Ρ‚Π°ΠΊ.
ВсСго нСсколько понятий: Π­Π”Π‘, напряТСниС,
Ρ‚ΠΎΠΊ ΠΈ сопротивлСниС; всСго Ρ‚Ρ€ΠΈ Π·Π°ΠΊΠΎΠ½Π°
элСктротСхники: Π·Π°ΠΊΠΎΠ½ Ома ΠΈ Π΄Π²Π° Π·Π°ΠΊΠΎΠ½Π°
ΠšΠΈΡ€Ρ…Π³ΠΎΡ„Π°, – Π΄Π°Π»ΠΈ Π½Π°ΠΌ Π²ΠΎΠ·ΠΌΠΎΠΆΠ½ΠΎΡΡ‚ΡŒ провСсти
ряд интСрСсных экспСримСнтов. И это
Π΄Π°Π»Π΅ΠΊΠΎ Π½Π΅ всС интСрСсныС экспСримСнты,
ΠΊΠΎΡ‚ΠΎΡ€Ρ‹Π΅ ΠΌΠΎΠΆΠ½ΠΎ Π±Ρ‹Π»ΠΎ Π±Ρ‹ провСсти. ΠŸΠΎΠΏΡ€ΠΎΠ±ΡƒΠΉΡ‚Π΅
ΡΠΎΡΡ‚Π°Π²Π»ΡΡ‚ΡŒ Ρ†Π΅ΠΏΠΈ ΠΈΠ· ΠΌΠ½ΠΎΠ³ΠΈΡ… Π±Π°Ρ‚Π°Ρ€Π΅Π΅ΠΊ ΠΈ
сопротивлСний, Π²ΠΊΠ»ΡŽΡ‡Π°Ρ ΠΈΡ… Ρ€Π°Π·Π½Ρ‹ΠΌΠΈ
способами, ΠΈ ΠΏΠΎΠΏΡ‹Ρ‚Π°ΠΉΡ‚Π΅ΡΡŒ ΠΎΡ‚Π²Π΅Ρ‚ΠΈΡ‚ΡŒ Π½Π°
вопрос ΠΎ ΠΏΠ°Π΄Π΅Π½ΠΈΠΈ напряТСния Π½Π° любом
ΠΈΠ· рСзисторов ΠΈ Ρ‚ΠΎΠΊΠ΅ Ρ‡Π΅Ρ€Π΅Π· Π½Π΅Π³ΠΎ! Π£Π²Π΅Ρ€Π΅Π½,
Π²Ρ‹ Π½Π°ΠΉΠ΄Π΅Ρ‚Π΅ ΠΌΠ½ΠΎΠ³ΠΎ ΠΈΠ½Ρ‚Π΅Ρ€Π΅ΡΠ½Π΅ΠΉΡˆΠΈΡ… Π²Π°Ρ€ΠΈΠ°Π½Ρ‚ΠΎΠ².

ΠŸΡ€Π΅ΠΆΠ΄Π΅,
Ρ‡Π΅ΠΌ ΠΏΡ€ΠΎΠ΄ΠΎΠ»ΠΆΠΈΡ‚ΡŒ рассказ, я Ρ…ΠΎΡ‡Ρƒ Π΅Ρ‰Π΅ Ρ€Π°Π·
ΠΎΠ±Ρ€Π°Ρ‚ΠΈΡ‚ΡŒ вашС Π²Π½ΠΈΠΌΠ°Π½ΠΈΠ΅ Π½Π° Ρ‚ΠΎ, Ρ‡Ρ‚ΠΎ
элСктричСство всСгда нСсСт с собой
Π½Π΅ΠΊΠΎΡ‚ΠΎΡ€ΡƒΡŽ ΠΎΠΏΠ°ΡΠ½ΠΎΡΡ‚ΡŒ. Π― ΡƒΠΆΠ΅ Π³ΠΎΠ²ΠΎΡ€ΠΈΠ», Ρ‡Ρ‚ΠΎ
Π½Π΅ΠΊΠΎΡ‚ΠΎΡ€Ρ‹Π΅ источники питания ΠΏΡ€ΠΈ
Π½Π΅Π°ΠΊΠΊΡƒΡ€Π°Ρ‚Π½ΠΎΠΌ ΠΎΠ±Ρ€Π°Ρ‰Π΅Π½ΠΈΠΈ с Π½ΠΈΠΌΠΈ ΠΌΠΎΠ³ΡƒΡ‚
привСсти ΠΊ Ρ‚Ρ€Π°Π²ΠΌΠ°ΠΌ, ΠΊΠ°ΠΊ аккумулятор
сотового Ρ‚Π΅Π»Π΅Ρ„ΠΎΠ½Π°, Π½ΠΎ Ρ€Π΅Ρ‡ΡŒ шла ΠΎΠ± ΠΎΠΆΠΎΠ³Π°Ρ…
ΠΏΡ€ΠΈ ΠΊΠΎΡ€ΠΎΡ‚ΠΊΠΎΠΌ Π·Π°ΠΌΡ‹ΠΊΠ°Π½ΠΈΠΈ. Π’Π΅ΠΏΠ΅Ρ€ΡŒ я Ρ…ΠΎΡ‡Ρƒ
ΡΠΊΠ°Π·Π°Ρ‚ΡŒ ΠΎ Π΄Ρ€ΡƒΠ³ΠΈΡ… опасностях. НСкоторыС
аккумуляторы, источники питания
ΠΌΠ½ΠΎΠ³ΠΎΠΊΡ€Π°Ρ‚Π½ΠΎΠ³ΠΎ примСнСния, ΠΎΡ‡Π΅Π½ΡŒ ΠΏΠΎΡ…ΠΎΠΆΠΈ
Π½Π° Π±Π°Ρ‚Π°Ρ€Π΅ΠΉΠΊΠΈ. Если Π½Π΅ ΠΎΠ±Ρ€Π°Ρ‚ΠΈΡ‚ΡŒ внимания
Π½Π° ΠΏΡ€Π΅Π΄ΠΎΡΡ‚Π΅Ρ€Π΅Π³Π°ΡŽΡ‰ΠΈΠ΅ надписи, Ссли
ΠΏΠΎΠΏΡ‹Ρ‚Π°Ρ‚ΡŒΡΡ Π·Π°Ρ€ΡΠΆΠ°Ρ‚ΡŒ Π±Π°Ρ‚Π°Ρ€Π΅ΠΉΠΊΠΈ, Ρ‚ΠΎ это
ΠΌΠΎΠΆΠ΅Ρ‚ Π²Ρ‹Π·Ρ‹Π²Π°Ρ‚ΡŒ Π²Ρ‹Ρ‚Π΅ΠΊΠ°Π½ΠΈΠ΅ ΠΈΠ· Π½ΠΈΡ…
элСктролита. А соврСмСнныС Π±Π°Ρ‚Π°Ρ€Π΅ΠΉΠΊΠΈ
Π² качСствС элСктролита ΠΌΠΎΠ³ΡƒΡ‚ ΠΈΡΠΏΠΎΠ»ΡŒΠ·ΠΎΠ²Π°Ρ‚ΡŒ
Ρ‰Π΅Π»ΠΎΡ‡ΠΈ. Попадая Π½Π° Ρ€ΡƒΠΊΠΈ, элСктролит Ρ‚ΠΎΠΆΠ΅
Π²Ρ‹Π·ΠΎΠ²Π΅Ρ‚ Π±ΠΎΠ»Π΅Π·Π½Π΅Π½Π½Ρ‹ΠΉ ΠΈ Π΄ΠΎΠ»Π³ΠΎ Π½Π΅ Π·Π°ΠΆΠΈΠ²Π°ΡŽΡ‰ΠΈΠΉ
ΠΎΠΆΠΎΠ³. ΠŸΡ€Π΅ΠΆΠ΄Π΅, Ρ‡Π΅ΠΌ ΠΏΡ‹Ρ‚Π°Ρ‚ΡŒΡΡ Π·Π°Ρ€ΡΠΆΠ°Ρ‚ΡŒ
Ρ‡Ρ‚ΠΎ-Π»ΠΈΠ±ΠΎ, слСдуСт ΠΏΡ€ΠΎΠ²Π΅Ρ€ΠΈΡ‚ΡŒ, ΠΏΠΎΠ΄Π»Π΅ΠΆΠΈΡ‚
Π»ΠΈ ΠΎΠ½ΠΎ заряду? И Π΅Ρ‰Π΅ Π½Π΅ΠΌΠ½ΠΎΠ³ΠΎ ΠΎ ΠΏΠΎΡ€Π°ΠΆΠ΅Π½ΠΈΠΈ
элСктричСским Ρ‚ΠΎΠΊΠΎΠΌ. Если Ρ‡Π΅Ρ€Π΅Π· Ρ‡Π΅Π»ΠΎΠ²Π΅ΠΊΠ°
ΠΏΡ€ΠΎΡ…ΠΎΠ΄ΠΈΡ‚ ΠΎΡ‡Π΅Π½ΡŒ нСбольшой Ρ‚ΠΎΠΊ Π² нСсколько
дСсятков ΠΌΠΈΠ»Π»ΠΈΠ°ΠΌΠΏΠ΅Ρ€, Ρ‚ΠΎ это ΠΌΠΎΠΆΠ΅Ρ‚
Π²Ρ‹Π·Ρ‹Π²Π°Ρ‚ΡŒ, Ссли ΠΈ Π½Π΅ ΡΠΌΠ΅Ρ€Ρ‚Π΅Π»ΡŒΠ½ΠΎΠ΅, Ρ‚ΠΎ
вСсьма опасноС ΠΏΠΎΡ€Π°ΠΆΠ΅Π½ΠΈΠ΅ элСктричСским
Ρ‚ΠΎΠΊΠΎΠΌ. ВсСго нСсколько дСсятков
ΠΌΠΈΠ»Π»ΠΈΠ°ΠΌΠΏΠ΅Ρ€! А Π±Π°Ρ‚Π°Ρ€Π΅ΠΉΠΊΠ° для Ρ„ΠΎΠ½Π°Ρ€ΠΈΠΊΠ°
ΠΌΠΎΠΆΠ΅Ρ‚ Π΄Π°Ρ‚ΡŒ Ρ‚ΠΎΠΊ Π² нСсколько Π°ΠΌΠΏΠ΅Ρ€! Опасна
Π»ΠΈ ΠΎΠ½Π°? Π—Π΄Π΅ΡΡŒ слСдуСт Π²ΡΠΏΠΎΠΌΠ½ΠΈΡ‚ΡŒ Π·Π°ΠΊΠΎΠ½
Ома. Если напряТСниС Π±Π°Ρ‚Π°Ρ€Π΅ΠΉΠΊΠΈ 1.5 Π’, Π°
сопротивлСниС внСшнСй Ρ†Π΅ΠΏΠΈ (Ρ‚ΠΎ Π΅ΡΡ‚ΡŒ,
Ρ‡Π΅Π»ΠΎΠ²Π΅ΠΊΠ°, ΠΊΠΎΡ‚ΠΎΡ€Ρ‹ΠΉ взялся ΠΎΠ΄Π½ΠΎΠΉ Ρ€ΡƒΠΊΠΎΠΉ
Π·Π° ΠΎΠ΄ΠΈΠ½ Π²Ρ‹Π²ΠΎΠ΄ Π±Π°Ρ‚Π°Ρ€Π΅ΠΉΠΊΠΈ, Π° Π΄Ρ€ΡƒΠ³ΠΎΠΉ Ρ€ΡƒΠΊΠΎΠΉ
Π·Π° Π΄Ρ€ΡƒΠ³ΠΎΠΉ Π²Ρ‹Π²ΠΎΠ΄) нСсколько дСсятков
тысяч ΠΎΠΌ, Ρ‚ΠΎ Ρ‚ΠΎΠΊ Π±ΡƒΠ΄Π΅Ρ‚ измСрятся Π΅Π΄ΠΈΠ½ΠΈΡ†Π°ΠΌΠΈ
ΠΌΠΈΠΊΡ€ΠΎΠ°ΠΌΠΏΠ΅Ρ€ ΠΈ Π²Ρ€Π΅Π΄Π° Π½Π΅ принСсСт.
Π‘ΠΎΠΏΡ€ΠΎΡ‚ΠΈΠ²Π»Π΅Π½ΠΈΠ΅ Ρ‡Π΅Π»ΠΎΠ²Π΅ΠΊΠ° Π² основном
зависит ΠΎΡ‚ ΠΌΠ½ΠΎΠ³ΠΈΡ… Π²Π½Π΅ΡˆΠ½ΠΈΡ… ΠΈ ΠΏΡ€ΠΈΡ€ΠΎΠ΄Π½Ρ‹Ρ…
Ρ„Π°ΠΊΡ‚ΠΎΡ€ΠΎΠ² ΠΈ составляСт нСсколько тысяч
ΠΎΠΌ, поэтому ΠΏΡ€ΠΈ ΠΌΠ°Π»Π΅Π½ΡŒΠΊΠΈΡ… напряТСниях
ΠΌΠΎΠΆΠ½ΠΎ Π½Π΅ слишком Π±Π΅ΡΠΏΠΎΠΊΠΎΠΈΡ‚ΡŒΡΡ ΠΎ ΠΏΠΎΡ€Π°ΠΆΠ΅Π½ΠΈΠΈ
элСктричСским Ρ‚ΠΎΠΊΠΎΠΌ. Но Ρ‚Π°ΠΊΠΎΠΉ ΠΏΠΎΠ΄Ρ…ΠΎΠ΄
ΠΌΠΎΠΆΠ΅Ρ‚ ΡΡ‹Π³Ρ€Π°Ρ‚ΡŒ Π·Π»ΡƒΡŽ ΡˆΡƒΡ‚ΠΊΡƒ, Ссли нСбольшоС
напряТСниС, бСзопасноС с Ρ‚ΠΎΡ‡ΠΊΠΈ зрСния
расчСтов, ΠΏΡ€ΠΎΠΉΠ΄Π΅Ρ‚ Ρ‡Π΅Ρ€Π΅Π· Ρ‡Π΅Π»ΠΎΠ²Π΅ΠΊΠ°
Π½Π΅ΡƒΠ΄Π°Ρ‡Π½Ρ‹ΠΌ ΠΎΠ±Ρ€Π°Π·ΠΎΠΌ. Π›ΡƒΡ‡ΡˆΠ΅ сразу Π²Ρ‹Ρ€Π°Π±ΠΎΡ‚Π°Ρ‚ΡŒ ΠΏΡ€ΠΈΠ²Ρ‹Ρ‡ΠΊΡƒ, Π΄Π°ΠΆΠ΅ работая с
Π±Π°Ρ‚Π°Ρ€Π΅ΠΉΠΊΠ°ΠΌΠΈ
ΠΎΡ‚ ΠΊΠ°Ρ€ΠΌΠ°Π½Π½ΠΎΠ³ΠΎ фонаря, Π½ΠΈΠΊΠΎΠ³Π΄Π° Π½Π΅ ΠΊΠ°ΡΠ°Ρ‚ΡŒΡΡ
Π΄Π²ΡƒΡ… полюсов ΠΎΠ΄Π½ΠΎΠ²Ρ€Π΅ΠΌΠ΅Π½Π½ΠΎ, Ссли Π΅ΡΡ‚ΡŒ
Π²ΠΎΠ·ΠΌΠΎΠΆΠ½ΠΎΡΡ‚ΡŒ, Ρ‚ΠΎ Π½Π΅ ΠΏΡ€ΠΎΠ²ΠΎΠ΄ΠΈΡ‚ΡŒ ΠΈΠ·ΠΌΠ΅Ρ€Π΅Π½ΠΈΠΉ
ΠΏΠΎΠ΄ напряТСниСм. НуТно ΠΈΠ·ΠΌΠ΅Ρ€ΠΈΡ‚ΡŒ напряТСниС
Π² схСмС, Π²Ρ‹ΠΊΠ»ΡŽΡ‡ΠΈΡ‚Π΅ ΠΏΠΈΡ‚Π°Π½ΠΈΠ΅, ΠΏΠΎΠ΄ΠΊΠ»ΡŽΡ‡ΠΈΡ‚Π΅
ΠΏΡ€ΠΈΠ±ΠΎΡ€ ΠΈ Π²ΠΊΠ»ΡŽΡ‡ΠΈΡ‚Π΅ ΠΏΠΈΡ‚Π°Π½ΠΈΠ΅. ΠΠ΅Π±Ρ€Π΅ΠΆΠ½ΠΎΡΡ‚ΡŒ,
ΠΏΡ€Π΅Π½Π΅Π±Ρ€Π΅ΠΆΠ΅Π½ΠΈΠ΅ ΠΏΡ€Π°Π²ΠΈΠ»Π°ΠΌΠΈ бСзопасности
ΠΌΠΎΠ³ΡƒΡ‚ навсСгда ΠΎΡ‚Π±ΠΈΡ‚ΡŒ ΠΎΡ…ΠΎΡ‚Ρƒ ΠΊ Ρ€Π°Π±ΠΎΡ‚Π΅
со схСмами. Но Π½Π΅Π±Ρ€Π΅ΠΆΠ½ΠΎΡΡ‚ΡŒ, всСгда
Π½Π΅Π±Ρ€Π΅ΠΆΠ½ΠΎΡΡ‚ΡŒ. Π‘Ρ…Π΅ΠΌΠ° здСсь Π½ΠΈ Π² Ρ‡Π΅ΠΌ Π½Π΅
Π²ΠΈΠ½ΠΎΠ²Π°Ρ‚Π°.

Π Π΅Π·ΡŽΠΌΠΈΡ€ΡƒΡ
свои прСдостСрСТСния, я Ρ…ΠΎΡ‡Ρƒ ΠΏΠΎΡΠΎΠ²Π΅Ρ‚ΠΎΠ²Π°Ρ‚ΡŒ
Π½Π°Ρ‡ΠΈΠ½Π°ΡŽΡ‰ΠΈΠΌ ΠΈΡΠΏΠΎΠ»ΡŒΠ·ΠΎΠ²Π°Ρ‚ΡŒ Π² экспСримСнтах
Π»ΠΈΠ±ΠΎ ΠΏΡ€ΠΎΡΡ‚Π΅ΠΉΡˆΠΈΠ΅ Π±Π°Ρ‚Π°Ρ€Π΅ΠΉΠΊΠΈ, Π»ΠΈΠ±ΠΎ
качСствСнныС (с Π·Π°Ρ‰ΠΈΡ‚ΠΎΠΉ) Π±Π»ΠΎΠΊΠΈ питания,
Π° всС ΡΠΎΠΌΠ½ΠΈΡ‚Π΅Π»ΡŒΠ½Ρ‹Π΅ экспСримСнты Π²Π½Π°Ρ‡Π°Π»Π΅
ΠΏΡ€ΠΎΠ²ΠΎΠ΄ΠΈΡ‚ΡŒ Π·Π° ΠΊΠΎΠΌΠΏΡŒΡŽΡ‚Π΅Ρ€ΠΎΠΌ, ΠΈΡΠΏΠΎΠ»ΡŒΠ·ΡƒΡ
ΠΏΡ€ΠΎΠ³Ρ€Π°ΠΌΠΌΡ‹ БАПР, Ρ‚Π°ΠΊΠΈΠ΅ ΠΊΠ°ΠΊ PSIM.

Насколько
ΠΎΠ½ΠΈ ΠΏΠΎΠ»Π΅Π·Π½Ρ‹ станСт понятно,
ΠΊΠΎΠ³Π΄Π° ΠΌΡ‹ Π½Π°Ρ‡Π½Π΅ΠΌ Π³ΠΎΠ²ΠΎΡ€ΠΈΡ‚ΡŒ ΠΎ ΠΏΠ΅Ρ€Π΅ΠΌΠ΅Π½Π½ΠΎΠΌ
Ρ‚ΠΎΠΊΠ΅.


Как ΠΏΠΈΡˆΠ΅Ρ‚ΡΡ Π·Π°ΠΊΠΎΠ½ ΠΎΠΌΠ° для участка Ρ†Π΅ΠΏΠΈ. Π—Π°ΠΊΠΎΠ½ ΠΎΠΌΠ° для ΠΏΠΎΠ»Π½ΠΎΠΉ Ρ†Π΅ΠΏΠΈ

Π—Π°ΠΊΠΎΠ½ Ома для ΠΏΠΎΠ»Π½ΠΎΠΉ Ρ†Π΅ΠΏΠΈ – эмпиричСский (ΠΏΠΎΠ»ΡƒΡ‡Π΅Π½Π½Ρ‹ΠΉ ΠΈΠ· экспСримСнта) Π·Π°ΠΊΠΎΠ½, ΠΊΠΎΡ‚ΠΎΡ€Ρ‹ΠΉ устанавливаСт связь ΠΌΠ΅ΠΆΠ΄Ρƒ силой Ρ‚ΠΎΠΊΠ°, элСктродвиТущСй силой (Π­Π”Π‘) ΠΈ внСшним ΠΈ Π²Π½ΡƒΡ‚Ρ€Π΅Π½Π½ΠΈΠΌ сопротивлСниСм Π² Ρ†Π΅ΠΏΠΈ.

ΠŸΡ€ΠΈ ΠΏΡ€ΠΎΠ²Π΅Π΄Π΅Π½ΠΈΠΈ Ρ€Π΅Π°Π»ΡŒΠ½Ρ‹Ρ… исслСдований элСктричСских характСристик Ρ†Π΅ΠΏΠ΅ΠΉ с постоянным Ρ‚ΠΎΠΊΠΎΠΌ Π½Π΅ΠΎΠ±Ρ…ΠΎΠ΄ΠΈΠΌΠΎ ΡƒΡ‡ΠΈΡ‚Ρ‹Π²Π°Ρ‚ΡŒ сопротивлСниС самого источника Ρ‚ΠΎΠΊΠ°. Π’Π°ΠΊΠΈΠΌ ΠΎΠ±Ρ€Π°Π·ΠΎΠΌ Π² Ρ„ΠΈΠ·ΠΈΠΊΠ΅ осущСствляСтся ΠΏΠ΅Ρ€Π΅Ρ…ΠΎΠ΄ ΠΎΡ‚ идСального источника Ρ‚ΠΎΠΊΠ° ΠΊ Ρ€Π΅Π°Π»ΡŒΠ½ΠΎΠΌΡƒ источнику Ρ‚ΠΎΠΊΠ°, Ρƒ ΠΊΠΎΡ‚ΠΎΡ€ΠΎΠ³ΠΎ Π΅ΡΡ‚ΡŒ своС сопротивлСниС (см. рис. 1).

Рис. 1. Π˜Π·ΠΎΠ±Ρ€Π°ΠΆΠ΅Π½ΠΈΠ΅ идСального ΠΈ Ρ€Π΅Π°Π»ΡŒΠ½ΠΎΠ³ΠΎ источников Ρ‚ΠΎΠΊΠ°

РассмотрСниС источника Ρ‚ΠΎΠΊΠ° с собствСнным сопротивлСниСм обязываСт ΠΈΡΠΏΠΎΠ»ΡŒΠ·ΠΎΠ²Π°Ρ‚ΡŒ Π·Π°ΠΊΠΎΠ½ Ома для ΠΏΠΎΠ»Π½ΠΎΠΉ Ρ†Π΅ΠΏΠΈ.

Π‘Ρ„ΠΎΡ€ΠΌΡƒΠ»ΠΈΡ€ΡƒΠ΅ΠΌ Π·Π°ΠΊΠΎΠ½Π° Ома для ΠΏΠΎΠ»Π½ΠΎΠΉ Ρ†Π΅ΠΏΠΈ Ρ‚Π°ΠΊ (см. рис. 2): сила Ρ‚ΠΎΠΊΠ° Π² ΠΏΠΎΠ»Π½ΠΎΠΉ Ρ†Π΅ΠΏΠΈ прямо ΠΏΡ€ΠΎΠΏΠΎΡ€Ρ†ΠΈΠΎΠ½Π°Π»ΡŒΠ½Π° Π­Π”Π‘ ΠΈ ΠΎΠ±Ρ€Π°Ρ‚Π½ΠΎ ΠΏΡ€ΠΎΠΏΠΎΡ€Ρ†ΠΈΠΎΠ½Π°Π»ΡŒΠ½Π° ΠΏΠΎΠ»Π½ΠΎΠΌΡƒ ΡΠΎΠΏΡ€ΠΎΡ‚ΠΈΠ²Π»Π΅Π½ΠΈΡŽ Ρ†Π΅ΠΏΠΈ, Π³Π΄Π΅ ΠΏΠΎΠ΄ ΠΏΠΎΠ»Π½Ρ‹ΠΌ сопротивлСниСм понимаСтся сумма Π²Π½Π΅ΡˆΠ½ΠΈΡ… ΠΈ Π²Π½ΡƒΡ‚Ρ€Π΅Π½Π½ΠΈΡ… сопротивлСний.

Рис. 2. Π‘Ρ…Π΅ΠΌΠ° Π·Π°ΠΊΠΎΠ½Π° Ома для ΠΏΠΎΠ»Π½ΠΎΠΉ Ρ†Π΅ΠΏΠΈ.

  • R – внСшнСС сопротивлСниС [Ом];
  • r – сопротивлСниС источника Π­Π”Π‘ (Π²Π½ΡƒΡ‚Ρ€Π΅Π½Π½Π΅Π΅) [Ом];
  • I – сила Ρ‚ΠΎΠΊΠ° [А];
  • Ρ– Π­Π”Π‘ источника Ρ‚ΠΎΠΊΠ° [Π’].

Рассмотрим Π½Π΅ΠΊΠΎΡ‚ΠΎΡ€Ρ‹Π΅ Π·Π°Π΄Π°Ρ‡ΠΈ Π½Π° Π΄Π°Π½Π½ΡƒΡŽ Ρ‚Π΅ΠΌΡƒ. Π—Π°Π΄Π°Ρ‡ΠΈ Π½Π° Π·Π°ΠΊΠΎΠ½ Ома для ΠΏΠΎΠ»Π½ΠΎΠΉ Ρ†Π΅ΠΏΠΈ, ΠΊΠ°ΠΊ ΠΏΡ€Π°Π²ΠΈΠ»ΠΎ, Π΄Π°ΡŽΡ‚ ΡƒΡ‡Π΅Π½ΠΈΠΊΠ°ΠΌ 10 класса, Ρ‡Ρ‚ΠΎΠ±Ρ‹ ΠΎΠ½ΠΈ ΠΌΠΎΠ³Π»ΠΈ Π»ΡƒΡ‡ΡˆΠ΅ ΡƒΡΠ²ΠΎΠΈΡ‚ΡŒ ΡƒΠΊΠ°Π·Π°Π½Π½ΡƒΡŽ Ρ‚Π΅ΠΌΡƒ.

I. ΠžΠΏΡ€Π΅Π΄Π΅Π»ΠΈΡ‚Π΅ силу Ρ‚ΠΎΠΊΠ° Π² Ρ†Π΅ΠΏΠΈ с Π»Π°ΠΌΠΏΠΎΡ‡ΠΊΠΎΠΉ, сопротивлСниСм 2,4 Ом ΠΈ источником Ρ‚ΠΎΠΊΠ°, Π­Π”Π‘ ΠΊΠΎΡ‚ΠΎΡ€ΠΎΠ³ΠΎ Ρ€Π°Π²Π½ΠΎ 10 Π’, Π° Π²Π½ΡƒΡ‚Ρ€Π΅Π½Π½Π΅Π΅ сопротивлСниС 0,1 Ом.

По ΠΎΠΏΡ€Π΅Π΄Π΅Π»Π΅Π½ΠΈΡŽ Π·Π°ΠΊΠΎΠ½Π° Ома для ΠΏΠΎΠ»Π½ΠΎΠΉ Ρ†Π΅ΠΏΠΈ, сила Ρ‚ΠΎΠΊΠ° Ρ€Π°Π²Π½Π°:

II. ΠžΠΏΡ€Π΅Π΄Π΅Π»ΠΈΡ‚ΡŒ Π²Π½ΡƒΡ‚Ρ€Π΅Π½Π½Π΅Π΅ сопротивлСниС источника Ρ‚ΠΎΠΊΠ° с Π­Π”Π‘ 52 Π’. Если извСстно, Ρ‡Ρ‚ΠΎ ΠΏΡ€ΠΈ ΠΏΠΎΠ΄ΠΊΠ»ΡŽΡ‡Π΅Π½ΠΈΠΈ этого источника Ρ‚ΠΎΠΊΠ° ΠΊ Ρ†Π΅ΠΏΠΈ с сопротивлСниСм 10 Ом Π°ΠΌΠΏΠ΅Ρ€ΠΌΠ΅Ρ‚Ρ€ ΠΏΠΎΠΊΠ°Π·Ρ‹Π²Π°Π΅Ρ‚ Π·Π½Π°Ρ‡Π΅Π½ΠΈΠ΅ 5 А.

Π—Π°ΠΏΠΈΡˆΠ΅ΠΌ Π·Π°ΠΊΠΎΠ½ Ома для ΠΏΠΎΠ»Π½ΠΎΠΉ Ρ†Π΅ΠΏΠΈ ΠΈ Π²Ρ‹Ρ€Π°Π·ΠΈΠΌ ΠΈΠ· Π½Π΅Π³ΠΎ Π²Π½ΡƒΡ‚Ρ€Π΅Π½Π½Π΅Π΅ сопротивлСниС:

III. ΠžΠ΄Π½Π°ΠΆΠ΄Ρ‹ школьник спросил Ρƒ учитСля ΠΏΠΎ Ρ„ΠΈΠ·ΠΈΠΊΠ΅: Β«ΠŸΠΎΡ‡Π΅ΠΌΡƒ Π±Π°Ρ‚Π°Ρ€Π΅ΠΉΠΊΠ° садится?Β» Как Π³Ρ€Π°ΠΌΠΎΡ‚Π½ΠΎ ΠΎΡ‚Π²Π΅Ρ‚ΠΈΡ‚ΡŒ Π½Π° Π΄Π°Π½Π½Ρ‹ΠΉ вопрос?

ΠœΡ‹ ΡƒΠΆΠ΅ Π·Π½Π°Π΅ΠΌ, Ρ‡Ρ‚ΠΎ Ρ€Π΅Π°Π»ΡŒΠ½Ρ‹ΠΉ источник ΠΎΠ±Π»Π°Π΄Π°Π΅Ρ‚ собствСнным сопротивлСниСм, ΠΊΠΎΡ‚ΠΎΡ€ΠΎΠ΅ обусловлСно Π»ΠΈΠ±ΠΎ сопротивлСниСм растворов элСктролитов для Π³Π°Π»ΡŒΠ²Π°Π½ΠΈΡ‡Π΅ΡΠΊΠΈΡ… элСмСнтов ΠΈ аккумуляторов, Π»ΠΈΠ±ΠΎ сопротивлСниСм ΠΏΡ€ΠΎΠ²ΠΎΠ΄Π½ΠΈΠΊΠΎΠ² для Π³Π΅Π½Π΅Ρ€Π°Ρ‚ΠΎΡ€ΠΎΠ². Богласно Π·Π°ΠΊΠΎΠ½Ρƒ Ома для ΠΏΠΎΠ»Π½ΠΎΠΉ Ρ†Π΅ΠΏΠΈ:

ΡΠ»Π΅Π΄ΠΎΠ²Π°Ρ‚Π΅Π»ΡŒΠ½ΠΎ, Ρ‚ΠΎΠΊ Π² Ρ†Π΅ΠΏΠΈ ΠΌΠΎΠΆΠ΅Ρ‚ ΡƒΠΌΠ΅Π½ΡŒΡˆΠ°Ρ‚ΡŒΡΡ Π»ΠΈΠ±ΠΎ ΠΈΠ·-Π·Π° ΡƒΠΌΠ΅Π½ΡŒΡˆΠ΅Π½ΠΈΡ Π­Π”Π‘, Π»ΠΈΠ±ΠΎ ΠΈΠ·-Π·Π° ΠΏΠΎΠ²Ρ‹ΡˆΠ΅Π½ΠΈΡ Π²Π½ΡƒΡ‚Ρ€Π΅Π½Π½Π΅Π³ΠΎ сопротивлСния. Π—Π½Π°Ρ‡Π΅Π½ΠΈΠ΅ Π­Π”Π‘ Ρƒ аккумулятора ΠΏΠΎΡ‡Ρ‚ΠΈ постоянный. Π‘Π»Π΅Π΄ΠΎΠ²Π°Ρ‚Π΅Π»ΡŒΠ½ΠΎ, Ρ‚ΠΎΠΊ Π² Ρ†Π΅ΠΏΠΈ пониТаСтся Π·Π° счСт ΠΏΠΎΠ²Ρ‹ΡˆΠ΅Π½ΠΈΡ Π²Π½ΡƒΡ‚Ρ€Π΅Π½Π½Π΅Π³ΠΎ сопротивлСния. Π˜Ρ‚Π°ΠΊ, Β«Π±Π°Ρ‚Π°Ρ€Π΅ΠΉΠΊΠ°Β» садится, Ρ‚Π°ΠΊ ΠΊΠ°ΠΊ Π΅Ρ‘ Π²Π½ΡƒΡ‚Ρ€Π΅Π½Π½Π΅Π΅ сопротивлСниС увСличиваСтся.

Π—Π°ΠΊΠΎΠ½
Ома для участка Ρ†Π΅ΠΏΠΈ: сила
Ρ‚ΠΎΠΊΠ°
I


Π½Π° участкС элСктричСской Ρ†Π΅ΠΏΠΈ прямо
ΠΏΡ€ΠΎΠΏΠΎΡ€Ρ†ΠΈΠΎΠ½Π°Π»ΡŒΠ½Π° Π½Π°ΠΏΡ€ΡΠΆΠ΅Π½ΠΈΡŽ
U


Π½Π° ΠΊΠΎΠ½Ρ†Π°Ρ… участка ΠΈ ΠΎΠ±Ρ€Π°Ρ‚Π½ΠΎ ΠΏΡ€ΠΎΠΏΠΎΡ€Ρ†ΠΈΠΎΠ½Π°Π»ΡŒΠ½Π°
Π΅Π³ΠΎ ΡΠΎΠΏΡ€ΠΎΡ‚ΠΈΠ²Π»Π΅Π½ΠΈΡŽ

R.

Π€ΠΎΡ€ΠΌΡƒΠ»Π°
Π·Π°ΠΊΠΎΠ½Π°:

I

=.
ΠžΡ‚ΡΡŽΠ΄Π° запишСм Ρ„ΠΎΡ€ΠΌΡƒΠ»Ρ‹U

=
IR


ΠΈ R
=

.

Рис.1.
Участок
Ρ†Π΅ΠΏΠΈ
Рис.2.
Полная
Ρ†Π΅ΠΏΡŒ

Π—Π°ΠΊΠΎΠ½
Ома для ΠΏΠΎΠ»Π½ΠΎΠΉ Ρ†Π΅ΠΏΠΈ: сила
Ρ‚ΠΎΠΊΠ°
I


ΠΏΠΎΠ»Π½ΠΎΠΉ элСктричСской Ρ†Π΅ΠΏΠΈ
Ρ€Π°Π²Π½Π°Π­Π”Π‘
(элСктродвиТущСй силС) источника Ρ‚ΠΎΠΊΠ°
Π•
,
Π΄Π΅Π»Π΅Π½Π½ΠΎΠΉ Π½Π° ΠΏΠΎΠ»Π½ΠΎΠ΅ сопротивлСниС Ρ†Π΅ΠΏΠΈ
(R
+ r).
ПолноС
сопротивлСниС Ρ†Π΅ΠΏΠΈ Ρ€Π°Π²Π½ΠΎ суммС
сопротивлСний внСшнСй Ρ†Π΅ΠΏΠΈ R

ΠΈ Π²Π½ΡƒΡ‚Ρ€Π΅Π½Π½Π΅Π³ΠΎ r

источника Ρ‚ΠΎΠΊΠ°.Π€ΠΎΡ€ΠΌΡƒΠ»Π°
Π·Π°ΠΊΠΎΠ½Π° I
=




.
На
рис. 1 ΠΈ 2 ΠΏΡ€ΠΈΠ²Π΅Π΄Π΅Π½Ρ‹ схСмы элСктричСских
Ρ†Π΅ΠΏΠ΅ΠΉ.

3. ΠŸΠΎΡΠ»Π΅Π΄ΠΎΠ²Π°Ρ‚Π΅Π»ΡŒΠ½ΠΎΠ΅ ΠΈ ΠΏΠ°Ρ€Π°Π»Π»Π΅Π»ΡŒΠ½ΠΎΠ΅ соСдинСниС ΠΏΡ€ΠΎΠ²ΠΎΠ΄Π½ΠΈΠΊΠΎΠ²

ΠŸΡ€ΠΎΠ²ΠΎΠ΄Π½ΠΈΠΊΠΈ
Π² элСктричСских цСпях ΠΌΠΎΠ³ΡƒΡ‚ ΡΠΎΠ΅Π΄ΠΈΠ½ΡΡ‚ΡŒΡΡ
ΠΏΠΎΡΠ»Π΅Π΄ΠΎΠ²Π°Ρ‚Π΅Π»ΡŒΠ½ΠΎ

ΠΈ ΠΏΠ°Ρ€Π°Π»Π»Π΅Π»ΡŒΠ½ΠΎ
.
БмСшанноС соСдинСниС сочСтаСт ΠΎΠ±Π° эти
соСдинСния.

Π‘ΠΎΠΏΡ€ΠΎΡ‚ΠΈΠ²Π»Π΅Π½ΠΈΠ΅,ΠΏΡ€ΠΈ
Π²ΠΊΠ»ΡŽΡ‡Π΅Π½ΠΈΠΈ ΠΊΠΎΡ‚ΠΎΡ€ΠΎΠ³ΠΎ
вмСсто всСх Π΄Ρ€ΡƒΠ³ΠΈΡ… ΠΏΡ€ΠΎΠ²ΠΎΠ΄Π½ΠΈΠΊΠΎΠ²,
находящихся ΠΌΠ΅ΠΆΠ΄Ρƒ двумя Ρ‚ΠΎΡ‡ΠΊΠ°ΠΌΠΈ Ρ†Π΅ΠΏΠΈ,
Ρ‚ΠΎΠΊ ΠΈ напряТСниС ΠΎΡΡ‚Π°ΡŽΡ‚ΡΡ Π½Π΅ΠΈΠ·ΠΌΠ΅Π½Π½Ρ‹ΠΌΠΈ,
Π½Π°Π·Ρ‹Π²Π°ΡŽΡ‚
эквивалСнтным
сопротивлСниСм

этих
ΠΏΡ€ΠΎΠ²ΠΎΠ΄Π½ΠΈΠΊΠΎΠ².

ΠŸΠΎΡΠ»Π΅Π΄ΠΎΠ²Π°Ρ‚Π΅Π»ΡŒΠ½ΠΎΠ΅ соСдинСниС

ΠŸΠΎΡΠ»Π΅Π΄ΠΎΠ²Π°Ρ‚Π΅Π»ΡŒΠ½Ρ‹ΠΌ
называСтся соСдинСниС, ΠΏΡ€ΠΈ ΠΊΠΎΡ‚ΠΎΡ€ΠΎΠΌ
ΠΊΠ°ΠΆΠ΄Ρ‹ΠΉ
ΠΏΡ€ΠΎΠ²ΠΎΠ΄Π½ΠΈΠΊ соСдиняСтся Ρ‚ΠΎΠ»ΡŒΠΊΠΎ с ΠΎΠ΄Π½ΠΈΠΌ
ΠΏΡ€Π΅Π΄Ρ‹Π΄ΡƒΡ‰ΠΈΠΌ ΠΈ ΠΎΠ΄Π½ΠΈΠΌ ΠΏΠΎΡΠ»Π΅Π΄ΡƒΡŽΡ‰ΠΈΠΌ
ΠΏΡ€ΠΎΠ²ΠΎΠ΄Π½ΠΈΠΊΠ°ΠΌΠΈ.

Как
слСдуСт ΠΈΠ· ΠΏΠ΅Ρ€Π²ΠΎΠ³ΠΎ ΠΏΡ€Π°Π²ΠΈΠ»Π°
ΠšΠΈΡ€Ρ…Π³ΠΎΡ„Π°
,
ΠΏΡ€ΠΈ ΠΏΠΎΡΠ»Π΅Π΄ΠΎΠ²Π°Ρ‚Π΅Π»ΡŒΠ½ΠΎΠΌ
соСдинСнии ΠΏΡ€ΠΎΠ²ΠΎΠ΄Π½ΠΈΠΊΠΎΠ² сила элСктричСского
Ρ‚ΠΎΠΊΠ°, ΠΏΡ€ΠΎΡ‚Π΅ΠΊΠ°ΡŽΡ‰Π΅Π³ΠΎ ΠΏΠΎ всСм ΠΏΡ€ΠΎΠ²ΠΎΠ΄Π½ΠΈΠΊΠ°ΠΌ,
ΠΎΠ΄ΠΈΠ½Π°ΠΊΠΎΠ²Π° (Π½Π° основании Π·Π°ΠΊΠΎΠ½Π° сохранСния
заряда).

1.
ΠŸΡ€ΠΈ ΠΏΠΎΡΠ»Π΅Π΄ΠΎΠ²Π°Ρ‚Π΅Π»ΡŒΠ½ΠΎΠΌ соСдинСнии

ΠΏΡ€ΠΎΠ²ΠΎΠ΄Π½ΠΈΠΊΠΎΠ²
(рис. 1)
сила
Ρ‚ΠΎΠΊΠ° Π²ΠΎ всСх ΠΏΡ€ΠΎΠ²ΠΎΠ΄Π½ΠΈΠΊΠ°Ρ… ΠΎΠ΄ΠΈΠ½Π°ΠΊΠΎΠ²Π°:
I

1
=
I

2
=

I

3

=

I

Рис.
1.ΠŸΠΎΡΠ»Π΅Π΄ΠΎΠ²Π°Ρ‚Π΅Π»ΡŒΠ½ΠΎΠ΅
соСдинСниС Π΄Π²ΡƒΡ… ΠΏΡ€ΠΎΠ²ΠΎΠ΄Π½ΠΈΠΊΠΎΠ².

2.
Богласно Π·Π°ΠΊΠΎΠ½Ρƒ Ома, напряТСнияU

1
ΠΈU

2
Π½Π°
ΠΏΡ€ΠΎΠ²ΠΎΠ΄Π½ΠΈΠΊΠ°Ρ… Ρ€Π°Π²Π½Ρ‹ U

1
=
IR

1
,
U

2
=
IR

2
,
U

3
=
IR

3
.

НапряТСниС
ΠΏΡ€ΠΈ ΠΏΠΎΡΠ»Π΅Π΄ΠΎΠ²Π°Ρ‚Π΅Π»ΡŒΠ½ΠΎΠΌ соСдинСнии
ΠΏΡ€ΠΎΠ²ΠΎΠ΄Π½ΠΈΠΊΠΎΠ² Ρ€Π°Π²Π½ΠΎ суммС напряТСний Π½Π°
ΠΎΡ‚Π΄Π΅Π»ΡŒΠ½Ρ‹Ρ… участках (ΠΏΡ€ΠΎΠ²ΠΎΠ΄Π½ΠΈΠΊΠ°Ρ…)
элСктричСской Ρ†Π΅ΠΏΠΈ.

U

=
U

1
+
U

2
+
U

3

ΠŸΠΎΠ·Π°ΠΊΠΎΠ½Ρƒ
Ома, напряТСния U
1,
U
2
Π½Π°
ΠΏΡ€ΠΎΠ²ΠΎΠ΄Π½ΠΈΠΊΠ°Ρ… Ρ€Π°Π²Π½Ρ‹U

1
=
IR

1
,
U

2
=
IR

2
,
Π’
соотвСтствии Π²Ρ‚ΠΎΡ€Ρ‹ΠΌ ΠΏΡ€Π°Π²ΠΈΠ»ΠΎΠΌ ΠšΠΈΡ€Ρ…Π³ΠΎΡ„Π°
напряТСниС Π½Π° всСм участкС:

U

=
U

1
+
U

2
=
IR

1
+
IR

2

=

I(R

1
+

R

2
)=
IΒ·R.

ΠŸΠΎΠ»ΡƒΡ‡Π°Π΅ΠΌ:
R

=
R

1
+
R

2

ΠžΠ±Ρ‰Π΅Π΅
напряТСниС
U


Π½Π° ΠΏΡ€ΠΎΠ²ΠΎΠ΄Π½ΠΈΠΊΠ°Ρ… Ρ€Π°Π²Π½ΠΎ суммС напряТСний
U

1
,
U

2
,
U

3

Ρ€Π°Π²Π½ΠΎ:
U

=
U

1
+
U

2
+
U

3
=
I

Β·

(R

1
+
R

2
+
R

3
)
=
IR

Π³Π΄Π΅
R

Π­ΠšΠ’
–
эквивалСнтноС

сопротивлСниС всСй Ρ†Π΅ΠΏΠΈ. ΠžΡ‚ΡΡŽΠ΄Π°: R

Π­ΠšΠ’

=
R

1
+
R

2
+
R

3

ΠŸΡ€ΠΈ
ΠΏΠΎΡΠ»Π΅Π΄ΠΎΠ²Π°Ρ‚Π΅Π»ΡŒΠ½ΠΎΠΌ соСдинСнии эквивалСнтноС
сопротивлСниС Ρ†Π΅ΠΏΠΈ Ρ€Π°Π²Π½ΠΎ суммС
сопротивлСний ΠΎΡ‚Π΄Π΅Π»ΡŒΠ½Ρ‹Ρ… участков Ρ†Π΅ΠΏΠΈ:
R

Π­ΠšΠ’
=
R

1
+
R

2
+
R

3
+…

Π­Ρ‚ΠΎΡ‚
Ρ€Π΅Π·ΡƒΠ»ΡŒΡ‚Π°Ρ‚ справСдлив для
любого числа

ΠΏΠΎΡΠ»Π΅Π΄ΠΎΠ²Π°Ρ‚Π΅Π»ΡŒΠ½ΠΎ соСдинСнных ΠΏΡ€ΠΎΠ²ΠΎΠ΄Π½ΠΈΠΊΠΎΠ².

Из
Π·Π°ΠΊΠΎΠ½Π° ΠžΠΌΠ°ΡΠ»Π΅Π΄ΡƒΠ΅Ρ‚:
ΠΏΡ€ΠΈ равСнствС сил Ρ‚ΠΎΠΊΠ° ΠΏΡ€ΠΈ ΠΏΠΎΡΠ»Π΅Π΄ΠΎΠ²Π°Ρ‚Π΅Π»ΡŒΠ½ΠΎΠΌ
соСдинСнии:

I

=
,
I

=
.
ΠžΡ‚ΡΡŽΠ΄Π°
=
ΠΈΠ»ΠΈ

=,
Ρ‚. Π΅. напряТСния Π½Π° ΠΎΡ‚Π΄Π΅Π»ΡŒΠ½Ρ‹Ρ… участках
Ρ†Π΅ΠΏΠΈ прямо ΠΏΡ€ΠΎΠΏΠΎΡ€Ρ†ΠΈΠΎΠ½Π°Π»ΡŒΠ½Ρ‹ сопротивлСниям
участков.

ΠŸΡ€ΠΈ
ΠΏΠΎΡΠ»Π΅Π΄ΠΎΠ²Π°Ρ‚Π΅Π»ΡŒΠ½ΠΎΠΌ соСдинСнии n
ΠΎΠ΄ΠΈΠ½Π°ΠΊΠΎΠ²Ρ‹Ρ…
ΠΏΡ€ΠΎΠ²ΠΎΠ΄Π½ΠΈΠΊΠΎΠ² ΠΎΠ±Ρ‰Π΅Π΅ напряТСниС Ρ€Π°Π²Π½ΠΎ
ΠΏΡ€ΠΎΠΈΠ·Π²Π΅Π΄Π΅Π½ΠΈΡŽ Π½Π°ΠΏΡ€ΡΠΆΠ΅Π½ΠΈΡŽ ΠΎΠ΄Π½ΠΎΠ³ΠΎ U 1
Π½Π°
ΠΈΡ… количСство n
:

U
ΠŸΠžΠ‘Π›Π•Π”
=
n

Β·
U
1
.
Аналогично
для сопротивлСний:

R

ΠŸΠžΠ‘Π›Π•Π”

=

n

Β·

R

1

ΠŸΡ€ΠΈ Ρ€Π°Π·ΠΌΡ‹ΠΊΠ°Π½ΠΈΠΈ
Ρ†Π΅ΠΏΠΈ ΠΎΠ΄Π½ΠΎΠ³ΠΎ ΠΈΠ· ΠΏΠΎΡΠ»Π΅Π΄ΠΎΠ²Π°Ρ‚Π΅Π»ΡŒΠ½ΠΎ
соСдинСнных ΠΏΠΎΡ‚Ρ€Π΅Π±ΠΈΡ‚Π΅Π»Π΅ΠΉ Ρ‚ΠΎΠΊ исчСзаСт
Π²ΠΎ всСй Ρ†Π΅ΠΏΠΈ, поэтому ΠΏΠΎΡΠ»Π΅Π΄ΠΎΠ²Π°Ρ‚Π΅Π»ΡŒΠ½ΠΎΠ΅
соСдинСниС Π½Π° ΠΏΡ€Π°ΠΊΡ‚ΠΈΠΊΠ΅ Π½Π΅ всСгда ΡƒΠ΄ΠΎΠ±Π½ΠΎ.

ЭлСктричСский Ρ‚ΠΎΠΊ ΠΈ опасноС напряТСниС Π½Π΅Π²ΠΎΠ·ΠΌΠΎΠΆΠ½ΠΎ ΡƒΡΠ»Ρ‹ΡˆΠ°Ρ‚ΡŒ (Π·Π° ΠΈΡΠΊΠ»ΡŽΡ‡Π΅Π½ΠΈΠ΅ΠΌ гудящих Π²Ρ‹ΡΠΎΠΊΠΎΠ²ΠΎΠ»ΡŒΡ‚Π½Ρ‹Ρ… Π»ΠΈΠ½ΠΈΠΉ ΠΈ элСктроустановок). Π’ΠΎΠΊΠΎΠ²Π΅Π΄ΡƒΡ‰ΠΈΠ΅ части, находящиСся ΠΏΠΎΠ΄ напряТСниСм, Π½ΠΈΡ‡Π΅ΠΌ Π½Π΅ ΠΎΡ‚Π»ΠΈΡ‡Π°ΡŽΡ‚ΡΡ ΠΏΠΎ Π²Π½Π΅ΡˆΠ½Π΅ΠΌΡƒ Π²ΠΈΠ΄Ρƒ.

НСвозмоТно ΡƒΠ·Π½Π°Ρ‚ΡŒ ΠΈΡ… ΠΈ ΠΏΠΎ Π·Π°ΠΏΠ°Ρ…Ρƒ, ΠΈ ΠΏΠΎΠ²Ρ‹ΡˆΠ΅Π½Π½ΠΎΠΉ Ρ‚Π΅ΠΌΠΏΠ΅Ρ€Π°Ρ‚ΡƒΡ€ΠΎΠΉ Π² ΡˆΡ‚Π°Ρ‚Π½Ρ‹Ρ… Ρ€Π΅ΠΆΠΈΠΌΠ°Ρ… Ρ€Π°Π±ΠΎΡ‚Ρ‹ ΠΎΠ½ΠΈ Π½Π΅ ΠΎΡ‚Π»ΠΈΡ‡Π°ΡŽΡ‚ΡΡ. Но Π²ΠΊΠ»ΡŽΡ‡Π°Π΅ΠΌ Π² Π±Π΅Π·ΠΌΠΎΠ»Π²Π½ΡƒΡŽ ΠΈ Ρ‚ΠΈΡ…ΡƒΡŽ Ρ€ΠΎΠ·Π΅Ρ‚ΠΊΡƒ пылСсос, Ρ‰Π΅Π»ΠΊΠ°Π΅ΠΌ Π²Ρ‹ΠΊΠ»ΡŽΡ‡Π°Ρ‚Π΅Π»Π΅ΠΌ — ΠΈ энСргия словно бСрСтся ΠΈΠ· Π½ΠΈΠΎΡ‚ΠΊΡƒΠ΄Π°, сама ΠΏΠΎ сСбС, ΠΌΠ°Ρ‚Π΅Ρ€ΠΈΠ°Π»ΠΈΠ·ΡƒΡΡΡŒ Π² Π²ΠΈΠ΄Π΅ ΡˆΡƒΠΌΠ° ΠΈ компрСссии Π²Π½ΡƒΡ‚Ρ€ΠΈ Π±Ρ‹Ρ‚ΠΎΠ²ΠΎΠ³ΠΎ ΠΏΡ€ΠΈΠ±ΠΎΡ€Π°.

ΠžΠΏΡΡ‚ΡŒ ΠΆΠ΅, Ссли ΠΌΡ‹ Π²ΠΎΡ‚ΠΊΠ½Π΅ΠΌ Π² Ρ€Π°Π·ΡŠΠ΅ΠΌΡ‹ Ρ€ΠΎΠ·Π΅Ρ‚ΠΊΠΈ Π΄Π²Π° гвоздя ΠΈ возьмСмся Π·Π° Π½ΠΈΡ…, Ρ‚ΠΎ Π±ΡƒΠΊΠ²Π°Π»ΡŒΠ½ΠΎ всСм своим Ρ‚Π΅Π»ΠΎΠΌ ΠΎΡ‰ΡƒΡ‚ΠΈΠΌ Ρ€Π΅Π°Π»ΡŒΠ½ΠΎΡΡ‚ΡŒ ΠΈ ΠΎΠ±ΡŠΠ΅ΠΊΡ‚ΠΈΠ²Π½ΠΎΡΡ‚ΡŒ сущСствования элСктричСского Ρ‚ΠΎΠΊΠ°. Π”Π΅Π»Π°Ρ‚ΡŒ это, ΠΊΠΎΠ½Π΅Ρ‡Π½ΠΎ, Π½Π°ΡΡ‚ΠΎΡΡ‚Π΅Π»ΡŒΠ½ΠΎ Π½Π΅ рСкомСндуСтся. Но ΠΏΡ€ΠΈΠΌΠ΅Ρ€Ρ‹ с пылСсосом ΠΈ гвоздями наглядно Π΄Π΅ΠΌΠΎΠ½ΡΡ‚Ρ€ΠΈΡ€ΡƒΡŽΡ‚ Π½Π°ΠΌ, Ρ‡Ρ‚ΠΎ ΠΈΠ·ΡƒΡ‡Π΅Π½ΠΈΠ΅ ΠΈ ΠΏΠΎΠ½ΠΈΠΌΠ°Π½ΠΈΠ΅ основных Π·Π°ΠΊΠΎΠ½ΠΎΠ² элСктротСхники способствуСт бСзопасности ΠΏΡ€ΠΈ ΠΎΠ±Ρ€Π°Ρ‰Π΅Π½ΠΈΠΈ с Π±Ρ‹Ρ‚ΠΎΠ²Ρ‹ΠΌ элСктричСством, Π° Ρ‚Π°ΠΊΠΆΠ΅ ΡƒΡΡ‚Ρ€Π°Π½Π΅Π½ΠΈΡŽ суСвСрных ΠΏΡ€Π΅Π΄ΡƒΠ±Π΅ΠΆΠ΄Π΅Π½ΠΈΠΉ, связанных с элСктричСским Ρ‚ΠΎΠΊΠΎΠΌ ΠΈ напряТСниСм.

Π˜Ρ‚Π°ΠΊ, рассмотрим ΠΎΠ΄ΠΈΠ½, самый Ρ†Π΅Π½Π½Ρ‹ΠΉ Π·Π°ΠΊΠΎΠ½ элСктротСхники, ΠΊΠΎΡ‚ΠΎΡ€Ρ‹ΠΉ ΠΏΠΎΠ»Π΅Π·Π½ΠΎ Π·Π½Π°Ρ‚ΡŒ. И попытаСмся ΡΠ΄Π΅Π»Π°Ρ‚ΡŒ это Π² ΠΊΠ°ΠΊ ΠΌΠΎΠΆΠ½ΠΎ Π±ΠΎΠ»Π΅Π΅ популярной Ρ„ΠΎΡ€ΠΌΠ΅.

Π—Π°ΠΊΠΎΠ½ Ома

1. Π”ΠΈΡ„Ρ„Π΅Ρ€Π΅Π½Ρ†ΠΈΠ°Π»ΡŒΠ½Π°Ρ Ρ„ΠΎΡ€ΠΌΠ° записи Π·Π°ΠΊΠΎΠ½Π° Ома

Π‘Π°ΠΌΡ‹ΠΉ Π³Π»Π°Π²Π½Ρ‹ΠΉ Π·Π°ΠΊΠΎΠ½ элСктротСхники — это, ΠΊΠΎΠ½Π΅Ρ‡Π½ΠΎ, Π·Π°ΠΊΠΎΠ½ Ома
. О Π΅Π³ΠΎ сущСствовании Π·Π½Π°ΡŽΡ‚ Π΄Π°ΠΆΠ΅ люди, Π½Π΅ ΠΈΠΌΠ΅ΡŽΡ‰ΠΈΠ΅ ΠΎΡ‚Π½ΠΎΡˆΠ΅Π½ΠΈΡ ΠΊ элСктротСхникС. Но ΠΌΠ΅ΠΆΠ΄Ρƒ Ρ‚Π΅ΠΌ вопрос «А знаСшь Π»ΠΈ Ρ‚Ρ‹ Π·Π°ΠΊΠΎΠ½ Ома?Β» Π² тСхничСских Π’Π£Π—Π°Ρ… являСтся Π»ΠΎΠ²ΡƒΡˆΠΊΠΎΠΉ для Π·Π°Ρ€Π²Π°Π²ΡˆΠΈΡ…ΡΡ ΠΈ самонадСянных ΡˆΠΊΠΎΠ»ΡΡ€ΠΎΠ². Π’ΠΎΠ²Π°Ρ€ΠΈΡ‰, разумССтся, ΠΎΡ‚Π²Π΅Ρ‡Π°Π΅Ρ‚, Ρ‡Ρ‚ΠΎ Π·Π°ΠΊΠΎΠ½ Ома Π·Π½Π°Π΅Ρ‚ ΠΎΡ‚Π»ΠΈΡ‡Π½ΠΎ, ΠΈ Ρ‚ΠΎΠ³Π΄Π° ΠΊ Π½Π΅ΠΌΡƒ ΠΎΠ±Ρ€Π°Ρ‰Π°ΡŽΡ‚ΡΡ с ΠΏΡ€ΠΎΡΡŒΠ±ΠΎΠΉ привСсти этот Π·Π°ΠΊΠΎΠ½ Π² Π΄ΠΈΡ„Ρ„Π΅Ρ€Π΅Π½Ρ†ΠΈΠ°Π»ΡŒΠ½ΠΎΠΉ Ρ„ΠΎΡ€ΠΌΠ΅. Π’ΡƒΡ‚-Ρ‚ΠΎ ΠΈ выясняСтся, Ρ‡Ρ‚ΠΎ ΡˆΠΊΠΎΠ»ΡΡ€Ρƒ ΠΈΠ»ΠΈ пСрвокурснику Π΅Ρ‰Π΅ ΡƒΡ‡ΠΈΡ‚ΡŒΡΡ ΠΈ ΡƒΡ‡ΠΈΡ‚ΡŒΡΡ.

Однако Π΄ΠΈΡ„Ρ„Π΅Ρ€Π΅Π½Ρ†ΠΈΠ°Π»ΡŒΠ½Π°Ρ Ρ„ΠΎΡ€ΠΌΠ° записи Π·Π°ΠΊΠΎΠ½Π° Ома Π½Π° ΠΏΡ€Π°ΠΊΡ‚ΠΈΠΊΠ΅ ΠΏΠΎΡ‡Ρ‚ΠΈ Π½Π΅ΠΏΡ€ΠΈΠΌΠ΅Π½ΠΈΠΌΠ°. Она ΠΎΡ‚Ρ€Π°ΠΆΠ°Π΅Ρ‚ Π·Π°Π²ΠΈΡΠΈΠΌΠΎΡΡ‚ΡŒ ΠΌΠ΅ΠΆΠ΄Ρƒ ΠΏΠ»ΠΎΡ‚Π½ΠΎΡΡ‚ΡŒΡŽ Ρ‚ΠΎΠΊΠ° ΠΈ Π½Π°ΠΏΡ€ΡΠΆΠ΅Π½Π½ΠΎΡΡ‚ΡŒΡŽ поля:

Π³Π΄Π΅ G — это ΠΏΡ€ΠΎΠ²ΠΎΠ΄ΠΈΠΌΠΎΡΡ‚ΡŒ Ρ†Π΅ΠΏΠΈ; Π• — Π½Π°ΠΏΡ€ΡΠΆΠ΅Π½Π½ΠΎΡΡ‚ΡŒ элСктричСского Ρ‚ΠΎΠΊΠ°.

ВсС это — ΠΏΠΎΠΏΡ‹Ρ‚ΠΊΠΈ Π²Ρ‹Ρ€Π°Π·ΠΈΡ‚ΡŒ элСктричСский Ρ‚ΠΎΠΊ, принимая Π²ΠΎ Π²Π½ΠΈΠΌΠ°Π½ΠΈΠ΅ Ρ‚ΠΎΠ»ΡŒΠΊΠΎ физичСскиС свойства ΠΌΠ°Ρ‚Π΅Ρ€ΠΈΠ°Π»Π° ΠΏΡ€ΠΎΠ²ΠΎΠ΄Π½ΠΈΠΊΠ°, Π±Π΅Π· ΡƒΡ‡Π΅Ρ‚Π° Π΅Π³ΠΎ гСомСтричСских ΠΏΠ°Ρ€Π°ΠΌΠ΅Ρ‚Ρ€ΠΎΠ² (Π΄Π»ΠΈΠ½Π°, Π΄ΠΈΠ°ΠΌΠ΅Ρ‚Ρ€ ΠΈ Ρ‚ΠΎΠΌΡƒ ΠΏΠΎΠ΄ΠΎΠ±Π½ΠΎΠ΅). Π”ΠΈΡ„Ρ„Π΅Ρ€Π΅Π½Ρ†ΠΈΠ°Π»ΡŒΠ½Π°Ρ Ρ„ΠΎΡ€ΠΌΠ° записи Π·Π°ΠΊΠΎΠ½Π° Ома — это чистая тСория, Π·Π½Π°Π½ΠΈΠ΅ Π΅Π΅ Π² Π±Ρ‹Ρ‚Ρƒ ΡΠΎΠ²Π΅Ρ€ΡˆΠ΅Π½Π½ΠΎ Π½Π΅ трСбуСтся.

2. Π˜Π½Ρ‚Π΅Π³Ρ€Π°Π»ΡŒΠ½Π°Ρ Ρ„ΠΎΡ€ΠΌΠ° записи Π·Π°ΠΊΠΎΠ½Π° Ома для участка Ρ†Π΅ΠΏΠΈ

ИноС Π΄Π΅Π»ΠΎ — ΠΈΠ½Ρ‚Π΅Π³Ρ€Π°Π»ΡŒΠ½Π°Ρ Ρ„ΠΎΡ€ΠΌΠ° записи. Она Ρ‚ΠΎΠΆΠ΅ ΠΈΠΌΠ΅Π΅Ρ‚ нСсколько разновидностСй. Π‘Π°ΠΌΠΎΠΉ популярной ΠΈΠ· Π½ΠΈΡ… являСтся Π·Π°ΠΊΠΎΠ½ Ома для участка Ρ†Π΅ΠΏΠΈ: I=U/R

Говоря ΠΏΠΎ-Π΄Ρ€ΡƒΠ³ΠΎΠΌΡƒ, Ρ‚ΠΎΠΊ Π² участкС Ρ†Π΅ΠΏΠΈ всСгда Ρ‚Π΅ΠΌ Π²Ρ‹ΡˆΠ΅, Ρ‡Π΅ΠΌ большС ΠΏΡ€ΠΈΠ»ΠΎΠΆΠ΅Π½Π½ΠΎΠ΅ ΠΊ этому участку напряТСниС ΠΈ Ρ‡Π΅ΠΌ мСньшС сопротивлСниС этого участка.

Π’ΠΎΡ‚ этот Β«Π²ΠΈΠ΄Β» Π·Π°ΠΊΠΎΠ½Π° Ома просто обязатСлСн ΠΊ запоминанию для всСх, ΠΊΠΎΠΌΡƒ Ρ…ΠΎΡ‚ΡŒ ΠΈΠ½ΠΎΠ³Π΄Π° приходится ΠΈΠΌΠ΅Ρ‚ΡŒ Π΄Π΅Π»ΠΎ с элСктричСством. Π‘Π»Π°Π³ΠΎ, ΠΈ Π·Π°Π²ΠΈΡΠΈΠΌΠΎΡΡ‚ΡŒ-Ρ‚ΠΎ совсСм простая. Π’Π΅Π΄ΡŒ напряТСниС Π² сСти ΠΌΠΎΠΆΠ½ΠΎ ΡΡ‡ΠΈΡ‚Π°Ρ‚ΡŒ Π½Π΅ΠΈΠ·ΠΌΠ΅Π½Π½Ρ‹ΠΌ. Для Ρ€ΠΎΠ·Π΅Ρ‚ΠΊΠΈ ΠΎΠ½ΠΎ Ρ€Π°Π²Π½ΠΎ 220 Π²ΠΎΠ»ΡŒΡ‚. ΠŸΠΎΡΡ‚ΠΎΠΌΡƒ получаСтся, Ρ‡Ρ‚ΠΎ Ρ‚ΠΎΠΊ Π² Ρ†Π΅ΠΏΠΈ зависит Ρ‚ΠΎΠ»ΡŒΠΊΠΎ ΠΎΡ‚ сопротивлСния Ρ†Π΅ΠΏΠΈ, ΠΏΠΎΠ΄ΠΊΠ»ΡŽΡ‡Π°Π΅ΠΌΠΎΠΉ ΠΊ Ρ€ΠΎΠ·Π΅Ρ‚ΠΊΠ΅. ΠžΡ‚ΡΡŽΠ΄Π° простая ΠΌΠΎΡ€Π°Π»ΡŒ: Π·Π° этим сопротивлСниСм Π½Π°Π΄ΠΎ ΡΠ»Π΅Π΄ΠΈΡ‚ΡŒ.

ΠšΠΎΡ€ΠΎΡ‚ΠΊΠΈΠ΅ замыкания, ΠΊΠΎΡ‚ΠΎΡ€Ρ‹Π΅ Ρƒ всСх Π½Π° слуху, ΡΠ»ΡƒΡ‡Π°ΡŽΡ‚ΡΡ ΠΈΠΌΠ΅Π½Π½ΠΎ ΠΏΠΎ ΠΏΡ€ΠΈΡ‡ΠΈΠ½Π΅ Π½ΠΈΠ·ΠΊΠΎΠ³ΠΎ сопротивлСния внСшнСй Ρ†Π΅ΠΏΠΈ. ΠŸΡ€Π΅Π΄ΠΏΠΎΠ»ΠΎΠΆΠΈΠΌ, Ρ‡Ρ‚ΠΎ ΠΈΠ·-Π·Π° Π½Π΅ΠΏΡ€Π°Π²ΠΈΠ»ΡŒΠ½ΠΎΠ³ΠΎ соСдинСния ΠΏΡ€ΠΎΠ²ΠΎΠ΄ΠΎΠ² Π² ΠΎΡ‚Π²Π΅Ρ‚Π²ΠΈΡ‚Π΅Π»ΡŒΠ½ΠΎΠΉ ΠΊΠΎΡ€ΠΎΠ±ΠΊΠ΅ Ρ„Π°Π·Π½Ρ‹ΠΉ ΠΈ Π½ΡƒΠ»Π΅Π²ΠΎΠΉ ΠΏΡ€ΠΎΠ²ΠΎΠ΄Π° оказались Π½Π°ΠΏΡ€ΡΠΌΡƒΡŽ соСдинСны ΠΌΠ΅ΠΆΠ΄Ρƒ собой. Π’ΠΎΠ³Π΄Π° сопротивлСниС участка Ρ†Π΅ΠΏΠΈ Ρ€Π΅Π·ΠΊΠΎ снизится практичСски Π΄ΠΎ нуля, Π° Ρ‚ΠΎΠΊ Ρ‚Π°ΠΊ ΠΆΠ΅ Ρ€Π΅Π·ΠΊΠΎ возрастСт Π΄ΠΎ ΠΎΡ‡Π΅Π½ΡŒ большой Π²Π΅Π»ΠΈΡ‡ΠΈΠ½Ρ‹. Если элСктропроводка Π²Ρ‹ΠΏΠΎΠ»Π½Π΅Π½Π° ΠΏΡ€Π°Π²ΠΈΠ»ΡŒΠ½ΠΎ, Ρ‚ΠΎ сработаСт автоматичСский Π²Ρ‹ΠΊΠ»ΡŽΡ‡Π°Ρ‚Π΅Π»ΡŒ, Π° Ссли Π΅Π³ΠΎ Π½Π΅Ρ‚, ΠΈΠ»ΠΈ ΠΎΠ½ нСисправСн ΠΈΠ»ΠΈ ΠΏΠΎΠ΄ΠΎΠ±Ρ€Π°Π½ Π½Π΅ΠΏΡ€Π°Π²ΠΈΠ»ΡŒΠ½ΠΎ, Ρ‚ΠΎ ΠΏΡ€ΠΎΠ²ΠΎΠ΄ Π½Π΅ справится с Π²ΠΎΠ·Ρ€ΠΎΡΡˆΠΈΠΌ Ρ‚ΠΎΠΊΠΎΠΌ, нагрССтся, расплавится ΠΈ, Π²ΠΎΠ·ΠΌΠΎΠΆΠ½ΠΎ, Π²Ρ‹Π·ΠΎΠ²Π΅Ρ‚ ΠΏΠΎΠΆΠ°Ρ€.

Но Π±Ρ‹Π²Π°Π΅Ρ‚, Ρ‡Ρ‚ΠΎ ΠΏΡ€ΠΈΠ±ΠΎΡ€Ρ‹, Π²ΠΊΠ»ΡŽΡ‡Π΅Π½Π½Ρ‹Π΅ Π² Ρ€ΠΎΠ·Π΅Ρ‚ΠΊΡƒ ΠΈ ΠΎΡ‚Ρ€Π°Π±ΠΎΡ‚Π°Π²ΡˆΠΈΠ΅ ΡƒΠΆΠ΅ Π΄Π°Π»Π΅ΠΊΠΎ Π½Π΅ ΠΎΠ΄ΠΈΠ½ час, становятся ΠΏΡ€ΠΈΡ‡ΠΈΠ½ΠΎΠΉ ΠΊΠΎΡ€ΠΎΡ‚ΠΊΠΎΠ³ΠΎ замыкания. Π’ΠΈΠΏΠΈΡ‡Π½Ρ‹ΠΉ случай — вСнтилятор, ΠΎΠ±ΠΌΠΎΡ‚ΠΊΠΈ двигатСля ΠΊΠΎΡ‚ΠΎΡ€ΠΎΠ³ΠΎ ΠΏΠΎΠ΄Π²Π΅Ρ€Π³Π»ΠΈΡΡŒ ΠΏΠ΅Ρ€Π΅Π³Ρ€Π΅Π²Ρƒ ΠΈΠ·-Π·Π° заклинивания лопастСй. Π˜Π·ΠΎΠ»ΡΡ†ΠΈΡ ΠΎΠ±ΠΌΠΎΡ‚ΠΎΠΊ двигатСля Π½Π΅ рассчитана Π½Π° ΡΠ΅Ρ€ΡŒΠ΅Π·Π½Ρ‹ΠΉ Π½Π°Π³Ρ€Π΅Π², ΠΎΠ½Π° быстро ΠΏΡ€ΠΈΡ…ΠΎΠ΄ΠΈΡ‚ Π² Π½Π΅Π³ΠΎΠ΄Π½ΠΎΡΡ‚ΡŒ. Π’ Ρ€Π΅Π·ΡƒΠ»ΡŒΡ‚Π°Ρ‚Π΅ ΠΏΠΎΡΠ²Π»ΡΡŽΡ‚ΡΡ ΠΌΠ΅ΠΆΠ²ΠΈΡ‚ΠΊΠΎΠ²Ρ‹Π΅ ΠΊΠΎΡ€ΠΎΡ‚ΠΊΠΈΠ΅ замыкания, ΠΊΠΎΡ‚ΠΎΡ€Ρ‹Π΅ ΡΠ½ΠΈΠΆΠ°ΡŽΡ‚ сопротивлСниС ΠΈ, Π² соотвСтствии с Π·Π°ΠΊΠΎΠ½ΠΎΠΌ Ома, Ρ‚Π°ΠΊΠΆΠ΅ Π²Π΅Π΄ΡƒΡ‚ ΠΊ ΡƒΠ²Π΅Π»ΠΈΡ‡Π΅Π½ΠΈΡŽ Ρ‚ΠΎΠΊΠ°.

ΠŸΠΎΠ²Ρ‹ΡˆΠ΅Π½Π½Ρ‹ΠΉ Ρ‚ΠΎΠΊ, Π² свою ΠΎΡ‡Π΅Ρ€Π΅Π΄ΡŒ, ΠΏΡ€ΠΈΠ²ΠΎΠ΄ΠΈΡ‚ ΠΈΠ·ΠΎΠ»ΡΡ†ΠΈΡŽ ΠΎΠ±ΠΌΠΎΡ‚ΠΎΠΊ Π² ΠΏΠΎΠ»Π½ΡƒΡŽ Π½Π΅Π³ΠΎΠ΄Π½ΠΎΡΡ‚ΡŒ, ΠΈ наступаСт ΡƒΠΆΠ΅ Π½Π΅ ΠΌΠ΅ΠΆΠ²ΠΈΡ‚ΠΊΠΎΠ²ΠΎΠ΅, Π° самоС настоящСС, ΠΏΠΎΠ»Π½ΠΎΡ†Π΅Π½Π½ΠΎΠ΅ ΠΊΠΎΡ€ΠΎΡ‚ΠΊΠΎΠ΅ Π·Π°ΠΌΡ‹ΠΊΠ°Π½ΠΈΠ΅. Π’ΠΎΠΊ ΠΈΠ΄Π΅Ρ‚ ΠΏΠΎΠΌΠΈΠΌΠΎ ΠΎΠ±ΠΌΠΎΡ‚ΠΎΠΊ, сразу ΠΈΠ· Ρ„Π°Π·Π½ΠΎΠ³ΠΎ Π² Π½ΡƒΠ»Π΅Π²ΠΎΠΉ ΠΏΡ€ΠΎΠ²ΠΎΠ΄. ΠŸΡ€Π°Π²Π΄Π°, всС сказанноС ΠΌΠΎΠΆΠ΅Ρ‚ ΡΠ»ΡƒΡ‡ΠΈΡ‚ΡŒΡΡ Ρ‚ΠΎΠ»ΡŒΠΊΠΎ с совсСм простым ΠΈ Π΄Π΅ΡˆΠ΅Π²Ρ‹ΠΌ вСнтилятором, Π½Π΅ ΠΎΠ±ΠΎΡ€ΡƒΠ΄ΠΎΠ²Π°Π½Π½Ρ‹ΠΌ Ρ‚Π΅ΠΏΠ»ΠΎΠ²ΠΎΠΉ Π·Π°Ρ‰ΠΈΡ‚ΠΎΠΉ.

Π—Π°ΠΊΠΎΠ½ Ома для ΠΏΠ΅Ρ€Π΅ΠΌΠ΅Π½Π½ΠΎΠ³ΠΎ Ρ‚ΠΎΠΊΠ°

Надо ΠΎΡ‚ΠΌΠ΅Ρ‚ΠΈΡ‚ΡŒ, Ρ‡Ρ‚ΠΎ привСдСнная запись Π·Π°ΠΊΠΎΠ½Π° Ома описываСт участок Ρ†Π΅ΠΏΠΈ с постоянным напряТСниСм. Π’ сСтях ΠΏΠ΅Ρ€Π΅ΠΌΠ΅Π½Π½ΠΎΠ³ΠΎ напряТСния сущСствуСт Π΄ΠΎΠΏΠΎΠ»Π½ΠΈΡ‚Π΅Π»ΡŒΠ½ΠΎΠ΅ Ρ€Π΅Π°ΠΊΡ‚ΠΈΠ²Π½ΠΎΠ΅ сопротивлСниС, Π° ΠΏΠΎΠ»Π½ΠΎΠ΅ сопротивлСниС ΠΏΡ€ΠΈΠΎΠ±Ρ€Π΅Ρ‚Π°Π΅Ρ‚ Π·Π½Π°Ρ‡Π΅Π½ΠΈΠ΅ ΠΊΠ²Π°Π΄Ρ€Π°Ρ‚Π½ΠΎΠ³ΠΎ корня ΠΈΠ· суммы ΠΊΠ²Π°Π΄Ρ€Π°Ρ‚ΠΎΠ² Π°ΠΊΡ‚ΠΈΠ²Π½ΠΎΠ³ΠΎ ΠΈ Ρ€Π΅Π°ΠΊΡ‚ΠΈΠ²Π½ΠΎΠ³ΠΎ сопротивлСния.

Π—Π°ΠΊΠΎΠ½ Ома для участка Ρ†Π΅ΠΏΠΈ ΠΏΠ΅Ρ€Π΅ΠΌΠ΅Π½Π½ΠΎΠ³ΠΎ Ρ‚ΠΎΠΊΠ° ΠΏΡ€ΠΈΠ½ΠΈΠΌΠ°Π΅Ρ‚ Π²ΠΈΠ΄: I=U/Z
,

Π³Π΄Π΅ Z — ΠΏΠΎΠ»Π½ΠΎΠ΅ сопротивлСниС Ρ†Π΅ΠΏΠΈ.

Но большоС Ρ€Π΅Π°ΠΊΡ‚ΠΈΠ²Π½ΠΎΠ΅ сопротивлСниС свойствСнно, ΠΏΡ€Π΅ΠΆΠ΄Π΅ всСго, ΠΌΠΎΡ‰Π½Ρ‹ΠΌ элСктричСским машинам ΠΈ силовой ΠΏΡ€Π΅ΠΎΠ±Ρ€Π°Π·ΠΎΠ²Π°Ρ‚Π΅Π»ΡŒΠ½ΠΎΠΉ Ρ‚Π΅Ρ…Π½ΠΈΠΊΠ΅. Π’Π½ΡƒΡ‚Ρ€Π΅Π½Π½Π΅Π΅ элСктричСскоС сопротивлСниС Π±Ρ‹Ρ‚ΠΎΠ²Ρ‹Ρ… ΠΏΡ€ΠΈΠ±ΠΎΡ€ΠΎΠ² ΠΈ ΡΠ²Π΅Ρ‚ΠΈΠ»ΡŒΠ½ΠΈΠΊΠΎΠ² практичСски ΠΏΠΎΠ»Π½ΠΎΡΡ‚ΡŒΡŽ являСтся Π°ΠΊΡ‚ΠΈΠ²Π½Ρ‹ΠΌ. ΠŸΠΎΡΡ‚ΠΎΠΌΡƒ Π² Π±Ρ‹Ρ‚Ρƒ для расчСтов ΠΌΠΎΠΆΠ½ΠΎ ΠΏΠΎΠ»ΡŒΠ·ΠΎΠ²Π°Ρ‚ΡŒΡΡ самой простой Ρ„ΠΎΡ€ΠΌΠΎΠΉ записи Π·Π°ΠΊΠΎΠ½Π° Ома: I=U/R.

3. Π˜Π½Ρ‚Π΅Π³Ρ€Π°Π»ΡŒΠ½Π°Ρ Ρ„ΠΎΡ€ΠΌΠ° записи для ΠΏΠΎΠ»Π½ΠΎΠΉ Ρ†Π΅ΠΏΠΈ

Π Π°Π· Π΅ΡΡ‚ΡŒ Ρ„ΠΎΡ€ΠΌΠ° записи Π·Π°ΠΊΠΎΠ½Π° для участка Ρ†Π΅ΠΏΠΈ, Ρ‚ΠΎ сущСствуСт ΠΈ Π·Π°ΠΊΠΎΠ½ Ома для ΠΏΠΎΠ»Π½ΠΎΠΉ Ρ†Π΅ΠΏΠΈ: I=E/(r+R)
.

Π—Π΄Π΅ΡΡŒ r — Π²Π½ΡƒΡ‚Ρ€Π΅Π½Π½Π΅Π΅ сопротивлСниС источника Π­Π”Π‘ сСти, Π° R — ΠΏΠΎΠ»Π½ΠΎΠ΅ сопротивлСниС самой Ρ†Π΅ΠΏΠΈ.

Π—Π° физичСской модСлью для ΠΈΠ»Π»ΡŽΡΡ‚Ρ€Π°Ρ†ΠΈΠΈ этого ΠΏΠΎΠ΄Π²ΠΈΠ΄Π° Π·Π°ΠΊΠΎΠ½Π° Ома Π΄Π°Π»Π΅ΠΊΠΎ Ρ…ΠΎΠ΄ΠΈΡ‚ΡŒ Π½Π΅ Π½Π°Π΄ΠΎ — это бортовая элСктричСская ΡΠ΅Ρ‚ΡŒ автомобиля, аккумулятор Π² ΠΊΠΎΡ‚ΠΎΡ€ΠΎΠΉ являСтся источником Π­Π”Π‘. НСльзя ΡΡ‡ΠΈΡ‚Π°Ρ‚ΡŒ, Ρ‡Ρ‚ΠΎ сопротивлСниС аккумулятора Ρ€Π°Π²Π½ΠΎ Π°Π±ΡΠΎΠ»ΡŽΡ‚Π½ΠΎΠΌΡƒ Π½ΡƒΠ»ΡŽ, поэтому Π΄Π°ΠΆΠ΅ ΠΏΡ€ΠΈ прямом Π·Π°ΠΌΡ‹ΠΊΠ°Π½ΠΈΠΈ ΠΌΠ΅ΠΆΠ΄Ρƒ Π΅Π³ΠΎ ΠΊΠ»Π΅ΠΌΠΌΠ°ΠΌΠΈ (отсутствии сопротивлСния R) Ρ‚ΠΎΠΊ вырастСт Π½Π΅ Π΄ΠΎ бСсконСчности, Π° просто Π΄ΠΎ высокого значСния. Однако этого высокого значСния, ΠΊΠΎΠ½Π΅Ρ‡Π½ΠΎ, Ρ…Π²Π°Ρ‚ΠΈΡ‚ для Ρ‚ΠΎΠ³ΠΎ, Ρ‡Ρ‚ΠΎΠ±Ρ‹ Π²Ρ‹Π·Π²Π°Ρ‚ΡŒ расплавлСниС ΠΏΡ€ΠΎΠ²ΠΎΠ΄ΠΎΠ² ΠΈ Π²ΠΎΠ·Π³ΠΎΡ€Π°Π½ΠΈΠ΅ обшивки Π°Π²Ρ‚ΠΎ. ΠŸΠΎΡΡ‚ΠΎΠΌΡƒ элСктричСскиС Ρ†Π΅ΠΏΠΈ Π°Π²Ρ‚ΠΎΠΌΠΎΠ±ΠΈΠ»Π΅ΠΉ Π·Π°Ρ‰ΠΈΡ‰Π°ΡŽΡ‚ ΠΎΡ‚ ΠΊΠΎΡ€ΠΎΡ‚ΠΊΠΎΠ³ΠΎ замыкания ΠΏΡ€ΠΈ ΠΏΠΎΠΌΠΎΡ‰ΠΈ ΠΏΡ€Π΅Π΄ΠΎΡ…Ρ€Π°Π½ΠΈΡ‚Π΅Π»Π΅ΠΉ.

Π’Π°ΠΊΠΎΠΉ Π·Π°Ρ‰ΠΈΡ‚Ρ‹ ΠΌΠΎΠΆΠ΅Ρ‚ ΠΎΠΊΠ°Π·Π°Ρ‚ΡŒΡΡ нСдостаточно, Ссли Π·Π°ΠΌΡ‹ΠΊΠ°Π½ΠΈΠ΅ ΠΏΡ€ΠΎΠΈΠ·ΠΎΠΉΠ΄Π΅Ρ‚ Π΄ΠΎ Π±Π»ΠΎΠΊΠ° ΠΏΡ€Π΅Π΄ΠΎΡ…Ρ€Π°Π½ΠΈΡ‚Π΅Π»Π΅ΠΉ ΠΎΡ‚Π½ΠΎΡΠΈΡ‚Π΅Π»ΡŒΠ½ΠΎ аккумулятора, ΠΈΠ»ΠΈ Ссли вовсС ΠΎΠ΄ΠΈΠ½ ΠΈΠ· ΠΏΡ€Π΅Π΄ΠΎΡ…Ρ€Π°Π½ΠΈΡ‚Π΅Π»Π΅ΠΉ Π·Π°ΠΌΠ΅Π½Π΅Π½ Π½Π° кусок ΠΌΠ΅Π΄Π½ΠΎΠΉ ΠΏΡ€ΠΎΠ²ΠΎΠ»ΠΎΠΊΠΈ. Π’ΠΎΠ³Π΄Π° спасСниС Ρ‚ΠΎΠ»ΡŒΠΊΠΎ Π² ΠΎΠ΄Π½ΠΎΠΌ — Π½Π΅ΠΎΠ±Ρ…ΠΎΠ΄ΠΈΠΌΠΎ ΠΊΠ°ΠΊ ΠΌΠΎΠΆΠ½ΠΎ быстрСС Ρ€Π°Π·ΠΎΡ€Π²Π°Ρ‚ΡŒ Ρ†Π΅ΠΏΡŒ ΠΏΠΎΠ»Π½ΠΎΡΡ‚ΡŒΡŽ, ΠΎΡ‚ΠΊΠΈΠ½ΡƒΠ² «массу», Ρ‚ΠΎ Π΅ΡΡ‚ΡŒ ΠΌΠΈΠ½ΡƒΡΠΎΠ²ΡƒΡŽ ΠΊΠ»Π΅ΠΌΠΌΡƒ.

4. Π˜Π½Ρ‚Π΅Π³Ρ€Π°Π»ΡŒΠ½Π°Ρ Ρ„ΠΎΡ€ΠΌΠ° записи Π·Π°ΠΊΠΎΠ½Π° Ома для участка Ρ†Π΅ΠΏΠΈ, содСрТащСго источник Π­Π”Π‘

Π‘Π»Π΅Π΄ΡƒΠ΅Ρ‚ ΡƒΠΏΠΎΠΌΡΠ½ΡƒΡ‚ΡŒ ΠΈ ΠΎ Ρ‚ΠΎΠΌ, Ρ‡Ρ‚ΠΎ Π΅ΡΡ‚ΡŒ ΠΈ Π΅Ρ‰Π΅ ΠΎΠ΄Π½Π° Ρ€Π°Π·Π½ΠΎΠ²ΠΈΠ΄Π½ΠΎΡΡ‚ΡŒ Π·Π°ΠΊΠΎΠ½Π° Ома — для участка Ρ†Π΅ΠΏΠΈ, содСрТащСго источник Π­Π”Π‘:

Π—Π΄Π΅ΡΡŒ U — это Ρ€Π°Π·Π½ΠΎΡΡ‚ΡŒ ΠΏΠΎΡ‚Π΅Π½Ρ†ΠΈΠ°Π»ΠΎΠ² Π² Π½Π°Ρ‡Π°Π»Π΅ ΠΈ Π² ΠΎΠΊΠΎΠ½Ρ‡Π°Π½ΠΈΠΈ рассматриваСмого участка Ρ†Π΅ΠΏΠΈ. Π—Π½Π°ΠΊ ΠΏΠ΅Ρ€Π΅Π΄ Π²Π΅Π»ΠΈΡ‡ΠΈΠ½ΠΎΠΉ Π­Π”Π‘ зависит ΠΎΡ‚ направлСнности Π΅Π΅ ΠΎΡ‚Π½ΠΎΡΠΈΡ‚Π΅Π»ΡŒΠ½ΠΎ напряТСния. Π’ΠΎΡΠΏΠΎΠ»ΡŒΠ·ΠΎΠ²Π°Ρ‚ΡŒΡΡ Π·Π°ΠΊΠΎΠ½ΠΎΠΌ Ома для участка Ρ†Π΅ΠΏΠΈ Π½Π΅Ρ€Π΅Π΄ΠΊΠΎ приходится ΠΏΡ€ΠΈ ΠΎΠΏΡ€Π΅Π΄Π΅Π»Π΅Π½ΠΈΠΈ ΠΏΠ°Ρ€Π°ΠΌΠ΅Ρ‚Ρ€ΠΎΠ² Ρ†Π΅ΠΏΠΈ, ΠΊΠΎΠ³Π΄Π° Ρ‡Π°ΡΡ‚ΡŒ схСмы нСдоступна для Π΄Π΅Ρ‚Π°Π»ΡŒΠ½ΠΎΠ³ΠΎ изучСния ΠΈ Π½Π΅ интСрСсуСт нас. Допустим, ΠΎΠ½Π° скрыта Π½Π΅Ρ€Π°Π·ΡŠΠ΅ΠΌΠ½Ρ‹ΠΌΠΈ дСталями корпуса. Π’ ΠΎΡΡ‚Π°Π²ΡˆΠ΅ΠΉΡΡ схСмС имССтся источник Π­Π”Π‘ ΠΈ элСмСнты с извСстным сопротивлСниСм. Π’ΠΎΠ³Π΄Π°, Π·Π°ΠΌΠ΅Ρ€ΠΈΠ² напряТСниС Π½Π° Π²Ρ…ΠΎΠ΄Π΅ нСизвСстной части схСмы, ΠΌΠΎΠΆΠ½ΠΎ Π²Ρ‹Ρ‡ΠΈΡΠ»ΠΈΡ‚ΡŒ Ρ‚ΠΎΠΊ, Π° послС этого — ΠΈ сопротивлСниС нСизвСстного элСмСнта.

Π’Ρ‹Π²ΠΎΠ΄Ρ‹

Π’Π°ΠΊΠΈΠΌ ΠΎΠ±Ρ€Π°Π·ΠΎΠΌ, ΠΌΡ‹ ΠΌΠΎΠΆΠ΅ΠΌ ΡƒΠ²ΠΈΠ΄Π΅Ρ‚ΡŒ, Ρ‡Ρ‚ΠΎ «простой» Π·Π°ΠΊΠΎΠ½ Ома Π΄Π°Π»Π΅ΠΊΠΎ Π½Π΅ Ρ‚Π°ΠΊ прост, ΠΊΠ°ΠΊ ΠΊΠΎΠΌΡƒ-Ρ‚ΠΎ, Π²ΠΎΠ·ΠΌΠΎΠΆΠ½ΠΎ, казалось. Зная всС Ρ„ΠΎΡ€ΠΌΡ‹ ΠΈΠ½Ρ‚Π΅Π³Ρ€Π°Π»ΡŒΠ½ΠΎΠΉ записи Π·Π°ΠΊΠΎΠ½ΠΎΠ² Ома, ΠΌΠΎΠΆΠ½ΠΎ ΠΏΠΎΠ½ΡΡ‚ΡŒ ΠΈ Π»Π΅Π³ΠΊΠΎ Π·Π°ΠΏΠΎΠΌΠ½ΠΈΡ‚ΡŒ ΠΌΠ½ΠΎΠ³ΠΈΠ΅ трСбования элСктробСзопасности, Π° Ρ‚Π°ΠΊΠΆΠ΅ приобрСсти ΡƒΠ²Π΅Ρ€Π΅Π½Π½ΠΎΡΡ‚ΡŒ Π² ΠΎΠ±Ρ€Π°Ρ‰Π΅Π½ΠΈΠΈ с элСктричСством.

ЀизичСский Π·Π°ΠΊΠΎΠ½ , ΠΎΠΏΡ€Π΅Π΄Π΅Π»ΡΡŽΡ‰ΠΈΠΉ связь (ΠΈΠ»ΠΈ элСктричСского напряТСния) с силой тока , ΠΏΡ€ΠΎΡ‚Π΅ΠΊΠ°ΡŽΡ‰Π΅Π³ΠΎ Π² ΠΏΡ€ΠΎΠ²ΠΎΠ΄Π½ΠΈΠΊΠ΅ , ΠΈ сопротивлСниСм ΠΏΡ€ΠΎΠ²ΠΎΠ΄Π½ΠΈΠΊΠ°. УстановлСн Π“Π΅ΠΎΡ€Π³ΠΎΠΌβ€…ΠžΠΌΠΎΠΌ Π² 1826β€…Π³ΠΎΠ΄Ρƒ ΠΈ Π½Π°Π·Π²Π°Π½ Π² Π΅Π³ΠΎ Ρ‡Π΅ΡΡ‚ΡŒ.

Π—Π°ΠΊΠΎΠ½ Ома для ΠΏΠ΅Ρ€Π΅ΠΌΠ΅Π½Π½ΠΎΠ³ΠΎ Ρ‚ΠΎΠΊΠ°

Π’Ρ‹ΡˆΠ΅ΠΏΡ€ΠΈΠ²Π΅Π΄Ρ‘Π½Π½Ρ‹Π΅ сообраТСния ΠΎ свойствах элСктричСской Ρ†Π΅ΠΏΠΈ ΠΏΡ€ΠΈ использовании источника (Π³Π΅Π½Π΅Ρ€Π°Ρ‚ΠΎΡ€Π°) с ΠΏΠ΅Ρ€Π΅ΠΌΠ΅Π½Π½ΠΎΠΉ Π²ΠΎ Π²Ρ€Π΅ΠΌΠ΅Π½ΠΈ Π­Π”Π‘ ΠΎΡΡ‚Π°ΡŽΡ‚ΡΡ справСдливыми. Π‘ΠΏΠ΅Ρ†ΠΈΠ°Π»ΡŒΠ½ΠΎΠΌΡƒ Ρ€Π°ΡΡΠΌΠΎΡ‚Ρ€Π΅Π½ΠΈΡŽ ΠΏΠΎΠ΄Π»Π΅ΠΆΠΈΡ‚ лишь ΡƒΡ‡Ρ‘Ρ‚ спСцифичСских свойств потрСбитСля, приводящих ΠΊ разноврСмённости достиТСния напряТСниСм ΠΈ Ρ‚ΠΎΠΊΠΎΠΌ своих ΠΌΠ°ΠΊΡΠΈΠΌΠ°Π»ΡŒΠ½Ρ‹Ρ… Π·Π½Π°Ρ‡Π΅Π½ΠΈΠΉ, Ρ‚ΠΎ Π΅ΡΡ‚ΡŒ ΡƒΡ‡Ρ‘Ρ‚Π° фазового сдвига .

Если Ρ‚ΠΎΠΊ являСтся ΡΠΈΠ½ΡƒΡΠΎΠΈΠ΄Π°Π»ΡŒΠ½Ρ‹ΠΌ с цикличСской частотой
Ο‰
{\displaystyle \omega }

, Π° Ρ†Π΅ΠΏΡŒ содСрТит Π½Π΅ Ρ‚ΠΎΠ»ΡŒΠΊΠΎ Π°ΠΊΡ‚ΠΈΠ²Π½Ρ‹Π΅, Π½ΠΎ ΠΈ Ρ€Π΅Π°ΠΊΡ‚ΠΈΠ²Π½Ρ‹Π΅ ΠΊΠΎΠΌΠΏΠΎΠ½Π΅Π½Ρ‚Ρ‹ (ёмкости , индуктивности), Ρ‚ΠΎ Π·Π°ΠΊΠΎΠ½ Ома обобщаСтся; Π²Π΅Π»ΠΈΡ‡ΠΈΠ½Ρ‹, входящиС Π² Π½Π΅Π³ΠΎ, становятся комплСксными:

U
=
I
β‹…
Z
{\displaystyle \mathbb {U} =\mathbb {I} \cdot Z}

  • U = U
    0 e
    i
    Ο‰t

    — напряТСниС ΠΈΠ»ΠΈ Ρ€Π°Π·Π½ΠΎΡΡ‚ΡŒ ΠΏΠΎΡ‚Π΅Π½Ρ†ΠΈΠ°Π»ΠΎΠ²,
  • I

    — сила Ρ‚ΠΎΠΊΠ°,
  • Z = Re
    βˆ’i
    Ξ΄
    — комплСксноС сопротивлСниС (элСктричСский импСданс),
  • R
    = √ R a
    2 + R r
    2

    — ΠΏΠΎΠ»Π½ΠΎΠ΅ сопротивлСниС,

  • R r
    = Ο‰L
    βˆ’ 1/(Ο‰C
    )
    — Ρ€Π΅Π°ΠΊΡ‚ΠΈΠ²Π½ΠΎΠ΅ сопротивлСниС (Ρ€Π°Π·Π½ΠΎΡΡ‚ΡŒ ΠΈΠ½Π΄ΡƒΠΊΡ‚ΠΈΠ²Π½ΠΎΠ³ΠΎ ΠΈ Смкостного),
  • R Π°

    — Π°ΠΊΡ‚ΠΈΠ²Π½ΠΎΠ΅ (омичСскоС) сопротивлСниС, Π½Π΅ зависящСС ΠΎΡ‚ частоты,
  • Ξ΄ = βˆ’ arctg (R r
    /R a
    )
    — сдвиг Ρ„Π°Π· ΠΌΠ΅ΠΆΠ΄Ρƒ напряТСниСм ΠΈ силой Ρ‚ΠΎΠΊΠ°. {i(\omega t+\varphi)},}
    Ρ‡Ρ‚ΠΎ
    Im
    ⁑
    U
    =
    U
    .
    {\displaystyle \operatorname {Im} \mathbb {U} =U.}
    Π’ΠΎΠ³Π΄Π° всС значСния Ρ‚ΠΎΠΊΠΎΠ² ΠΈ напряТСний Π² схСмС Π½Π°Π΄ΠΎ ΡΡ‡ΠΈΡ‚Π°Ρ‚ΡŒ ΠΊΠ°ΠΊ
    F
    =
    Im
    ⁑
    F
    {\displaystyle F=\operatorname {Im} \mathbb {F} }

    ЗдравствуйтС, ΡƒΠ²Π°ΠΆΠ°Π΅ΠΌΡ‹Π΅ Ρ‡ΠΈΡ‚Π°Ρ‚Π΅Π»ΠΈ сайта Β«Π—Π°ΠΌΠ΅Ρ‚ΠΊΠΈ элСктрика»..

    БСгодня ΠΎΡ‚ΠΊΡ€Ρ‹Π²Π°ΡŽ Π½ΠΎΠ²Ρ‹ΠΉ Ρ€Π°Π·Π΄Π΅Π» Π½Π° сайтС ΠΏΠΎΠ΄ Π½Π°Π·Π²Π°Π½ΠΈΠ΅ΠΌ .

    Π’ этом Ρ€Π°Π·Π΄Π΅Π»Π΅ я ΠΏΠΎΡΡ‚Π°Ρ€Π°ΡŽΡΡŒ Π² наглядной ΠΈ простой Ρ„ΠΎΡ€ΠΌΠ΅ ΠΎΠ±ΡŠΡΡΠ½ΠΈΡ‚ΡŒ Π’Π°ΠΌ вопросы элСктротСхники. Π‘ΠΊΠ°ΠΆΡƒ сразу, Ρ‡Ρ‚ΠΎ Π΄Π°Π»Π΅ΠΊΠΎ ΡƒΠ³Π»ΡƒΠ±Π»ΡΡ‚ΡŒΡΡ Π² тСорСтичСскиС знания ΠΌΡ‹ Π½Π΅ Π±ΡƒΠ΄Π΅ΠΌ, Π½ΠΎ Π²ΠΎΡ‚ с основами познакомимся Π² достаточном порядкС.

    ΠŸΠ΅Ρ€Π²ΠΎΠ΅, с Ρ‡Π΅ΠΌ я Ρ…ΠΎΡ‡Ρƒ Вас ΠΏΠΎΠ·Π½Π°ΠΊΠΎΠΌΠΈΡ‚ΡŒ, это с Π·Π°ΠΊΠΎΠ½ΠΎΠΌ Ома для участка Ρ†Π΅ΠΏΠΈ. Π­Ρ‚ΠΎ самый основной Π·Π°ΠΊΠΎΠ½, ΠΊΠΎΡ‚ΠΎΡ€Ρ‹ΠΉ Π΄ΠΎΠ»ΠΆΠ΅Π½ Π·Π½Π°Ρ‚ΡŒ ΠΊΠ°ΠΆΠ΄Ρ‹ΠΉ .

    Π—Π½Π°Π½ΠΈΠ΅ этого Π·Π°ΠΊΠΎΠ½Π° ΠΏΠΎΠ·Π²ΠΎΠ»ΠΈΡ‚ Π½Π°ΠΌ бСспрСпятствСнно ΠΈ Π±Π΅Π·ΠΎΡˆΠΈΠ±ΠΎΡ‡Π½ΠΎ ΠΎΠΏΡ€Π΅Π΄Π΅Π»ΡΡ‚ΡŒ значСния силы Ρ‚ΠΎΠΊΠ°, напряТСния (разности ΠΏΠΎΡ‚Π΅Π½Ρ†ΠΈΠ°Π»ΠΎΠ²) ΠΈ сопротивлСния Π½Π° участкС Ρ†Π΅ΠΏΠΈ.

    ΠšΡ‚ΠΎ Ρ‚Π°ΠΊΠΎΠΉ Ом? НСмного истории

    Π—Π°ΠΊΠΎΠ½ Ома ΠΎΡ‚ΠΊΡ€Ρ‹Π» всСм извСстный Π½Π΅ΠΌΠ΅Ρ†ΠΊΠΈΠΉ Ρ„ΠΈΠ·ΠΈΠΊ Π“Π΅ΠΎΡ€Π³ Π‘ΠΈΠΌΠΎΠ½ Ом Π² 1826 Π³ΠΎΠ΄Ρƒ. Π’ΠΎΡ‚ Ρ‚Π°ΠΊ ΠΎΠ½ выглядСл.

    Π’ΡΡŽ Π±ΠΈΠΎΠ³Ρ€Π°Ρ„ΠΈΡŽ Π“Π΅ΠΎΡ€Π³Π° Ома я Ρ€Π°ΡΡΠΊΠ°Π·Ρ‹Π²Π°Ρ‚ΡŒ Π’Π°ΠΌ Π½Π΅ Π±ΡƒΠ΄Ρƒ. ΠŸΡ€ΠΎ это Π’Ρ‹ ΠΌΠΎΠΆΠ΅Ρ‚Π΅ ΡƒΠ·Π½Π°Ρ‚ΡŒ Π½Π° Π΄Ρ€ΡƒΠ³ΠΈΡ… рСсурсах Π±ΠΎΠ»Π΅Π΅ ΠΏΠΎΠ΄Ρ€ΠΎΠ±Π½ΠΎ.

    Π‘ΠΊΠ°ΠΆΡƒ Ρ‚ΠΎΠ»ΡŒΠΊΠΎ самоС Π³Π»Π°Π²Π½ΠΎΠ΅.

    Π•Π³ΠΎ ΠΈΠΌΠ΅Π½Π΅ΠΌ Π½Π°Π·Π²Π°Π½ самый основной Π·Π°ΠΊΠΎΠ½ элСктротСхники, ΠΊΠΎΡ‚ΠΎΡ€Ρ‹ΠΉ ΠΌΡ‹ Π°ΠΊΡ‚ΠΈΠ²Π½ΠΎ примСняСм Π² слоТных расчСтах ΠΏΡ€ΠΈ ΠΏΡ€ΠΎΠ΅ΠΊΡ‚ΠΈΡ€ΠΎΠ²Π°Π½ΠΈΠΈ, Π½Π° производствС ΠΈ Π² Π±Ρ‹Ρ‚Ρƒ.

    Π—Π°ΠΊΠΎΠ½ Ома для ΠΎΠ΄Π½ΠΎΡ€ΠΎΠ΄Π½ΠΎΠ³ΠΎ участка Ρ†Π΅ΠΏΠΈ выглядит ΡΠ»Π΅Π΄ΡƒΡŽΡ‰ΠΈΠΌ ΠΎΠ±Ρ€Π°Π·ΠΎΠΌ:

    I – Π·Π½Π°Ρ‡Π΅Π½ΠΈΠ΅ Ρ‚ΠΎΠΊΠ°, ΠΈΠ΄ΡƒΡ‰Π΅Π³ΠΎ Ρ‡Π΅Ρ€Π΅Π· участок Ρ†Π΅ΠΏΠΈ (измСряСтся Π² Π°ΠΌΠΏΠ΅Ρ€Π°Ρ…)

    U – Π·Π½Π°Ρ‡Π΅Π½ΠΈΠ΅ напряТСния Π½Π° участкС Ρ†Π΅ΠΏΠΈ (измСряСтся Π² Π²ΠΎΠ»ΡŒΡ‚Π°Ρ…)

    R – Π·Π½Π°Ρ‡Π΅Π½ΠΈΠ΅ сопротивлСния участка Ρ†Π΅ΠΏΠΈ (измСряСтся Π² ΠžΠΌΠ°Ρ…)

    Если Ρ„ΠΎΡ€ΠΌΡƒΠ»Ρƒ ΠΎΠ±ΡŠΡΡΠ½ΠΈΡ‚ΡŒ словами, Ρ‚ΠΎ получится, Ρ‡Ρ‚ΠΎ сила Ρ‚ΠΎΠΊΠ° ΠΏΡ€ΠΎΠΏΠΎΡ€Ρ†ΠΈΠΎΠ½Π°Π»ΡŒΠ½Π°Ρ Π½Π°ΠΏΡ€ΡΠΆΠ΅Π½ΠΈΡŽ ΠΈ ΠΎΠ±Ρ€Π°Ρ‚Π½ΠΎ ΠΏΡ€ΠΎΠΏΠΎΡ€Ρ†ΠΈΠΎΠ½Π°Π»ΡŒΠ½Π° ΡΠΎΠΏΡ€ΠΎΡ‚ΠΈΠ²Π»Π΅Π½ΠΈΡŽ участка Ρ†Π΅ΠΏΠΈ.

    ΠŸΡ€ΠΎΠ²Π΅Π΄Π΅ΠΌ экспСримСнт

    Π§Ρ‚ΠΎΠ±Ρ‹ ΠΏΠΎΠ½ΡΡ‚ΡŒ Ρ„ΠΎΡ€ΠΌΡƒΠ»Ρƒ Π½Π΅ Π½Π° словах, Π° Π½Π° Π΄Π΅Π»Π΅, Π½Π΅ΠΎΠ±Ρ…ΠΎΠ΄ΠΈΠΌΠΎ ΡΠΎΠ±Ρ€Π°Ρ‚ΡŒ ΡΠ»Π΅Π΄ΡƒΡŽΡ‰ΡƒΡŽ схСму:

    ЦСль этой ΡΡ‚Π°Ρ‚ΡŒΠΈ β€” это ΠΏΠΎΠΊΠ°Π·Π°Ρ‚ΡŒ наглядно, ΠΊΠ°ΠΊ ΠΈΡΠΏΠΎΠ»ΡŒΠ·ΠΎΠ²Π°Ρ‚ΡŒ Π·Π°ΠΊΠΎΠ½ Ома для участка Ρ†Π΅ΠΏΠΈ. ΠŸΠΎΡΡ‚ΠΎΠΌΡƒ я Π½Π° своСм Ρ€Π°Π±ΠΎΡ‡Π΅ΠΌ стСндС собрал эту схСму. Π‘ΠΌΠΎΡ‚Ρ€ΠΈΡ‚Π΅ Π½ΠΈΠΆΠ΅ ΠΊΠ°ΠΊ ΠΎΠ½Π° выглядит.

    Π‘ ΠΏΠΎΠΌΠΎΡ‰ΡŒΡŽ ΠΊΠ»ΡŽΡ‡Π° управлСния (избирания) ΠΌΠΎΠΆΠ½ΠΎ Π²Ρ‹Π±Ρ€Π°Ρ‚ΡŒ, Π»ΠΈΠ±ΠΎ постоянноС напряТСниС, Π»ΠΈΠ±ΠΎ ΠΏΠ΅Ρ€Π΅ΠΌΠ΅Π½Π½ΠΎΠ΅ напряТСниС Π½Π° Π²Ρ‹Ρ…ΠΎΠ΄Π΅. Π’ нашСм случаС ΠΈΡΠΏΠΎΠ»ΡŒΠ·ΡƒΠ΅Ρ‚ΡΡ постоянноС напряТСния. Π£Ρ€ΠΎΠ²Π΅Π½ΡŒ напряТСния я мСняю с ΠΏΠΎΠΌΠΎΡ‰ΡŒΡŽ Π»Π°Π±ΠΎΡ€Π°Ρ‚ΠΎΡ€Π½ΠΎΠ³ΠΎ автотрансформатора (ЛАВР).

    Π’ нашСм экспСримСнтС я Π±ΡƒΠ΄Ρƒ ΠΈΡΠΏΠΎΠ»ΡŒΠ·ΠΎΠ²Π°Ρ‚ΡŒ напряТСниС Π½Π° участкС Ρ†Π΅ΠΏΠΈ, Ρ€Π°Π²Π½ΠΎΠ΅ 220 (Π’). ΠšΠΎΠ½Ρ‚Ρ€ΠΎΠ»ΡŒ напряТСния Π½Π° Π²Ρ‹Ρ…ΠΎΠ΄Π΅ смотрим ΠΏΠΎ Π²ΠΎΠ»ΡŒΡ‚ΠΌΠ΅Ρ‚Ρ€Ρƒ.

    Π’Π΅ΠΏΠ΅Ρ€ΡŒ ΠΌΡ‹ ΠΏΠΎΠ»Π½ΠΎΡΡ‚ΡŒΡŽ Π³ΠΎΡ‚ΠΎΠ²Ρ‹ провСсти ΡΠ°ΠΌΠΎΡΡ‚ΠΎΡΡ‚Π΅Π»ΡŒΠ½ΠΎ экспСримСнт ΠΈ ΠΏΡ€ΠΎΠ²Π΅Ρ€ΠΈΡ‚ΡŒ Π·Π°ΠΊΠΎΠ½ Ома Π² Π΄Π΅ΠΉΡΡ‚Π²ΠΈΡ‚Π΅Π»ΡŒΠ½ΠΎΡΡ‚ΠΈ.

    НиТС я ΠΏΡ€ΠΈΠ²Π΅Π΄Ρƒ 3 ΠΏΡ€ΠΈΠΌΠ΅Ρ€Π°. Π’ ΠΊΠ°ΠΆΠ΄ΠΎΠΌ ΠΏΡ€ΠΈΠΌΠ΅Ρ€Π΅ ΠΌΡ‹ Π±ΡƒΠ΄Π΅ΠΌ ΠΎΠΏΡ€Π΅Π΄Π΅Π»ΡΡ‚ΡŒ ΠΈΡΠΊΠΎΠΌΡƒΡŽ Π²Π΅Π»ΠΈΡ‡ΠΈΠ½Ρƒ 2 ΠΌΠ΅Ρ‚ΠΎΠ΄Π°ΠΌΠΈ: с ΠΏΠΎΠΌΠΎΡ‰ΡŒΡŽ Ρ„ΠΎΡ€ΠΌΡƒΠ»Ρ‹ ΠΈ практичСским ΠΏΡƒΡ‚Π΅ΠΌ.

    ΠŸΡ€ΠΈΠΌΠ΅Ρ€ β„– 1

    Π’ ΠΏΠ΅Ρ€Π²ΠΎΠΌ ΠΏΡ€ΠΈΠΌΠ΅Ρ€Π΅ Π½Π°ΠΌ Π½ΡƒΠΆΠ½ΠΎ Π½Π°ΠΉΡ‚ΠΈ Ρ‚ΠΎΠΊ (I) Π² Ρ†Π΅ΠΏΠΈ, зная Π²Π΅Π»ΠΈΡ‡ΠΈΠ½Ρƒ источника постоянного напряТСния ΠΈ Π²Π΅Π»ΠΈΡ‡ΠΈΠ½Ρƒ сопротивлСния свСтодиодной Π»Π°ΠΌΠΏΠΎΡ‡ΠΊΠΈ.

    НапряТСниС источника постоянного напряТСния составляСт U = 220 (Π’)
    . Π‘ΠΎΠΏΡ€ΠΎΡ‚ΠΈΠ²Π»Π΅Π½ΠΈΠ΅ свСтодиодной Π»Π°ΠΌΠΏΠΎΡ‡ΠΊΠΈ Ρ€Π°Π²Π½ΠΎ R = 40740 (Ом)
    .

    Π‘ ΠΏΠΎΠΌΠΎΡ‰ΡŒΡŽ Ρ„ΠΎΡ€ΠΌΡƒΠ»Ρ‹ Π½Π°ΠΉΠ΄Π΅ΠΌ Ρ‚ΠΎΠΊ Π² Ρ†Π΅ΠΏΠΈ:

    I = U/R = 220 / 40740 = 0,0054 (А)

    ΠŸΠΎΠ΄ΠΊΠ»ΡŽΡ‡Π°Π΅ΠΌ ΠΏΠΎΡΠ»Π΅Π΄ΠΎΠ²Π°Ρ‚Π΅Π»ΡŒΠ½ΠΎ свСтодиодной Π»Π°ΠΌΠΏΠΎΡ‡ΠΊΠ΅ , Π²ΠΊΠ»ΡŽΡ‡Π΅Π½Π½Ρ‹ΠΉ Π² Ρ€Π΅ΠΆΠΈΠΌΠ΅ Π°ΠΌΠΏΠ΅Ρ€ΠΌΠ΅Ρ‚Ρ€, ΠΈ замСряСм Ρ‚ΠΎΠΊ Π² Ρ†Π΅ΠΏΠΈ.

    На дисплСС ΠΌΡƒΠ»ΡŒΡ‚ΠΈΠΌΠ΅Ρ‚Ρ€Π° ΠΏΠΎΠΊΠ°Π·Π°Π½ Ρ‚ΠΎΠΊ Ρ†Π΅ΠΏΠΈ. Π•Π³ΠΎ Π·Π½Π°Ρ‡Π΅Π½ΠΈΠ΅ Ρ€Π°Π²Π½ΠΎ 5,4 (мА) ΠΈΠ»ΠΈ 0,0054 (А), Ρ‡Ρ‚ΠΎ соотвСтствуСт Ρ‚ΠΎΠΊΡƒ, Π½Π°ΠΉΠ΄Π΅Π½Π½ΠΎΠΌΡƒ ΠΏΠΎ Ρ„ΠΎΡ€ΠΌΡƒΠ»Π΅.

    ΠŸΡ€ΠΈΠΌΠ΅Ρ€ β„– 2

    Π’ΠΎ Π²Ρ‚ΠΎΡ€ΠΎΠΌ ΠΏΡ€ΠΈΠΌΠ΅Ρ€Π΅ Π½Π°ΠΌ Π½ΡƒΠΆΠ½ΠΎ Π½Π°ΠΉΡ‚ΠΈ напряТСниС (U) участка Ρ†Π΅ΠΏΠΈ, зная Π²Π΅Π»ΠΈΡ‡ΠΈΠ½Ρƒ Ρ‚ΠΎΠΊΠ° Π² Ρ†Π΅ΠΏΠΈ ΠΈ Π²Π΅Π»ΠΈΡ‡ΠΈΠ½Ρƒ сопротивлСния свСтодиодной Π»Π°ΠΌΠΏΠΎΡ‡ΠΊΠΈ.

    I = 0,0054 (А)

    R = 40740 (Ом)

    Π‘ ΠΏΠΎΠΌΠΎΡ‰ΡŒΡŽ Ρ„ΠΎΡ€ΠΌΡƒΠ»Ρ‹ Π½Π°ΠΉΠ΄Π΅ΠΌ напряТСниС участка Ρ†Π΅ΠΏΠΈ:

    U = I*R = 0,0054 *40740 = 219,9 (Π’) = 220 (Π’)

    А Ρ‚Π΅ΠΏΠ΅Ρ€ΡŒ ΠΏΡ€ΠΎΠ²Π΅Ρ€ΠΈΠΌ ΠΏΠΎΠ»ΡƒΡ‡Π΅Π½Π½Ρ‹ΠΉ Ρ€Π΅Π·ΡƒΠ»ΡŒΡ‚Π°Ρ‚ практичСским ΠΏΡƒΡ‚Π΅ΠΌ.

    ΠŸΠΎΠ΄ΠΊΠ»ΡŽΡ‡Π°Π΅ΠΌ ΠΏΠ°Ρ€Π°Π»Π»Π΅Π»ΡŒΠ½ΠΎ свСтодиодной Π»Π°ΠΌΠΏΠΎΡ‡ΠΊΠ΅ ΠΌΡƒΠ»ΡŒΡ‚ΠΈΠΌΠ΅Ρ‚Ρ€, Π²ΠΊΠ»ΡŽΡ‡Π΅Π½Π½Ρ‹ΠΉ Π² Ρ€Π΅ΠΆΠΈΠΌΠ΅ Π²ΠΎΠ»ΡŒΡ‚ΠΌΠ΅Ρ‚Ρ€, ΠΈ замСряСм напряТСниС.

    На дисплСС ΠΌΡƒΠ»ΡŒΡ‚ΠΈΠΌΠ΅Ρ‚Ρ€Π° ΠΏΠΎΠΊΠ°Π·Π°Π½Π° Π²Π΅Π»ΠΈΡ‡ΠΈΠ½Π° ΠΈΠ·ΠΌΠ΅Ρ€Π΅Π½Π½ΠΎΠ³ΠΎ напряТСния. Π•Π³ΠΎ Π·Π½Π°Ρ‡Π΅Π½ΠΈΠ΅ Ρ€Π°Π²Π½ΠΎ 220 (Π’), Ρ‡Ρ‚ΠΎ соотвСтствуСт Π½Π°ΠΏΡ€ΡΠΆΠ΅Π½ΠΈΡŽ, Π½Π°ΠΉΠ΄Π΅Π½Π½ΠΎΠΌΡƒ ΠΏΠΎ Ρ„ΠΎΡ€ΠΌΡƒΠ»Π΅ Π·Π°ΠΊΠΎΠ½Π° Ома для участка Ρ†Π΅ΠΏΠΈ.

    ΠŸΡ€ΠΈΠΌΠ΅Ρ€ β„– 3

    Π’ Ρ‚Ρ€Π΅Ρ‚ΡŒΠ΅ΠΌ ΠΏΡ€ΠΈΠΌΠ΅Ρ€Π΅ Π½Π°ΠΌ Π½ΡƒΠΆΠ½ΠΎ Π½Π°ΠΉΡ‚ΠΈ сопротивлСниС (R) участка Ρ†Π΅ΠΏΠΈ, зная Π²Π΅Π»ΠΈΡ‡ΠΈΠ½Ρƒ Ρ‚ΠΎΠΊΠ° Π² Ρ†Π΅ΠΏΠΈ ΠΈ Π²Π΅Π»ΠΈΡ‡ΠΈΠ½Ρƒ напряТСния участка Ρ†Π΅ΠΏΠΈ.

    I = 0,0054 (А)

    U = 220 (Π’)

    ΠžΠΏΡΡ‚ΡŒ Ρ‚Π°ΠΊΠΈ, Π²ΠΎΡΠΏΠΎΠ»ΡŒΠ·ΡƒΠ΅ΠΌΡΡ Ρ„ΠΎΡ€ΠΌΡƒΠ»ΠΎΠΉ ΠΈ Π½Π°ΠΉΠ΄Π΅ΠΌ сопротивлСниС участка Ρ†Π΅ΠΏΠΈ:

    R = U/
    I = 220/0,0054 = 40740,7 (Ом)

    А Ρ‚Π΅ΠΏΠ΅Ρ€ΡŒ ΠΏΡ€ΠΎΠ²Π΅Ρ€ΠΈΠΌ ΠΏΠΎΠ»ΡƒΡ‡Π΅Π½Π½Ρ‹ΠΉ Ρ€Π΅Π·ΡƒΠ»ΡŒΡ‚Π°Ρ‚ практичСским ΠΏΡƒΡ‚Π΅ΠΌ.

    Π‘ΠΎΠΏΡ€ΠΎΡ‚ΠΈΠ²Π»Π΅Π½ΠΈΠ΅ свСтодиодной Π»Π°ΠΌΠΏΠΎΡ‡ΠΊΠΈ ΠΌΡ‹ измСряСм с ΠΏΠΎΠΌΠΎΡ‰ΡŒΡŽ ΠΈΠ»ΠΈ ΠΌΡƒΠ»ΡŒΡ‚ΠΈΠΌΠ΅Ρ‚Ρ€Π°.

    ΠŸΠΎΠ»ΡƒΡ‡Π΅Π½Π½ΠΎΠ΅ Π·Π½Π°Ρ‡Π΅Π½ΠΈΠ΅ составило R = 40740 (Ом)
    , Ρ‡Ρ‚ΠΎ соотвСтствуСт ΡΠΎΠΏΡ€ΠΎΡ‚ΠΈΠ²Π»Π΅Π½ΠΈΡŽ, Π½Π°ΠΉΠ΄Π΅Π½Π½ΠΎΠΌΡƒ ΠΏΠΎ Ρ„ΠΎΡ€ΠΌΡƒΠ»Π΅.

    Как Π»Π΅Π³ΠΊΠΎ Π·Π°ΠΏΠΎΠΌΠ½ΠΈΡ‚ΡŒ Π—Π°ΠΊΠΎΠ½ Ома для участка Ρ†Π΅ΠΏΠΈ!!!

    Π§Ρ‚ΠΎΠ±Ρ‹ Π½Π΅ ΠΏΡƒΡ‚Π°Ρ‚ΡŒΡΡ ΠΈ Π»Π΅Π³ΠΊΠΎ Π·Π°ΠΏΠΎΠΌΠ½ΠΈΡ‚ΡŒ Ρ„ΠΎΡ€ΠΌΡƒΠ»Ρƒ, ΠΌΠΎΠΆΠ½ΠΎ Π²ΠΎΡΠΏΠΎΠ»ΡŒΠ·ΠΎΠ²Π°Ρ‚ΡŒΡΡ нСбольшой подсказкой, ΠΊΠΎΡ‚ΠΎΡ€ΡƒΡŽ Π’Ρ‹ ΠΌΠΎΠΆΠ΅Ρ‚Π΅ ΡΠ΄Π΅Π»Π°Ρ‚ΡŒ ΡΠ°ΠΌΠΎΡΡ‚ΠΎΡΡ‚Π΅Π»ΡŒΠ½ΠΎ.

    НарисуйтС Ρ‚Ρ€Π΅ΡƒΠ³ΠΎΠ»ΡŒΠ½ΠΈΠΊ ΠΈ Π²ΠΏΠΈΡˆΠΈΡ‚Π΅ Π² Π½Π΅Π³ΠΎ ΠΏΠ°Ρ€Π°ΠΌΠ΅Ρ‚Ρ€Ρ‹ элСктричСской Ρ†Π΅ΠΏΠΈ, согласно рисунка Π½ΠΈΠΆΠ΅. Π£ Вас Π΄ΠΎΠ»ΠΆΠ½ΠΎ получится Π²ΠΎΡ‚ Ρ‚Π°ΠΊ.

    Как этим ΠΏΠΎΠ»ΡŒΠ·ΠΎΠ²Π°Ρ‚ΡŒΡΡ?

    ΠŸΠΎΠ»ΡŒΠ·ΠΎΠ²Π°Ρ‚ΡŒΡΡ Ρ‚Ρ€Π΅ΡƒΠ³ΠΎΠ»ΡŒΠ½ΠΈΠΊΠΎΠΌ-подсказкой ΠΎΡ‡Π΅Π½ΡŒ Π»Π΅Π³ΠΊΠΎ ΠΈ просто. Π—Π°ΠΊΡ€Ρ‹Π²Π°Π΅Ρ‚Π΅ своим ΠΏΠ°Π»ΡŒΡ†Π΅ΠΌ, Ρ‚ΠΎΡ‚ ΠΏΠ°Ρ€Π°ΠΌΠ΅Ρ‚Ρ€ Ρ†Π΅ΠΏΠΈ, ΠΊΠΎΡ‚ΠΎΡ€Ρ‹ΠΉ Π½Π΅ΠΎΠ±Ρ…ΠΎΠ΄ΠΈΠΌΠΎ Π½Π°ΠΉΡ‚ΠΈ.

    Если ΠΎΡΡ‚Π°Π²ΡˆΠΈΠ΅ΡΡ Π½Π° Ρ‚Ρ€Π΅ΡƒΠ³ΠΎΠ»ΡŒΠ½ΠΈΠΊΠ΅ ΠΏΠ°Ρ€Π°ΠΌΠ΅Ρ‚Ρ€Ρ‹ располоТСны Π½Π° ΠΎΠ΄Π½ΠΎΠΌ ΡƒΡ€ΠΎΠ²Π½Π΅, Ρ‚ΠΎ Π·Π½Π°Ρ‡ΠΈΡ‚ ΠΈΡ… Π½Π΅ΠΎΠ±Ρ…ΠΎΠ΄ΠΈΠΌΠΎ ΠΏΠ΅Ρ€Π΅ΠΌΠ½ΠΎΠΆΠΈΡ‚ΡŒ.

    Если ΠΆΠ΅ ΠΎΡΡ‚Π°Π²ΡˆΠΈΠ΅ΡΡ Π½Π° Ρ‚Ρ€Π΅ΡƒΠ³ΠΎΠ»ΡŒΠ½ΠΈΠΊΠ΅ ΠΏΠ°Ρ€Π°ΠΌΠ΅Ρ‚Ρ€Ρ‹ располоТСны Π½Π° Ρ€Π°Π·Π½ΠΎΠΌ ΡƒΡ€ΠΎΠ²Π½Π΅, Ρ‚ΠΎ Ρ‚ΠΎΠ³Π΄Π° Π½Π΅ΠΎΠ±Ρ…ΠΎΠ΄ΠΈΠΌΠΎ Ρ€Π°Π·Π΄Π΅Π»ΠΈΡ‚ΡŒ Π²Π΅Ρ€Ρ…Π½ΠΈΠΉ ΠΏΠ°Ρ€Π°ΠΌΠ΅Ρ‚Ρ€ Π½Π° Π½ΠΈΠΆΠ½ΠΈΠΉ.

    Π‘ ΠΏΠΎΠΌΠΎΡ‰ΡŒΡŽ Ρ‚Ρ€Π΅ΡƒΠ³ΠΎΠ»ΡŒΠ½ΠΈΠΊΠ°-подсказки Π’Ρ‹ Π½Π΅ Π±ΡƒΠ΄Π΅Ρ‚Π΅ ΠΏΡƒΡ‚Π°Ρ‚ΡŒΡΡ Π² Ρ„ΠΎΡ€ΠΌΡƒΠ»Π΅. Но Π»ΡƒΡ‡ΡˆΠ΅ всС Ρ‚Π°ΠΊΠΈ Π΅Π΅ Π²Ρ‹ΡƒΡ‡ΠΈΡ‚ΡŒ, ΠΊΠ°ΠΊ Ρ‚Π°Π±Π»ΠΈΡ†Ρƒ умноТСния.

    Π’Ρ‹Π²ΠΎΠ΄Ρ‹

    Π’ Π·Π°Π²Π΅Ρ€ΡˆΠ΅Π½ΠΈΠΈ ΡΡ‚Π°Ρ‚ΡŒΠΈ сдСлаю Π²Ρ‹Π²ΠΎΠ΄.

    ЭлСктричСский Ρ‚ΠΎΠΊ β€” это Π½Π°ΠΏΡ€Π°Π²Π»Π΅Π½Π½Ρ‹ΠΉ ΠΏΠΎΡ‚ΠΎΠΊ элСктронов ΠΎΡ‚ Ρ‚ΠΎΡ‡ΠΊΠΈ Π’ с ΠΏΠΎΡ‚Π΅Π½Ρ†ΠΈΠ°Π»ΠΎΠΌ минус ΠΊ Ρ‚ΠΎΡ‡ΠΊΠ΅ А с ΠΏΠΎΡ‚Π΅Π½Ρ†ΠΈΠ°Π»ΠΎΠΌ плюс. И Ρ‡Π΅ΠΌ Π²Ρ‹ΡˆΠ΅ Ρ€Π°Π·Π½ΠΎΡΡ‚ΡŒ ΠΏΠΎΡ‚Π΅Π½Ρ†ΠΈΠ°Π»ΠΎΠ² ΠΌΠ΅ΠΆΠ΄Ρƒ этими Ρ‚ΠΎΡ‡ΠΊΠ°ΠΌΠΈ, Ρ‚Π΅ΠΌ большС элСктронов пСрСмСстится ΠΈΠ· Ρ‚ΠΎΡ‡ΠΊΠΈ Π’ Π² Ρ‚ΠΎΡ‡ΠΊΡƒ А, Ρ‚. Π΅. Ρ‚ΠΎΠΊ Π² Ρ†Π΅ΠΏΠΈ увСличится, ΠΏΡ€ΠΈ условии, Ρ‡Ρ‚ΠΎ сопротивлСниС Ρ†Π΅ΠΏΠΈ останСтся Π½Π΅ΠΈΠ·ΠΌΠ΅Π½Π½Ρ‹ΠΌ.

    Но сопротивлСниС Π»Π°ΠΌΠΏΠΎΡ‡ΠΊΠΈ противодСйствуСт ΠΏΡ€ΠΎΡ‚Π΅ΠΊΠ°Π½ΠΈΡŽ элСктричСского Ρ‚ΠΎΠΊΠ°. И Ρ‡Π΅ΠΌ большС сопротивлСниС Π² Ρ†Π΅ΠΏΠΈ (ΠΏΠΎΡΠ»Π΅Π΄ΠΎΠ²Π°Ρ‚Π΅Π»ΡŒΠ½ΠΎΠ΅ соСдинСниС Π½Π΅ΡΠΊΠΎΠ»ΡŒΠΊΠΈΡ… Π»Π°ΠΌΠΏΠΎΡ‡Π΅ΠΊ), Ρ‚Π΅ΠΌ мСньшС Π±ΡƒΠ΄Π΅Ρ‚ Ρ‚ΠΎΠΊ Π² Ρ†Π΅ΠΏΠΈ, ΠΏΡ€ΠΈ Π½Π΅ΠΈΠ·ΠΌΠ΅Π½Π½ΠΎΠΌ напряТСнии сСти.

    P.S. Π’ΡƒΡ‚ Π² ΠΈΠ½Ρ‚Π΅Ρ€Π½Π΅Ρ‚Π΅ нашСл ΡΠΌΠ΅ΡˆΠ½ΡƒΡŽ, Π½ΠΎ ΠΏΠΎΡΡΠ½ΡΡŽΡ‰ΡƒΡŽ ΠΊΠ°Ρ€ΠΈΠΊΠ°Ρ‚ΡƒΡ€Ρƒ Π½Π° Ρ‚Π΅ΠΌΡƒ Π·Π°ΠΊΠΎΠ½Π° Ома для участка Ρ†Π΅ΠΏΠΈ.

    ВСст ΠΏΠΎ Ρ‚Π΅ΠΌΠ΅ «Π—Π°ΠΊΠΎΠ½ Ома для ΠΏΠΎΠ»Π½ΠΎΠΉ Ρ†Π΅ΠΏΠΈ».

    ВСст ΠΏΠΎ Ρ‚Π΅ΠΌΠ΅ «Π—Π°ΠΊΠΎΠ½ Ома для ΠΏΠΎΠ»Π½ΠΎΠΉ Ρ†Π΅ΠΏΠΈ».

    тСст
    ΠΏΠΎ Ρ„ΠΈΠ·ΠΈΠΊΠ΅ 10 класс

    Β 

    Π—Π°ΠΊΠΎΠ½ Ома для ΠΏΠΎΠ»Π½ΠΎΠΉ Ρ†Π΅ΠΏΠΈ опрСдСляСт Π·Π½Π°Ρ‡Π΅Π½ΠΈΠ΅ Ρ‚ΠΎΠΊΠ° Π²
    Ρ€Π΅Π°Π»ΡŒΠ½ΠΎΠΉ Ρ†Π΅ΠΏΠΈ, ΠΊΠΎΡ‚ΠΎΡ€Ρ‹ΠΉ зависит Π½Π΅ Ρ‚ΠΎΠ»ΡŒΠΊΠΎ ΠΎΡ‚ сопротивлСния Π½Π°Π³Ρ€ΡƒΠ·ΠΊΠΈ, Π½ΠΎ ΠΈ ΠΎΡ‚
    сопротивлСния самого источника Ρ‚ΠΎΠΊΠ°. Π”Ρ€ΡƒΠ³ΠΎΠ΅ Π½Π°Π·Π²Π°Π½ΠΈΠ΅ этого Π·Π°ΠΊΠΎΠ½Π° — Π·Π°ΠΊΠΎΠ½ Ома
    для Π·Π°ΠΌΠΊΠ½ΡƒΡ‚ΠΎΠΉ Ρ†Π΅ΠΏΠΈ. Рассмотрим смысл Π·Π°ΠΊΠΎΠ½Π° Ома для ΠΏΠΎΠ»Π½ΠΎΠΉ Ρ†Π΅ΠΏΠΈ Π±ΠΎΠ»Π΅Π΅ ΠΏΠΎΠ΄Ρ€ΠΎΠ±Π½ΠΎ
    пройдя тСст.

    Β 

    Π’Π°Ρ€ΠΈΠ°Π½Ρ‚
    1.

    1.Β Β Β Β Β 
    Аккумулятор с Π­Π”Π‘ 35 B ΠΈ Π²Π½ΡƒΡ‚Ρ€Π΅Π½Π½ΠΈΠΌ
    сопротивлСниСм 2 Ом Π·Π°ΠΌΠΊΠ½ΡƒΡ‚ Π½Π° ΡΠΏΠΈΡ€Π°Π»ΡŒ с сопротивлСниСм 5 Ом. Какой Ρ‚ΠΎΠΊ ΠΏΠΎΠΊΠ°ΠΆΠ΅Ρ‚
    Π°ΠΌΠΏΠ΅Ρ€ΠΌΠ΅Ρ‚Ρ€, Π²ΠΊΠ»ΡŽΡ‡Ρ‘Π½Π½Ρ‹ΠΉ Π² Ρ†Π΅ΠΏΡŒ. (Π²Π½ΡƒΡ‚Ρ€Π΅Π½Π½ΠΈΠΌ сопротивлСниСм
    Π°ΠΌΠΏΠ΅Ρ€ΠΌΠ΅Ρ‚Ρ€Π° ΠΏΡ€Π΅Π½Π΅Π±Ρ€Π΅Ρ‡ΡŒ).

    a.Β Β Β Β Β Β Β Β 
    5 А;

    b.Β Β Β Β Β Β Β 
    7 А;

    c.Β Β Β Β Β Β Β Β 
    9 А;

    d.Β Β Β Β Β Β Β 
    10,5 А;

    e.Β Β Β Β Β Β Β Β 
    2 А.

    2.Β Β Β Β Β 
    Аккумулятор с Π²Π½ΡƒΡ‚Ρ€Π΅Π½Π½ΠΈΠΌ сопротивлСниСм
    0,4 Ом ΠΈ Π­Π”Π‘ 4 B Π·Π°ΠΌΠΊΠ½ΡƒΡ‚ ΡΠΏΠΈΡ€Π°Π»ΡŒΡŽ ΠΈΠ· ΠΏΡ€ΠΎΠ²ΠΎΠ»ΠΎΠΊΠΎΠΉ сСчСниСм 1 ΠΌΠΌ2 ΠΈ
    ΡƒΠ΄Π΅Π»ΡŒΠ½Ρ‹ΠΌ сопротивлСниСм 0,0000008 ΠžΠΌβ€’ΠΌ. НайдитС Π΄Π»ΠΈΠ½Ρƒ спирали, Ссли сила Ρ‚ΠΎΠΊΠ° Π²
    Ρ†Π΅ΠΏΠΈ 2 А.

    a.Β Β Β Β Β Β Β Β 
    2,3 ΠΌ;

    b.Β Β Β Β Β Β Β 
    2 ΠΌ;

    c.Β Β Β Β Β Β Β Β 
    1,5 ΠΌ;

    d.Β Β Β Β Β Β Β 
    15 ΠΌ;

    e.Β Β Β Β Β Β Β Β 
    13 ΠΌ.

    3.Β Β Β Β Β 
    Если ΠΊ Π³Π°Π»ΡŒΠ²Π°Π½ΠΈΡ‡Π΅ΡΠΊΠΎΠΌΡƒ элСмСнту с Π­Π”Π‘ 5
    B ΠΈ Π²Π½ΡƒΡ‚Ρ€Π΅Π½Π½ΠΈΠΌ сопротивлСниСм 1 Ом ΠΏΠΎΠ΄ΠΊΠ»ΡŽΡ‡ΠΈΡ‚ΡŒ Π°ΠΌΠΏΠ΅Ρ€ΠΌΠ΅Ρ‚Ρ€, Ρ‚ΠΎ ΠΎΠ½ ΠΏΠΎΠΊΠ°ΠΆΠ΅Ρ‚ силу
    Ρ‚ΠΎΠΊΠ° 1 A. ΠžΠΏΡ€Π΅Π΄Π΅Π»ΠΈΡ‚Π΅ Π²Π½ΡƒΡ‚Ρ€Π΅Π½Π½Π΅Π΅ сопротивлСниС Π°ΠΌΠΏΠ΅Ρ€ΠΌΠ΅Ρ‚Ρ€Π°.

    a.Β Β Β Β Β Β 
    2 Ом;

    b.Β Β Β Β Β 
    1,5 Ом;

    c.Β Β Β Β Β Β 
    4 Ом;

    d.Β Β Β Β Β 
    3,1 Ом;

    e.Β Β Β Β Β Β 
    5,8 Ом;

    1. Π‘ΠΏΠΈΡ€Π°Π»ΡŒ
      сопротивлСниСм 4 Ом, ΠΏΠΎΠ΄ΠΊΠ»ΡŽΡ‡ΠΈΠ»ΠΈ ΠΊ Π³Π°Π»ΡŒΠ²Π°Π½ΠΈΡ‡Π΅ΡΠΊΠΎΠΌΡƒ элСмСнту с Π­Π”Π‘ 6 B.
      АмпСрмСтр ΠΏΠΎΠΊΠ°Π·Π°Π» Ρ‚ΠΎΠΊ Π² спирали 0,6 A. НайдитС Ρ‚ΠΎΠΊ ΠΊΠΎΡ€ΠΎΡ‚ΠΊΠΎΠ³ΠΎ замыкания Π³Π°Π»ΡŒΠ²Π°Π½ΠΈΡ‡Π΅ΡΠΊΠΎΠ³ΠΎ
      элСмСнта. (Π²Π½ΡƒΡ‚Ρ€Π΅Π½Π½ΠΈΠΌ сопротивлСниСм Π°ΠΌΠΏΠ΅Ρ€ΠΌΠ΅Ρ‚Ρ€Π° ΠΏΡ€Π΅Π½Π΅Π±Ρ€Π΅Ρ‡ΡŒ).

    a.Β Β Β Β Β Β Β Β 
    5,5 А;

    b.Β Β Β Β Β Β Β 
    6,5 А;

    c.Β Β Β Β Β Β Β Β 
    0,3 А;

    d.Β Β Β Β Β Β Β 
    2,4 А;

    e.Β Β Β Β Β Β Β Β 
    1 А.

    1. ΠŸΡ€ΠΈ
      ΠΏΠΎΠ΄ΠΊΠ»ΡŽΡ‡Π΅Π½ΠΈΠΈ аккумулятора с Π­Π”Π‘ 10 B ΠΊ рСзистору, напряТСниС Π½Π° ΠΏΠΎΠ»ΡŽΡΠ°Ρ…
      источника Π²ΠΎΠ»ΡŒΡ‚ΠΌΠ΅Ρ‚Ρ€ ΠΏΠΎΠΊΠ°Π·Π°Π» 8 B, Π° Π°ΠΌΠΏΠ΅Ρ€ΠΌΠ΅Ρ‚Ρ€ ΠΏΠΎΠΊΠ°Π·Π°Π» силу Ρ‚ΠΎΠΊΠ° Π² Ρ†Π΅ΠΏΠΈ 4 А.
      НайдитС Π²Π½ΡƒΡ‚Ρ€Π΅Π½Π½Π΅Π΅ сопротивлСниС источника. (Π²Π½ΡƒΡ‚Ρ€Π΅Π½Π½ΠΈΠΌ сопротивлСниСм
      Π°ΠΌΠΏΠ΅Ρ€ΠΌΠ΅Ρ‚Ρ€Π° ΠΏΡ€Π΅Π½Π΅Π±Ρ€Π΅Ρ‡ΡŒ).

    a.Β Β Β Β Β Β Β Β 
    5,7 Ом;

    b.Β Β Β Β Β Β Β 
    1,9 Ом;

    c.Β Β Β Β Β Β Β Β 
    3 Ом;

    d.Β Β Β Β Β Β Β 
    0,3 Ом;

    e.Β Β Β Β Β Β Β Β 
    0,5 Ом

    Β 

    6.Β Β Β Β Β 
    Π’Π½ΡƒΡ‚Ρ€Π΅Π½Π½Π΅Π΅ сопротивлСниС Π³Π°Π»ΡŒΠ²Π°Π½ΠΈΡ‡Π΅ΡΠΊΠΎΠ³ΠΎ
    элСмСнта с Π­Π”Π‘ 3,6 B Ρ€Π°Π²Π½ΠΎ 0,4 Ом. К Π³Π°Π»ΡŒΠ²Π°Π½ΠΈΡ‡Π΅ΡΠΊΠΎΠΌΡƒ элСмСнту ΠΏΠΎΠ΄ΠΊΠ»ΡŽΡ‡Π΅Π½Ρ‹
    ΠΏΠ°Ρ€Π°Π»Π»Π΅Π»ΡŒΠ½ΠΎ Ρ‚Ρ€ΠΈ спирали сопротивлСниСм ΠΏΠΎ 3 Ом каТдая. НайдитС Ρ€Π°Π·Π½ΠΎΡΡ‚ΡŒ
    ΠΏΠΎΡ‚Π΅Π½Ρ†ΠΈΠ°Π»ΠΎΠ² Π½Π° ΠΊΠ»Π΅ΠΌΠΌΠ°Ρ… Π³Π°Π»ΡŒΠ²Π°Π½ΠΈΡ‡Π΅ΡΠΊΠΎΠ³ΠΎ элСмСнта?

    a.Β Β Β Β Β Β Β Β 
    2,6 Π’;

    b.Β Β Β Β Β Β Β 
    4,3 Π’;

    c.Β Β Β Β Β Β Β Β 
    3,9 Π’;

    d. Β Β Β Β Β Β Β 
    1,4 Π’;

    e.Β Β Β Β Β Β Β Β 
    0,9 Π’.

    7.Β Β Β Β Β 
    Π’ Ρ†Π΅ΠΏΠΈ, состоящСй ΠΈΠ· аккумулятора с Π­Π”Π‘
    8 Π’ ΠΈ Π²Π½ΡƒΡ‚Ρ€Π΅Π½Π½ΠΈΠΌ сопротивлСниСм 1 Ом. ΠŸΠΎΠ΄ΠΊΠ»ΡŽΡ‡ΠΈΠ»ΠΈ ΠΏΡ€ΠΎΠ²ΠΎΠ»ΠΎΡ‡Π½Ρ‹ΠΉ рСостат, Π²ΠΎΠ·Π½ΠΈΠΊ
    Ρ‚ΠΎΠΊ Π² Ρ†Π΅ΠΏΠΈ 2 А. Какой Π±ΡƒΠ΄Π΅Ρ‚ сила Ρ‚ΠΎΠΊΠ° Π² Ρ†Π΅ΠΏΠΈ, Ссли сопротивлСниС ΠΏΡ€ΠΎΠ²ΠΎΠ»ΠΎΡ‡Π½ΠΎΠ³ΠΎ рСостата
    ΡƒΠΌΠ΅Π½ΡŒΡˆΠΈΡ‚ΡŒ Π² 2 Ρ€Π°Π·Π°?

    a.Β Β Β Β Β Β Β Β 
    5,1 А;

    b.Β Β Β Β Β Β Β 
    3,3 А;

    c.Β Β Β Β Β Β Β Β 
    1,4 А;

    d.Β Β Β Β Β Β Β 
    3,2 А;

    e.Β Β Β Β Β Β Β Β 
    2,7 А

    8.Β Β Β Β Β 
    Аккумулятор с Π­Π”Π‘ 12 B ΠΈ Π²Π½ΡƒΡ‚Ρ€Π΅Π½Π½ΠΈΠΌ
    сопротивлСниСм 2 Ом ΠΏΠΈΡ‚Π°Π΅Ρ‚ 4 ΠΏΠ°Ρ€Π°Π»Π»Π΅Π»ΡŒΠ½ΠΎ соСдинСнных Π»Π°ΠΌΠΏΠΎΡ‡ΠΊΠΈ ΠΏΠΎ 8 Ом ΠΊΠ°ΠΆΠ΄ΠΎΠ΅.
    ΠžΠΏΡ€Π΅Π΄Π΅Π»ΠΈΡ‚Π΅ напряТСниС Π½Π° ΠΊΠ°ΠΆΠ΄ΠΎΠΉ Π»Π°ΠΌΠΏΠΎΡ‡ΠΊΠ΅.

    a.Β Β Β Β Β Β Β Β 
    2,1 Π’;

    b.Β Β Β Β Β Β Β 
    6 Π’;

    c.Β Β Β Β Β Β Β Β 
    10,6 Π’;

    d.Β Β Β Β Β Β Β 
    8,5 Π’;

    e.Β Β Β Β Β Β Β Β 
    4,7 Π’.

    9.Β Β Β Β Β 
    К Π°ΠΌΠΏΠ΅Ρ€ΠΌΠ΅Ρ‚Ρ€Ρƒ, с Π²Π½ΡƒΡ‚Ρ€Π΅Π½Π½ΠΈΠΌ
    сопротивлСниСм 0,1 Ом, ΠΏΠΎΠ΄ΠΊΠ»ΡŽΡ‡Π΅Π½ ΡˆΡƒΠ½Ρ‚ (ΠΏΡ€ΠΎΠ²ΠΎΠ»ΠΎΡ‡Π½Ρ‹ΠΉ рСзистор) сопротивлСниСм
    0,0111 Ом. ΠžΠΏΡ€Π΅Π΄Π΅Π»ΠΈΡ‚Π΅ силу Ρ‚ΠΎΠΊΠ°, ΠΏΡ€ΠΎΡ‚Π΅ΠΊΠ°ΡŽΡ‰Π΅Π³ΠΎ Ρ‡Π΅Ρ€Π΅Π· Π°ΠΌΠΏΠ΅Ρ€ΠΌΠ΅Ρ‚Ρ€, Ссли сила Ρ‚ΠΎΠΊΠ° Π²
    ΠΎΠ±Ρ‰Π΅ΠΉ Ρ†Π΅ΠΏΠΈ 0,27 А.

    a.Β Β Β Β Β Β Β Β 
    2,7 А;   

    b.Β Β Β Β Β Β Β 
    0,27 А;   

    c.Β Β Β Β Β Β Β Β 
    0,027 А;   

    d. Β Β Β Β Β Β Β 
    0,0027 А;

    e.Β Β Β Β Β Β Β Β 
    0,00027 А.

    10.Β  Β Π’ ΠΊΠ°ΠΊΠΈΡ… Π»Π°ΠΌΠΏΠ°Ρ… Π½ΠΈΡ‚ΡŒ накаливания Ρ‚ΠΎΠ½ΡŒΡˆΠ΅: Π² Π±ΠΎΠ»Π΅Π΅ ΠΈΠ»ΠΈ ΠΌΠ΅Π½Π΅Π΅
    ΠΌΠΎΡ‰Π½Ρ‹Ρ…?

    a.Β Β Β Β Β Β Β Β 
    Π’ ΠΌΠ΅Π½Π΅Π΅ ΠΌΠΎΡ‰Π½Ρ‹Ρ… Π½ΠΈΡ‚ΡŒ накаливания Ρ‚ΠΎΠ½ΡŒΡˆΠ΅,
    большС сопротивлСниС Π½ΠΈΡ‚ΠΈ;

    b.Β Β Β Β Β Β Β 
    Π’ Π±ΠΎΠ»Π΅Π΅ ΠΌΠΎΡ‰Π½Ρ‹Ρ… Π½ΠΈΡ‚ΡŒ накаливания Ρ‚ΠΎΠ½ΡŒΡˆΠ΅,
    мСньшС сила Ρ‚ΠΎΠΊΠ°;

    c.Β Β Β Β Β Β Β Β 
    Π’ ΠΌΠ΅Π½Π΅Π΅ ΠΌΠΎΡ‰Π½Ρ‹Ρ… Π½ΠΈΡ‚ΡŒ накаливания Ρ‚ΠΎΠ»Ρ‰Π΅,
    большС сила Ρ‚ΠΎΠΊΠ°;

    d.Β Β Β Β Β Β Β 
    Π’ Π±ΠΎΠ»Π΅Π΅ ΠΌΠΎΡ‰Π½Ρ‹Ρ… Π½ΠΈΡ‚ΡŒ накаливания Ρ‚ΠΎΠ½ΡŒΡˆΠ΅,
    мСньшС сопротивлСниС Π½ΠΈΡ‚ΠΈ;

    e.Β Β Β Β Β Β Β Β 
    НСт ΠΏΡ€Π°Π²ΠΈΠ»ΡŒΠ½ΠΎΠ³ΠΎ ΠΎΡ‚Π²Π΅Ρ‚Π°.

    Β 

    Β 

    Β 

    Β 

    Π’Π°Ρ€ΠΈΠ°Π½Ρ‚ 2.

    Β 

    1.Β Β Β Β Β Β Β Β Β Β Β Β Β Β Β 
    ΠžΠΏΡ€Π΅Π΄Π΅Π»ΠΈΡ‚Π΅ силу Ρ‚ΠΎΠΊΠ° Π² Ρ†Π΅ΠΏΠΈ, содСрТащСй Π³Π°Π»ΡŒΠ²Π°Π½ΠΈΡ‡Π΅ΡΠΊΠΈΠΉ элСмСнт с Π­Π”Π‘,
    Ρ€Π°Π²Π½ΠΎΠΉ 6 Π’, ΠΈ Π²Π½ΡƒΡ‚Ρ€Π΅Π½Π½ΠΈΠΌ сопротивлСниСм 0,5 Ом ΠΏΡ€ΠΈ ΠΏΠΎΠ΄ΠΊΠ»ΡŽΡ‡Π΅Π½ΠΈΠΈ Π²ΠΎ внСшнСй Ρ†Π΅ΠΏΠΈ Π»Π°ΠΌΠΏΠΎΡ‡ΠΊΠΈ
    с сопротивлСниСм 2,5 Ом.

    a)     1 А;

    b)     2 А;

    c)     0,5 А;

    d)     3 А;

    e)     10 А;

    2.Β Β Β Β Β Β Β Β Β Β Β Β Β Β Β 
    ЭлСктричСский Ρ‡Π°ΠΉΠ½ΠΈΠΊ, Π²ΠΊΠ»ΡŽΡ‡Π΅Π½Π½Ρ‹ΠΉ Π² ΡΠ΅Ρ‚ΡŒ напряТСниСм 220 Π’, потрСбляСт
    Ρ‚ΠΎΠΊ 1,2 А. ΠžΠΏΡ€Π΅Π΄Π΅Π»ΠΈΡ‚Π΅ сопротивлСниС Π²ΠΎΠ΄ΠΎΠ½Π°Π³Ρ€Π΅Π²Π°Ρ‚Π΅Π»ΡŒΠ½ΠΎΠ³ΠΎ элСмСнта Ρ‡Π°ΠΉΠ½ΠΈΠΊΠ°.

    a)     183,3 Ом;

    b)     100 Ом;

    c)     330 Ом;

    d)     44,5 Ом;

    e)     220,7 Ом;

    3.Β Β Β Β Β Β Β Β Β Β Β Β Β Β Β 
    Π­Π”Π‘ Π³Π°Π»ΡŒΠ²Π°Π½ΠΈΡ‡Π΅ΡΠΊΠΎΠ³ΠΎ элСмСнта 3 Π’,
    Π΅Π³ΠΎ Π²Π½ΡƒΡ‚Ρ€Π΅Π½Π½Π΅Π΅ сопротивлСниС Ρ€Π°Π²Π½ΠΎ1 Ом, сопротивлСния рСзисторов R1
    = R2 = 1,75 Ом, R3 = 2 Ом, R4 = 6 Ом. Π§Π΅ΠΌΡƒ
    Ρ€Π°Π²Π½Π° сила Ρ‚ΠΎΠΊΠ°, ΠΏΡ€ΠΎΡ‚Π΅ΠΊΠ°ΡŽΡ‰Π΅Π³ΠΎ Ρ‡Π΅Ρ€Π΅Π· рСзистор R4? (см. рис)

    a)Β Β Β Β Β  0,5
    А;

    b)     0,045 А;

    c)      2 А;

    d)     0,35 А;

    e)Β Β Β Β Β  0,125
    А;

    4.Β Β Β Β Β Β Β Β Β Β Β Β Β Β Β 
    ΠžΠΏΡ€Π΅Π΄Π΅Π»ΠΈΡ‚Π΅ сопротивлСниС Π²ΠΎΠ΄ΠΎΠ½Π°Π³Ρ€Π΅Π²Π°Ρ‚Π΅Π»ΡŒΠ½ΠΎΠ³ΠΎ тэна, Ссли ΠΏΡ€ΠΈ напряТСнии
    110 Π’ сила Ρ‚ΠΎΠΊΠ° Π² Π½Ρ‘ΠΌ 2 А.

    a)     60 Ом;

    b)     35 Ом;

    c)     55 Ом;

    d)     100 Ом;

    e)     135 Ом;

    5.Β Β Β Β Β Β Β Β Β Β Β Β Β Β Β 
    Π“Π°Π»ΡŒΠ²Π°Π½ΠΈΡ‡Π΅ΡΠΊΠΈΠΉ элСмСнт ΠΈΠΌΠ΅Π΅Ρ‚ Π­Π”Π‘, Ρ€Π°Π²Π½Ρ‹ΠΉ 2,2 Π’ ΠΈ Π²Π½ΡƒΡ‚Ρ€Π΅Π½Π½Π΅Π΅
    сопротивлСниС r =1 Ом. Он Π·Π°ΠΌΠΊΠ½ΡƒΡ‚ Π½Π° Π»Π°ΠΌΠΏΠΎΡ‡ΠΊΡƒ сопротивлСниС, ΠΊΠΎΡ‚ΠΎΡ€ΠΎΠΉ Ρ€Π°Π²Π½ΠΎ R=9 Ом.
    Какой Π±ΡƒΠ΄Π΅Ρ‚ ΠΏΡ€ΠΈ этом сила Ρ‚ΠΎΠΊΠ° Π² Ρ†Π΅ΠΏΠΈ?

    a)     0,22 А;

    b)     0,15 А;

    c)     0,5 А;

    d)     2 А;

    e)     1 А;

    6. Β Β Β Β Β Β Β Β Β Β Β Β Β Β Β 
    Π”Π²Π΅ Π»Π°ΠΌΠΏΠΎΡ‡ΠΊΠΈ, сопротивлСния ΠΊΠΎΡ‚ΠΎΡ€Ρ‹Ρ… R1=10
    Ом ΠΈ R2=40 Ом, соСдинСны ΠΏΠΎΡΠ»Π΅Π΄ΠΎΠ²Π°Ρ‚Π΅Π»ΡŒΠ½ΠΎ ΠΈ
    ΠΏΠΎΠ΄ΠΊΠ»ΡŽΡ‡Π΅Π½Ρ‹ ΠΊ аккумулятору с Π­Π”Π‘ Ρ€Π°Π²Π½Ρ‹ΠΌ 15 Π’. ΠžΠΏΡ€Π΅Π΄Π΅Π»ΠΈΡ‚Π΅ Π²Π½ΡƒΡ‚Ρ€Π΅Π½Π½Π΅Π΅
    сопротивлСниС аккумулятора, Ссли сила Ρ‚ΠΎΠΊΠ° Π² Ρ†Π΅ΠΏΠΈ I=0,29
    А.

    a)     1,25 А;

    b)     0,34 А;

    c)     2,15 А;

    d)     1,72 А;

    e)     3,3 А;

    7.Β Β Β Β Β Β Β Β Β Β Β Β Β Β Β 
    Найти силу Ρ‚ΠΎΠΊΠ° Π² элСктричСской Ρ†Π΅ΠΏΠΈ, Ссли извСстно, Ρ‡Ρ‚ΠΎ сопротивлСниС
    Ρ†Π΅ΠΏΠΈ 11 Ом, Π° источник, ΠΏΠΎΠ΄ΠΊΠ»ΡŽΡ‡Π΅Π½Π½Ρ‹ΠΉ ΠΊ Π½Π΅ΠΉ, ΠΈΠΌΠ΅Π΅Ρ‚ Π­Π”Π‘ 12 Π’ ΠΈ Π²Π½ΡƒΡ‚Ρ€Π΅Π½Π½Π΅Π΅
    сопротивлСниС 1 Ом.

    a)     1 А;

    b)     2 А;

    c)     3 А;

    d)     4 А;

    e)     5 А;

    8.Β Β Β Β Β Β Β Β Β Β Β Β Β Β Β 
    Аккумулятор ΠΏΠΎΠ΄ΠΊΠ»ΡŽΡ‡Π΅Π½ ΠΊ рСзистору сопротивлСниСм 10 Ом с ΠΏΠΎΠΌΠΎΡ‰ΡŒΡŽ ΠΌΠ΅Π΄Π½ΠΎΠΉ спирали
    Π΄Π»ΠΈΠ½ΠΎΠΉ 1 ΠΌ ΠΈ ΠΏΠ»ΠΎΡ‰Π°Π΄ΡŒΡŽ ΠΏΠΎΠΏΠ΅Ρ€Π΅Ρ‡Π½ΠΎΠ³ΠΎ сСчСния 1 ΠΌΠΌ2. Найти силу Ρ‚ΠΎΠΊΠ°,
    зная, Ρ‡Ρ‚ΠΎ Π­Π”Π‘ аккумулятора Ρ€Π°Π²Π½ΠΎ 12 Π’, Π° Π²Π½ΡƒΡ‚Ρ€Π΅Π½Π½Π΅Π΅ сопротивлСниС 1,9825 Ом.

    a)     0,5 А;

    b)     0,8 А;

    c)     1 А;

    d)     1,5 А;

    e)     2 А;

    9. Β Β Β Β Β Β Β Β Β Β Β Β Β Β Β 
    ΠŸΡ€ΠΈ внСшнСм сопротивлСнии 3,75 Ом Π² Ρ†Π΅ΠΏΠΈ ΠΈΠ΄Π΅Ρ‚ Ρ‚ΠΎΠΊ 0,5 А. Когда Π² Ρ†Π΅ΠΏΡŒ
    Π²Π²Π΅Π»ΠΈ Π΅Ρ‰Π΅ сопротивлСниС 1 Ом, сила Ρ‚ΠΎΠΊΠ° стала Ρ€Π°Π²Π½ΠΎΠΉ 0,4 А. Найти Π­Π”Π‘ ΠΈ
    Π²Π½ΡƒΡ‚Ρ€Π΅Π½Π½Π΅Π΅ сопротивлСниС Π³Π°Π»ΡŒΠ²Π°Π½ΠΈΡ‡Π΅ΡΠΊΠΎΠ³ΠΎ элСмСнта.

    a)Β Β Β Β  1 Π’; 0,5
    Ом;

    b)Β Β Β Β  2 Π’; 0,25
    Ом;

    c)Β Β Β Β  1 Π’, 3 Ом;

    d)Β Β Β Β  3 Π’; 1,5
    Ом;

    e)Β Β Β Β  5 Π’; 4 Ом;

    10.Β Β Β Β Β Β Β Β Β Β Β 
    ΠžΠΏΡ€Π΅Π΄Π΅Π»ΠΈΡ‚ΡŒ Ρ‚ΠΎΠΊ ΠΏΡ€ΠΈ ΠΊΠΎΡ€ΠΎΡ‚ΠΊΠΎΠΌ Π·Π°ΠΌΡ‹ΠΊΠ°Π½ΠΈΠΈ аккумулятора, Ссли ΠΏΡ€ΠΈ внСшнСм
    сопротивлСнии R1=50 Ом Ρ‚ΠΎΠΊ Π² Ρ†Π΅ΠΏΠΈ I1= 0,2 А, Π° ΠΏΡ€ΠΈ сопротивлСнии R2=
    110 Ом Ρ‚ΠΎΠΊ I2= 0,1 А.

    a)     1400 мА;

    b)     1000 мА;

    c)     3000 мА;

    d)     1600 мА;

    e)     1200 мА;

    Β 

    Β 

    Β 

    ΠžΡ‚Π²Π΅Ρ‚Ρ‹: Π’Π°Ρ€ΠΈΠ°Π½Ρ‚ 1: 1 Π°; 2 b;
    3 с; 4 С; 5 С; 6 а; 7
    d; 8 b; 9 с; 10 a.

    Β Β Β Β Β Β Β Β Β Β Β Β Β Β Β Β  Π’Π°Ρ€ΠΈΠ°Π½Ρ‚ 2: 1 b;
    2 а; 3 С; 4 с; 5 а; 6
    d; 7 а; 8 с; 9 b; 10 С.


    Π‘ΠΊΠ°Ρ‡Π°Π½ΠΎ с www.znanio.ru

    Π—Π°Π΄Π°Ρ‡ΠΈ ΠΏΠΎ Ρ„ΠΈΠ·ΠΈΠΊΠ΅ ΠΈ ΠΌΠ°Ρ‚Π΅ΠΌΠ°Ρ‚ΠΈΠΊΠ΅ с Ρ€Π΅ΡˆΠ΅Π½ΠΈΡΠΌΠΈ ΠΈ ΠΎΡ‚Π²Π΅Ρ‚Π°ΠΌΠΈ

    Π—Π°Π΄Π°Ρ‡Π° ΠΏΠΎ Ρ„ΠΈΠ·ΠΈΠΊΠ΅ — 9017

    Зная, Ρ‡Ρ‚ΠΎ ΠΏΠΎΠΊΠ°Π·Π°Ρ‚Π΅Π»ΡŒ прСломлСния $n$ водяных ΠΏΠ°Ρ€ΠΎΠ² ΠΏΡ€ΠΈ Π½ΠΎΡ€ΠΌΠ°Π»ΡŒΠ½Ρ‹Ρ… условиях Ρ€Π°Π²Π΅Π½ 1,000252 ΠΈ Ρ‡Ρ‚ΠΎ ΠΌΠΎΠ»Π΅ΠΊΡƒΠ»Π° Π²ΠΎΠ΄Ρ‹ ΠΎΠ±Π»Π°Π΄Π°Π΅Ρ‚ элСктричСским ΠΌΠΎΠΌΠ΅Π½Ρ‚ΠΎΠΌ $p = 6,1 \cdot 10^{-30} Кл \cdot ΠΌ$, ΠΎΠΏΡ€Π΅Π΄Π΅Π»ΠΈΡ‚ΡŒ, ΠΊΠ°ΠΊΡƒΡŽ долю ΠΎΡ‚ ΠΎΠ±Ρ‰Π΅ΠΉ поляризуСмости (элСктронной ΠΈ ΠΎΡ€ΠΈΠ΅Π½Ρ‚Π°Ρ†ΠΈΠΎΠ½Π½ΠΎΠΉ) составляСт элСктронная ΠΏΠΎΠ»ΡΡ€ΠΈΠ·ΡƒΠ΅ΠΌΠΎΡΡ‚ΡŒ ΠΌΠΎΠ»Π΅ΠΊΡƒΠ»Ρ‹. { \circ} Π‘$. Найти сопротивлСниС $R$ ΠΏΡ€ΠΎΠ²ΠΎΠ΄Π½ΠΈΠΊΠ°, считая Π³Ρ€Π°Π΄ΠΈΠ΅Π½Ρ‚ Ρ‚Π΅ΠΌΠΏΠ΅Ρ€Π°Ρ‚ΡƒΡ€Ρ‹ вдоль Π΅Π³ΠΎ оси постоянным.


    ΠŸΠΎΠ΄Ρ€ΠΎΠ±Π½Π΅Π΅

    Π—Π°Π΄Π°Ρ‡Π° ΠΏΠΎ Ρ„ΠΈΠ·ΠΈΠΊΠ΅ — 9021

    К источнику Ρ‚ΠΎΠΊΠ° с Π­Π”Π‘ $\mathcal{E} = 1,5 Π’$ присоСдинили ΠΊΠ°Ρ‚ΡƒΡˆΠΊΡƒ с сопротивлСниСм $R = 0,1 Ом$. АмпСрмСтр ΠΏΠΎΠΊΠ°Π·Π°Π» силу Ρ‚ΠΎΠΊΠ°, Ρ€Π°Π²Π½ΡƒΡŽ $I_{1} = 0,5 А$. Когда ΠΊ источнику Ρ‚ΠΎΠΊΠ° присоСдинили ΠΏΠΎΡΠ»Π΅Π΄ΠΎΠ²Π°Ρ‚Π΅Π»ΡŒΠ½ΠΎ Π΅Ρ‰Π΅ ΠΎΠ΄ΠΈΠ½ источник Ρ‚ΠΎΠΊΠ° с Ρ‚Π°ΠΊΠΎΠΉ ΠΆΠ΅ Π­Π”Π‘, Ρ‚ΠΎ сила Ρ‚ΠΎΠΊΠ° $I$ Π² Ρ‚ΠΎΠΉ ΠΆΠ΅ ΠΊΠ°Ρ‚ΡƒΡˆΠΊΠ΅ оказалась Ρ€Π°Π²Π½ΠΎΠΉ 0,4 А. ΠžΠΏΡ€Π΅Π΄Π΅Π»ΠΈΡ‚ΡŒ Π²Π½ΡƒΡ‚Ρ€Π΅Π½Π½ΠΈΠ΅ сопротивлСния $r_{1}$ ΠΈ $r_{2}$ ΠΏΠ΅Ρ€Π²ΠΎΠ³ΠΎ ΠΈ Π²Ρ‚ΠΎΡ€ΠΎΠ³ΠΎ источников Ρ‚ΠΎΠΊΠ°.


    ΠŸΠΎΠ΄Ρ€ΠΎΠ±Π½Π΅Π΅

    Π—Π°Π΄Π°Ρ‡Π° ΠΏΠΎ Ρ„ΠΈΠ·ΠΈΠΊΠ΅ — 9022

    Π˜ΠΌΠ΅Π΅Ρ‚ΡΡ $N$ ΠΎΠ΄ΠΈΠ½Π°ΠΊΠΎΠ²Ρ‹Ρ… Π³Π°Π»ΡŒΠ²Π°Π½ΠΈΡ‡Π΅ΡΠΊΠΈΡ… элСмСнтов с Π­Π”Π‘ $\mathcal{E}$ ΠΈ Π²Π½ΡƒΡ‚Ρ€Π΅Π½Π½ΠΈΠΌ сопротивлСниСм $r_{i}$ ΠΊΠ°ΠΆΠ΄Ρ‹ΠΉ. Из этих элСмСнтов трСбуСтся ΡΠΎΠ±Ρ€Π°Ρ‚ΡŒ Π±Π°Ρ‚Π°Ρ€Π΅ΡŽ, ΡΠΎΡΡ‚ΠΎΡΡ‰ΡƒΡŽ ΠΈΠ· Π½Π΅ΡΠΊΠΎΠ»ΡŒΠΊΠΈΡ… ΠΏΠ°Ρ€Π°Π»Π»Π΅Π»ΡŒΠ½ΠΎ соСдинСнных Π³Ρ€ΡƒΠΏΠΏ, содСрТащих ΠΏΠΎ $n$ ΠΏΠΎΡΠ»Π΅Π΄ΠΎΠ²Π°Ρ‚Π΅Π»ΡŒΠ½ΠΎ соСдинСнных элСмСнтов. ΠŸΡ€ΠΈ Ρ‚Π°ΠΊΠΎΠΌ Π·Π½Π°Ρ‡Π΅Π½ΠΈΠΈ $n$ сила Ρ‚ΠΎΠΊΠ° $I$ Π²ΠΎ внСшнСй Ρ†Π΅ΠΏΠΈ, ΠΈΠΌΠ΅ΡŽΡ‰Π΅ΠΉ сопротивлСниС $R$, Π±ΡƒΠ΄Π΅Ρ‚ максимальной? Π§Π΅ΠΌΡƒ Π±ΡƒΠ΄Π΅Ρ‚ Ρ€Π°Π²Π½ΠΎ Π²Π½ΡƒΡ‚Ρ€Π΅Π½Π½Π΅Π΅ сопротивлСниС $R_{i}$ Π±Π°Ρ‚Π°Ρ€Π΅ΠΈ ΠΏΡ€ΠΈ этом Π·Π½Π°Ρ‡Π΅Π½ΠΈΠΈ $n$?


    ΠŸΠΎΠ΄Ρ€ΠΎΠ±Π½Π΅Π΅

    Π—Π°Π΄Π°Ρ‡Π° ΠΏΠΎ Ρ„ΠΈΠ·ΠΈΠΊΠ΅ — 9023

    Π”Π°Π½Ρ‹ 12 элСмСнтов с Π­Π”Π‘ $\mathcal{E} = 1,5 Π’$ ΠΈ Π²Π½ΡƒΡ‚Ρ€Π΅Π½Π½Π΅ΠΌ сопротивлСниСм $r = 0,4 Ом$. Как Π½ΡƒΠΆΠ½ΠΎ ΡΠΎΠ΅Π΄ΠΈΠ½ΠΈΡ‚ΡŒ эти элСмСнты, Ρ‡Ρ‚ΠΎΠ±Ρ‹ ΠΏΠΎΠ»ΡƒΡ‡ΠΈΡ‚ΡŒ ΠΎΡ‚ собранной ΠΈΠ· Π½ΠΈΡ… Π±Π°Ρ‚Π°Ρ€Π΅ΠΈ Π½Π°ΠΈΠ±ΠΎΠ»ΡŒΡˆΡƒΡŽ силу Ρ‚ΠΎΠΊΠ° Π²ΠΎ внСшнСй Ρ†Π΅ΠΏΠΈ, ΠΈΠΌΠ΅ΡŽΡ‰Π΅ΠΉ сопротивлСниС $R = 0,3 Ом$? ΠžΠΏΡ€Π΅Π΄Π΅Π»ΠΈΡ‚ΡŒ ΠΌΠ°ΠΊΡΠΈΠΌΠ°Π»ΡŒΠ½ΡƒΡŽ силу Ρ‚ΠΎΠΊΠ° $I_{max}$.


    ΠŸΠΎΠ΄Ρ€ΠΎΠ±Π½Π΅Π΅

    Π—Π°Π΄Π°Ρ‡Π° ΠΏΠΎ Ρ„ΠΈΠ·ΠΈΠΊΠ΅ — 9024

    Π”Π²Π° элСмСнта ($\mathcal{E}_{1} =l,2 Π’, r_{1} = 0,1 Ом; \mathcal{E}_{2} = 0,9 Π’, r_{2} = 0,3 Ом$) соСдинСны ΠΎΠ΄Π½ΠΎΠΈΠΌΠ΅Π½Π½Ρ‹ΠΌΠΈ полюсами. Π‘ΠΎΠΏΡ€ΠΎΡ‚ΠΈΠ²Π»Π΅Π½ΠΈΠ΅ $R$ ΡΠΎΠ΅Π΄ΠΈΠ½ΠΈΡ‚Π΅Π»ΡŒΠ½Ρ‹Ρ… ΠΏΡ€ΠΎΠ²ΠΎΠ΄ΠΎΠ² Ρ€Π°Π²Π½ΠΎ 0,2 Ом. ΠžΠΏΡ€Π΅Π΄Π΅Π»ΠΈΡ‚ΡŒ силу Ρ‚ΠΎΠΊΠ° $I$ Π² Ρ†Π΅ΠΏΠΈ.


    ΠŸΠΎΠ΄Ρ€ΠΎΠ±Π½Π΅Π΅

    Π—Π°Π΄Π°Ρ‡Π° ΠΏΠΎ Ρ„ΠΈΠ·ΠΈΠΊΠ΅ — 9025

    Π”Π²Π΅ Π±Π°Ρ‚Π°Ρ€Π΅ΠΈ аккумуляторов ($\mathcal{E}_{1} = 10 Π’, r_{1} = 1 Ом; \mathcal{E}_{2} =8 Π’, r_{2} = 2 Ом$) ΠΈ рСостат ($R = 6 Ом$) соСдинСны, ΠΊΠ°ΠΊ ΠΏΠΎΠΊΠ°Π·Π°Π½ΠΎ Π½Π° рис. Найти силу Ρ‚ΠΎΠΊΠ° Π² батарСях ΠΈ рСостатС.


    ΠŸΠΎΠ΄Ρ€ΠΎΠ±Π½Π΅Π΅

    Π—Π°Π΄Π°Ρ‡Π° ΠΏΠΎ Ρ„ΠΈΠ·ΠΈΠΊΠ΅ — 9026

    Π’Ρ€ΠΈ батарСис Π­Π”Π‘ $\mathcal{E}_{1} = 12 Π’, \mathcal{E}_{2} = 5 Π’$ ΠΈ $\mathcal{E} = 10 Π’$ ΠΈ ΠΎΠ΄ΠΈΠ½Π°ΠΊΠΎΠ²Ρ‹ΠΌΠΈ Π²Π½ΡƒΡ‚Ρ€Π΅Π½Π½ΠΈΠΌΠΈ сопротивлСниями $r$, Ρ€Π°Π²Π½Ρ‹ΠΌΠΈ 1 Ом, соСдинСны ΠΌΠ΅ΠΆΠ΄Ρƒ собой ΠΎΠ΄Π½ΠΎΠΈΠΌΠ΅Π½Π½Ρ‹ΠΌΠΈ полюсами. Π‘ΠΎΠΏΡ€ΠΎΡ‚ΠΈΠ²Π»Π΅Π½ΠΈΠ΅ ΡΠΎΠ΅Π΄ΠΈΠ½ΠΈΡ‚Π΅Π»ΡŒΠ½Ρ‹Ρ… ΠΏΡ€ΠΎΠ²ΠΎΠ΄ΠΎΠ² Π½ΠΈΡ‡Ρ‚ΠΎΠΆΠ½ΠΎ ΠΌΠ°Π»ΠΎ. ΠžΠΏΡ€Π΅Π΄Π΅Π»ΠΈΡ‚ΡŒ силы Ρ‚ΠΎΠΊΠΎΠ² $I$, ΠΈΠ΄ΡƒΡ‰ΠΈΡ… Ρ‡Π΅Ρ€Π΅Π· ΠΊΠ°ΠΆΠ΄ΡƒΡŽ Π±Π°Ρ‚Π°Ρ€Π΅ΡŽ.


    ΠŸΠΎΠ΄Ρ€ΠΎΠ±Π½Π΅Π΅

    Π—Π°Π΄Π°Ρ‡Π° ΠΏΠΎ Ρ„ΠΈΠ·ΠΈΠΊΠ΅ — 9027

    Π­Π”Π‘ Π±Π°Ρ‚Π°Ρ€Π΅ΠΈ аккумуляторов $\mathcal{E} = 12 Π’$, сила Ρ‚ΠΎΠΊΠ° $I$ ΠΊΠΎΡ€ΠΎΡ‚ΠΊΠΎΠ³ΠΎ замыкания Ρ€Π°Π²Π½Π° 5 А. ΠšΠ°ΠΊΡƒΡŽ Π½Π°ΠΈΠ±ΠΎΠ»ΡŒΡˆΡƒΡŽ ΠΌΠΎΡ‰Π½ΠΎΡΡ‚ΡŒ $P_{max}$ ΠΌΠΎΠΆΠ½ΠΎ ΠΏΠΎΠ»ΡƒΡ‡ΠΈΡ‚ΡŒ Π²ΠΎ внСшнСй Ρ†Π΅ΠΏΠΈ, соСдинСнной с Ρ‚Π°ΠΊΠΎΠΉ Π±Π°Ρ‚Π°Ρ€Π΅Π΅ΠΉ?


    ΠŸΠΎΠ΄Ρ€ΠΎΠ±Π½Π΅Π΅

    Π—Π°Π΄Π°Ρ‡Π° ΠΏΠΎ Ρ„ΠΈΠ·ΠΈΠΊΠ΅ — 9028

    Π­Π”Π‘ $\mathcal{E}$ Π±Π°Ρ‚Π°Ρ€Π΅ΠΈ Ρ€Π°Π²Π½Π° 20 Π’. Π‘ΠΎΠΏΡ€ΠΎΡ‚ΠΈΠ²Π»Π΅Π½ΠΈΠ΅ $R$ внСшнСй Ρ†Π΅ΠΏΠΈ Ρ€Π°Π²Π½ΠΎ 2 Ом, сила Ρ‚ΠΎΠΊΠ° $I = 4 А$. Найти ΠšΠŸΠ” Π±Π°Ρ‚Π°Ρ€Π΅ΠΈ. ΠŸΡ€ΠΈ ΠΊΠ°ΠΊΠΎΠΌ Π·Π½Π°Ρ‡Π΅Π½ΠΈΠΈ внСшнСго сопротивлСния $R$ ΠšΠŸΠ” Π±ΡƒΠ΄Π΅Ρ‚ Ρ€Π°Π²Π΅Π½ 99%?


    ΠŸΠΎΠ΄Ρ€ΠΎΠ±Π½Π΅Π΅

    Π—Π°Π΄Π°Ρ‡Π° ΠΏΠΎ Ρ„ΠΈΠ·ΠΈΠΊΠ΅ — 9029

    К Π·Π°ΠΆΠΈΠΌΠ°ΠΌ Π±Π°Ρ‚Π°Ρ€Π΅ΠΈ аккумуляторов присоСдинСн Π½Π°Π³Ρ€Π΅Π²Π°Ρ‚Π΅Π»ΡŒ. Π­Π”Π‘ $\mathcal{E}$ Π±Π°Ρ‚Π°Ρ€Π΅ΠΈ Ρ€Π°Π²Π½Π° 24 Π’, Π²Π½ΡƒΡ‚Ρ€Π΅Π½Π½Π΅Π΅ сопротивлСниС $r = 1 Ом$. ΠΠ°Π³Ρ€Π΅Π²Π°Ρ‚Π΅Π»ΡŒ, Π²ΠΊΠ»ΡŽΡ‡Π΅Π½Π½Ρ‹ΠΉ Π² Ρ†Π΅ΠΏΡŒ, потрСбляСт ΠΌΠΎΡ‰Π½ΠΎΡΡ‚ΡŒ $P = 80 Π’Ρ‚$. Π’Ρ‹Ρ‡ΠΈΡΠ»ΠΈΡ‚ΡŒ силу Ρ‚ΠΎΠΊΠ° $I$ Π² Ρ†Π΅ΠΏΠΈ ΠΈ ΠšΠŸΠ” $\eta$ нагрСватСля.


    ΠŸΠΎΠ΄Ρ€ΠΎΠ±Π½Π΅Π΅

    Π—Π°Π΄Π°Ρ‡Π° ΠΏΠΎ Ρ„ΠΈΠ·ΠΈΠΊΠ΅ — 9030

    Π‘ΠΈΠ»Π° Ρ‚ΠΎΠΊΠ° Π² ΠΏΡ€ΠΎΠ²ΠΎΠ΄Π½ΠΈΠΊΠ΅ сопротивлСниСм $R = 12 Ом$ Ρ€Π°Π²Π½ΠΎΠΌΠ΅Ρ€Π½ΠΎ ΡƒΠ±Ρ‹Π²Π°Π΅Ρ‚ ΠΎΡ‚ $I_{0} = 5 А$ Π΄ΠΎ $I=0$ Π² Ρ‚Π΅Ρ‡Π΅Π½ΠΈΠ΅ Π²Ρ€Π΅ΠΌΠ΅Π½ΠΈ $t= 10 с$. КакоС количСство Ρ‚Π΅ΠΏΠ»ΠΎΡ‚Ρ‹ $Q$ выдСляСтся Π² этом ΠΏΡ€ΠΎΠ²ΠΎΠ΄Π½ΠΈΠΊΠ΅ Π·Π° ΡƒΠΊΠ°Π·Π°Π½Π½Ρ‹ΠΉ ΠΏΡ€ΠΎΠΌΠ΅ΠΆΡƒΡ‚ΠΎΠΊ Π²Ρ€Π΅ΠΌΠ΅Π½ΠΈ?


    ΠŸΠΎΠ΄Ρ€ΠΎΠ±Π½Π΅Π΅

    Π—Π°Π΄Π°Ρ‡Π° ΠΏΠΎ Ρ„ΠΈΠ·ΠΈΠΊΠ΅ — 9031

    По ΠΏΡ€ΠΎΠ²ΠΎΠ΄Π½ΠΈΠΊΡƒ сопротивлСниСм $R=3 Ом$ Ρ‚Π΅Ρ‡Π΅Ρ‚ Ρ‚ΠΎΠΊ, сила ΠΊΠΎΡ‚ΠΎΡ€ΠΎΠ³ΠΎ возрастаСт. ΠšΠΎΠ»ΠΈΡ‡Π΅ΡΡ‚Π²ΠΎ Ρ‚Π΅ΠΏΠ»ΠΎΡ‚Ρ‹ $Q$, Π²Ρ‹Π΄Π΅Π»ΠΈΠ²ΡˆΠ΅Π΅ΡΡ Π² ΠΏΡ€ΠΎΠ²ΠΎΠ΄Π½ΠΈΠΊΠ΅ Π·Π° врСмя $\tau = 8 с$, Ρ€Π°Π²Π½ΠΎ 200 Π”ΠΆ. ΠžΠΏΡ€Π΅Π΄Π΅Π»ΠΈΡ‚ΡŒ количСство элСктричСства $q$, ΠΏΡ€ΠΎΡ‚Π΅ΠΊΡˆΠ΅Π΅ Π·Π° это врСмя ΠΏΠΎ ΠΏΡ€ΠΎΠ²ΠΎΠ΄Π½ΠΈΠΊΡƒ. Π’ ΠΌΠΎΠΌΠ΅Π½Ρ‚ Π²Ρ€Π΅ΠΌΠ΅Π½ΠΈ, принятый Π·Π° Π½Π°Ρ‡Π°Π»ΡŒΠ½Ρ‹ΠΉ, сила Ρ‚ΠΎΠΊΠ° Π² ΠΏΡ€ΠΎΠ²ΠΎΠ΄Π½ΠΈΠΊΠ΅ Ρ€Π°Π²Π½Π° Π½ΡƒΠ»ΡŽ.


    ΠŸΠΎΠ΄Ρ€ΠΎΠ±Π½Π΅Π΅

    Как Ρ€Π°ΡΡΡ‡ΠΈΡ‚Π°Ρ‚ΡŒ Π­Π”Π‘ | Sciencing

    ОбновлСно 2 ноября 2020 Π³.

    Π›ΠΈ ДТонсон

    ЭлСктродвиТущая сила (Π­Π”Π‘) — понятиС Π½Π΅Π·Π½Π°ΠΊΠΎΠΌΠΎΠ΅ для Π±ΠΎΠ»ΡŒΡˆΠΈΠ½ΡΡ‚Π²Π° людСй, Π½ΠΎ ΠΎΠ½ΠΎ тСсно связано с Π±ΠΎΠ»Π΅Π΅ Π·Π½Π°ΠΊΠΎΠΌΡ‹ΠΌ понятиСм напряТСния. ПониманиС Ρ€Π°Π·Π½ΠΈΡ†Ρ‹ ΠΌΠ΅ΠΆΠ΄Ρƒ Π½ΠΈΠΌΠΈ ΠΈ Ρ‚ΠΎΠ³ΠΎ, Ρ‡Ρ‚ΠΎ ΠΎΠ·Π½Π°Ρ‡Π°Π΅Ρ‚ Π­Π”Π‘, Π΄Π°Π΅Ρ‚ Π²Π°ΠΌ инструмСнты, Π½Π΅ΠΎΠ±Ρ…ΠΎΠ΄ΠΈΠΌΡ‹Π΅ для Ρ€Π΅ΡˆΠ΅Π½ΠΈΡ ΠΌΠ½ΠΎΠ³ΠΈΡ… ΠΏΡ€ΠΎΠ±Π»Π΅ΠΌ Π² Ρ„ΠΈΠ·ΠΈΠΊΠ΅ ΠΈ элСктроникС, Π° Ρ‚Π°ΠΊΠΆΠ΅ Π·Π½Π°ΠΊΠΎΠΌΠΈΡ‚ с ΠΊΠΎΠ½Ρ†Π΅ΠΏΡ†ΠΈΠ΅ΠΉ Π²Π½ΡƒΡ‚Ρ€Π΅Π½Π½Π΅Π³ΠΎ сопротивлСния Π±Π°Ρ‚Π°Ρ€Π΅ΠΈ. Π­Π”Π‘ сообщаСт Π²Π°ΠΌ напряТСниС Π±Π°Ρ‚Π°Ρ€Π΅ΠΈ Π±Π΅Π· ΡƒΠΌΠ΅Π½ΡŒΡˆΠ΅Π½ΠΈΡ Π²Π½ΡƒΡ‚Ρ€Π΅Π½Π½Π΅Π³ΠΎ сопротивлСния, ΠΊΠ°ΠΊ ΠΏΡ€ΠΈ ΠΎΠ±Ρ‹Ρ‡Π½Ρ‹Ρ… измСрСниях разности ΠΏΠΎΡ‚Π΅Π½Ρ†ΠΈΠ°Π»ΠΎΠ².Π’Ρ‹ ΠΌΠΎΠΆΠ΅Ρ‚Π΅ Ρ€Π°ΡΡΡ‡ΠΈΡ‚Π°Ρ‚ΡŒ Π΅Π³ΠΎ нСсколькими способами, Π² зависимости ΠΎΡ‚ Ρ‚ΠΎΠ³ΠΎ, какая информация Ρƒ вас Π΅ΡΡ‚ΡŒ.

    TL; DR (слишком Π΄ΠΎΠ»Π³ΠΎ; Π½Π΅ Ρ‡ΠΈΡ‚Π°Π»)

    РассчитайтС Π­Π”Π‘ ΠΏΠΎ Ρ„ΠΎΡ€ΠΌΡƒΠ»Π΅:

    Ξ΅ = V + Ir

    Π—Π΄Π΅ΡΡŒ (Π’) ΠΎΠ·Π½Π°Ρ‡Π°Π΅Ρ‚ напряТСниС элСмСнта, (I) ΠΎΠ·Π½Π°Ρ‡Π°Π΅Ρ‚ Ρ‚ΠΎΠΊ Π² Ρ†Π΅ΠΏΠΈ, Π° (r) ΠΎΠ·Π½Π°Ρ‡Π°Π΅Ρ‚ Π²Π½ΡƒΡ‚Ρ€Π΅Π½Π½Π΅Π΅ сопротивлСниС ячСйки.

    Π§Ρ‚ΠΎ Ρ‚Π°ΠΊΠΎΠ΅ Π­Π”Π‘?

    ЭлСктродвиТущая сила — это Ρ€Π°Π·Π½ΠΎΡΡ‚ΡŒ ΠΏΠΎΡ‚Π΅Π½Ρ†ΠΈΠ°Π»ΠΎΠ² (Ρ‚. Π•. НапряТСниС) Π½Π° ΠΊΠ»Π΅ΠΌΠΌΠ°Ρ… Π±Π°Ρ‚Π°Ρ€Π΅ΠΈ ΠΏΡ€ΠΈ отсутствии Ρ‚ΠΎΠΊΠ°.ΠœΠΎΠΆΠ΅Ρ‚ ΠΏΠΎΠΊΠ°Π·Π°Ρ‚ΡŒΡΡ, Ρ‡Ρ‚ΠΎ это Π½Π΅ ΠΈΠΌΠ΅Π΅Ρ‚ значСния, Π½ΠΎ каТдая батарСя ΠΈΠΌΠ΅Π΅Ρ‚ Β«Π²Π½ΡƒΡ‚Ρ€Π΅Π½Π½Π΅Π΅ сопротивлСниС». Π­Ρ‚ΠΎ ΠΏΠΎΡ…ΠΎΠΆΠ΅ Π½Π° ΠΎΠ±Ρ‹Ρ‡Π½ΠΎΠ΅ сопротивлСниС, ΠΊΠΎΡ‚ΠΎΡ€ΠΎΠ΅ сниТаСт Ρ‚ΠΎΠΊ Π² Ρ†Π΅ΠΏΠΈ, Π½ΠΎ ΠΎΠ½ΠΎ сущСствуСт Π²Π½ΡƒΡ‚Ρ€ΠΈ самой Π±Π°Ρ‚Π°Ρ€Π΅ΠΈ. Π­Ρ‚ΠΎ связано с Ρ‚Π΅ΠΌ, Ρ‡Ρ‚ΠΎ ΠΌΠ°Ρ‚Π΅Ρ€ΠΈΠ°Π»Ρ‹, ΠΈΠ· ΠΊΠΎΡ‚ΠΎΡ€Ρ‹Ρ… состоят элСмСнты Π² Π±Π°Ρ‚Π°Ρ€Π΅Π΅, ΠΈΠΌΠ΅ΡŽΡ‚ собствСнноС сопротивлСниС (Ρ‚Π°ΠΊ ΠΊΠ°ΠΊ практичСски всС ΠΌΠ°Ρ‚Π΅Ρ€ΠΈΠ°Π»Ρ‹ ΠΈΠΌΠ΅ΡŽΡ‚).

    Когда Ρ‡Π΅Ρ€Π΅Π· элСмСнт Π½Π΅ Ρ‚Π΅Ρ‡Π΅Ρ‚ Ρ‚ΠΎΠΊ, это Π²Π½ΡƒΡ‚Ρ€Π΅Π½Π½Π΅Π΅ сопротивлСниС Π½ΠΈΡ‡Π΅Π³ΠΎ Π½Π΅ мСняСт, ΠΏΠΎΡ‚ΠΎΠΌΡƒ Ρ‡Ρ‚ΠΎ Π½Π΅Ρ‚ Ρ‚ΠΎΠΊΠ°, ΠΊΠΎΡ‚ΠΎΡ€Ρ‹ΠΉ ΠΌΠΎΠ³ Π±Ρ‹ Π·Π°ΠΌΠ΅Π΄Π»ΠΈΡ‚ΡŒ Π΅Π³ΠΎ. Π’ Π½Π΅ΠΊΠΎΡ‚ΠΎΡ€ΠΎΠΌ смыслС, Π­Π”Π‘ ΠΌΠΎΠΆΠ½ΠΎ Ρ€Π°ΡΡΠΌΠ°Ρ‚Ρ€ΠΈΠ²Π°Ρ‚ΡŒ ΠΊΠ°ΠΊ ΠΌΠ°ΠΊΡΠΈΠΌΠ°Π»ΡŒΠ½ΡƒΡŽ Ρ€Π°Π·Π½ΠΎΡΡ‚ΡŒ ΠΏΠΎΡ‚Π΅Π½Ρ†ΠΈΠ°Π»ΠΎΠ² Π½Π° ΠΊΠ»Π΅ΠΌΠΌΠ°Ρ… Π² идСальной ситуации, ΠΈ Π½Π° ΠΏΡ€Π°ΠΊΡ‚ΠΈΠΊΠ΅ ΠΎΠ½Π° всСгда большС, Ρ‡Π΅ΠΌ напряТСниС аккумулятора.

    УравнСния для расчСта Π­Π”Π‘

    Π•ΡΡ‚ΡŒ Π΄Π²Π° основных уравнСния для расчСта Π­Π”Π‘. НаиболСС Ρ„ΡƒΠ½Π΄Π°ΠΌΠ΅Π½Ρ‚Π°Π»ΡŒΠ½ΠΎΠ΅ ΠΎΠΏΡ€Π΅Π΄Π΅Π»Π΅Π½ΠΈΠ΅ — это количСство Π΄ΠΆΠΎΡƒΠ»Π΅ΠΉ энСргии (E), ΠΊΠΎΡ‚ΠΎΡ€ΠΎΠ΅ ΠΊΠ°ΠΆΠ΄Ρ‹ΠΉ ΠΊΡƒΠ»ΠΎΠ½ заряда (Q) Π·Π°Π±ΠΈΡ€Π°Π΅Ρ‚ ΠΏΡ€ΠΈ ΠΏΡ€ΠΎΡ…ΠΎΠΆΠ΄Π΅Π½ΠΈΠΈ Ρ‡Π΅Ρ€Π΅Π· ячСйку:

    Π“Π΄Π΅ (Ξ΅) — символ элСктродвиТущСй силы, (E) — энСргия Π² Ρ†Π΅ΠΏΠΈ, Π° (Q) — заряд Ρ†Π΅ΠΏΠΈ.Если Π²Ρ‹ Π·Π½Π°Π΅Ρ‚Π΅ Ρ€Π΅Π·ΡƒΠ»ΡŒΡ‚ΠΈΡ€ΡƒΡŽΡ‰ΡƒΡŽ ΡΠ½Π΅Ρ€Π³ΠΈΡŽ ΠΈ количСство заряда, проходящСго Ρ‡Π΅Ρ€Π΅Π· ячСйку, это самый простой способ Π²Ρ‹Ρ‡ΠΈΡΠ»ΠΈΡ‚ΡŒ Π­Π”Π‘, Π½ΠΎ Π² Π±ΠΎΠ»ΡŒΡˆΠΈΠ½ΡΡ‚Π²Π΅ случаСв Ρƒ вас Π½Π΅Ρ‚ этой ΠΈΠ½Ρ„ΠΎΡ€ΠΌΠ°Ρ†ΠΈΠΈ.

    ВмСсто этого Π²Ρ‹ ΠΌΠΎΠΆΠ΅Ρ‚Π΅ ΠΈΡΠΏΠΎΠ»ΡŒΠ·ΠΎΠ²Π°Ρ‚ΡŒ ΠΎΠΏΡ€Π΅Π΄Π΅Π»Π΅Π½ΠΈΠ΅, большС ΠΏΠΎΡ…ΠΎΠΆΠ΅Π΅ Π½Π° Π·Π°ΠΊΠΎΠ½ Ома (V = IR). Π­Ρ‚ΠΎ ΠΌΠΎΠΆΠ΅Ρ‚ Π±Ρ‹Ρ‚ΡŒ Π²Ρ‹Ρ€Π°ΠΆΠ΅Π½ΠΎ ΠΊΠ°ΠΊ:

    \ epsilon = I (R + r)

    , Π³Π΄Π΅ (I) ΠΎΠ·Π½Π°Ρ‡Π°Π΅Ρ‚ Ρ‚ΠΎΠΊ, (R) — сопротивлСниС рассматриваСмой Ρ†Π΅ΠΏΠΈ, Π° (r) — Π²Π½ΡƒΡ‚Ρ€Π΅Π½Π½Π΅Π΅ сопротивлСниС ячСйки. Π Π°ΡΡˆΠΈΡ€Π΅Π½ΠΈΠ΅ этого ΠΏΠΎΠΊΠ°Π·Ρ‹Π²Π°Π΅Ρ‚ Ρ‚Π΅ΡΠ½ΡƒΡŽ связь с Π·Π°ΠΊΠΎΠ½ΠΎΠΌ Ома:

    \ epsilon = IR + Ir = V + Ir

    Π­Ρ‚ΠΎ ΠΏΠΎΠΊΠ°Π·Ρ‹Π²Π°Π΅Ρ‚, Ρ‡Ρ‚ΠΎ Π²Ρ‹ ΠΌΠΎΠΆΠ΅Ρ‚Π΅ Ρ€Π°ΡΡΡ‡ΠΈΡ‚Π°Ρ‚ΡŒ Π­Π”Π‘, Ссли Π²Ρ‹ Π·Π½Π°Π΅Ρ‚Π΅ напряТСниС Π½Π° ΠΊΠ»Π΅ΠΌΠΌΠ°Ρ… (напряТСниС, ΠΈΡΠΏΠΎΠ»ΡŒΠ·ΡƒΠ΅ΠΌΠΎΠ΅ Π² Ρ€Π΅Π°Π»ΡŒΠ½Ρ‹Ρ… ситуациях) , ΠΏΡ€ΠΎΡ‚Π΅ΠΊΠ°ΡŽΡ‰ΠΈΠΉ Ρ‚ΠΎΠΊ ΠΈ Π²Π½ΡƒΡ‚Ρ€Π΅Π½Π½Π΅Π΅ сопротивлСниС ячСйки.

    Как Ρ€Π°ΡΡΡ‡ΠΈΡ‚Π°Ρ‚ΡŒ Π­Π”Π‘: ΠΏΡ€ΠΈΠΌΠ΅Ρ€

    Π’ качСствС ΠΏΡ€ΠΈΠΌΠ΅Ρ€Π° ΠΏΡ€Π΅Π΄ΡΡ‚Π°Π²ΡŒΡ‚Π΅, Ρ‡Ρ‚ΠΎ Ρƒ вас Π΅ΡΡ‚ΡŒ Ρ†Π΅ΠΏΡŒ с Ρ€Π°Π·Π½ΠΎΡΡ‚ΡŒΡŽ ΠΏΠΎΡ‚Π΅Π½Ρ†ΠΈΠ°Π»ΠΎΠ² 3,2 Π’, ΠΏΡ€ΠΎΡ‚Π΅ΠΊΠ°ΡŽΡ‰ΠΈΠΌ Ρ‚ΠΎΠΊΠΎΠΌ 0,6 А ΠΈ Π²Π½ΡƒΡ‚Ρ€Π΅Π½Π½ΠΈΠΌ сопротивлСниСм Π±Π°Ρ‚Π°Ρ€Π΅ΠΈ 0,5 Ом. Π˜ΡΠΏΠΎΠ»ΡŒΠ·ΡƒΡ Ρ„ΠΎΡ€ΠΌΡƒΠ»Ρƒ Π²Ρ‹ΡˆΠ΅:

    \ epsilon = V + Ir = 3.2 \ text {V} + (0.6 \ text {A}) (0.5 \ text {} \ Omega) = 3.5 \ text {V}

    Π˜Ρ‚Π°ΠΊ, Π­Π”Π‘ этой Ρ†Π΅ΠΏΠΈ составляСт 3,5 Π’.

    ДвиТущая Π­Π”Π‘ | Π€ΠΈΠ·ΠΈΠΊΠ°

    ЦСль обучСния

    К ΠΊΠΎΠ½Ρ†Ρƒ этого Ρ€Π°Π·Π΄Π΅Π»Π° Π²Ρ‹ смоТСтС:

    • Π’Ρ‹Ρ‡ΠΈΡΠ»ΠΈΡ‚ΡŒ Π­Π”Π‘, силу, ΠΌΠ°Π³Π½ΠΈΡ‚Π½ΠΎΠ΅ ΠΏΠΎΠ»Π΅ ΠΈ Ρ€Π°Π±ΠΎΡ‚Ρƒ, ΠΎΠ±ΡƒΡΠ»ΠΎΠ²Π»Π΅Π½Π½ΡƒΡŽ Π΄Π²ΠΈΠΆΠ΅Π½ΠΈΠ΅ΠΌ ΠΎΠ±ΡŠΠ΅ΠΊΡ‚Π° Π² ΠΌΠ°Π³Π½ΠΈΡ‚Π½ΠΎΠΌ ΠΏΠΎΠ»Π΅.

    Как ΠΌΡ‹ Π²ΠΈΠ΄Π΅Π»ΠΈ, любоС ΠΈΠ·ΠΌΠ΅Π½Π΅Π½ΠΈΠ΅ ΠΌΠ°Π³Π½ΠΈΡ‚Π½ΠΎΠ³ΠΎ ΠΏΠΎΡ‚ΠΎΠΊΠ° ΠΈΠ½Π΄ΡƒΡ†ΠΈΡ€ΡƒΠ΅Ρ‚ ΠΏΡ€ΠΎΡ‚ΠΈΠ²ΠΎΠ΄Π΅ΠΉΡΡ‚Π²ΡƒΡŽΡ‰ΡƒΡŽ этому измСнСнию Π­Π”Π‘ — процСсс, извСстный ΠΊΠ°ΠΊ индукция. Π”Π²ΠΈΠΆΠ΅Π½ΠΈΠ΅ — ΠΎΠ΄Π½Π° ΠΈΠ· основных ΠΏΡ€ΠΈΡ‡ΠΈΠ½ ΠΈΠ½Π΄ΡƒΠΊΡ†ΠΈΠΈ. НапримСр, ΠΌΠ°Π³Π½ΠΈΡ‚, ΠΏΠ΅Ρ€Π΅ΠΌΠ΅Ρ‰Π΅Π½Π½Ρ‹ΠΉ ΠΊ ΠΊΠ°Ρ‚ΡƒΡˆΠΊΠ΅, ΠΈΠ½Π΄ΡƒΡ†ΠΈΡ€ΡƒΠ΅Ρ‚ Π­Π”Π‘, Π° ΠΊΠ°Ρ‚ΡƒΡˆΠΊΠ°, пСрСмСщСнная ΠΊ ΠΌΠ°Π³Π½ΠΈΡ‚Ρƒ, создаСт Π°Π½Π°Π»ΠΎΠ³ΠΈΡ‡Π½ΡƒΡŽ Π­Π”Π‘. Π’ этом Ρ€Π°Π·Π΄Π΅Π»Π΅ ΠΌΡ‹ сосрСдоточимся Π½Π° Π΄Π²ΠΈΠΆΠ΅Π½ΠΈΠΈ Π² ΠΌΠ°Π³Π½ΠΈΡ‚Π½ΠΎΠΌ ΠΏΠΎΠ»Π΅, ΠΊΠΎΡ‚ΠΎΡ€ΠΎΠ΅ являСтся стационарным ΠΎΡ‚Π½ΠΎΡΠΈΡ‚Π΅Π»ΡŒΠ½ΠΎ Π—Π΅ΠΌΠ»ΠΈ, производя Ρ‚ΠΎ, Ρ‡Ρ‚ΠΎ Π² ΠΎΠ±Ρ‰ΠΈΡ… Ρ‡Π΅Ρ€Ρ‚Π°Ρ… называСтся Π­Π”Π‘ двиТСния . Одна ситуация, ΠΊΠΎΠ³Π΄Π° Π²ΠΎΠ·Π½ΠΈΠΊΠ°Π΅Ρ‚ Π΄Π²ΠΈΠ³Π°Ρ‚Π΅Π»ΡŒΠ½Π°Ρ Π­Π”Π‘, извСстна ΠΊΠ°ΠΊ эффСкт Π₯ΠΎΠ»Π»Π° ΠΈ ΡƒΠΆΠ΅ Π±Ρ‹Π»Π° исслСдована.Заряды, двиТущиСся Π² ΠΌΠ°Π³Π½ΠΈΡ‚Π½ΠΎΠΌ ΠΏΠΎΠ»Π΅, ΠΈΡΠΏΡ‹Ρ‚Ρ‹Π²Π°ΡŽΡ‚ ΠΌΠ°Π³Π½ΠΈΡ‚Π½ΡƒΡŽ силу F = qvB sin ΞΈ , которая ΠΏΠ΅Ρ€Π΅ΠΌΠ΅Ρ‰Π°Π΅Ρ‚ ΠΏΡ€ΠΎΡ‚ΠΈΠ²ΠΎΠΏΠΎΠ»ΠΎΠΆΠ½Ρ‹Π΅ заряды Π² ΠΏΡ€ΠΎΡ‚ΠΈΠ²ΠΎΠΏΠΎΠ»ΠΎΠΆΠ½Ρ‹Ρ… направлСниях ΠΈ создаСт em f = Bβ„“v . ΠœΡ‹ Π²ΠΈΠ΄Π΅Π»ΠΈ, Ρ‡Ρ‚ΠΎ Ρƒ эффСкта Π₯ΠΎΠ»Π»Π° Π΅ΡΡ‚ΡŒ прилоТСния, Π²ΠΊΠ»ΡŽΡ‡Π°Ρ измСрСния B ΠΈ v . Π’Π΅ΠΏΠ΅Ρ€ΡŒ ΠΌΡ‹ ΡƒΠ²ΠΈΠ΄ΠΈΠΌ, Ρ‡Ρ‚ΠΎ эффСкт Π₯ΠΎΠ»Π»Π° являСтся ΠΎΠ΄Π½ΠΈΠΌ ΠΈΠ· аспСктов Π±ΠΎΠ»Π΅Π΅ ΡˆΠΈΡ€ΠΎΠΊΠΎΠ³ΠΎ явлСния ΠΈΠ½Π΄ΡƒΠΊΡ†ΠΈΠΈ, ΠΈ ΠΌΡ‹ ΠΎΠ±Π½Π°Ρ€ΡƒΠΆΠΈΠΌ, Ρ‡Ρ‚ΠΎ Π­Π”Π‘ двиТСния ΠΌΠΎΠΆΠ΅Ρ‚ ΠΈΡΠΏΠΎΠ»ΡŒΠ·ΠΎΠ²Π°Ρ‚ΡŒΡΡ ΠΊΠ°ΠΊ источник энСргии. Рассмотрим ΡΠΈΡ‚ΡƒΠ°Ρ†ΠΈΡŽ, ΠΏΠΎΠΊΠ°Π·Π°Π½Π½ΡƒΡŽ Π½Π° рисункС 1.Π‘Ρ‚Π΅Ρ€ΠΆΠ΅Π½ΡŒ пСрСмСщаСтся со ΡΠΊΠΎΡ€ΠΎΡΡ‚ΡŒΡŽ v ΠΏΠΎ ΠΏΠ°Ρ€Π΅ проводящих Ρ€Π΅Π»ΡŒΡΠΎΠ², Ρ€Π°Π·Π΄Π΅Π»Π΅Π½Π½Ρ‹Ρ… расстояниСм β„“ Π² ΠΎΠ΄Π½ΠΎΡ€ΠΎΠ΄Π½ΠΎΠΌ ΠΌΠ°Π³Π½ΠΈΡ‚Π½ΠΎΠΌ ΠΏΠΎΠ»Π΅ B . Π Π΅ΠΉΠΊΠΈ Π½Π΅ΠΏΠΎΠ΄Π²ΠΈΠΆΠ½Ρ‹ ΠΎΡ‚Π½ΠΎΡΠΈΡ‚Π΅Π»ΡŒΠ½ΠΎ B ΠΈ ΠΏΠΎΠ΄ΠΊΠ»ΡŽΡ‡Π΅Π½Ρ‹ ΠΊ Π½Π΅ΠΏΠΎΠ΄Π²ΠΈΠΆΠ½ΠΎΠΌΡƒ рСзистору R . РСзистором ΠΌΠΎΠΆΠ΅Ρ‚ Π±Ρ‹Ρ‚ΡŒ Ρ‡Ρ‚ΠΎ ΡƒΠ³ΠΎΠ΄Π½ΠΎ, ΠΎΡ‚ Π»Π°ΠΌΠΏΠΎΡ‡ΠΊΠΈ Π΄ΠΎ Π²ΠΎΠ»ΡŒΡ‚ΠΌΠ΅Ρ‚Ρ€Π°. Рассмотрим ΠΏΠ»ΠΎΡ‰Π°Π΄ΡŒ, ΠΎΠ³Ρ€Π°Π½ΠΈΡ‡Π΅Π½Π½ΡƒΡŽ ΠΏΠΎΠ΄Π²ΠΈΠΆΠ½Ρ‹ΠΌ стСрТнСм, Π½Π°ΠΏΡ€Π°Π²Π»ΡΡŽΡ‰ΠΈΠΌΠΈ ΠΈ рСзистором. B пСрпСндикулярно этой области, ΠΈ ΠΏΠ»ΠΎΡ‰Π°Π΄ΡŒ увСличиваСтся ΠΏΠΎ ΠΌΠ΅Ρ€Π΅ пСрСмСщСния стСрТня. Π’Π°ΠΊΠΈΠΌ ΠΎΠ±Ρ€Π°Π·ΠΎΠΌ, увСличиваСтся ΠΌΠ°Π³Π½ΠΈΡ‚Π½Ρ‹ΠΉ ΠΏΠΎΡ‚ΠΎΠΊ ΠΌΠ΅ΠΆΠ΄Ρƒ Ρ€Π΅Π»ΡŒΡΠ°ΠΌΠΈ, стСрТнСм ΠΈ рСзистором.Когда ΠΏΠΎΡ‚ΠΎΠΊ измСняСтся, Π²ΠΎΠ·Π½ΠΈΠΊΠ°Π΅Ρ‚ Π­Π”Π‘ согласно Π·Π°ΠΊΠΎΠ½Ρƒ ΠΈΠ½Π΄ΡƒΠΊΡ†ΠΈΠΈ ЀарадСя.

    Рис. 1. (a) ΠœΠ΅ΠΆΠ΄Ρƒ Ρ€Π΅Π»ΡŒΡΠ°ΠΌΠΈ Π²ΠΎΠ·Π½ΠΈΠΊΠ°Π΅Ρ‚ Π­Π”Π‘ двиТСния = B β„“ v , ΠΊΠΎΠ³Π΄Π° этот ΡΡ‚Π΅Ρ€ΠΆΠ΅Π½ΡŒ пСрСмСщаСтся Π²ΠΏΡ€Π°Π²ΠΎ Π² ΠΎΠ΄Π½ΠΎΡ€ΠΎΠ΄Π½ΠΎΠΌ ΠΌΠ°Π³Π½ΠΈΡ‚Π½ΠΎΠΌ ΠΏΠΎΠ»Π΅. ΠœΠ°Π³Π½ΠΈΡ‚Π½ΠΎΠ΅ ΠΏΠΎΠ»Π΅ B, находится Π²Π½ΡƒΡ‚Ρ€ΠΈ страницы, пСрпСндикулярно двиТущСмуся ΡΡ‚Π΅Ρ€ΠΆΠ½ΡŽ ΠΈ Ρ€Π΅Π»ΡŒΡΠ°ΠΌ ΠΈ, ΡΠ»Π΅Π΄ΠΎΠ²Π°Ρ‚Π΅Π»ΡŒΠ½ΠΎ, ΠΊ области, ΠΎΠΊΡ€ΡƒΠΆΠ°ΡŽΡ‰Π΅ΠΉ ΠΈΡ…. (Π±) Π—Π°ΠΊΠΎΠ½ Π›Π΅Π½Ρ†Π° опрСдСляСт направлСния ΠΈΠ½Π΄ΡƒΡ†ΠΈΡ€ΠΎΠ²Π°Π½Π½ΠΎΠ³ΠΎ поля ΠΈ Ρ‚ΠΎΠΊΠ°, Π° Ρ‚Π°ΠΊΠΆΠ΅ ΠΏΠΎΠ»ΡΡ€Π½ΠΎΡΡ‚ΡŒ Π½Π°Π²Π΅Π΄Π΅Π½Π½ΠΎΠΉ Π­Π”Π‘.ΠŸΠΎΡΠΊΠΎΠ»ΡŒΠΊΡƒ ΠΏΠΎΡ‚ΠΎΠΊ увСличиваСтся, ΠΈΠ½Π΄ΡƒΡ†ΠΈΡ€ΠΎΠ²Π°Π½Π½ΠΎΠ΅ ΠΏΠΎΠ»Π΅ Π½Π°ΠΏΡ€Π°Π²Π»Π΅Π½ΠΎ Π² ΠΏΡ€ΠΎΡ‚ΠΈΠ²ΠΎΠΏΠΎΠ»ΠΎΠΆΠ½ΠΎΠΌ Π½Π°ΠΏΡ€Π°Π²Π»Π΅Π½ΠΈΠΈ ΠΈΠ»ΠΈ Π·Π° ΠΏΡ€Π΅Π΄Π΅Π»Ρ‹ страницы. RHR-2 Π΄Π°Π΅Ρ‚ ΡƒΠΊΠ°Π·Π°Π½Π½ΠΎΠ΅ Π½Π°ΠΏΡ€Π°Π²Π»Π΅Π½ΠΈΠ΅ Ρ‚ΠΎΠΊΠ°, ΠΈ ΠΏΠΎΠ»ΡΡ€Π½ΠΎΡΡ‚ΡŒ стСрТня Π±ΡƒΠ΄Π΅Ρ‚ ΡƒΠΏΡ€Π°Π²Π»ΡΡ‚ΡŒ Ρ‚Π°ΠΊΠΈΠΌ Ρ‚ΠΎΠΊΠΎΠΌ. RHR-1 Ρ‚Π°ΠΊΠΆΠ΅ ΡƒΠΊΠ°Π·Ρ‹Π²Π°Π΅Ρ‚ Π½Π° Ρ‚Π°ΠΊΡƒΡŽ ​​ТС ΠΏΠΎΠ»ΡΡ€Π½ΠΎΡΡ‚ΡŒ стСрТня. (ΠžΠ±Ρ€Π°Ρ‚ΠΈΡ‚Π΅ Π²Π½ΠΈΠΌΠ°Π½ΠΈΠ΅, Ρ‡Ρ‚ΠΎ символ Π±ΡƒΠΊΠ²Ρ‹ E, ΠΈΡΠΏΠΎΠ»ΡŒΠ·ΡƒΠ΅ΠΌΡ‹ΠΉ Π² эквивалСнтной схСмС Π² Π½ΠΈΠΆΠ½Π΅ΠΉ части части (b), прСдставляСт собой Π­Π”Π‘. )

    Π§Ρ‚ΠΎΠ±Ρ‹ Π½Π°ΠΉΡ‚ΠΈ Π²Π΅Π»ΠΈΡ‡ΠΈΠ½Ρƒ Π­Π”Π‘, ΠΈΠ½Π΄ΡƒΡ†ΠΈΡ€ΠΎΠ²Π°Π½Π½ΠΎΠΉ вдоль двиТущСгося стСрТня, Π²ΠΎΡΠΏΠΎΠ»ΡŒΠ·ΡƒΠ΅ΠΌΡΡ Π·Π°ΠΊΠΎΠ½ΠΎΠΌ ΠΈΠ½Π΄ΡƒΠΊΡ†ΠΈΠΈ ЀарадСя Π±Π΅Π· Π·Π½Π°ΠΊΠ°:

    [латСкс] \ text {emf} = \ text {N} \ frac {\ Delta \ Phi} {\ Delta t} \\ [/ latex].

    Π—Π΄Π΅ΡΡŒ ΠΈ Π΄Π°Π»Π΅Π΅ Β«Π­Π”Π‘Β» ΠΎΠ·Π½Π°Ρ‡Π°Π΅Ρ‚ Π²Π΅Π»ΠΈΡ‡ΠΈΠ½Ρƒ Π­Π”Π‘. Π’ этом ΡƒΡ€Π°Π²Π½Π΅Π½ΠΈΠΈ N = 1 ΠΈ ΠΏΠΎΡ‚ΠΎΠΊ Ξ¦ = BA cos ΞΈ . Π£ нас ΞΈ = 0ΒΊ ΠΈ cos ΞΈ = 1, Ρ‚Π°ΠΊ ΠΊΠ°ΠΊ B пСрпСндикулярно A . Π’Π΅ΠΏΠ΅Ρ€ΡŒ Ξ” Ξ¦ = Ξ” ( BA ) = B Ξ” A , ΠΏΠΎΡΠΊΠΎΠ»ΡŒΠΊΡƒ B являСтся ΠΎΠ΄Π½ΠΎΡ€ΠΎΠ΄Π½Ρ‹ΠΌ. ΠžΠ±Ρ€Π°Ρ‚ΠΈΡ‚Π΅ Π²Π½ΠΈΠΌΠ°Π½ΠΈΠ΅, Ρ‡Ρ‚ΠΎ ΠΏΠ»ΠΎΡ‰Π°Π΄ΡŒ, замСтаСмая стСрТнСм, составляСт Ξ” A = β„“ Ξ” x .Π’Π²ΠΎΠ΄ этих Π²Π΅Π»ΠΈΡ‡ΠΈΠ½ Π² Π²Ρ‹Ρ€Π°ΠΆΠ΅Π½ΠΈΠ΅ для Π­Π”Π‘ Π΄Π°Π΅Ρ‚

    [латСкс] \ text {emf} = \ frac {B \ Delta A} {\ Delta t} = B \ frac {\ ell \ Delta x} {\ Delta t} \\ [/ latex].

    НаконСц, ΠΎΠ±Ρ€Π°Ρ‚ΠΈΡ‚Π΅ Π²Π½ΠΈΠΌΠ°Π½ΠΈΠ΅, Ρ‡Ρ‚ΠΎ Ξ” x / Ξ” t = v , ΡΠΊΠΎΡ€ΠΎΡΡ‚ΡŒ стСрТня. Π’Π²ΠΎΠ΄ этого Π² послСднСС Π²Ρ‹Ρ€Π°ΠΆΠ΅Π½ΠΈΠ΅ ΠΏΠΎΠΊΠ°Π·Ρ‹Π²Π°Π΅Ρ‚, Ρ‡Ρ‚ΠΎ

    Π­Π”Π‘ = Bβ„“v ( B , β„“ ΠΈ v пСрпСндикулярно)

    — Π­Π”Π‘ двиТСния. Π­Ρ‚ΠΎ Ρ‚ΠΎ ΠΆΠ΅ Π²Ρ‹Ρ€Π°ΠΆΠ΅Π½ΠΈΠ΅, Ρ‡Ρ‚ΠΎ ΠΈ для эффСкта Π₯ΠΎΠ»Π»Π° Ρ€Π°Π½Π΅Π΅.

    БоСдинСния: объСдинСниС сил

    ΠœΠ΅ΠΆΠ΄Ρƒ элСктричСской ΠΈ ΠΌΠ°Π³Π½ΠΈΡ‚Π½ΠΎΠΉ силой сущСствуСт мноТСство связСй. Π’ΠΎΡ‚ Ρ„Π°ΠΊΡ‚, Ρ‡Ρ‚ΠΎ двиТущССся элСктричСскоС ΠΏΠΎΠ»Π΅ создаСт ΠΌΠ°Π³Π½ΠΈΡ‚Π½ΠΎΠ΅ ΠΏΠΎΠ»Π΅, ΠΈ, Π½Π°ΠΎΠ±ΠΎΡ€ΠΎΡ‚, двиТущССся ΠΌΠ°Π³Π½ΠΈΡ‚Π½ΠΎΠ΅ ΠΏΠΎΠ»Π΅ создаСт элСктричСскоС ΠΏΠΎΠ»Π΅, являСтся Ρ‡Π°ΡΡ‚ΡŒΡŽ Ρ‚ΠΎΠ³ΠΎ, ΠΏΠΎΡ‡Π΅ΠΌΡƒ элСктричСскиС ΠΈ ΠΌΠ°Π³Π½ΠΈΡ‚Π½Ρ‹Π΅ силы Ρ‚Π΅ΠΏΠ΅Ρ€ΡŒ ΡΡ‡ΠΈΡ‚Π°ΡŽΡ‚ΡΡ Ρ€Π°Π·Π»ΠΈΡ‡Π½Ρ‹ΠΌΠΈ проявлСниями ΠΎΠ΄Π½ΠΎΠΉ ΠΈ Ρ‚ΠΎΠΉ ΠΆΠ΅ силы. Π­Ρ‚ΠΎ классичСскоС объСдинСниС элСктричСских ΠΈ ΠΌΠ°Π³Π½ΠΈΡ‚Π½Ρ‹Ρ… сил Π² Ρ‚Π°ΠΊ Π½Π°Π·Ρ‹Π²Π°Π΅ΠΌΡƒΡŽ ΡΠ»Π΅ΠΊΡ‚Ρ€ΠΎΠΌΠ°Π³Π½ΠΈΡ‚Π½ΡƒΡŽ силу являСтся источником вдохновСния для соврСмСнных усилий ΠΏΠΎ объСдинСнию Π΄Ρ€ΡƒΠ³ΠΈΡ… основных сил.

    Π§Ρ‚ΠΎΠ±Ρ‹ Π½Π°ΠΉΡ‚ΠΈ Π½Π°ΠΏΡ€Π°Π²Π»Π΅Π½ΠΈΠ΅ ΠΈΠ½Π΄ΡƒΡ†ΠΈΡ€ΠΎΠ²Π°Π½Π½ΠΎΠ³ΠΎ поля, Π½Π°ΠΏΡ€Π°Π²Π»Π΅Π½ΠΈΠ΅ Ρ‚ΠΎΠΊΠ° ΠΈ ΠΏΠΎΠ»ΡΡ€Π½ΠΎΡΡ‚ΡŒ Π½Π°Π²Π΅Π΄Π΅Π½Π½ΠΎΠΉ Π­Π”Π‘, ΠΌΡ‹ примСняСм Π·Π°ΠΊΠΎΠ½ Π›Π΅Π½Ρ†Π°, ΠΊΠ°ΠΊ объяснСно Π² Π—Π°ΠΊΠΎΠ½Π΅ ΠΈΠ½Π΄ΡƒΠΊΡ†ΠΈΠΈ ЀарадСя: Π—Π°ΠΊΠΎΠ½ Π›Π΅Π½Ρ†Π°. (Π‘ΠΌ. Рис. 1 (b).) ΠŸΠΎΡ‚ΠΎΠΊ увСличиваСтся, Ρ‚Π°ΠΊ ΠΊΠ°ΠΊ увСличиваСтся ΠΏΠ»ΠΎΡ‰Π°Π΄ΡŒ Π·Π°ΠΌΠΊΠ½ΡƒΡ‚ΠΎΠΉ области. Π’Π°ΠΊΠΈΠΌ ΠΎΠ±Ρ€Π°Π·ΠΎΠΌ, ΠΈΠ½Π΄ΡƒΡ†ΠΈΡ€ΠΎΠ²Π°Π½Π½ΠΎΠ΅ ΠΏΠΎΠ»Π΅ Π΄ΠΎΠ»ΠΆΠ½ΠΎ ΠΏΡ€ΠΎΡ‚ΠΈΠ²ΠΎΡΡ‚ΠΎΡΡ‚ΡŒ ΡΡƒΡ‰Π΅ΡΡ‚Π²ΡƒΡŽΡ‰Π΅ΠΌΡƒ ΠΈ Π±Ρ‹Ρ‚ΡŒ Π²Π½Π΅ страницы. Π’Π°ΠΊΠΈΠΌ ΠΎΠ±Ρ€Π°Π·ΠΎΠΌ, RHR-2 Ρ‚Ρ€Π΅Π±ΡƒΠ΅Ρ‚, Ρ‡Ρ‚ΠΎΠ±Ρ‹ I Π²Ρ€Π°Ρ‰Π°Π»ΠΈΡΡŒ ΠΏΡ€ΠΎΡ‚ΠΈΠ² часовой стрСлки, Ρ‡Ρ‚ΠΎ, Π² свою ΠΎΡ‡Π΅Ρ€Π΅Π΄ΡŒ, ΠΎΠ·Π½Π°Ρ‡Π°Π΅Ρ‚, Ρ‡Ρ‚ΠΎ вСрхняя Ρ‡Π°ΡΡ‚ΡŒ стСрТня ΠΏΠΎΠ»ΠΎΠΆΠΈΡ‚Π΅Π»ΡŒΠ½Π°, ΠΊΠ°ΠΊ ΠΏΠΎΠΊΠ°Π·Π°Π½ΠΎ.

    Π­Π”Π‘ двиТСния Ρ‚Π°ΠΊΠΆΠ΅ Π²ΠΎΠ·Π½ΠΈΠΊΠ°Π΅Ρ‚, Ссли ΠΌΠ°Π³Π½ΠΈΡ‚Π½ΠΎΠ΅ ΠΏΠΎΠ»Π΅ двиТСтся ΠΈ ΡΡ‚Π΅Ρ€ΠΆΠ΅Π½ΡŒ (ΠΈΠ»ΠΈ Π΄Ρ€ΡƒΠ³ΠΎΠΉ ΠΎΠ±ΡŠΠ΅ΠΊΡ‚) Π½Π΅ΠΏΠΎΠ΄Π²ΠΈΠΆΠ΅Π½ ΠΎΡ‚Π½ΠΎΡΠΈΡ‚Π΅Π»ΡŒΠ½ΠΎ Π—Π΅ΠΌΠ»ΠΈ (ΠΈΠ»ΠΈ Π½Π΅ΠΊΠΎΡ‚ΠΎΡ€ΠΎΠ³ΠΎ Π½Π°Π±Π»ΡŽΠ΄Π°Ρ‚Π΅Π»Ρ).ΠœΡ‹ Π²ΠΈΠ΄Π΅Π»ΠΈ ΠΏΡ€ΠΈΠΌΠ΅Ρ€ этого Π² ситуации, ΠΊΠΎΠ³Π΄Π° двиТущийся ΠΌΠ°Π³Π½ΠΈΡ‚ ΠΈΠ½Π΄ΡƒΡ†ΠΈΡ€ΡƒΠ΅Ρ‚ Π­Π”Π‘ Π² Π½Π΅ΠΏΠΎΠ΄Π²ΠΈΠΆΠ½ΠΎΠΉ ΠΊΠ°Ρ‚ΡƒΡˆΠΊΠ΅. Π’Π°ΠΆΠ½ΠΎ ΠΎΡ‚Π½ΠΎΡΠΈΡ‚Π΅Π»ΡŒΠ½ΠΎΠ΅ Π΄Π²ΠΈΠΆΠ΅Π½ΠΈΠ΅. Π’ этих Π½Π°Π±Π»ΡŽΠ΄Π΅Π½ΠΈΡΡ… обнаруТиваСтся связь ΠΌΠ΅ΠΆΠ΄Ρƒ ΠΌΠ°Π³Π½ΠΈΡ‚Π½Ρ‹ΠΌ ΠΈ элСктричСским полями. ДвиТущССся ΠΌΠ°Π³Π½ΠΈΡ‚Π½ΠΎΠ΅ ΠΏΠΎΠ»Π΅ создаСт элСктричСскоС ΠΏΠΎΠ»Π΅ Π·Π° счСт Π½Π°Π²Π΅Π΄Π΅Π½Π½ΠΎΠΉ Π­Π”Π‘. ΠœΡ‹ ΡƒΠΆΠ΅ Π²ΠΈΠ΄Π΅Π»ΠΈ, Ρ‡Ρ‚ΠΎ двиТущССся элСктричСскоС ΠΏΠΎΠ»Π΅ создаСт ΠΌΠ°Π³Π½ΠΈΡ‚Π½ΠΎΠ΅ ΠΏΠΎΠ»Π΅: двиТущийся заряд ΠΏΠΎΠ΄Ρ€Π°Π·ΡƒΠΌΠ΅Π²Π°Π΅Ρ‚ двиТущССся элСктричСскоС ΠΏΠΎΠ»Π΅, Π° двиТущийся заряд создаСт ΠΌΠ°Π³Π½ΠΈΡ‚Π½ΠΎΠ΅ ΠΏΠΎΠ»Π΅.

    Π­Π”Π‘ двиТСния Π² слабом ΠΌΠ°Π³Π½ΠΈΡ‚Π½ΠΎΠΌ ΠΏΠΎΠ»Π΅ Π—Π΅ΠΌΠ»ΠΈ ΠΎΠ±Ρ‹Ρ‡Π½ΠΎ Π½Π΅ ΠΎΡ‡Π΅Π½ΡŒ Π²Π΅Π»ΠΈΠΊΠΈ, ΠΈΠ½Π°Ρ‡Π΅ ΠΌΡ‹ ΠΌΠΎΠ³Π»ΠΈ Π±Ρ‹ Π·Π°ΠΌΠ΅Ρ‚ΠΈΡ‚ΡŒ напряТСниС вдоль мСталличСских стСрТнСй, Ρ‚Π°ΠΊΠΈΡ… ΠΊΠ°ΠΊ ΠΎΡ‚Π²Π΅Ρ€Ρ‚ΠΊΠ°, Π²ΠΎ врСмя ΠΎΠ±Ρ‹Ρ‡Π½Ρ‹Ρ… Π΄Π²ΠΈΠΆΠ΅Π½ΠΈΠΉ.НапримСр, простой расчСт Π­Π”Π‘ двиТСния стСрТня Π΄Π»ΠΈΠ½ΠΎΠΉ 1 ΠΌ, двиТущСгося со ΡΠΊΠΎΡ€ΠΎΡΡ‚ΡŒΡŽ 3,0 ΠΌ / с пСрпСндикулярно полю Π—Π΅ΠΌΠ»ΠΈ, Π΄Π°Π΅Ρ‚ Π­Π”Π‘ = Bβ„“v = (5,0 Γ— 10 βˆ’5 Π’Π») (1,0 ΠΌ) (3,0 ΠΌ / с) = 150 ΠΌΠΊΠ’. Π­Ρ‚ΠΎ нСбольшоС Π·Π½Π°Ρ‡Π΅Π½ΠΈΠ΅ согласуСтся с ΠΎΠΏΡ‹Ρ‚ΠΎΠΌ. Однако Π΅ΡΡ‚ΡŒ Π²ΠΏΠ΅Ρ‡Π°Ρ‚Π»ΡΡŽΡ‰Π΅Π΅ ΠΈΡΠΊΠ»ΡŽΡ‡Π΅Π½ΠΈΠ΅. Π’ 1992 ΠΈ 1996 Π³ΠΎΠ΄Π°Ρ… с космичСским ΡˆΠ°Ρ‚Ρ‚Π»ΠΎΠΌ Π±Ρ‹Π»ΠΈ прСдприняты ΠΏΠΎΠΏΡ‹Ρ‚ΠΊΠΈ ΡΠΎΠ·Π΄Π°Ρ‚ΡŒ большиС Π΄Π²ΠΈΠ³Π°Ρ‚Π΅Π»ΡŒΠ½Ρ‹Π΅ Π­Π”Π‘. ΠŸΡ€ΠΈΠ²ΡΠ·Π°Π½Π½Ρ‹ΠΉ спутник Π΄ΠΎΠ»ΠΆΠ΅Π½ Π±Ρ‹Π» Π±Ρ‹Ρ‚ΡŒ Π²Ρ‹ΠΏΡƒΡ‰Π΅Π½ Π½Π° ΠΏΡ€ΠΎΠ²ΠΎΠ΄Π΅ Π΄Π»ΠΈΠ½ΠΎΠΉ 20 ΠΊΠΌ, ΠΊΠ°ΠΊ ΠΏΠΎΠΊΠ°Π·Π°Π½ΠΎ Π½Π° рисункС 2, для создания Π­Π”Π‘ 5 ΠΊΠ’ Π·Π° счСт двиТСния с ΠΎΡ€Π±ΠΈΡ‚Π°Π»ΡŒΠ½ΠΎΠΉ ΡΠΊΠΎΡ€ΠΎΡΡ‚ΡŒΡŽ Ρ‡Π΅Ρ€Π΅Π· ΠΏΠΎΠ»Π΅ Π—Π΅ΠΌΠ»ΠΈ.Π­Ρ‚Ρƒ Π­Π”Π‘ ΠΌΠΎΠΆΠ½ΠΎ Π±Ρ‹Π»ΠΎ Π±Ρ‹ ΠΈΡΠΏΠΎΠ»ΡŒΠ·ΠΎΠ²Π°Ρ‚ΡŒ для прСобразования Π½Π΅ΠΊΠΎΡ‚ΠΎΡ€ΠΎΠΉ кинСтичСской ΠΈ ΠΏΠΎΡ‚Π΅Π½Ρ†ΠΈΠ°Π»ΡŒΠ½ΠΎΠΉ энСргии ΡˆΠ°Ρ‚Ρ‚Π»Π° Π² ΡΠ»Π΅ΠΊΡ‚Ρ€ΠΈΡ‡Π΅ΡΠΊΡƒΡŽ, Ссли Π±Ρ‹ ΠΌΠΎΠΆΠ½ΠΎ Π±Ρ‹Π»ΠΎ ΡΠΎΠ·Π΄Π°Ρ‚ΡŒ ΠΏΠΎΠ»Π½ΡƒΡŽ схСму. Π§Ρ‚ΠΎΠ±Ρ‹ Π·Π°ΠΌΠΊΠ½ΡƒΡ‚ΡŒ Ρ†Π΅ΠΏΡŒ, нСподвиТная ионосфСра Π΄ΠΎΠ»ΠΆΠ½Π° Π±Ρ‹Π»Π° ΠΎΠ±Π΅ΡΠΏΠ΅Ρ‡ΠΈΡ‚ΡŒ ΠΎΠ±Ρ€Π°Ρ‚Π½Ρ‹ΠΉ ΠΏΡƒΡ‚ΡŒ для протСкания Ρ‚ΠΎΠΊΠ°. (Π˜ΠΎΠ½ΠΎΡΡ„Π΅Ρ€Π° — это разрСТСнная ΠΈ частично ионизированная атмосфСра Π½Π° ΠΎΡ€Π±ΠΈΡ‚Π°Π»ΡŒΠ½Ρ‹Ρ… высотах. Она ΠΏΡ€ΠΎΠ²ΠΎΠ΄ΠΈΡ‚ ΠΈΠ·-Π·Π° ΠΈΠΎΠ½ΠΈΠ·Π°Ρ†ΠΈΠΈ. Π˜ΠΎΠ½ΠΎΡΡ„Π΅Ρ€Π° выполняСт Ρ‚Ρƒ ΠΆΠ΅ Ρ„ΡƒΠ½ΠΊΡ†ΠΈΡŽ, Ρ‡Ρ‚ΠΎ ΠΈ Π½Π΅ΠΏΠΎΠ΄Π²ΠΈΠΆΠ½Ρ‹Π΅ Ρ€Π΅Π»ΡŒΡΡ‹ ΠΈ ΡΠΎΠ΅Π΄ΠΈΠ½ΠΈΡ‚Π΅Π»ΡŒΠ½Ρ‹ΠΉ рСзистор Π½Π° рисункС 1, Π±Π΅Π· ΠΊΠΎΡ‚ΠΎΡ€Ρ‹Ρ… Π½Π΅ Π±Ρ‹Π»ΠΎ Π±Ρ‹ ΠΏΠΎΠ»Π½ΠΎΠΉ Ρ†Π΅ΠΏΠΈ.) ЗатягиваниС Ρ‚ΠΎΠΊΠ° Π² ΠΊΠ°Π±Π΅Π»Π΅ ΠΈΠ·-Π·Π° ΠΌΠ°Π³Π½ΠΈΡ‚Π½ΠΎΠΉ силы F = Iβ„“B sin ΞΈ выполняСт Ρ€Π°Π±ΠΎΡ‚Ρƒ, которая ΡƒΠΌΠ΅Π½ΡŒΡˆΠ°Π΅Ρ‚ ΠΊΠΈΠ½Π΅Ρ‚ΠΈΡ‡Π΅ΡΠΊΡƒΡŽ ΠΈ ΠΏΠΎΡ‚Π΅Π½Ρ†ΠΈΠ°Π»ΡŒΠ½ΡƒΡŽ ΡΠ½Π΅Ρ€Π³ΠΈΡŽ ΡˆΠ°Ρ‚Ρ‚Π»Π° ΠΈ позволяСт ΠΏΡ€Π΅ΠΎΠ±Ρ€Π°Π·ΠΎΠ²Ρ‹Π²Π°Ρ‚ΡŒ Π΅Π΅ Π² ΡΠ»Π΅ΠΊΡ‚Ρ€ΠΈΡ‡Π΅ΡΠΊΡƒΡŽ. Оба тСста Π±Ρ‹Π»ΠΈ Π±Π΅Π·ΡƒΡΠΏΠ΅ΡˆΠ½Ρ‹ΠΌΠΈ. Π’ ΠΏΠ΅Ρ€Π²ΠΎΠΌ случаС кабСль зависал, ΠΈ Π΅Π³ΠΎ ΠΌΠΎΠΆΠ½ΠΎ Π±Ρ‹Π»ΠΎ ΠΏΡ€ΠΎΡ‚ΡΠ½ΡƒΡ‚ΡŒ Ρ‚ΠΎΠ»ΡŒΠΊΠΎ Π½Π° ΠΏΠ°Ρ€Ρƒ сотСн ΠΌΠ΅Ρ‚Ρ€ΠΎΠ²; Π²ΠΎ Π²Ρ‚ΠΎΡ€ΠΎΠΌ трос оборвался ΠΏΡ€ΠΈ ΠΏΠΎΡ‡Ρ‚ΠΈ ΠΏΠΎΠ»Π½ΠΎΠΌ растяТСнии. Π‘Π»Π΅Π΄ΡƒΡŽΡ‰ΠΈΠΉ ΠΏΡ€ΠΈΠΌΠ΅Ρ€ ΠΏΠΎΠΊΠ°Π·Ρ‹Π²Π°Π΅Ρ‚ ΠΏΡ€ΠΈΠ½Ρ†ΠΈΠΏΠΈΠ°Π»ΡŒΠ½ΡƒΡŽ ΠΎΡΡƒΡ‰Π΅ΡΡ‚Π²ΠΈΠΌΠΎΡΡ‚ΡŒ.

    ΠŸΡ€ΠΈΠΌΠ΅Ρ€ 1. РасчСт большой Π­Π”Π‘ двиТСния ΠΎΠ±ΡŠΠ΅ΠΊΡ‚Π° Π½Π° ΠΎΡ€Π±ΠΈΡ‚Π΅

    Рисунок 2.Π­Π”Π‘ двиТСния ΠΊΠ°ΠΊ ΠΏΡ€Π΅ΠΎΠ±Ρ€Π°Π·ΠΎΠ²Π°Π½ΠΈΠ΅ элСктроэнСргии для космичСского Ρ‡Π΅Π»Π½ΠΎΠΊΠ° являСтся ΠΌΠΎΡ‚ΠΈΠ²Π°Ρ†ΠΈΠ΅ΠΉ для экспСримСнта с привязанным спутником. Богласно ΠΏΡ€ΠΎΠ³Π½ΠΎΠ·Π°ΠΌ, Π­Π”Π‘ 5 ΠΊΠ’ Π±ΡƒΠ΄Π΅Ρ‚ ΠΈΠ½Π΄ΡƒΡ†ΠΈΡ€ΠΎΠ²Π°Ρ‚ΡŒΡΡ Π² тросС Π΄Π»ΠΈΠ½ΠΎΠΉ 20 ΠΊΠΌ ΠΏΡ€ΠΈ Π΄Π²ΠΈΠΆΠ΅Π½ΠΈΠΈ с ΠΎΡ€Π±ΠΈΡ‚Π°Π»ΡŒΠ½ΠΎΠΉ ΡΠΊΠΎΡ€ΠΎΡΡ‚ΡŒΡŽ Π² ΠΌΠ°Π³Π½ΠΈΡ‚Π½ΠΎΠΌ ΠΏΠΎΠ»Π΅ Π—Π΅ΠΌΠ»ΠΈ. Π‘Ρ…Π΅ΠΌΠ° замыкаСтся ΠΎΠ±Ρ€Π°Ρ‚Π½Ρ‹ΠΌ Ρ‚Ρ€Π°ΠΊΡ‚ΠΎΠΌ Ρ‡Π΅Ρ€Π΅Π· Π½Π΅ΠΏΠΎΠ΄Π²ΠΈΠΆΠ½ΡƒΡŽ ионосфСру.

    РассчитайтС Π­Π”Π‘ двиТСния, ΠΈΠ½Π΄ΡƒΡ†ΠΈΡ€ΠΎΠ²Π°Π½Π½ΡƒΡŽ вдоль ΠΏΡ€ΠΎΠ²ΠΎΠ΄Π½ΠΈΠΊΠ° Π΄Π»ΠΈΠ½ΠΎΠΉ 20,0 ΠΊΠΌ, двиТущСгося с ΠΎΡ€Π±ΠΈΡ‚Π°Π»ΡŒΠ½ΠΎΠΉ ΡΠΊΠΎΡ€ΠΎΡΡ‚ΡŒΡŽ 7,80 ΠΊΠΌ / с пСрпСндикулярно ΠΌΠ°Π³Π½ΠΈΡ‚Π½ΠΎΠΌΡƒ полю Π—Π΅ΠΌΠ»ΠΈ 5,00 Γ— 10 βˆ’5 Π’Π».{3} \ text {V} \ end {array} \\ [/ latex].

    ΠžΠ±ΡΡƒΠΆΠ΄Π΅Π½ΠΈΠ΅

    ΠŸΠΎΠ»ΡƒΡ‡Π΅Π½Π½ΠΎΠ΅ Π·Π½Π°Ρ‡Π΅Π½ΠΈΠ΅ ΠΏΡ€Π΅Π²Ρ‹ΡˆΠ°Π΅Ρ‚ ΠΈΠ·ΠΌΠ΅Ρ€Π΅Π½Π½ΠΎΠ΅ напряТСниС 5 ΠΊΠ’ для экспСримСнта с ΡˆΠ°Ρ‚Ρ‚Π»ΠΎΠΌ, ΠΏΠΎΡΠΊΠΎΠ»ΡŒΠΊΡƒ фактичСскоС ΠΎΡ€Π±ΠΈΡ‚Π°Π»ΡŒΠ½ΠΎΠ΅ Π΄Π²ΠΈΠΆΠ΅Π½ΠΈΠ΅ троса Π½Π΅ пСрпСндикулярно полю Π—Π΅ΠΌΠ»ΠΈ. Π—Π½Π°Ρ‡Π΅Π½ΠΈΠ΅ 7,80 ΠΊΠ’ являСтся максимальной Π­Π”Π‘, ΠΏΠΎΠ»ΡƒΡ‡Π΅Π½Π½ΠΎΠΉ ΠΏΡ€ΠΈ ΞΈ = 90ΒΊ ΠΈ sin ΞΈ = 1.

    Π‘Π²ΠΎΠ΄ΠΊΠ° Ρ€Π°Π·Π΄Π΅Π»Π°

    • Π­Π”Π‘, индуцированная Π΄Π²ΠΈΠΆΠ΅Π½ΠΈΠ΅ΠΌ ΠΎΡ‚Π½ΠΎΡΠΈΡ‚Π΅Π»ΡŒΠ½ΠΎ ΠΌΠ°Π³Π½ΠΈΡ‚Π½ΠΎΠ³ΠΎ поля B , называСтся Π­Π”Π‘ двиТСния ΠΈ опрСдСляСтся Π²Ρ‹Ρ€Π°ΠΆΠ΅Π½ΠΈΠ΅ΠΌ

      Π­Π”Π‘ = Bβ„“v ( B , β„“ ΠΈ v пСрпСндикулярно)

      , Π³Π΄Π΅ β„“ — Π΄Π»ΠΈΠ½Π° ΠΎΠ±ΡŠΠ΅ΠΊΡ‚Π°, двиТущСгося со ΡΠΊΠΎΡ€ΠΎΡΡ‚ΡŒΡŽ v ΠΎΡ‚Π½ΠΎΡΠΈΡ‚Π΅Π»ΡŒΠ½ΠΎ поля.

    ΠšΠΎΠ½Ρ†Π΅ΠΏΡ‚ΡƒΠ°Π»ΡŒΠ½Ρ‹Π΅ вопросы

    1. ΠŸΠΎΡ‡Π΅ΠΌΡƒ Ρ‡Π°ΡΡ‚ΡŒ схСмы Π΄ΠΎΠ»ΠΆΠ½Π° ΠΏΠ΅Ρ€Π΅ΠΌΠ΅Ρ‰Π°Ρ‚ΡŒΡΡ ΠΎΡ‚Π½ΠΎΡΠΈΡ‚Π΅Π»ΡŒΠ½ΠΎ Π΄Ρ€ΡƒΠ³ΠΈΡ… частСй, Ρ‡Ρ‚ΠΎΠ±Ρ‹ ΠΌΠΎΠΆΠ½ΠΎ Π±Ρ‹Π»ΠΎ ΠΈΡΠΏΠΎΠ»ΡŒΠ·ΠΎΠ²Π°Ρ‚ΡŒ Π­Π”Π‘ двиТСния? Рассмотрим, Π½Π°ΠΏΡ€ΠΈΠΌΠ΅Ρ€, Ρ‡Ρ‚ΠΎ Ρ€Π΅Π»ΡŒΡΡ‹ Π½Π° рисункС 1 Π½Π΅ΠΏΠΎΠ΄Π²ΠΈΠΆΠ½Ρ‹ ΠΎΡ‚Π½ΠΎΡΠΈΡ‚Π΅Π»ΡŒΠ½ΠΎ ΠΌΠ°Π³Π½ΠΈΡ‚Π½ΠΎΠ³ΠΎ поля, Π² Ρ‚ΠΎ врСмя ΠΊΠ°ΠΊ ΡΡ‚Π΅Ρ€ΠΆΠ΅Π½ΡŒ двиТСтся.
    2. ΠœΠΎΡ‰Π½ΡƒΡŽ ΠΈΠ½Π΄ΡƒΠΊΡ†ΠΈΠΎΠ½Π½ΡƒΡŽ ΠΏΡƒΡˆΠΊΡƒ ΠΌΠΎΠΆΠ½ΠΎ ΡΠ΄Π΅Π»Π°Ρ‚ΡŒ, помСстив мСталличСский Ρ†ΠΈΠ»ΠΈΠ½Π΄Ρ€ Π²Π½ΡƒΡ‚Ρ€ΠΈ ΠΊΠ°Ρ‚ΡƒΡˆΠΊΠΈ солСноида. Π¦ΠΈΠ»ΠΈΠ½Π΄Ρ€ ΠΏΡ€ΠΈΠ½ΡƒΠ΄ΠΈΡ‚Π΅Π»ΡŒΠ½ΠΎ выталкиваСтся ΠΏΡ€ΠΈ быстром Π²ΠΊΠ»ΡŽΡ‡Π΅Π½ΠΈΠΈ элСктромагнитного Ρ‚ΠΎΠΊΠ°. Π˜ΡΠΏΠΎΠ»ΡŒΠ·ΡƒΠΉΡ‚Π΅ Π·Π°ΠΊΠΎΠ½Ρ‹ ЀарадСя ΠΈ Π›Π΅Π½Ρ†Π°, Ρ‡Ρ‚ΠΎΠ±Ρ‹ ΠΎΠ±ΡŠΡΡΠ½ΠΈΡ‚ΡŒ, ΠΊΠ°ΠΊ это Ρ€Π°Π±ΠΎΡ‚Π°Π΅Ρ‚. ΠŸΠΎΡ‡Π΅ΠΌΡƒ Ρ†ΠΈΠ»ΠΈΠ½Π΄Ρ€ ΠΌΠΎΠΆΠ΅Ρ‚ ΡΡ‚Π°Ρ‚ΡŒ Π°ΠΊΡ‚ΠΈΠ²Π½Ρ‹ΠΌ / горячим ΠΏΡ€ΠΈ выстрСлС ΠΈΠ· ΠΏΡƒΡˆΠΊΠΈ?
    3. Π˜Π½Π΄ΡƒΠΊΡ†ΠΈΠΎΠ½Π½Π°Ρ ΠΏΠ»ΠΈΡ‚Π° Π½Π°Π³Ρ€Π΅Π²Π°Π΅Ρ‚ ΠΊΠ°ΡΡ‚Ρ€ΡŽΠ»ΡŽ с ΠΊΠ°Ρ‚ΡƒΡˆΠΊΠΎΠΉ, ΠΏΠΎ ΠΊΠΎΡ‚ΠΎΡ€ΠΎΠΉ ΠΏΡ€ΠΎΡ…ΠΎΠ΄ΠΈΡ‚ ΠΏΠ΅Ρ€Π΅ΠΌΠ΅Π½Π½Ρ‹ΠΉ Ρ‚ΠΎΠΊ, располоТСнной ΠΏΠΎΠ΄ ΠΊΠ°ΡΡ‚Ρ€ΡŽΠ»Π΅ΠΉ (ΠΈ Π±Π΅Π· горячСй повСрхности). ΠœΠΎΠΆΠ΅Ρ‚ Π»ΠΈ ΠΏΠΎΠ²Π΅Ρ€Ρ…Π½ΠΎΡΡ‚ΡŒ ΠΏΠ΅Ρ‡ΠΈ Π±Ρ‹Ρ‚ΡŒ ΠΏΡ€ΠΎΠ²ΠΎΠ΄Π½ΠΈΠΊΠΎΠΌ? ΠŸΠΎΡ‡Π΅ΠΌΡƒ Π½Π΅ Ρ€Π°Π±ΠΎΡ‚Π°Π΅Ρ‚ ΠΊΠ°Ρ‚ΡƒΡˆΠΊΠ° постоянного Ρ‚ΠΎΠΊΠ°?
    4. ΠžΠ±ΡŠΡΡΠ½ΠΈΡ‚Π΅, ΠΊΠ°ΠΊ ΠΌΠΎΠΆΠ½ΠΎ Ρ€Π°Π·ΠΌΠΎΡ€ΠΎΠ·ΠΈΡ‚ΡŒ Π·Π°ΠΌΠ΅Ρ€Π·ΡˆΡƒΡŽ Π²ΠΎΠ΄ΠΎΠΏΡ€ΠΎΠ²ΠΎΠ΄Π½ΡƒΡŽ Ρ‚Ρ€ΡƒΠ±Ρƒ, Π½Π°ΠΌΠΎΡ‚Π°Π² Π½Π° Π½Π΅Π΅ ΠΊΠ°Ρ‚ΡƒΡˆΠΊΡƒ, ΠΏΠΎ ΠΊΠΎΡ‚ΠΎΡ€ΠΎΠΉ ΠΏΡ€ΠΎΡ…ΠΎΠ΄ΠΈΡ‚ ΠΏΠ΅Ρ€Π΅ΠΌΠ΅Π½Π½Ρ‹ΠΉ Ρ‚ΠΎΠΊ. Π˜ΠΌΠ΅Π΅Ρ‚ Π»ΠΈ Π·Π½Π°Ρ‡Π΅Π½ΠΈΠ΅, являСтся Π»ΠΈ Ρ‚Ρ€ΡƒΠ±Π° ΠΏΡ€ΠΎΠ²ΠΎΠ΄Π½ΠΈΠΊΠΎΠΌ? Объясни.

    Π—Π°Π΄Π°Ρ‡ΠΈ ΠΈ упраТнСния

    1.Π˜ΡΠΏΠΎΠ»ΡŒΠ·ΡƒΠΉΡ‚Π΅ Π·Π°ΠΊΠΎΠ½ ЀарадСя, Π·Π°ΠΊΠΎΠ½ Π›Π΅Π½Ρ†Π° ΠΈ RHR-1, Ρ‡Ρ‚ΠΎΠ±Ρ‹ ΠΏΠΎΠΊΠ°Π·Π°Ρ‚ΡŒ, Ρ‡Ρ‚ΠΎ магнитная сила, Π΄Π΅ΠΉΡΡ‚Π²ΡƒΡŽΡ‰Π°Ρ Π½Π° Ρ‚ΠΎΠΊ Π² двиТущСмся стСрТнС Π½Π° рисункС 1, Π½Π°ΠΏΡ€Π°Π²Π»Π΅Π½Π° ​​в ΠΏΡ€ΠΎΡ‚ΠΈΠ²ΠΎΠΏΠΎΠ»ΠΎΠΆΠ½ΠΎΠΌ Π½Π°ΠΏΡ€Π°Π²Π»Π΅Π½ΠΈΠΈ ΠΎΡ‚ Π΅Π³ΠΎ скорости.

    2. Если Π² спутниковом тросС, ΠΏΠΎΠΊΠ°Π·Π°Π½Π½ΠΎΠΌ Π½Π° рисункС 2, Ρ‚Π΅Ρ‡Π΅Ρ‚ Ρ‚ΠΎΠΊ, ΠΈΡΠΏΠΎΠ»ΡŒΠ·ΡƒΠΉΡ‚Π΅ Π·Π°ΠΊΠΎΠ½ ЀарадСя, Π·Π°ΠΊΠΎΠ½ Π›Π΅Π½Ρ†Π° ΠΈ RHR-1, Ρ‡Ρ‚ΠΎΠ±Ρ‹ ΠΏΠΎΠΊΠ°Π·Π°Ρ‚ΡŒ, Ρ‡Ρ‚ΠΎ Π½Π° трос дСйствуСт магнитная сила Π² Π½Π°ΠΏΡ€Π°Π²Π»Π΅Π½ΠΈΠΈ, ΠΏΡ€ΠΎΡ‚ΠΈΠ²ΠΎΠΏΠΎΠ»ΠΎΠΆΠ½ΠΎΠΌ Π΅Π³ΠΎ скорости.

    3. (a) Π Π΅Π°ΠΊΡ‚ΠΈΠ²Π½Ρ‹ΠΉ самолСт с Ρ€Π°Π·ΠΌΠ°Ρ…ΠΎΠΌ ΠΊΡ€Ρ‹Π»Π° 75,0 ΠΌ Π»Π΅Ρ‚ΠΈΡ‚ со ΡΠΊΠΎΡ€ΠΎΡΡ‚ΡŒΡŽ 280 ΠΌ / с. Какая Π­Π”Π‘ Π²ΠΎΠ·Π½ΠΈΠΊΠ°Π΅Ρ‚ ΠΌΠ΅ΠΆΠ΄Ρƒ Π·Π°ΠΊΠΎΠ½Ρ†ΠΎΠ²ΠΊΠ°ΠΌΠΈ ΠΊΡ€Ρ‹Π»ΡŒΠ΅Π², Ссли Π²Π΅Ρ€Ρ‚ΠΈΠΊΠ°Π»ΡŒΠ½Π°Ρ ΡΠΎΡΡ‚Π°Π²Π»ΡΡŽΡ‰Π°Ρ поля Π—Π΅ΠΌΠ»ΠΈ Ρ€Π°Π²Π½Π° 3?00 Γ— 10 βˆ’5 Π’? (Π±) ΠœΠΎΠΆΠ΅Ρ‚ Π»ΠΈ Π­Π”Π‘ Ρ‚Π°ΠΊΠΎΠΉ Π²Π΅Π»ΠΈΡ‡ΠΈΠ½Ρ‹ ΠΈΠΌΠ΅Ρ‚ΡŒ ΠΊΠ°ΠΊΠΈΠ΅-Π»ΠΈΠ±ΠΎ послСдствия? Объясни.

    4. (a) ΠžΡ‚Π²Π΅Ρ€Ρ‚ΠΊΠ° для Ρ†Π²Π΅Ρ‚Π½Ρ‹Ρ… ΠΌΠ΅Ρ‚Π°Π»Π»ΠΎΠ² ΠΈΡΠΏΠΎΠ»ΡŒΠ·ΡƒΠ΅Ρ‚ΡΡ Π² ΠΌΠ°Π³Π½ΠΈΡ‚Π½ΠΎΠΌ ΠΏΠΎΠ»Π΅ 2,00 Π’Π». Какая максимальная Π­Π”Π‘ ΠΌΠΎΠΆΠ΅Ρ‚ Π±Ρ‹Ρ‚ΡŒ ΠΈΠ½Π΄ΡƒΡ†ΠΈΡ€ΠΎΠ²Π°Π½Π° Π½Π° Π΅Π³ΠΎ Π΄Π»ΠΈΠ½Π΅ 12,0 см, ΠΊΠΎΠ³Π΄Π° ΠΎΠ½ двиТСтся со ΡΠΊΠΎΡ€ΠΎΡΡ‚ΡŒΡŽ 6,00 ΠΌ / с? Π±) ВСроятно Π»ΠΈ, Ρ‡Ρ‚ΠΎ эта Π­Π”Π‘ Π±ΡƒΠ΄Π΅Ρ‚ ΠΈΠΌΠ΅Ρ‚ΡŒ ΠΊΠ°ΠΊΠΈΠ΅-Π»ΠΈΠ±ΠΎ послСдствия ΠΈΠ»ΠΈ Π΄Π°ΠΆΠ΅ Π±ΡƒΠ΄Π΅Ρ‚ Π·Π°ΠΌΠ΅Ρ‡Π΅Π½Π°?

    5. Π‘ ΠΊΠ°ΠΊΠΎΠΉ ΡΠΊΠΎΡ€ΠΎΡΡ‚ΡŒΡŽ Π΄ΠΎΠ»ΠΆΠ΅Π½ Π΄Π²ΠΈΠ³Π°Ρ‚ΡŒΡΡ ΡΠΊΠΎΠ»ΡŒΠ·ΡΡ‰ΠΈΠΉ ΡΡ‚Π΅Ρ€ΠΆΠ΅Π½ΡŒ Π½Π° РисункС 1, Ρ‡Ρ‚ΠΎΠ±Ρ‹ ΡΠΎΠ·Π΄Π°Ρ‚ΡŒ Π­Π”Π‘ 1,00 Π’ Π² ΠΏΠΎΠ»Π΅ 1,50 Π’Π», учитывая, Ρ‡Ρ‚ΠΎ Π΄Π»ΠΈΠ½Π° стСрТня Ρ€Π°Π²Π½Π° 30.0 см?

    6. Π¨Ρ‚Π°Π½Π³Π° Π΄Π»ΠΈΠ½ΠΎΠΉ 12,0 см Π½Π° РисункС 1 двиТСтся со ΡΠΊΠΎΡ€ΠΎΡΡ‚ΡŒΡŽ 4,00 ΠΌ / с. Какова Π½Π°ΠΏΡ€ΡΠΆΠ΅Π½Π½ΠΎΡΡ‚ΡŒ ΠΌΠ°Π³Π½ΠΈΡ‚Π½ΠΎΠ³ΠΎ поля ΠΏΡ€ΠΈ Π½Π°Π²Π΅Π΄Π΅Π½ΠΈΠΈ Π­Π”Π‘ 95,0 Π’.

    7. Π”ΠΎΠΊΠ°ΠΆΠΈΡ‚Π΅, Ρ‡Ρ‚ΠΎ ΠΊΠΎΠ³Π΄Π° B , β„“ ΠΈ v Π½Π΅ Π²Π·Π°ΠΈΠΌΠ½ΠΎ пСрпСндикулярны, Π­Π”Π‘ двиТСния опрСдСляСтся ΠΊΠ°ΠΊ Π­Π”Π‘ = Bβ„“v sin ΞΈ . Если v пСрпСндикулярно B , Ρ‚ΠΎΠ³Π΄Π° ΞΈ — это ΡƒΠ³ΠΎΠ» ΠΌΠ΅ΠΆΠ΄Ρƒ β„“ ΠΈ B . Если β„“ пСрпСндикулярно B , Ρ‚ΠΎΠ³Π΄Π° ΞΈ — это ΡƒΠ³ΠΎΠ» ΠΌΠ΅ΠΆΠ΄Ρƒ v ΠΈ B .

    8. Π’ΠΎ врСмя ΠΏΠΎΠ»Π΅Ρ‚Π° космичСского Ρ‡Π΅Π»Π½ΠΎΠΊΠ° Π² августС 1992 Π³. ΡƒΠ΄Π°Π»ΠΎΡΡŒ Π²Ρ‹ΠΏΡƒΡΡ‚ΠΈΡ‚ΡŒ Ρ‚ΠΎΠ»ΡŒΠΊΠΎ 250 ΠΌ проводящСго троса, рассмотрСнного Π² ΠŸΡ€ΠΈΠΌΠ΅Ρ€Π΅ 1 (Π²Ρ‹ΡˆΠ΅). Π­Π”Π‘ двиТСния 40,0 Π’ Π³Π΅Π½Π΅Ρ€ΠΈΡ€ΠΎΠ²Π°Π»Π°ΡΡŒ Π² ΠΏΠΎΠ»Π΅ Π—Π΅ΠΌΠ»ΠΈ 5,00 Γ— 10 βˆ’5 Π’Π» ΠΏΡ€ΠΈ Π΄Π²ΠΈΠΆΠ΅Π½ΠΈΠΈ со ΡΠΊΠΎΡ€ΠΎΡΡ‚ΡŒΡŽ 7,80 Γ— 10 3 ΠΌ / с. Каков ΡƒΠ³ΠΎΠ» ΠΌΠ΅ΠΆΠ΄Ρƒ ΡΠΊΠΎΡ€ΠΎΡΡ‚ΡŒΡŽ ΡˆΠ°Ρ‚Ρ‚Π»Π° ΠΈ ΠΏΠΎΠ»Π΅ΠΌ Π—Π΅ΠΌΠ»ΠΈ, Ссли ΠΏΡ€Π΅Π΄ΠΏΠΎΠ»ΠΎΠΆΠΈΡ‚ΡŒ, Ρ‡Ρ‚ΠΎ ΠΏΡ€ΠΎΠ²ΠΎΠ΄Π½ΠΈΠΊ пСрпСндикулярСн полю?

    9. Integrated Concepts Π’Ρ‹Π²Π΅Π΄ΠΈΡ‚Π΅ Π²Ρ‹Ρ€Π°ΠΆΠ΅Π½ΠΈΠ΅ для Ρ‚ΠΎΠΊΠ° Π² систСмС, ΠΏΠΎΠ΄ΠΎΠ±Π½ΠΎΠΉ ΠΏΠΎΠΊΠ°Π·Π°Π½Π½ΠΎΠΉ Π½Π° рисункС 1, ΠΏΡ€ΠΈ ΡΠ»Π΅Π΄ΡƒΡŽΡ‰ΠΈΡ… условиях.Π‘ΠΎΠΏΡ€ΠΎΡ‚ΠΈΠ²Π»Π΅Π½ΠΈΠ΅ ΠΌΠ΅ΠΆΠ΄Ρƒ Ρ€Π΅Π»ΡŒΡΠ°ΠΌΠΈ составляСт R , Ρ€Π΅Π»ΡŒΡΡ‹ ΠΈ ΠΏΠΎΠ΄Π²ΠΈΠΆΠ½Ρ‹ΠΉ ΡΡ‚Π΅Ρ€ΠΆΠ΅Π½ΡŒ ΠΈΠ΄Π΅Π½Ρ‚ΠΈΡ‡Π½Ρ‹ ΠΏΠΎ ΡΠ΅Ρ‡Π΅Π½ΠΈΡŽ A, ΠΈ ΠΈΠΌΠ΅ΡŽΡ‚ ΠΎΠ΄ΠΈΠ½Π°ΠΊΠΎΠ²ΠΎΠ΅ ΡƒΠ΄Π΅Π»ΡŒΠ½ΠΎΠ΅ сопротивлСниС ρ . РасстояниС ΠΌΠ΅ΠΆΠ΄Ρƒ Ρ€Π΅Π»ΡŒΡΠ°ΠΌΠΈ l, Π° ΡΡ‚Π΅Ρ€ΠΆΠ΅Π½ΡŒ двиТСтся с постоянной ΡΠΊΠΎΡ€ΠΎΡΡ‚ΡŒΡŽ v пСрпСндикулярно ΠΎΠ΄Π½ΠΎΡ€ΠΎΠ΄Π½ΠΎΠΌΡƒ полю B . Π’ Π½ΡƒΠ»Π΅Π²ΠΎΠΉ ΠΌΠΎΠΌΠ΅Π½Ρ‚ Π²Ρ€Π΅ΠΌΠ΅Π½ΠΈ двиТущийся ΡΡ‚Π΅Ρ€ΠΆΠ΅Π½ΡŒ находится рядом с сопротивлСниСм R .

    10. Integrated Concepts ΠŸΡ€ΠΈΠ²ΡΠ·Π°Π½Π½Ρ‹ΠΉ спутник Π½Π° РисункС 2 ΠΈΠΌΠ΅Π΅Ρ‚ массу 525 ΠΊΠ³ ΠΈ находится Π² ΠΊΠΎΠ½Ρ†Π΅ 20-Π³ΠΎ.КабСль Π΄Π»ΠΈΠ½ΠΎΠΉ 0 ΠΊΠΌ, Π΄ΠΈΠ°ΠΌΠ΅Ρ‚Ρ€ΠΎΠΌ 2,50 ΠΌΠΌ с ΠΏΡ€ΠΎΡ‡Π½ΠΎΡΡ‚ΡŒΡŽ Π½Π° Ρ€Π°Π·Ρ€Ρ‹Π², ΠΊΠ°ΠΊ ΡΡ‚Π°Π»ΡŒ. (Π°) Насколько растягиваСтся кабСль, Ссли ΠΏΡ€ΠΈΠ»ΠΎΠΆΠΈΡ‚ΡŒ усилиС 100 Н, Ρ‡Ρ‚ΠΎΠ±Ρ‹ Π²Ρ‚ΡΠ½ΡƒΡ‚ΡŒ спутник? (ΠŸΡ€Π΅Π΄ΠΏΠΎΠ»ΠΎΠΆΠΈΠΌ, Ρ‡Ρ‚ΠΎ спутник ΠΈ ΡˆΠ°Ρ‚Ρ‚Π» находятся Π½Π° ΠΎΠ΄Π½ΠΎΠΉ высотС Π½Π°Π΄ Π—Π΅ΠΌΠ»Π΅ΠΉ.) (B) Какова эффСктивная силовая постоянная кабСля? (c) Бколько энСргии хранится Π² Π½Π΅ΠΌ ΠΏΡ€ΠΈ растяТСнии силой 100 Н.

    11. Integrated Concepts ΠŸΡ€ΠΈΠ²ΡΠ·Π°Π½Π½Ρ‹ΠΉ спутник, обсуТдаСмый Π² этом ΠΌΠΎΠ΄ΡƒΠ»Π΅, Π²Ρ‹Ρ€Π°Π±Π°Ρ‚Ρ‹Π²Π°Π΅Ρ‚ 5,00 ΠΊΠ’ ΠΈ Ρ‚ΠΎΠΊ 10.0 А Ρ‚Π΅Ρ‡Π΅Ρ‚. (Π°) ΠšΠ°ΠΊΡƒΡŽ силу ΠΌΠ°Π³Π½ΠΈΡ‚Π½ΠΎΠ³ΠΎ сопротивлСния это создаСт, Ссли систСма двиТСтся со ΡΠΊΠΎΡ€ΠΎΡΡ‚ΡŒΡŽ 7,80 ΠΊΠΌ / с? (Π±) Бколько кинСтичСской энСргии ΡƒΡ…ΠΎΠ΄ΠΈΡ‚ ΠΈΠ· систСмы Π·Π° 1,00 Ρ‡, Ссли Π½Π΅ ΡƒΡ‡ΠΈΡ‚Ρ‹Π²Π°Ρ‚ΡŒ ΠΊΠ°ΠΊΠΈΠ΅-Π»ΠΈΠ±ΠΎ измСнСния высоты ΠΈΠ»ΠΈ скорости Π·Π° это врСмя? (c) Каково ΠΈΠ·ΠΌΠ΅Π½Π΅Π½ΠΈΠ΅ скорости, Ссли масса систСмы составляСт 100 000 ΠΊΠ³? (d) ΠžΠ±ΡΡƒΠ΄ΠΈΡ‚Π΅ долгосрочныС послСдствия (Π½Π°ΠΏΡ€ΠΈΠΌΠ΅Ρ€, Π½Π΅Π΄Π΅Π»ΡŒΠ½Ρ‹ΠΉ ΠΏΠΎΠ»Π΅Ρ‚) Π½Π° ΠΎΡ€Π±ΠΈΡ‚Π΅ космичСского Ρ‡Π΅Π»Π½ΠΎΠΊΠ°, ΠΎΡ‚ΠΌΠ΅Ρ‚ΠΈΠ², ΠΊΠ°ΠΊΠΎΠΉ эффСкт ΠΈΠΌΠ΅Π΅Ρ‚ сниТСниС скорости, ΠΈ ΠΎΡ†Π΅Π½ΠΈΠ² Π²Π΅Π»ΠΈΡ‡ΠΈΠ½Ρƒ этого эффСкта.

    Π˜Π·Π±Ρ€Π°Π½Π½Ρ‹Π΅ Ρ€Π΅ΡˆΠ΅Π½ΠΈΡ ΠΏΡ€ΠΎΠ±Π»Π΅ΠΌ ΠΈ упраТнСния

    1.(Π°) 0,630 Π’ (Π±) НСт, это ΠΎΡ‡Π΅Π½ΡŒ малая Π­Π”Π‘.

    5. 2,22 м / с

    11. (Π°) 10,0 Н (Π±) 2,81 Γ— 10 8 Π”ΠΆ (Π²) 0,36 ΠΌ / с (Π³) Для нСдСльной миссии (168 часов) ΠΈΠ·ΠΌΠ΅Π½Π΅Π½ΠΈΠ΅ скорости Π±ΡƒΠ΄Π΅Ρ‚ 60 ΠΌ / с, ΠΈΠ»ΠΈ ΠΏΡ€ΠΈΠΌΠ΅Ρ€Π½ΠΎ 1%. Π’ ΠΎΠ±Ρ‰Π΅ΠΌ, ΡƒΠΌΠ΅Π½ΡŒΡˆΠ΅Π½ΠΈΠ΅ скорости ΠΏΡ€ΠΈΠ²Π΅Π΄Π΅Ρ‚ ΠΊ Ρ‚ΠΎΠΌΡƒ, Ρ‡Ρ‚ΠΎ ΠΎΡ€Π±ΠΈΡ‚Π° Π½Π°Ρ‡Π½Π΅Ρ‚ Π²Ρ€Π°Ρ‰Π°Ρ‚ΡŒΡΡ ΠΏΠΎ спирали Π²Π½ΡƒΡ‚Ρ€ΡŒ, ΠΏΠΎΡ‚ΠΎΠΌΡƒ Ρ‡Ρ‚ΠΎ скорости большС Π½Π΅ Π±ΡƒΠ΄Π΅Ρ‚ достаточно для поддСрТания ΠΊΡ€ΡƒΠ³ΠΎΠ²ΠΎΠΉ ΠΎΡ€Π±ΠΈΡ‚Ρ‹. ДолгосрочныС послСдствия состоят Π² Ρ‚ΠΎΠΌ, Ρ‡Ρ‚ΠΎ ΡˆΠ°Ρ‚Ρ‚Π»Ρƒ потрСбуСтся Π½Π΅ΠΌΠ½ΠΎΠ³ΠΎ большС Ρ‚ΠΎΠΏΠ»ΠΈΠ²Π° для поддСрТания ΠΆΠ΅Π»Π°Π΅ΠΌΠΎΠΉ скорости, Π² ΠΏΡ€ΠΎΡ‚ΠΈΠ²Π½ΠΎΠΌ случаС ΠΎΡ€Π±ΠΈΡ‚Π° Π±ΡƒΠ΄Π΅Ρ‚ слСгка Π·Π°ΠΊΡ€ΡƒΡ‡ΠΈΠ²Π°Ρ‚ΡŒΡΡ Π²Π½ΡƒΡ‚Ρ€ΡŒ.Π’.

    Π”Π°Π½Π½Ρ‹Π΅ ΠΌΠΎΠΆΠ½ΠΎ ввСсти Π² любоС ΠΈΠ· ΠΏΠΎΠ»Π΅ΠΉ. Когда Π²Ρ‹ Π·Π°ΠΊΠΎΠ½Ρ‡ΠΈΠ»ΠΈ Π²Π²ΠΎΠ΄ Π΄Π°Π½Π½Ρ‹Ρ…, Ρ‰Π΅Π»ΠΊΠ½ΠΈΡ‚Π΅ количСство, ΠΊΠΎΡ‚ΠΎΡ€ΠΎΠ΅ Π²Ρ‹ Ρ…ΠΎΡ‚ΠΈΡ‚Π΅ Π²Ρ‹Ρ‡ΠΈΡΠ»ΠΈΡ‚ΡŒ, Π² Π°ΠΊΡ‚ΠΈΠ²Π½ΠΎΠΉ Ρ„ΠΎΡ€ΠΌΡƒΠ»Π΅ Π²Ρ‹ΡˆΠ΅. ΠšΠΎΠ»ΠΈΡ‡Π΅ΡΡ‚Π²ΠΎ Π½Π΅ Π±ΡƒΠ΄Π΅Ρ‚ ΠΏΡ€ΠΈΠ½ΡƒΠ΄ΠΈΡ‚Π΅Π»ΡŒΠ½ΠΎ согласованным, ΠΏΠΎΠΊΠ° Π²Ρ‹ Π½Π΅ Π½Π°ΠΆΠΌΠ΅Ρ‚Π΅ Π½Π° Π²Ρ‹Π±ΠΎΡ€. Для Π½Π΅ΠΎΠΏΡ€Π΅Π΄Π΅Π»Π΅Π½Π½Ρ‹Ρ… ΠΏΠ°Ρ€Π°ΠΌΠ΅Ρ‚Ρ€ΠΎΠ² Π±ΡƒΠ΄ΡƒΡ‚ Π²Π²Π΅Π΄Π΅Π½Ρ‹ значСния ΠΏΠΎ ΡƒΠΌΠΎΠ»Ρ‡Π°Π½ΠΈΡŽ, Π½ΠΎ всС значСния ΠΌΠΎΠ³ΡƒΡ‚ Π±Ρ‹Ρ‚ΡŒ ΠΈΠ·ΠΌΠ΅Π½Π΅Π½Ρ‹.

    ПослС Ρ‚ΠΎΠ³ΠΎ, ΠΊΠ°ΠΊ Π²Ρ‹ рассчитали Π³Π΅Π½Π΅Ρ€ΠΈΡ€ΡƒΠ΅ΠΌΠΎΠ΅ напряТСниС, Ρ€Π°Π·ΡƒΠΌΠ½Ρ‹ΠΌ ΠΏΠΎΡΠ»Π΅Π΄ΡƒΡŽΡ‰ΠΈΠΌ вопросом Π±ΡƒΠ΄Π΅Ρ‚ «Какой Ρ‚ΠΎΠΊ ΠΈ ΠΌΠΎΡ‰Π½ΠΎΡΡ‚ΡŒ я ΠΌΠΎΠ³Ρƒ ΠΏΠΎΠ»ΡƒΡ‡ΠΈΡ‚ΡŒ ΠΎΡ‚ Π³Π΅Π½Π΅Ρ€Π°Ρ‚ΠΎΡ€Π°?Β». НСсмотря Π½Π° Ρ‚ΠΎ, Ρ‡Ρ‚ΠΎ это Π½Π΅ Π±ΡƒΠ΄Π΅Ρ‚ ΠΏΡ€Π°ΠΊΡ‚ΠΈΡ‡Π½ΠΎΠΉ Π³Π΅ΠΎΠΌΠ΅Ρ‚Ρ€ΠΈΠ΅ΠΉ Π³Π΅Π½Π΅Ρ€Π°Ρ‚ΠΎΡ€Π°, ΠΎΠ½Π° ΠΌΠΎΠΆΠ΅Ρ‚ ΡΠ»ΡƒΠΆΠΈΡ‚ΡŒ
    идСализация для обсуТдСния ΠΏΡ€ΠΈΠ½Ρ†ΠΈΠΏΠΎΠ² Π³Π΅Π½Π΅Ρ€Π°Ρ†ΠΈΠΈ напряТСния ΠΏΡ€ΠΈ взаимодСйствии с ΠΌΠ°Π³Π½ΠΈΡ‚Π½Ρ‹ΠΌ ΠΏΠΎΠ»Π΅ΠΌ. ΠŸΡ€ΠΈΠ½ΠΈΠΌΠ°Ρ это
    простая гСомСтрия, элСктричСский Ρ‚ΠΎΠΊ Π² Π°ΠΌΠΏΠ΅Ρ€Π°Ρ…, Π³Π΅Π½Π΅Ρ€ΠΈΡ€ΡƒΠ΅ΠΌΡ‹ΠΉ ΠΏΡ€ΠΈ ΠΏΠ΅Ρ€Π΅ΠΌΠ΅Ρ‰Π΅Π½ΠΈΠΈ ΠΏΡ€ΠΎΠ²ΠΎΠ΄Π°
    Ρ‡Π΅Ρ€Π΅Π· ΠΌΠ°Π³Π½ΠΈΡ‚Π½ΠΎΠ΅ ΠΏΠΎΠ»Π΅ Π±ΡƒΠ΄Π΅Ρ‚ ΠΎΠΏΡ€Π΅Π΄Π΅Π»ΡΡ‚ΡŒΡΡ сопротивлСниСм Ρ†Π΅ΠΏΠΈ, ΠΊ ΠΊΠΎΡ‚ΠΎΡ€ΠΎΠΉ ΠΎΠ½ ΠΏΠΎΠ΄ΠΊΠ»ΡŽΡ‡Π΅Π½.
    связаны ΠΏΠΎ Π·Π°ΠΊΠΎΠ½Ρƒ Ома I = V / R. Если Π²Ρ‹ Π³Π΅Π½Π΅Ρ€ΠΈΡ€ΠΎΠ²Π°Π»ΠΈ 10 Π²ΠΎΠ»ΡŒΡ‚ ΠΈ Π±Ρ‹Π»ΠΈ ΠΏΠΎΠ΄ΠΊΠ»ΡŽΡ‡Π΅Π½Ρ‹ ΠΊ Ρ†Π΅ΠΏΠΈ
    сопротивлСниС 1 Ом, Ρ€Π΅Π·ΡƒΠ»ΡŒΡ‚ΠΈΡ€ΡƒΡŽΡ‰ΠΈΠΉ Ρ‚ΠΎΠΊ Π±ΡƒΠ΄Π΅Ρ‚ 10 Π°ΠΌΠΏΠ΅Ρ€, Π° пСрСдаваСмая ΠΌΠΎΡ‰Π½ΠΎΡΡ‚ΡŒ P = VI = 10 Π²ΠΎΠ»ΡŒΡ‚ x 10
    Π°ΠΌΠΏΠ΅Ρ€Ρ‹ = 100 Π’Ρ‚ (см. ΡΠΎΠΎΡ‚Π½ΠΎΡˆΠ΅Π½ΠΈΠ΅ мощности).Но бСсплатного ΠΎΠ±Π΅Π΄Π° Π½Π΅ Π±Ρ‹Π²Π°Π΅Ρ‚, ΠΈ Π²Π°ΠΌ придСтся усСрднСС Π΄Π²ΠΈΠ³Π°Ρ‚ΡŒΡΡ.
    ΠΏΡ€ΠΎΠ²ΠΎΠ΄ Ρ‡Π΅Ρ€Π΅Π· ΠΌΠ°Π³Π½ΠΈΡ‚Π½ΠΎΠ΅ ΠΏΠΎΠ»Π΅ с Ρ‚Π°ΠΊΠΎΠΉ ΡΠΊΠΎΡ€ΠΎΡΡ‚ΡŒΡŽ — ΠΏΠΎ сути, Π²Ρ‹ Ρ‚ΠΎΡ€Π³ΡƒΠ΅Ρ‚Π΅ мСханичСской энСргиСй толкания
    для Π²Ρ‹Ρ€Π°Π±ΠΎΡ‚ΠΊΠΈ элСктричСской энСргии, всСгда ΠΎΠ³Ρ€Π°Π½ΠΈΡ‡ΠΈΠ²Π°ΡΡΡŒ ΠΏΡ€ΠΈΠ½Ρ†ΠΈΠΏΠΎΠΌ сохранСния энСргии. Π’Π°ΠΌ придСтся
    ΠΏΡ€ΠΈΠ»ΠΎΠΆΠΈΡ‚Π΅ (ΠΏΠΎ ΠΊΡ€Π°ΠΉΠ½Π΅ΠΉ ΠΌΠ΅Ρ€Π΅) 100 Π’Ρ‚ мСханичСской мощности наТатия, Ρ‡Ρ‚ΠΎΠ±Ρ‹ ΠΏΠΎΠ»ΡƒΡ‡ΠΈΡ‚ΡŒ 100 Π’Ρ‚ элСктроэнСргии.
    ΠŸΡ€Π°ΠΊΡ‚ΠΈΡ‡Π΅ΡΠΊΠΈΠ΅ Π³Π΅Π½Π΅Ρ€Π°Ρ‚ΠΎΡ€Ρ‹ ΠΏΠΎΡ‡Ρ‚ΠΈ всСгда ΠΈΡΠΏΠΎΠ»ΡŒΠ·ΡƒΡŽΡ‚ Π³Π΅ΠΎΠΌΠ΅Ρ‚Ρ€ΠΈΡŽ Π²Ρ€Π°Ρ‰Π°ΡŽΡ‰Π΅ΠΉΡΡ ΠΊΠ°Ρ‚ΡƒΡˆΠΊΠΈ, Π° ΠΊΡ€ΡƒΠΏΠ½ΠΎΠΌΠ°ΡΡˆΡ‚Π°Π±Π½Ρ‹Π΅ Π³Π΅Π½Π΅Ρ€Π°Ρ‚ΠΎΡ€Ρ‹ энСргии ΠΈΡΠΏΠΎΠ»ΡŒΠ·ΡƒΡŽΡ‚ Ρ‡Ρ‚ΠΎ-Ρ‚ΠΎ Π²Ρ€ΠΎΠ΄Π΅ ΠΏΠ°Ρ€ΠΎΠ²ΠΎΠΉ Ρ‚ΡƒΡ€Π±ΠΈΠ½Ρ‹ ΠΈΠ»ΠΈ водяной Ρ‚ΡƒΡ€Π±ΠΈΠ½Ρ‹ для ΠΏΠΎΠ²ΠΎΡ€ΠΎΡ‚Π° ΠΊΠ°Ρ‚ΡƒΡˆΠΊΠΈ с ΠΏΡ€ΠΎΠ²ΠΎΠ»ΠΎΠΊΠΎΠΉ Π²
    ΠΌΠ°Π³Π½ΠΈΡ‚Π½ΠΎΠ΅ ΠΏΠΎΠ»Π΅, ΠΏΠΎΠ»ΡƒΡ‡Π°ΡŽΡ‰Π΅Π΅ напряТСниС, Π³Π΅Π½Π΅Ρ€ΠΈΡ€ΡƒΠ΅ΠΌΠΎΠ΅ с ΠΎΠ±Π΅ΠΈΡ… сторон Π²Ρ€Π°Ρ‰Π°ΡŽΡ‰Π΅ΠΉΡΡ ΠΊΠ°Ρ‚ΡƒΡˆΠΊΠΈ.

    Если ΡƒΠΊΠ°Π·Π°Π½Π½Ρ‹ΠΉ Π²Ρ‹ΡˆΠ΅ Π³Π΅Π½Π΅Ρ€Π°Ρ‚ΠΎΡ€ Π±Ρ‹Π» ΠΏΠΎΠ΄ΠΊΠ»ΡŽΡ‡Π΅Π½ ΠΊ Ρ†Π΅ΠΏΠΈ с сопротивлСниСм R = Ом,

    элСктричСский Ρ‚ΠΎΠΊ Π±ΡƒΠ΄Π΅Ρ‚ I = V / R = Π°ΠΌΠΏΠ΅Ρ€Ρ‹ для скорости, пСрпСндикулярной B.

    ΠœΠΎΡ‰Π½ΠΎΡΡ‚ΡŒ, подаваСмая Π² схСму, Π±ΡƒΠ΄Π΅Ρ‚ P = VI = Π’Ρ‚.

    Π’ идСальном случаС, ΠΊΠΎΠ³Π΄Π° Π½Π΅ Π±Ρ‹Π»ΠΎ ΠΏΠΎΡ‚Π΅Ρ€ΡŒ, мСханичСская ΠΌΠΎΡ‰Π½ΠΎΡΡ‚ΡŒ P = Fv, нСобходимая для проталкивания ΠΏΡ€ΠΎΠ²ΠΎΠ΄Π° Ρ‡Π΅Ρ€Π΅Π· ΠΌΠ°Π³Π½ΠΈΡ‚Π½ΠΎΠ΅ ΠΏΠΎΠ»Π΅, Π±Ρ‹Π»Π° Π±Ρ‹ Ρ€Π°Π²Π½Π° элСктричСской мощности. Для ΡƒΠΊΠ°Π·Π°Π½Π½ΠΎΠΉ Π²Ρ‹ΡˆΠ΅ скорости трСбуСмая сила Ρ€Π°Π²Π½Π°

    .

    ИдСальная минимально нСобходимая сила:

    F = P / v = Π½ΡŒΡŽΡ‚ΠΎΠ½Ρ‹ = Ρ„ΡƒΠ½Ρ‚Ρ‹.

    Π—Π°ΠΊΠΎΠ½ Ома ΠΈ ΡΠΎΠΎΡ‚Π½ΠΎΡˆΠ΅Π½ΠΈΠ΅ V-I-R

    Π’ Ρ„ΠΈΠ·ΠΈΠΊΠ΅ Π΅ΡΡ‚ΡŒ ΠΎΠΏΡ€Π΅Π΄Π΅Π»Π΅Π½Π½Ρ‹Π΅ Ρ„ΠΎΡ€ΠΌΡƒΠ»Ρ‹, ΠΊΠΎΡ‚ΠΎΡ€Ρ‹Π΅ Π½Π°ΡΡ‚ΠΎΠ»ΡŒΠΊΠΎ ΡΠΈΠ»ΡŒΠ½Ρ‹ ΠΈ распространСны, Ρ‡Ρ‚ΠΎ Π΄ΠΎΡΡ‚ΠΈΠ³Π°ΡŽΡ‚ уровня общСизвСстных Π·Π½Π°Π½ΠΈΠΉ. Π‘Ρ‚ΡƒΠ΄Π΅Π½Ρ‚, ΠΈΠ·ΡƒΡ‡Π°ΡŽΡ‰ΠΈΠΉ Ρ„ΠΈΠ·ΠΈΠΊΡƒ, записывал Ρ‚Π°ΠΊΠΈΠ΅ Ρ„ΠΎΡ€ΠΌΡƒΠ»Ρ‹ ΡΡ‚ΠΎΠ»ΡŒΠΊΠΎ Ρ€Π°Π·, Ρ‡Ρ‚ΠΎ Π·Π°ΠΏΠΎΠΌΠΈΠ½Π°Π» ΠΈΡ…, Π΄Π°ΠΆΠ΅ Π½Π΅ ΠΏΡ‹Ρ‚Π°ΡΡΡŒ. БСзусловно, для профСссионалов Π² этой области Ρ‚Π°ΠΊΠΈΠ΅ Ρ„ΠΎΡ€ΠΌΡƒΠ»Ρ‹ Π½Π°ΡΡ‚ΠΎΠ»ΡŒΠΊΠΎ Π²Π°ΠΆΠ½Ρ‹, Ρ‡Ρ‚ΠΎ ΠΎΡΡ‚Π°ΡŽΡ‚ΡΡ Π² ΠΈΡ… сознании. Π’ области соврСмСнной Ρ„ΠΈΠ·ΠΈΠΊΠΈ E = m β€’ c 2 . Π’ области Π½ΡŒΡŽΡ‚ΠΎΠ½ΠΎΠ²ΡΠΊΠΎΠΉ ΠΌΠ΅Ρ…Π°Π½ΠΈΠΊΠΈ сущСствуСт F net = m β€’ a. Π’ области Π²ΠΎΠ»Π½ΠΎΠ²ΠΎΠΉ ΠΌΠ΅Ρ…Π°Π½ΠΈΠΊΠΈ v = f β€’ Ξ». А Π² области Ρ‚Π΅ΠΊΡƒΡ‰Π΅Π³ΠΎ элСктричСства Ξ”V = I β€’ R.

    ΠŸΡ€Π΅ΠΎΠ±Π»Π°Π΄Π°ΡŽΡ‰ΠΈΠΌ ΡƒΡ€Π°Π²Π½Π΅Π½ΠΈΠ΅ΠΌ, ΠΊΠΎΡ‚ΠΎΡ€ΠΎΠ΅ ΠΏΡ€ΠΎΠ½ΠΈΠ·Ρ‹Π²Π°Π΅Ρ‚ ΠΈΠ·ΡƒΡ‡Π΅Π½ΠΈΠ΅ элСктричСских Ρ†Π΅ΠΏΠ΅ΠΉ, являСтся ΡƒΡ€Π°Π²Π½Π΅Π½ΠΈΠ΅

    Ξ”V = I β€’ R

    Π”Ρ€ΡƒΠ³ΠΈΠΌΠΈ словами, Ρ€Π°Π·Π½ΠΎΡΡ‚ΡŒ элСктричСских ΠΏΠΎΡ‚Π΅Π½Ρ†ΠΈΠ°Π»ΠΎΠ² ΠΌΠ΅ΠΆΠ΄Ρƒ двумя Ρ‚ΠΎΡ‡ΠΊΠ°ΠΌΠΈ Π² Ρ†Π΅ΠΏΠΈ ( Ξ”V ) эквивалСнтна ΠΏΡ€ΠΎΠΈΠ·Π²Π΅Π΄Π΅Π½ΠΈΡŽ Ρ‚ΠΎΠΊΠ° ΠΌΠ΅ΠΆΠ΄Ρƒ этими двумя Ρ‚ΠΎΡ‡ΠΊΠ°ΠΌΠΈ ( I ) ΠΈ ΠΎΠ±Ρ‰Π΅Π³ΠΎ сопротивлСния всСх элСктричСских устройств, ΠΏΡ€ΠΈΡΡƒΡ‚ΡΡ‚Π²ΡƒΡŽΡ‰ΠΈΡ… ΠΌΠ΅ΠΆΠ΄Ρƒ этими двумя Ρ‚ΠΎΡ‡ΠΊΠ°ΠΌΠΈ ( R ).Π’ ΠΎΡΡ‚Π°Π»ΡŒΠ½ΠΎΠΉ части этого Ρ€Π°Π·Π΄Π΅Π»Π° ЀизичСского класса это ΡƒΡ€Π°Π²Π½Π΅Π½ΠΈΠ΅ станСт Π½Π°ΠΈΠ±ΠΎΠ»Π΅Π΅ распространСнным ΡƒΡ€Π°Π²Π½Π΅Π½ΠΈΠ΅ΠΌ, ΠΊΠΎΡ‚ΠΎΡ€ΠΎΠ΅ ΠΌΡ‹ Π²ΠΈΠ΄ΠΈΠΌ. Π­Ρ‚ΠΎ ΡƒΡ€Π°Π²Π½Π΅Π½ΠΈΠ΅, часто Π½Π°Π·Ρ‹Π²Π°Π΅ΠΌΠΎΠ΅ ΡƒΡ€Π°Π²Π½Π΅Π½ΠΈΠ΅ΠΌ Π·Π°ΠΊΠΎΠ½Π° Ома , являСтся ΠΌΠΎΡ‰Π½Ρ‹ΠΌ прСдсказатСлСм взаимосвязи ΠΌΠ΅ΠΆΠ΄Ρƒ Ρ€Π°Π·Π½ΠΎΡΡ‚ΡŒΡŽ ΠΏΠΎΡ‚Π΅Π½Ρ†ΠΈΠ°Π»ΠΎΠ², Ρ‚ΠΎΠΊΠΎΠΌ ΠΈ сопротивлСниСм.

    Π—Π°ΠΊΠΎΠ½ Ома ΠΊΠ°ΠΊ ΠΏΡ€Π΅Π΄ΡΠΊΠ°Π·Π°Ρ‚Π΅Π»ΡŒ Ρ‚ΠΎΠΊΠ°

    Π£Ρ€Π°Π²Π½Π΅Π½ΠΈΠ΅ Π·Π°ΠΊΠΎΠ½Π° Ома ΠΌΠΎΠΆΠ½ΠΎ ΠΏΠ΅Ρ€Π΅Ρ„ΠΎΡ€ΠΌΡƒΠ»ΠΈΡ€ΠΎΠ²Π°Ρ‚ΡŒ ΠΈ Π²Ρ‹Ρ€Π°Π·ΠΈΡ‚ΡŒ ΠΊΠ°ΠΊ

    Π’ качСствС уравнСния это слуТит алгСбраичСским Ρ€Π΅Ρ†Π΅ΠΏΡ‚ΠΎΠΌ для вычислСния Ρ‚ΠΎΠΊΠ°, Ссли извСстны Ρ€Π°Π·Π½ΠΎΡΡ‚ΡŒ элСктричСских ΠΏΠΎΡ‚Π΅Π½Ρ†ΠΈΠ°Π»ΠΎΠ² ΠΈ сопротивлСниС. Π’Π΅ΠΌ Π½Π΅ ΠΌΠ΅Π½Π΅Π΅, хотя это ΡƒΡ€Π°Π²Π½Π΅Π½ΠΈΠ΅ слуТит ΠΌΠΎΡ‰Π½Ρ‹ΠΌ Ρ€Π΅Ρ†Π΅ΠΏΡ‚ΠΎΠΌ Ρ€Π΅ΡˆΠ΅Π½ΠΈΡ ΠΏΡ€ΠΎΠ±Π»Π΅ΠΌ, это Π³ΠΎΡ€Π°Π·Π΄ΠΎ большС. Π­Ρ‚ΠΎ ΡƒΡ€Π°Π²Π½Π΅Π½ΠΈΠ΅ ΡƒΠΊΠ°Π·Ρ‹Π²Π°Π΅Ρ‚ Π½Π° Π΄Π²Π΅ ΠΏΠ΅Ρ€Π΅ΠΌΠ΅Π½Π½Ρ‹Π΅, ΠΊΠΎΡ‚ΠΎΡ€Ρ‹Π΅ ΠΌΠΎΠ³ΡƒΡ‚ ΠΏΠΎΠ²Π»ΠΈΡΡ‚ΡŒ Π½Π° Π²Π΅Π»ΠΈΡ‡ΠΈΠ½Ρƒ Ρ‚ΠΎΠΊΠ° Π² Ρ†Π΅ΠΏΠΈ. Π’ΠΎΠΊ Π² Ρ†Π΅ΠΏΠΈ прямо ΠΏΡ€ΠΎΠΏΠΎΡ€Ρ†ΠΈΠΎΠ½Π°Π»Π΅Π½ разности элСктричСских ΠΏΠΎΡ‚Π΅Π½Ρ†ΠΈΠ°Π»ΠΎΠ², ΠΏΡ€ΠΈΠ»ΠΎΠΆΠ΅Π½Π½ΠΎΠΉ ΠΊ Π΅Π΅ ΠΊΠΎΠ½Ρ†Π°ΠΌ, ΠΈ ΠΎΠ±Ρ€Π°Ρ‚Π½ΠΎ ΠΏΡ€ΠΎΠΏΠΎΡ€Ρ†ΠΈΠΎΠ½Π°Π»Π΅Π½ ΠΎΠ±Ρ‰Π΅ΠΌΡƒ ΡΠΎΠΏΡ€ΠΎΡ‚ΠΈΠ²Π»Π΅Π½ΠΈΡŽ внСшнСй Ρ†Π΅ΠΏΠΈ. Π§Π΅ΠΌ большС напряТСниС Π±Π°Ρ‚Π°Ρ€Π΅ΠΈ (Ρ‚ΠΎ Π΅ΡΡ‚ΡŒ Ρ€Π°Π·Π½ΠΎΡΡ‚ΡŒ элСктричСских ΠΏΠΎΡ‚Π΅Π½Ρ†ΠΈΠ°Π»ΠΎΠ²), Ρ‚Π΅ΠΌ большС Ρ‚ΠΎΠΊ. И Ρ‡Π΅ΠΌ большС сопротивлСниС, Ρ‚Π΅ΠΌ мСньшС Ρ‚ΠΎΠΊ.Заряд ΠΈΠ΄Π΅Ρ‚ с наибольшСй ΡΠΊΠΎΡ€ΠΎΡΡ‚ΡŒΡŽ, ΠΊΠΎΠ³Π΄Π° напряТСниС аккумулятора увСличиваСтся, Π° сопротивлСниС ΡƒΠΌΠ΅Π½ΡŒΡˆΠ°Π΅Ρ‚ΡΡ. ЀактичСски, Π΄Π²ΡƒΠΊΡ€Π°Ρ‚Π½ΠΎΠ΅ ΡƒΠ²Π΅Π»ΠΈΡ‡Π΅Π½ΠΈΠ΅ напряТСния Π±Π°Ρ‚Π°Ρ€Π΅ΠΈ ΠΏΡ€ΠΈΠ²Π΅Π»ΠΎ Π±Ρ‹ ΠΊ Π΄Π²ΡƒΠΊΡ€Π°Ρ‚Π½ΠΎΠΌΡƒ ΡƒΠ²Π΅Π»ΠΈΡ‡Π΅Π½ΠΈΡŽ Ρ‚ΠΎΠΊΠ° (Ссли всС ΠΎΡΡ‚Π°Π»ΡŒΠ½Ρ‹Π΅ Ρ„Π°ΠΊΡ‚ΠΎΡ€Ρ‹ ΠΎΡΡ‚Π°ΡŽΡ‚ΡΡ Ρ€Π°Π²Π½Ρ‹ΠΌΠΈ). А ΡƒΠ²Π΅Π»ΠΈΡ‡Π΅Π½ΠΈΠ΅ сопротивлСния Π½Π°Π³Ρ€ΡƒΠ·ΠΊΠΈ Π² Π΄Π²Π° Ρ€Π°Π·Π° ΠΏΡ€ΠΈΠ²Π΅Π΄Π΅Ρ‚ ΠΊ ΡƒΠΌΠ΅Π½ΡŒΡˆΠ΅Π½ΠΈΡŽ Ρ‚ΠΎΠΊΠ° Π² Π΄Π²Π° Ρ€Π°Π·Π° Π΄ΠΎ ΠΏΠΎΠ»ΠΎΠ²ΠΈΠ½Ρ‹ Π΅Π³ΠΎ ΠΏΠ΅Ρ€Π²ΠΎΠ½Π°Ρ‡Π°Π»ΡŒΠ½ΠΎΠ³ΠΎ значСния.

    ΠŸΡ€ΠΈΠ²Π΅Π΄Π΅Π½Π½Π°Ρ Π½ΠΈΠΆΠ΅ Ρ‚Π°Π±Π»ΠΈΡ†Π° ΠΈΠ»Π»ΡŽΡΡ‚Ρ€ΠΈΡ€ΡƒΠ΅Ρ‚ это ΡΠΎΠΎΡ‚Π½ΠΎΡˆΠ΅Π½ΠΈΠ΅ ΠΊΠ°ΠΊ качСствСнно, Ρ‚Π°ΠΊ ΠΈ количСствСнно для Π½Π΅ΡΠΊΠΎΠ»ΡŒΠΊΠΈΡ… Ρ†Π΅ΠΏΠ΅ΠΉ с Ρ€Π°Π·Π»ΠΈΡ‡Π½Ρ‹ΠΌΠΈ напряТСниями ΠΈ сопротивлСниСм Π±Π°Ρ‚Π°Ρ€Π΅ΠΉ.

    Π‘Ρ‚Ρ€ΠΎΠΊΠΈ 1, 2 ΠΈ 3 ΠΏΠΎΠΊΠ°Π·Ρ‹Π²Π°ΡŽΡ‚, Ρ‡Ρ‚ΠΎ ΡƒΠ΄Π²ΠΎΠ΅Π½ΠΈΠ΅ ΠΈ ΡƒΡ‚Ρ€ΠΎΠ΅Π½ΠΈΠ΅ напряТСния Π±Π°Ρ‚Π°Ρ€Π΅ΠΈ ΠΏΡ€ΠΈΠ²ΠΎΠ΄ΠΈΡ‚ ΠΊ ΡƒΠ΄Π²ΠΎΠ΅Π½ΠΈΡŽ ΠΈ ΡƒΡ‚Ρ€ΠΎΠ΅Π½ΠΈΡŽ Ρ‚ΠΎΠΊΠ° Π² Ρ†Π΅ΠΏΠΈ. Π‘Ρ€Π°Π²Π½Π΅Π½ΠΈΠ΅ строк 1 ΠΈ 4 ΠΈΠ»ΠΈ строк 2 ΠΈ 5 ΠΏΠΎΠΊΠ°Π·Ρ‹Π²Π°Π΅Ρ‚, Ρ‡Ρ‚ΠΎ ΡƒΠ΄Π²ΠΎΠ΅Π½ΠΈΠ΅ ΠΎΠ±Ρ‰Π΅Π³ΠΎ сопротивлСния слуТит для ΡƒΠΌΠ΅Π½ΡŒΡˆΠ΅Π½ΠΈΡ Π²Π΄Π²ΠΎΠ΅ Ρ‚ΠΎΠΊΠ° Π² Ρ†Π΅ΠΏΠΈ.

    ΠŸΠΎΡΠΊΠΎΠ»ΡŒΠΊΡƒ Π½Π° Ρ‚ΠΎΠΊ Π² Ρ†Π΅ΠΏΠΈ влияСт сопротивлСниС, Π² цСпях элСктроприборов часто ΠΈΡΠΏΠΎΠ»ΡŒΠ·ΡƒΡŽΡ‚ΡΡ рСзисторы, Ρ‡Ρ‚ΠΎΠ±Ρ‹ Π²Π»ΠΈΡΡ‚ΡŒ Π½Π° Π²Π΅Π»ΠΈΡ‡ΠΈΠ½Ρƒ Ρ‚ΠΎΠΊΠ°, ΠΏΡ€ΠΈΡΡƒΡ‚ΡΡ‚Π²ΡƒΡŽΡ‰Π΅Π³ΠΎ Π² Π΅Π΅ Ρ€Π°Π·Π»ΠΈΡ‡Π½Ρ‹Ρ… ΠΊΠΎΠΌΠΏΠΎΠ½Π΅Π½Ρ‚Π°Ρ….УвСличивая ΠΈΠ»ΠΈ ΡƒΠΌΠ΅Π½ΡŒΡˆΠ°Ρ Π²Π΅Π»ΠΈΡ‡ΠΈΠ½Ρƒ сопротивлСния Π² ΠΊΠΎΠ½ΠΊΡ€Π΅Ρ‚Π½ΠΎΠΉ Π²Π΅Ρ‚Π²ΠΈ схСмы, ΠΏΡ€ΠΎΠΈΠ·Π²ΠΎΠ΄ΠΈΡ‚Π΅Π»ΡŒ ΠΌΠΎΠΆΠ΅Ρ‚ ΡƒΠ²Π΅Π»ΠΈΡ‡ΠΈΠ²Π°Ρ‚ΡŒ ΠΈΠ»ΠΈ ΡƒΠΌΠ΅Π½ΡŒΡˆΠ°Ρ‚ΡŒ Π²Π΅Π»ΠΈΡ‡ΠΈΠ½Ρƒ Ρ‚ΠΎΠΊΠ° Π² этой Π²Π΅Ρ‚Π²ΠΈ . ΠšΡƒΡ…ΠΎΠ½Π½Ρ‹Π΅ ΠΏΡ€ΠΈΠ±ΠΎΡ€Ρ‹, Ρ‚Π°ΠΊΠΈΠ΅ ΠΊΠ°ΠΊ элСктричСскиС миксСры ΠΈ ΠΏΠ΅Ρ€Π΅ΠΊΠ»ΡŽΡ‡Π°Ρ‚Π΅Π»ΠΈ свСта, Ρ€Π°Π±ΠΎΡ‚Π°ΡŽΡ‚, измСняя Ρ‚ΠΎΠΊ Π½Π°Π³Ρ€ΡƒΠ·ΠΊΠΈ, увСличивая ΠΈΠ»ΠΈ ΡƒΠΌΠ΅Π½ΡŒΡˆΠ°Ρ сопротивлСниС Ρ†Π΅ΠΏΠΈ. НаТатиС Ρ€Π°Π·Π»ΠΈΡ‡Π½Ρ‹Ρ… ΠΊΠ½ΠΎΠΏΠΎΠΊ Π½Π° элСктричСском ΠΌΠΈΠΊΡˆΠ΅Ρ€Π΅ ΠΌΠΎΠΆΠ΅Ρ‚ ΠΈΠ·ΠΌΠ΅Π½ΠΈΡ‚ΡŒ Ρ€Π΅ΠΆΠΈΠΌ с ΠΌΠΈΠΊΡˆΠΈΡ€ΠΎΠ²Π°Π½ΠΈΡ Π½Π° Π²Π·Π±ΠΈΠ²Π°Π½ΠΈΠ΅, ΡƒΠΌΠ΅Π½ΡŒΡˆΠΈΠ² сопротивлСниС ΠΈ ΠΏΠΎΠ·Π²ΠΎΠ»ΠΈΠ² Π±ΠΎΠ»ΡŒΡˆΠ΅ΠΌΡƒ Ρ‚ΠΎΠΊΡƒ ΠΏΡ€ΠΈΡΡƒΡ‚ΡΡ‚Π²ΠΎΠ²Π°Ρ‚ΡŒ Π² миксСрС.Π’ΠΎΡ‡Π½ΠΎ Ρ‚Π°ΠΊ ΠΆΠ΅ ΠΏΠΎΠ²ΠΎΡ€ΠΎΡ‚ Ρ€ΡƒΡ‡ΠΊΠΈ рСгулятора яркости ΠΌΠΎΠΆΠ΅Ρ‚ ΡƒΠ²Π΅Π»ΠΈΡ‡ΠΈΡ‚ΡŒ сопротивлСниС Π΅Π³ΠΎ встроСнного рСзистора ΠΈ, Ρ‚Π°ΠΊΠΈΠΌ ΠΎΠ±Ρ€Π°Π·ΠΎΠΌ, ΡƒΠΌΠ΅Π½ΡŒΡˆΠΈΡ‚ΡŒ Ρ‚ΠΎΠΊ.

    На схСмС Π½ΠΈΠΆΠ΅ ΠΈΠ·ΠΎΠ±Ρ€Π°ΠΆΠ΅Π½Π° ΠΏΠ°Ρ€Π° Ρ†Π΅ΠΏΠ΅ΠΉ, содСрТащих источник напряТСния (аккумуляторная батарСя), рСзистор (Π»Π°ΠΌΠΏΠΎΡ‡ΠΊΠ°) ΠΈ Π°ΠΌΠΏΠ΅Ρ€ΠΌΠ΅Ρ‚Ρ€ (для измСрСния Ρ‚ΠΎΠΊΠ°). Π’ ΠΊΠ°ΠΊΠΎΠΉ Ρ†Π΅ΠΏΠΈ Ρƒ Π»Π°ΠΌΠΏΠΎΡ‡ΠΊΠΈ наибольшСС сопротивлСниС? НаТмитС ΠΊΠ½ΠΎΠΏΠΊΡƒ Β«ΠŸΠΎΡΠΌΠΎΡ‚Ρ€Π΅Ρ‚ΡŒ ΠΎΡ‚Π²Π΅Ρ‚Β», Ρ‡Ρ‚ΠΎΠ±Ρ‹ ΡƒΠ±Π΅Π΄ΠΈΡ‚ΡŒΡΡ, Ρ‡Ρ‚ΠΎ Π²Ρ‹ ΠΏΡ€Π°Π²Ρ‹.

    Π£Ρ€Π°Π²Π½Π΅Π½ΠΈΠ΅ Π·Π°ΠΊΠΎΠ½Π° Ома часто исслСдуСтся Π² физичСских лабораториях с использованиСм рСзистора, аккумуляторной Π±Π°Ρ‚Π°Ρ€Π΅ΠΈ, Π°ΠΌΠΏΠ΅Ρ€ΠΌΠ΅Ρ‚Ρ€Π° ΠΈ Π²ΠΎΠ»ΡŒΡ‚ΠΌΠ΅Ρ‚Ρ€Π°.АмпСрмСтр — это устройство, ΠΈΡΠΏΠΎΠ»ΡŒΠ·ΡƒΠ΅ΠΌΠΎΠ΅ для измСрСния силы Ρ‚ΠΎΠΊΠ° Π² Π·Π°Π΄Π°Π½Π½ΠΎΠΌ мСстС. Π’ΠΎΠ»ΡŒΡ‚ΠΌΠ΅Ρ‚Ρ€ — это устройство, оснащСнноС Π΄Π°Ρ‚Ρ‡ΠΈΠΊΠ°ΠΌΠΈ, ΠΊΠΎΡ‚ΠΎΡ€Ρ‹Ρ… ΠΌΠΎΠΆΠ½ΠΎ ΠΏΡ€ΠΈΠΊΠΎΡΠ½ΡƒΡ‚ΡŒΡΡ ΠΊ Π΄Π²ΡƒΠΌ Ρ‚ΠΎΡ‡ΠΊΠ°ΠΌ Ρ†Π΅ΠΏΠΈ, Ρ‡Ρ‚ΠΎΠ±Ρ‹ ΠΎΠΏΡ€Π΅Π΄Π΅Π»ΠΈΡ‚ΡŒ Ρ€Π°Π·Π½ΠΎΡΡ‚ΡŒ элСктричСских ΠΏΠΎΡ‚Π΅Π½Ρ†ΠΈΠ°Π»ΠΎΠ² Π² этих мСстах. ИзмСняя количСство ячССк Π² аккумуляторной Π±Π°Ρ‚Π°Ρ€Π΅Π΅, ΠΌΠΎΠΆΠ½ΠΎ ΠΈΠ·ΠΌΠ΅Π½ΡΡ‚ΡŒ Ρ€Π°Π·Π½ΠΎΡΡ‚ΡŒ элСктричСских ΠΏΠΎΡ‚Π΅Π½Ρ†ΠΈΠ°Π»ΠΎΠ² Π²ΠΎ внСшнСй Ρ†Π΅ΠΏΠΈ. Π’ΠΎΠ»ΡŒΡ‚ΠΌΠ΅Ρ‚Ρ€ ΠΌΠΎΠΆΠ΅Ρ‚ ΠΈΡΠΏΠΎΠ»ΡŒΠ·ΠΎΠ²Π°Ρ‚ΡŒΡΡ для опрСдСлСния этой разности ΠΏΠΎΡ‚Π΅Π½Ρ†ΠΈΠ°Π»ΠΎΠ², Π° Π°ΠΌΠΏΠ΅Ρ€ΠΌΠ΅Ρ‚Ρ€ ΠΌΠΎΠΆΠ΅Ρ‚ ΠΈΡΠΏΠΎΠ»ΡŒΠ·ΠΎΠ²Π°Ρ‚ΡŒΡΡ для опрСдСлСния Ρ‚ΠΎΠΊΠ°, связанного с этим Ξ”V.К Π±Π°Ρ‚Π°Ρ€Π΅ΠΉΠ½ΠΎΠΌΡƒ Π±Π»ΠΎΠΊΡƒ ΠΌΠΎΠΆΠ½ΠΎ Π΄ΠΎΠ±Π°Π²ΠΈΡ‚ΡŒ Π±Π°Ρ‚Π°Ρ€Π΅ΡŽ, ΠΈ процСсс ΠΌΠΎΠΆΠ½ΠΎ ΠΏΠΎΠ²Ρ‚ΠΎΡ€ΠΈΡ‚ΡŒ нСсколько Ρ€Π°Π·, Ρ‡Ρ‚ΠΎΠ±Ρ‹ ΠΏΠΎΠ»ΡƒΡ‡ΠΈΡ‚ΡŒ Π½Π°Π±ΠΎΡ€ Π΄Π°Π½Π½Ρ‹Ρ… I-Ξ”V. Π“Ρ€Π°Ρ„ΠΈΠΊ зависимости I ΠΎΡ‚ Ξ”V даст линию с ΠΊΡ€ΡƒΡ‚ΠΈΠ·Π½ΠΎΠΉ, эквивалСнтной ΠΎΠ±Ρ€Π°Ρ‚Π½ΠΎΠΉ Π²Π΅Π»ΠΈΡ‡ΠΈΠ½Π΅ сопротивлСния рСзистора. Π­Ρ‚ΠΎ Π·Π½Π°Ρ‡Π΅Π½ΠΈΠ΅ ΠΌΠΎΠΆΠ½ΠΎ ΡΡ€Π°Π²Π½ΠΈΡ‚ΡŒ с заявлСнным ΠΏΡ€ΠΎΠΈΠ·Π²ΠΎΠ΄ΠΈΡ‚Π΅Π»Π΅ΠΌ Π·Π½Π°Ρ‡Π΅Π½ΠΈΠ΅ΠΌ, Ρ‡Ρ‚ΠΎΠ±Ρ‹ ΠΎΠΏΡ€Π΅Π΄Π΅Π»ΠΈΡ‚ΡŒ Ρ‚ΠΎΡ‡Π½ΠΎΡΡ‚ΡŒ Π»Π°Π±ΠΎΡ€Π°Ρ‚ΠΎΡ€Π½Ρ‹Ρ… Π΄Π°Π½Π½Ρ‹Ρ… ΠΈ ΡΠΏΡ€Π°Π²Π΅Π΄Π»ΠΈΠ²ΠΎΡΡ‚ΡŒ уравнСния Π·Π°ΠΊΠΎΠ½Π° Ома.

    Π’Π΅Π»ΠΈΡ‡ΠΈΠ½Ρ‹, символы, уравнСния ΠΈ Π΅Π΄ΠΈΠ½ΠΈΡ†Ρ‹!

    ВСндСнция ΡƒΠ΄Π΅Π»ΡΡ‚ΡŒ Π²Π½ΠΈΠΌΠ°Π½ΠΈΠ΅ Π΅Π΄ΠΈΠ½ΠΈΡ†Π°ΠΌ — Π½Π΅ΠΎΡ‚ΡŠΠ΅ΠΌΠ»Π΅ΠΌΠ°Ρ Ρ‡Π΅Ρ€Ρ‚Π° любого Ρ…ΠΎΡ€ΠΎΡˆΠ΅Π³ΠΎ студСнта-Ρ„ΠΈΠ·ΠΈΠΊΠ°.МногиС трудности, связанныС с Ρ€Π΅ΡˆΠ΅Π½ΠΈΠ΅ΠΌ ΠΏΡ€ΠΎΠ±Π»Π΅ΠΌ, ΠΌΠΎΠ³ΡƒΡ‚ Π±Ρ‹Ρ‚ΡŒ связаны с Ρ‚Π΅ΠΌ, Ρ‡Ρ‚ΠΎ Π½Π΅ ΡƒΠ΄Π΅Π»ΠΈΠ»ΠΈ Π²Π½ΠΈΠΌΠ°Π½ΠΈΠ΅ подраздСлСниям. ΠŸΠΎΡΠΊΠΎΠ»ΡŒΠΊΡƒ всС большС ΠΈ большС элСктричСских Π²Π΅Π»ΠΈΡ‡ΠΈΠ½ ΠΈ ΠΈΡ… ΡΠΎΠΎΡ‚Π²Π΅Ρ‚ΡΡ‚Π²ΡƒΡŽΡ‰ΠΈΡ… мСтричСских Π΅Π΄ΠΈΠ½ΠΈΡ† вводится Π² этот Ρ€Π°Π·Π΄Π΅Π» ΡƒΡ‡Π΅Π±Π½ΠΎΠ³ΠΎ пособия «ЀизичСский класс», становится всС Π±ΠΎΠ»Π΅Π΅ Π²Π°ΠΆΠ½Ρ‹ΠΌ ΠΎΡ€Π³Π°Π½ΠΈΠ·ΠΎΠ²Π°Ρ‚ΡŒ ΠΈΠ½Ρ„ΠΎΡ€ΠΌΠ°Ρ†ΠΈΡŽ Π² своСй Π³ΠΎΠ»ΠΎΠ²Π΅. Π’ Ρ‚Π°Π±Π»ΠΈΡ†Π΅ Π½ΠΈΠΆΠ΅ пСрСчислСны Π½Π΅ΠΊΠΎΡ‚ΠΎΡ€Ρ‹Π΅ ΠΈΠ· Π²Π²Π΅Π΄Π΅Π½Π½Ρ‹Ρ… Π½Π° Π΄Π°Π½Π½Ρ‹ΠΉ ΠΌΠΎΠΌΠ΅Π½Ρ‚ количСств. Π‘ΠΈΠΌΠ²ΠΎΠ», ΡƒΡ€Π°Π²Π½Π΅Π½ΠΈΠ΅ ΠΈ ΡΠΎΠΎΡ‚Π²Π΅Ρ‚ΡΡ‚Π²ΡƒΡŽΡ‰ΠΈΠ΅ мСтричСскиС Π΅Π΄ΠΈΠ½ΠΈΡ†Ρ‹ Ρ‚Π°ΠΊΠΆΠ΅ ΡƒΠΊΠ°Π·Π°Π½Ρ‹ для ΠΊΠ°ΠΆΠ΄ΠΎΠΉ Π²Π΅Π»ΠΈΡ‡ΠΈΠ½Ρ‹.Π‘Ρ‹Π»ΠΎ Π±Ρ‹ Ρ€Π°Π·ΡƒΠΌΠ½ΠΎ часто ΠΎΠ±Ρ€Π°Ρ‰Π°Ρ‚ΡŒΡΡ ΠΊ этому списку ΠΈΠ»ΠΈ Π΄Π°ΠΆΠ΅ Π΄Π΅Π»Π°Ρ‚ΡŒ свою копию ΠΈ Π΄ΠΎΠ±Π°Π²Π»ΡΡ‚ΡŒ Π΅Π΅ ΠΏΠΎ ΠΌΠ΅Ρ€Π΅ развития модуля. НСкоторыС студСнты ΡΡ‡ΠΈΡ‚Π°ΡŽΡ‚ ΠΏΠΎΠ»Π΅Π·Π½Ρ‹ΠΌ ΡΠΎΡΡ‚Π°Π²ΠΈΡ‚ΡŒ пятый столбСц, Π² ΠΊΠΎΡ‚ΠΎΡ€ΠΎΠΌ приводится ΠΎΠΏΡ€Π΅Π΄Π΅Π»Π΅Π½ΠΈΠ΅ ΠΊΠ°ΠΆΠ΄ΠΎΠΉ Π²Π΅Π»ΠΈΡ‡ΠΈΠ½Ρ‹.

    Кол-Π²ΠΎ Π‘ΠΈΠΌΠ²ΠΎΠ» Π£Ρ€Π°Π²Π½Π΅Π½ΠΈΠ΅ (я) Бтандартная мСтричСская Π΅Π΄ΠΈΠ½ΠΈΡ†Π° Π”Ρ€ΡƒΠ³ΠΈΠ΅ Π΅Π΄ΠΈΠ½ΠΈΡ†Ρ‹
    Π Π°Π·Π½ΠΎΡΡ‚ΡŒ ΠΏΠΎΡ‚Π΅Π½Ρ†ΠΈΠ°Π»ΠΎΠ²

    (Π³.k.a. напряТСниС)

    Ξ”V Ξ”V = Ξ”PE / Q

    Ξ”V = I β€’ R

    Π’ΠΎΠ»ΡŒΡ‚ (Π’) J / C
    Ρ‚ΠΎΠΊ я I = Q / Ρ‚

    I = Ξ”V / R

    АмпСры (А) Π£ΡΠΈΠ»ΠΈΡ‚Π΅Π»ΡŒ ΠΈΠ»ΠΈ К / с

    ΠΈΠ»ΠΈ Π’ / Ом

    ΠœΠΎΡ‰Π½ΠΎΡΡ‚ΡŒ ΠΏ P = Ξ”PE / Ρ‚

    (Π΅Ρ‰Π΅ прСдстоит)

    Π’Π°Ρ‚Ρ‚ (Π’Ρ‚) Π”ΠΆ / с
    Π‘ΠΎΠΏΡ€ΠΎΡ‚ΠΈΠ²Π»Π΅Π½ΠΈΠ΅ Ρ€ R = ρ β€’ L / A

    R = Ξ”V / I

    Ом (Ом) Π’ / А
    ЭнСргия E ΠΈΠ»ΠΈ Ξ”PE Ξ”PE = Ξ”V β€’ Q

    Ξ”PE = P β€’ t

    Π”ΠΆΠΎΡƒΠ»ΡŒ (Π”ΠΆ) V β€’ C ΠΈΠ»ΠΈ

    Π’Ρ‚ β€’ с

    (ΠžΠ±Ρ€Π°Ρ‚ΠΈΡ‚Π΅ Π²Π½ΠΈΠΌΠ°Π½ΠΈΠ΅, Ρ‡Ρ‚ΠΎ символ Π΅Π΄ΠΈΠ½ΠΈΡ†Ρ‹ C прСдставляСт собой ΠΊΡƒΠ»ΠΎΠ½Ρ‹. )

    Π’ ΡΠ»Π΅Π΄ΡƒΡŽΡ‰Π΅ΠΌ Ρ€Π°Π·Π΄Π΅Π»Π΅ Π£Ρ€ΠΎΠΊΠ° 3 ΠΌΡ‹ Π΅Ρ‰Π΅ Ρ€Π°Π· рассмотрим ΠΊΠΎΠ»ΠΈΡ‡Π΅ΡΡ‚Π²Π΅Π½Π½ΡƒΡŽ ΠΌΠΎΡ‰Π½ΠΎΡΡ‚ΡŒ. НовоС ΡƒΡ€Π°Π²Π½Π΅Π½ΠΈΠ΅ мощности Π±ΡƒΠ΄Π΅Ρ‚ Π²Π²Π΅Π΄Π΅Π½ΠΎ ΠΏΡƒΡ‚Π΅ΠΌ объСдинСния Π΄Π²ΡƒΡ… (ΠΈΠ»ΠΈ Π±ΠΎΠ»Π΅Π΅) ΡƒΡ€Π°Π²Π½Π΅Π½ΠΈΠΉ Π² ΠΏΡ€ΠΈΠ²Π΅Π΄Π΅Π½Π½ΠΎΠΉ Π²Ρ‹ΡˆΠ΅ Ρ‚Π°Π±Π»ΠΈΡ†Π΅.

    ΠœΡ‹ Ρ…ΠΎΡ‚Π΅Π»ΠΈ Π±Ρ‹ ΠΏΡ€Π΅Π΄Π»ΠΎΠΆΠΈΡ‚ΡŒ …

    Π—Π°Ρ‡Π΅ΠΌ просто Ρ‡ΠΈΡ‚Π°Ρ‚ΡŒ ΠΎΠ± этом ΠΈ ΠΊΠΎΠ³Π΄Π° ΠΌΠΎΠΆΠ½ΠΎ с этим Π²Π·Π°ΠΈΠΌΠΎΠ΄Π΅ΠΉΡΡ‚Π²ΠΎΠ²Π°Ρ‚ΡŒ? ВзаимодСйствиС — это ΠΈΠΌΠ΅Π½Π½ΠΎ Ρ‚ΠΎ, Ρ‡Ρ‚ΠΎ Π²Ρ‹ Π΄Π΅Π»Π°Π΅Ρ‚Π΅, ΠΊΠΎΠ³Π΄Π° ΠΈΡΠΏΠΎΠ»ΡŒΠ·ΡƒΠ΅Ρ‚Π΅ ΠΎΠ΄Π½Ρƒ ΠΈΠ· ΠΈΠ½Ρ‚Π΅Ρ€Π°ΠΊΡ‚ΠΈΠ²Π½Ρ‹Ρ… Ρ„ΡƒΠ½ΠΊΡ†ΠΈΠΉ The Physics Classroom.ΠœΡ‹ Ρ…ΠΎΡ‚Π΅Π»ΠΈ Π±Ρ‹ ΠΏΡ€Π΅Π΄Π»ΠΎΠΆΠΈΡ‚ΡŒ Π²Π°ΠΌ ΡΠΎΠ²ΠΌΠ΅ΡΡ‚ΠΈΡ‚ΡŒ Ρ‡Ρ‚Π΅Π½ΠΈΠ΅ этой страницы с использованиСм нашСго ΠΈΠ½Ρ‚Π΅Ρ€Π°ΠΊΡ‚ΠΈΠ²Π½ΠΎΠ³ΠΎ срСдства построСния Ρ†Π΅ΠΏΠ΅ΠΉ постоянного Ρ‚ΠΎΠΊΠ°. Π’Ρ‹ ΠΌΠΎΠΆΠ΅Ρ‚Π΅ Π½Π°ΠΉΡ‚ΠΈ Π΅Π³ΠΎ Π² Ρ€Π°Π·Π΄Π΅Π»Π΅ Physics Interactives Π½Π° нашСм сайтС. ΠŸΠΎΡΡ‚Ρ€ΠΎΠΈΡ‚Π΅Π»ΡŒ Ρ†Π΅ΠΏΠ΅ΠΉ постоянного Ρ‚ΠΎΠΊΠ° прСдоставляСт учащСмуся Π½Π°Π±ΠΎΡ€ для построСния Π²ΠΈΡ€Ρ‚ΡƒΠ°Π»ΡŒΠ½Ρ‹Ρ… Ρ†Π΅ΠΏΠ΅ΠΉ. Π‘ Π»Π΅Π³ΠΊΠΎΡΡ‚ΡŒΡŽ ΠΏΠ΅Ρ€Π΅Ρ‚Π°Ρ‰ΠΈΡ‚Π΅ источник напряТСния, рСзисторы ΠΈ ΠΏΡ€ΠΎΠ²ΠΎΠ΄Π° Π½Π° Ρ€Π°Π±ΠΎΡ‡Π΅Π΅ мСсто. Π‘ΠΎΠ΅Π΄ΠΈΠ½ΠΈΡ‚Π΅ ΠΈΡ…, ΠΈ Ρƒ вас Π±ΡƒΠ΄Π΅Ρ‚ схСма. Π”ΠΎΠ±Π°Π²ΡŒΡ‚Π΅ Π°ΠΌΠΏΠ΅Ρ€ΠΌΠ΅Ρ‚Ρ€ для измСрСния Ρ‚ΠΎΠΊΠ° ΠΈ ΠΈΡΠΏΠΎΠ»ΡŒΠ·ΡƒΠΉΡ‚Π΅ Π΄Π°Ρ‚Ρ‡ΠΈΠΊΠΈ напряТСния для опрСдСлСния падСния напряТСния. Π­Ρ‚ΠΎ Ρ‚Π°ΠΊ просто. И Π½Π΅ Π½ΡƒΠΆΠ½ΠΎ Π±Π΅ΡΠΏΠΎΠΊΠΎΠΈΡ‚ΡŒΡΡ ΠΎ ΠΏΠΎΡ€Π°ΠΆΠ΅Π½ΠΈΠΈ элСктричСским Ρ‚ΠΎΠΊΠΎΠΌ (Ссли, ΠΊΠΎΠ½Π΅Ρ‡Π½ΠΎ, Π²Ρ‹ Π½Π΅ Ρ‡ΠΈΡ‚Π°Π΅Ρ‚Π΅ это Π² Π²Π°Π½Π½ΠΎΠΉ).

    ΠŸΡ€ΠΎΠ²Π΅Ρ€ΡŒΡ‚Π΅ своС ΠΏΠΎΠ½ΠΈΠΌΠ°Π½ΠΈΠ΅

    1. Π§Ρ‚ΠΎ ΠΈΠ· пСрСчислСнного Π½ΠΈΠΆΠ΅ ΠΏΡ€ΠΈΠ²Π΅Π΄Π΅Ρ‚ ΠΊ ΡƒΠΌΠ΅Π½ΡŒΡˆΠ΅Π½ΠΈΡŽ Ρ‚ΠΎΠΊΠ° Π² элСктричСской Ρ†Π΅ΠΏΠΈ? Π’Ρ‹Π±Π΅Ρ€ΠΈΡ‚Π΅ всС, Ρ‡Ρ‚ΠΎ ΠΏΠΎΠ΄Ρ…ΠΎΠ΄ΠΈΡ‚.

    Π°. ΡƒΠΌΠ΅Π½ΡŒΡˆΠΈΡ‚ΡŒ напряТСниС

    Π³. ΡƒΠΌΠ΅Π½ΡŒΡˆΠΈΡ‚ΡŒ сопротивлСниС

    Π³. ΡƒΠ²Π΅Π»ΠΈΡ‡ΠΈΡ‚ΡŒ напряТСниС

    Π³.ΡƒΠ²Π΅Π»ΠΈΡ‡ΠΈΡ‚ΡŒ сопротивлСниС

    2. ΠžΠΏΡ€Π΅Π΄Π΅Π»Π΅Π½Π½Π°Ρ элСктричСская Ρ†Π΅ΠΏΡŒ содСрТит аккумулятор ΠΈΠ· Ρ‚Ρ€Π΅Ρ… элСмСнтов, ΠΏΡ€ΠΎΠ²ΠΎΠ΄Π° ΠΈ Π»Π°ΠΌΠΏΠΎΡ‡ΠΊΡƒ. Из-Π·Π° Ρ‡Π΅Π³ΠΎ Π»Π°ΠΌΠΏΠ° Π±ΡƒΠ΄Π΅Ρ‚ ΡΠ²Π΅Ρ‚ΠΈΡ‚ΡŒ ΠΌΠ΅Π½Π΅Π΅ ярко? Π’Ρ‹Π±Π΅Ρ€ΠΈΡ‚Π΅ всС, Ρ‡Ρ‚ΠΎ ΠΏΠΎΠ΄Ρ…ΠΎΠ΄ΠΈΡ‚.

    Π°. ΡƒΠ²Π΅Π»ΠΈΡ‡ΠΈΡ‚ΡŒ напряТСниС Π±Π°Ρ‚Π°Ρ€Π΅ΠΈ (Π΄ΠΎΠ±Π°Π²ΠΈΡ‚ΡŒ Π΅Ρ‰Π΅ ΠΎΠ΄Π½Ρƒ ячСйку)

    Π³. ΡƒΠΌΠ΅Π½ΡŒΡˆΠΈΡ‚ΡŒ напряТСниС аккумулятора (ΡƒΠ΄Π°Π»ΠΈΡ‚ΡŒ элСмСнт)

    Π³.ΡƒΠΌΠ΅Π½ΡŒΡˆΠΈΡ‚ΡŒ сопротивлСниС Ρ†Π΅ΠΏΠΈ

    Π³. ΡƒΠ²Π΅Π»ΠΈΡ‡ΠΈΡ‚ΡŒ сопротивлСниС Ρ†Π΅ΠΏΠΈ

    3. Π‘ΠΊΠΎΡ€Π΅Π΅ всСго, вас ΠΏΡ€Π΅Π΄ΡƒΠΏΡ€Π΅Π΄ΠΈΠ»ΠΈ, Ρ‡Ρ‚ΠΎΠ±Ρ‹ Π²Ρ‹ Π½Π΅ ΠΏΡ€ΠΈΠΊΠ°ΡΠ°Π»ΠΈΡΡŒ ΠΊ элСктроприборам ΠΈΠ»ΠΈ Π΄Π°ΠΆΠ΅ ΠΊ элСктричСским Ρ€ΠΎΠ·Π΅Ρ‚ΠΊΠ°ΠΌ ΠΌΠΎΠΊΡ€Ρ‹ΠΌΠΈ Ρ€ΡƒΠΊΠ°ΠΌΠΈ. Π’Π°ΠΊΠΎΠΉ ΠΊΠΎΠ½Ρ‚Π°ΠΊΡ‚ Π±ΠΎΠ»Π΅Π΅ опасСн, ΠΊΠΎΠ³Π΄Π° ваши Ρ€ΡƒΠΊΠΈ ΠΌΠΎΠΊΡ€Ρ‹Π΅ (Π° Π½Π΅ сухиС), ΠΏΠΎΡ‚ΠΎΠΌΡƒ Ρ‡Ρ‚ΠΎ ΠΌΠΎΠΊΡ€Ρ‹Π΅ Ρ€ΡƒΠΊΠΈ Π²Ρ‹Π·Ρ‹Π²Π°ΡŽΡ‚ ____.

    Π°.напряТСниС Ρ†Π΅ΠΏΠΈ Π΄ΠΎΠ»ΠΆΠ½ΠΎ Π±Ρ‹Ρ‚ΡŒ Π²Ρ‹ΡˆΠ΅

    Π³. напряТСниС Ρ†Π΅ΠΏΠΈ Π΄ΠΎΠ»ΠΆΠ½ΠΎ Π±Ρ‹Ρ‚ΡŒ Π½ΠΈΠΆΠ΅

    Π³. вашС сопротивлСниС Π±ΡƒΠ΄Π΅Ρ‚ Π²Ρ‹ΡˆΠ΅

    Π³. вашС сопротивлСниС Π΄ΠΎΠ»ΠΆΠ½ΠΎ Π±Ρ‹Ρ‚ΡŒ Π½ΠΈΠΆΠ΅

    e. Ρ‚ΠΎΠΊ Ρ‡Π΅Ρ€Π΅Π· тСбя Π±ΡƒΠ΄Π΅Ρ‚ Π½ΠΈΠΆΠ΅

    4. Если Π±Ρ‹ сопротивлСниС Ρ†Π΅ΠΏΠΈ Π±Ρ‹Π»ΠΎ ΡƒΡ‚Ρ€ΠΎΠ΅Π½ΠΎ, Ρ‚ΠΎ Ρ‚ΠΎΠΊ Ρ‡Π΅Ρ€Π΅Π· Ρ†Π΅ΠΏΡŒ Π±Ρ‹Π» Π±Ρ‹ ____.

    Π°. Ρ‚Ρ€Π΅Ρ‚ΡŒ ΠΎΡ‚

    Π³. Π²Ρ‚Ρ€ΠΎΠ΅ большС

    Π³. Π±Π΅Π· ΠΈΠ·ΠΌΠ΅Π½Π΅Π½ΠΈΠΉ

    Π³. … Π΅Ρ€ΡƒΠ½Π΄Π°! Π‘Π΄Π΅Π»Π°Ρ‚ΡŒ Ρ‚Π°ΠΊΠΎΠΉ ΠΏΡ€ΠΎΠ³Π½ΠΎΠ· Π½Π΅Π²ΠΎΠ·ΠΌΠΎΠΆΠ½ΠΎ.

    5. Если напряТСниС Π² Ρ†Π΅ΠΏΠΈ ΡƒΠ²Π΅Π»ΠΈΡ‡ΠΈΡ‚ΡŒ Π² Ρ‡Π΅Ρ‚Ρ‹Ρ€Π΅ Ρ€Π°Π·Π°, Ρ‚ΠΎ Ρ‚ΠΎΠΊ Π² Ρ†Π΅ΠΏΠΈ Π±ΡƒΠ΄Π΅Ρ‚ ____.

    Π°.Ρ‡Π΅Ρ‚Π²Π΅Ρ€Ρ‚ΡŒ ΠΎΡ‚

    Π³. Π² Ρ‡Π΅Ρ‚Ρ‹Ρ€Π΅ Ρ€Π°Π·Π° большС

    Π³. Π±Π΅Π· ΠΈΠ·ΠΌΠ΅Π½Π΅Π½ΠΈΠΉ

    Π³. … Π΅Ρ€ΡƒΠ½Π΄Π°! Π‘Π΄Π΅Π»Π°Ρ‚ΡŒ Ρ‚Π°ΠΊΠΎΠΉ ΠΏΡ€ΠΎΠ³Π½ΠΎΠ· Π½Π΅Π²ΠΎΠ·ΠΌΠΎΠΆΠ½ΠΎ.

    6. Π‘Ρ…Π΅ΠΌΠ° соСдинСна с Π±Π»ΠΎΠΊΠΎΠΌ питания, рСзистором ΠΈ Π°ΠΌΠΏΠ΅Ρ€ΠΌΠ΅Ρ‚Ρ€ΠΎΠΌ (для измСрСния Ρ‚ΠΎΠΊΠ°). АмпСрмСтр ΠΏΠΎΠΊΠ°Π·Ρ‹Π²Π°Π΅Ρ‚ Π·Π½Π°Ρ‡Π΅Π½ΠΈΠ΅ Ρ‚ΠΎΠΊΠ° 24 мА (миллиАмпСр). ΠžΠΏΡ€Π΅Π΄Π΅Π»ΠΈΡ‚Π΅ Π½ΠΎΠ²Ρ‹ΠΉ Ρ‚ΠΎΠΊ, Ссли напряТСниС источника питания Π±Ρ‹Π»ΠΎ…

    Π°. … увСличился Π² 2 Ρ€Π°Π·Π°, Π° сопротивлСниС ΠΎΡΡ‚Π°Π»ΠΎΡΡŒ постоянным.

    Π³. … увСличился Π² 3 Ρ€Π°Π·Π°, Π° сопротивлСниС ΠΎΡΡ‚Π°Π»ΠΎΡΡŒ постоянным.

    Π³. … ΡƒΠΌΠ΅Π½ΡŒΡˆΠΈΠ»ΠΎΡΡŒ Π² 2 Ρ€Π°Π·Π°, Π° сопротивлСниС ΠΎΡΡ‚Π°Π»ΠΎΡΡŒ постоянным.

    Π³. … ΠΎΡΡ‚Π°Π²Π°Π»ΠΎΡΡŒ постоянным, Π° сопротивлСниС ΡƒΠ²Π΅Π»ΠΈΡ‡ΠΈΠ²Π°Π»ΠΎΡΡŒ Π² 2 Ρ€Π°Π·Π°.

    e. … ΠΎΡΡ‚Π°Π²Π°Π»ΠΎΡΡŒ постоянным, Π° сопротивлСниС ΡƒΠ²Π΅Π»ΠΈΡ‡ΠΈΠ²Π°Π»ΠΎΡΡŒ Π² 4 Ρ€Π°Π·Π°.

    Ρ„…. ΠΎΡΡ‚Π°Π²Π°Π»ΠΎΡΡŒ постоянным, Π° сопротивлСниС ΡƒΠΌΠ΅Π½ΡŒΡˆΠ°Π»ΠΎΡΡŒ Π² 2 Ρ€Π°Π·Π°.

    Π³. … ΡƒΠ²Π΅Π»ΠΈΡ‡ΠΈΠ»ΠΎΡΡŒ Π² 2 Ρ€Π°Π·Π°, Π° сопротивлСниС ΡƒΠ²Π΅Π»ΠΈΡ‡ΠΈΠ»ΠΎΡΡŒ Π² 2 Ρ€Π°Π·Π°.

    Ρ‡. … ΡƒΠ²Π΅Π»ΠΈΡ‡ΠΈΠ»ΠΎΡΡŒ Π² 3 Ρ€Π°Π·Π°, Π° сопротивлСниС ΡƒΠΌΠ΅Π½ΡŒΡˆΠΈΠ»ΠΎΡΡŒ Π² 2 Ρ€Π°Π·Π°.

    ΠΈ. … ΡƒΠΌΠ΅Π½ΡŒΡˆΠΈΠ»ΠΎΡΡŒ Π² 2 Ρ€Π°Π·Π°, Π° сопротивлСниС ΡƒΠ²Π΅Π»ΠΈΡ‡ΠΈΠ»ΠΎΡΡŒ Π² 2 Ρ€Π°Π·Π°.

    7.Π˜ΡΠΏΠΎΠ»ΡŒΠ·ΡƒΠΉΡ‚Π΅ ΡƒΡ€Π°Π²Π½Π΅Π½ΠΈΠ΅ Π·Π°ΠΊΠΎΠ½Π° Ома, Ρ‡Ρ‚ΠΎΠ±Ρ‹ Π΄Π°Ρ‚ΡŒ числовыС ΠΎΡ‚Π²Π΅Ρ‚Ρ‹ Π½Π° ΡΠ»Π΅Π΄ΡƒΡŽΡ‰ΠΈΠ΅ вопросы:

    Π°. ЭлСктричСскоС устройство с сопротивлСниСм 3,0 Ом ΠΏΠΎΠ·Π²ΠΎΠ»ΠΈΡ‚ ΠΏΡ€ΠΎΡ‚Π΅ΠΊΠ°Ρ‚ΡŒ Ρ‡Π΅Ρ€Π΅Π· Π½Π΅Π³ΠΎ Ρ‚ΠΎΠΊΡƒ 4,0 А, Ссли Π½Π° устройствС Π½Π°Π±Π»ΡŽΠ΄Π°Π΅Ρ‚ΡΡ ΠΏΠ°Π΄Π΅Π½ΠΈΠ΅ напряТСния ________ Π’ΠΎΠ»ΡŒΡ‚.

    Π³. Когда Π½Π° элСктричСский Π½Π°Π³Ρ€Π΅Π²Π°Ρ‚Π΅Π»ΡŒ подаСтся напряТСниС 120 Π’, Ρ‡Π΅Ρ€Π΅Π· Π½Π°Π³Ρ€Π΅Π²Π°Ρ‚Π΅Π»ΡŒ Π±ΡƒΠ΄Π΅Ρ‚ ΠΏΡ€ΠΎΡ‚Π΅ΠΊΠ°Ρ‚ΡŒ Ρ‚ΠΎΠΊ 10,0 А, Ссли сопротивлСниС составляСт ________ Ом.

    Π³. Π€ΠΎΠ½Π°Ρ€ΠΈΠΊ, ΠΊΠΎΡ‚ΠΎΡ€Ρ‹ΠΉ питаСтся ΠΎΡ‚ 3 Π’ ΠΈ ΠΈΡΠΏΠΎΠ»ΡŒΠ·ΡƒΠ΅Ρ‚ Π»Π°ΠΌΠΏΡƒ с сопротивлСниСм 60 Ом, Π±ΡƒΠ΄Π΅Ρ‚ ΠΈΠΌΠ΅Ρ‚ΡŒ Ρ‚ΠΎΠΊ ________ Π°ΠΌΠΏΠ΅Ρ€.

    8. Π˜ΡΠΏΠΎΠ»ΡŒΠ·ΡƒΠΉΡ‚Π΅ ΡƒΡ€Π°Π²Π½Π΅Π½ΠΈΠ΅ Π·Π°ΠΊΠΎΠ½Π° Ома для опрСдСлСния Π½Π΅Π΄ΠΎΡΡ‚Π°ΡŽΡ‰ΠΈΡ… Π·Π½Π°Ρ‡Π΅Π½ΠΈΠΉ Π² ΡΠ»Π΅Π΄ΡƒΡŽΡ‰ΠΈΡ… схСмах.

    9. Π‘ΠΌ. Вопрос 8 Π²Ρ‹ΡˆΠ΅. Π’ схСмах схСм A ΠΈ B ΠΊΠ°ΠΊΠΎΠΉ ΠΌΠ΅Ρ‚ΠΎΠ΄ использовался для управлСния Ρ‚ΠΎΠΊΠΎΠΌ Π² схСмах? А Π² схСмах схСм C ΠΈ D ΠΊΠ°ΠΊΠΎΠΉ ΠΌΠ΅Ρ‚ΠΎΠ΄ использовался для контроля Ρ‚ΠΎΠΊΠ° Π² схСмах?

    БАМАЯ ПРОБВАЯ Π¦Π•ΠŸΠ¬:

    БАМАЯ ПРОБВАЯ Π¦Π•ΠŸΠ¬:

    Π—ΠΠšΠžΠ ОМА

    БАМАЯ ПРОБВАЯ Π¦Π•ΠŸΠ¬:

    ΠœΡ‹ ΠΌΠΎΠΆΠ΅ΠΌ Π·Π°ΡΡ‚Π°Π²ΠΈΡ‚ΡŒ Ρ‚ΠΎΠΊ Ρ‚Π΅Ρ‡ΡŒ ΠΏΠΎ ΠΊΡ€ΡƒΠ³Ρƒ (Ρ†Π΅ΠΏΠΈ)
    ΠΏΡƒΡ‚Π΅ΠΌ соСдинСния ΠΊΠ»Π΅ΠΌΠΌ аккумулятора вмСстС. Π­Ρ‚ΠΎ растопит
    ΠΏΡ€ΠΎΠ²ΠΎΠ΄, Ρ€Π°Π·ΠΆΠΈΠ³Π°ΠΉ искры ΠΈ, Π²ΠΎΠ·ΠΌΠΎΠΆΠ½ΠΎ, Ρ€Π°Π·ΠΆΠΈΠ³Π°ΠΉ огонь, Ρ‚Π°ΠΊ Ρ‡Ρ‚ΠΎ Π½Π΅ Π΄Π΅Π»Π°ΠΉ этого.
    ВмСсто этого ΠΏΠΎΠ΄ΠΊΠ»ΡŽΡ‡ΠΈΡ‚Π΅ Ρ‡Ρ‚ΠΎ-Π½ΠΈΠ±ΡƒΠ΄ΡŒ для управлСния Ρ‚ΠΎΠΊΠΎΠΌ. Π‘ΠΏΠΎΡΠΎΠ±Π½ΠΎΡΡ‚ΡŒ ΠΊ
    ΡƒΠΏΡ€Π°Π²Π»ΡΡŽΡ‰ΠΈΠΉ Ρ‚ΠΎΠΊ называСтся сопротивлСниСм, ΠΈ всС ΠΌΠ°Ρ‚Π΅Ρ€ΠΈΠ°Π»Ρ‹ ΠΈΠΌΠ΅ΡŽΡ‚ Π΅Π³ΠΎ
    Π² Π½Π΅ΠΊΠΎΡ‚ΠΎΡ€ΠΎΠΉ стСпСни — Π½Π° самом Π΄Π΅Π»Π΅ ΠΌΡ‹ классифицируСм ΠΌΠ°Ρ‚Π΅Ρ€ΠΈΠ°Π»Ρ‹ ΠΏΠΎ ΠΈΡ…
    сопротивлСниС: с ΠΎΡ‡Π΅Π½ΡŒ Π½ΠΈΠ·ΠΊΠΈΠΌ сопротивлСниСм — ΠΏΡ€ΠΎΠ²ΠΎΠ΄Π½ΠΈΠΊΠΈ, с
    ΠΌΠ½ΠΎΠ³ΠΎ сопротивлСния — изоляторы. Π•ΡΡ‚ΡŒ устройства, Π½Π°Π·Ρ‹Π²Π°Π΅ΠΌΡ‹Π΅ рСзисторами
    ΠΊΠΎΡ‚ΠΎΡ€Ρ‹Π΅ ΠΈΡΠΏΠΎΠ»ΡŒΠ·ΡƒΡŽΡ‚ΡΡ Π² элСктронных Π³Π°Π΄ΠΆΠ΅Ρ‚Π°Ρ… — ΠΎΠ½ΠΈ ΠΈΠΌΠ΅ΡŽΡ‚ сопротивлСниС, ΠΊΠΎΡ‚ΠΎΡ€ΠΎΠ΅
    Ρ‡Ρ‚ΠΎ-Ρ‚ΠΎ срСднСС ΠΌΠ΅ΠΆΠ΄Ρƒ ΠΏΡ€ΠΎΠ²ΠΎΠ΄ΠΈΠΌΠΎΡΡ‚ΡŒΡŽ ΠΈ изоляциСй ΠΈ прСдсказуСмо.Π’Π°ΠΊ
    Π²ΠΎΡ‚ бСзопасная Ρ†Π΅ΠΏΡŒ:

    БатарСя ΠΈΠΌΠ΅Π΅Ρ‚ ΠΎΠΏΡ€Π΅Π΄Π΅Π»Π΅Π½Π½ΠΎΠ΅ Π½Π°ΠΆΠ°Ρ‚ΠΈΠ΅,
    называСтся элСктродвиТущСй силой ΠΈΠ»ΠΈ Π­Π”Π‘. Π­Ρ‚ΠΎ измСряСтся Π² Π΅Π΄ΠΈΠ½ΠΈΡ†Π°Ρ…, Π½Π°Π·Ρ‹Π²Π°Π΅ΠΌΡ‹Ρ…
    Π²ΠΎΠ»ΡŒΡ‚ .
    Π­Π”Π‘ (часто Π½Π°Π·Ρ‹Π²Π°Π΅ΠΌΡƒΡŽ напряТСниСм) ΡƒΠΊΠ°Π·Ρ‹Π²Π°Π΅ΠΌ Π² Ρ„ΠΎΡ€ΠΌΡƒΠ»Π°Ρ… Π±ΡƒΠΊΠ²ΠΎΠΉ
    E .
    НапряТСниС Π΄ΠΎΠ»ΠΆΠ½ΠΎ Π±Ρ‹Ρ‚ΡŒ ΠΈΠ·ΠΌΠ΅Ρ€Π΅Π½ΠΎ ΠΌΠ΅ΠΆΠ΄Ρƒ двумя Ρ‚ΠΎΡ‡ΠΊΠ°ΠΌΠΈ Ρ†Π΅ΠΏΠΈ Π²
    Ρ‚Π°ΠΊΠΈΠΌ ΠΆΠ΅ ΠΎΠ±Ρ€Π°Π·ΠΎΠΌ измСряСтся высота ΠΌΠ΅ΠΆΠ΄Ρƒ двумя Ρ‚ΠΎΡ‡ΠΊΠ°ΠΌΠΈ Π½Π°
    сторона Π³ΠΎΡ€Ρ‹. НСт Ρ‚Π°ΠΊΠΎΠ³ΠΎ понятия, ΠΊΠ°ΠΊ Β«0 Π²ΠΎΠ»ΡŒΡ‚Β», ΠΊΡ€ΠΎΠΌΠ΅
    напряТСниС ΠΌΠ΅ΠΆΠ΄Ρƒ двумя Ρ‚ΠΎΡ‡ΠΊΠ°ΠΌΠΈ Ρ€Π°Π²Π½ΠΎ 0, Ссли ΠΎΠ½ΠΈ ΠΏΠΎΠ΄ΠΊΠ»ΡŽΡ‡Π΅Π½Ρ‹
    всС вмСстС.

    РСзистор ΠΈΠΌΠ΅Π΅Ρ‚ ΠΎΠΏΡ€Π΅Π΄Π΅Π»Π΅Π½Π½ΡƒΡŽ Π²Π΅Π»ΠΈΡ‡ΠΈΠ½Ρƒ
    Π‘ΠΎΠΏΡ€ΠΎΡ‚ΠΈΠ²Π»Π΅Π½ΠΈΠ΅, ΠΈΠ·ΠΌΠ΅Ρ€Π΅Π½Π½ΠΎΠ΅ Π² Π΅Π΄ΠΈΠ½ΠΈΡ†Π°Ρ…, называСтся Ом . ΠœΡ‹ ΡƒΠΊΠ°Π·Ρ‹Π²Π°Π΅ΠΌ
    сопротивлСниС Π² Ρ„ΠΎΡ€ΠΌΡƒΠ»Π°Ρ… с Π±ΡƒΠΊΠ²ΠΎΠΉ R .

    Когда Ρ‚ΠΎΠΊ Ρ‚Π΅Ρ‡Π΅Ρ‚, ΠΌΡ‹ измСряСм Π΅Π³ΠΎ Π²
    Π΅Π΄ΠΈΠ½ΠΈΡ†Ρ‹ ΠΎΠ±ΠΎΠ·Π½Π°Ρ‡Π°ΡŽΡ‚ Π°ΠΌΠΏΠ΅Ρ€ , ΠΈ ΠΎΠ±ΠΎΠ·Π½Π°Ρ‡Π°ΡŽΡ‚ Π΅Π³ΠΎ Π±ΡƒΠΊΠ²ΠΎΠΉ I .

    Π­Ρ‚ΠΈ Ρ‚Ρ€ΠΈ связаны простой Ρ„ΠΎΡ€ΠΌΡƒΠ»ΠΎΠΉ
    называСтся Π·Π°ΠΊΠΎΠ½ΠΎΠΌ Ома:

    I = E / R

    ВакТС написано E =
    IR
    ΠΈΠ»ΠΈ R = E / I.

    Π­Ρ‚ΠΎ Π³ΠΎΠ²ΠΎΡ€ΠΈΡ‚ Π½Π°ΠΌ Ρ‚ΠΎΠΊ, Ссли ΠΌΡ‹ Π·Π½Π°Π΅ΠΌ
    напряТСниС ΠΈ сопротивлСниС, ΠΈΠ»ΠΈ напряТСниС, Ссли ΠΌΡ‹ Π·Π½Π°Π΅ΠΌ Ρ‚ΠΎΠΊ ΠΈ
    сопротивлСниС, ΠΈΠ»ΠΈ сопротивлСниС, Ссли ΠΌΡ‹ Π·Π½Π°Π΅ΠΌ Ρ‚ΠΎΠΊ ΠΈ напряТСниС. Если
    это каТСтся Π²Π°ΠΌ Π½Π΅ΠΌΠ½ΠΎΠ³ΠΎ Π·Π°ΠΌΠΊΠ½ΡƒΡ‚Ρ‹ΠΌ, Π²Ρ‹ ΠΏΡ€Π°Π²Ρ‹. ΠœΡ‹ ΠΌΠΎΠΆΠ΅ΠΌ ΠΈΠ·ΠΌΠ΅Ρ€ΠΈΡ‚ΡŒ
    Ρ‚ΠΎΠΊ силой ΠΌΠ°Π³Π½ΠΈΡ‚Π½ΠΎΠ³ΠΎ поля, ΠΊΠΎΡ‚ΠΎΡ€ΠΎΠ΅ ΠΎΠ½ Π±ΡƒΠ΄Π΅Ρ‚ Π³Π΅Π½Π΅Ρ€ΠΈΡ€ΠΎΠ²Π°Ρ‚ΡŒ, Π½ΠΎ Ρ‚Π°ΠΌ
    Π½Π΅ являСтся ΠΊΡ€ΠΈΡ‚Π΅Ρ€ΠΈΠ΅ΠΌ для измСрСния напряТСния, ΠΊΡ€ΠΎΠΌΠ΅ ΠΊΠ°ΠΊ ΡƒΠ²ΠΈΠ΄Π΅Ρ‚ΡŒ, сколько Ρ‚ΠΎΠΊΠ° Ρ‚Π΅Ρ‡Π΅Ρ‚
    Ρ‡Π΅Ρ€Π΅Π· извСстноС сопротивлСниС. А ΠΊΠ°ΠΊ ΡƒΠ·Π½Π°Ρ‚ΡŒ сопротивлСниС? ΠœΡ‹ примСняСм
    извСстноС напряТСниС ΠΈ посмотритС, сколько Ρ‚ΠΎΠΊΠ° Ρ‚Π΅Ρ‡Π΅Ρ‚.

    ΠžΠΏΡ€Π΅Π΄Π΅Π»Π΅Π½ΠΈΠ΅ Π΅Π΄ΠΈΠ½ΠΈΡ† Ρ‚Π°ΠΊΠΆΠ΅ ΠΊΡ€ΡƒΠ³Π»ΠΎΠ΅: 1
    Π°ΠΌΠΏΠ΅Ρ€ — это количСство Ρ‚ΠΎΠΊΠ°, ΠΏΡ€ΠΎΡ‚Π΅ΠΊΠ°ΡŽΡ‰Π΅Π³ΠΎ Ρ‡Π΅Ρ€Π΅Π· рСзистор сопротивлСниСм 1 Ом.
    Ссли ΠΏΡ€ΠΈΠ»ΠΎΠΆΠ΅Π½ΠΎ 1 Π²ΠΎΠ»ΡŒΡ‚.


    Π”Π²Π° рСзистора ΠΏΠΎΡΠ»Π΅Π΄ΠΎΠ²Π°Ρ‚Π΅Π»ΡŒΠ½ΠΎ:

    Π‘ΡƒΠ΄Π΅Ρ‚ Π½Π΅ΠΌΠ½ΠΎΠ³ΠΎ слоТнСС, Ссли Π΅ΡΡ‚ΡŒ
    Π΄Π²Π° рСзистора:

    Какой Π±Ρ‹ Π½ΠΈ Π±Ρ‹Π» Ρ‚ΠΎΠΊ, ΠΎΠ½ Ρ‚Π°ΠΊΠΎΠΉ ΠΆΠ΅ Π² A,
    B ΠΈ C. (Π‘ΠΎΠ»ΡŒΡˆΠ΅ Π½Π΅ΠΊΡƒΠ΄Π° Ρ‚Π΅Ρ‡ΡŒ Ρ‚ΠΎΠΊ).

    НапряТСниС ΠΌΠ΅ΠΆΠ΄Ρƒ A ΠΈ C Ρ€Π°Π²Π½ΠΎ этому
    ΠΌΠ΅ΠΆΠ΄Ρƒ A ΠΈ B добавляСтся ΠΊ Ρ‚ΠΎΠΌΡƒ, Ρ‡Ρ‚ΠΎ ΠΌΠ΅ΠΆΠ΄Ρƒ B ΠΈ C.

    E AC =
    E AB + E BC

    НапряТСния ΡΠΊΠ»Π°Π΄Ρ‹Π²Π°ΡŽΡ‚ΡΡ, ΠΊΠ°ΠΊ ΠΈ высота
    Π΄ΠΎΠΌ — это сумма высот Π΅Π³ΠΎ этаТСй.

    НапряТСниС Π½Π° ΠΊΠ°ΠΆΠ΄ΠΎΠΌ рСзисторС Ρ€Π°Π²Π½ΠΎ
    ΠΏΡ€ΠΎΠΏΠΎΡ€Ρ†ΠΈΠΎΠ½Π°Π»ΡŒΠ½ΠΎ ΡΠΎΠΏΡ€ΠΎΡ‚ΠΈΠ²Π»Π΅Π½ΠΈΡŽ ΠΊΠ°ΠΆΠ΄ΠΎΠ³ΠΎ рСзистора.

    E AB /
    R
    1 = E BC /
    R
    2

    Π’ΠΈΠ΄ΠΈΡ‚Π΅ Π»ΠΈ, Π·Π°ΠΊΠΎΠ½ Ома Π²Π΅Ρ€Π΅Π½ для ΠΊΠ°ΠΆΠ΄ΠΎΠΉ части
    схСму, Π° Ρ‚Π°ΠΊΠΆΠ΅ схСму Π² Ρ†Π΅Π»ΠΎΠΌ.Какой Π±Ρ‹ Ρ‚ΠΎΠΊ Π½ΠΈ Ρ‚Π΅ΠΊ,
    ΠΎΠ½ ΠΎΠ΄ΠΈΠ½Π°ΠΊΠΎΠ² Π½Π° ΠΊΠ°ΠΆΠ΄ΠΎΠΌ рСзисторС, поэтому напряТСниС Π±ΡƒΠ΄Π΅Ρ‚ Ρ€Π΅Π³ΡƒΠ»ΠΈΡ€ΠΎΠ²Π°Ρ‚ΡŒΡΡ
    самих сСбя.

    Π‘ΡƒΠΌΠΌΠ°Ρ€Π½ΠΎΠ΅ сопротивлСниС R 1
    +
    R 2


    Π”Π²Π° ΠΏΠ°Ρ€Π°Π»Π»Π΅Π»ΡŒΠ½Ρ‹Ρ… рСзистора

    Π’ΠΎΠΊ Ρ‡Π΅Ρ€Π΅Π· А Ρ€Π°Π²Π΅Π½ Ρ‚ΠΎΠΊΡƒ
    Ρ‡Π΅Ρ€Π΅Π· B плюс Ρ‚ΠΎΠΊ Ρ‡Π΅Ρ€Π΅Π· C. Π’ΠΎΠΊ раздСляСтся ΠΈ ΠΈΠ΄Π΅Ρ‚
    вмСстС, ΠΊΠ°ΠΊ Π²ΠΎΠ΄Π°, тСкущая Π²ΠΎΠΊΡ€ΡƒΠ³ острова.

    НапряТСниС Π½Π° R 1 Ρ‚Π°ΠΊΠΎΠ΅ ΠΆΠ΅, ΠΊΠ°ΠΊ Π½Π°
    напряТСниС Π½Π° R 2.

    E AB = E AC,
    Ρ‚Π°ΠΊ I B
    R 1 =
    I C R 2 ΠΈ
    I B /
    R
    2 =
    I C / R 1

    Π”Ρ€ΡƒΠ³ΠΈΠΌΠΈ словами, Ρ‚ΠΎΠΊ Ρ‡Π΅Ρ€Π΅Π· ΠΊΠ°ΠΆΠ΄Ρ‹ΠΉ
    рСзистор ΠΎΠ±Ρ€Π°Ρ‚Π½ΠΎ ΠΏΡ€ΠΎΠΏΠΎΡ€Ρ†ΠΈΠΎΠ½Π°Π»Π΅Π½ Π½ΠΎΠΌΠΈΠ½Π°Π»Π°ΠΌ рСзисторов.
    Π’Π°ΠΊΠΆΠ΅ Π²Π°ΠΆΠ½ΠΎ ΠΏΠΎΠΌΠ½ΠΈΡ‚ΡŒ
    рСзистор высокого Π½ΠΎΠΌΠΈΠ½Π°Π»Π° ΠΏΡ€ΠΎΡ…ΠΎΠ΄ΠΈΡ‚ Ρ‡Π΅Ρ€Π΅Π· нСбольшой
    Ρ‚Π΅ΠΊΡƒΡ‰ΠΈΠΉ.

    ΠœΡ‹ ΠΌΠΎΠΆΠ΅ΠΌ Ρ€Π΅ΡˆΠΈΡ‚ΡŒ Π²Ρ‹ΡˆΠ΅ΡƒΠΊΠ°Π·Π°Π½Π½ΡƒΡŽ ΠΏΡ€ΠΎΠ±Π»Π΅ΠΌΡƒ для ΠΏΠΎΠ»Π½ΠΎΠ³ΠΎ Ρ‚ΠΎΠΊΠ°
    ( I B +
    I C) ΠΈ ΠΏΠΎΠ»ΡƒΡ‡ΠΈΡ‚Π΅ эквивалСнт
    сопротивлСниС для Π΄Π²ΡƒΡ… рСзисторов:

    Π’ частном случаС, ΠΊΠΎΠ³Π΄Π° рСзисторы
    Π’ΠΎ ΠΆΠ΅, эквивалСнтноС сопротивлСниС — R 1 /2. Π­Ρ‚ΠΎ появляСтся Ρ‡Π°Ρ‰Π΅, Ρ‡Π΅ΠΌ Π²Ρ‹
    ΠΌΠΎΠΆΠ½ΠΎ Π±Ρ‹Π»ΠΎ ΠΎΠΆΠΈΠ΄Π°Ρ‚ΡŒ.

    Π’ Π΄Ρ€ΡƒΠ³ΠΎΠΌ особом случаС, ΠΊΠΎΠ³Π΄Π° R2 большС, Ρ‡Π΅ΠΌ
    100-ΠΊΡ€Π°Ρ‚Π½ΠΎΠ΅ Π·Π½Π°Ρ‡Π΅Π½ΠΈΠ΅ R1, R2 составляСт Ρ‚Π°ΠΊΡƒΡŽ β€‹β€‹Π½Π΅Π±ΠΎΠ»ΡŒΡˆΡƒΡŽ Ρ‡Π°ΡΡ‚ΡŒ
    Ρ‚ΠΎΠΊ, ΠΊΠΎΡ‚ΠΎΡ€Ρ‹ΠΉ ΠΌΡ‹ Π½Π΅ ΡƒΠ΄ΠΎΡΡƒΠΆΠΈΠ»ΠΈΡΡŒ Π²ΠΊΠ»ΡŽΡ‡Π°Ρ‚ΡŒ Π² расчСты.
    Π’ΠΎΠ³Π΄Π° ΠΌΡ‹ Π³ΠΎΠ²ΠΎΡ€ΠΈΠΌ, Ρ‡Ρ‚ΠΎ R2 Π½Π΅ Π·Π°Π³Ρ€ΡƒΠΆΠ°Π΅Ρ‚ Ρ†Π΅ΠΏΠΈ.


    Π’ΠΎΡ‚ Π±ΠΎΠ»Π΅Π΅ слоТный ΠΏΡ€ΠΈΠΌΠ΅Ρ€:

    R1 — ΡΠΏΠ΅Ρ†ΠΈΠ°Π»ΡŒΠ½Ρ‹ΠΉ Ρ‚ΠΈΠΏ рСзистора
    с Ρ€Π΅Π³ΡƒΠ»ΠΈΡ€ΡƒΠ΅ΠΌΡ‹ΠΌ ΠΊΡ€Π°Π½ΠΎΠΌ посСрСдинС.Π­Ρ‚ΠΎ Π΄Π΅ΠΉΡΡ‚Π²ΠΈΡ‚Π΅Π»ΡŒΠ½ΠΎ заставляСт R1 вСсти сСбя
    ΠΊΠ°ΠΊ Π΄Π²Π° ΠΏΠΎΡΠ»Π΅Π΄ΠΎΠ²Π°Ρ‚Π΅Π»ΡŒΠ½ΠΎ Π²ΠΊΠ»ΡŽΡ‡Π΅Π½Π½Ρ‹Ρ… рСзистора. Если ΠΌΡ‹ скаТСм, Ρ‡Ρ‚ΠΎ R2 Π² 100 Ρ€Π°Π· большС R1, ΠΌΡ‹ ΠΌΠΎΠΆΠ΅ΠΌ
    ΠΎΡΡ‚Π°Π²ΡŒΡ‚Π΅ это Π² расчСтах ΠΈ ​​обнаруТитС, Ρ‡Ρ‚ΠΎ напряТСниС E2 Π±ΡƒΠ΄Π΅Ρ‚
    Π½Π°ΠΏΡ€ΡΠΌΡƒΡŽ зависит ΠΎΡ‚ полоТСния ΠΊΡ€Π°Π½Π°.

    Если Π±Ρ‹ R2 Π±Ρ‹Π» сопоставим ΠΏΠΎ стоимости с R1, ΠΌΡ‹ Π±Ρ‹
    Π½ΡƒΠΆΠ½ΠΎ Π²Ρ‹Ρ‡ΠΈΡΠ»ΠΈΡ‚ΡŒ, сначала Ρ€Π΅ΡˆΠΈΠ² R2 ΠΈ ниТнюю Ρ‡Π°ΡΡ‚ΡŒ R1 ΠΊΠ°ΠΊ
    Π΄Π²Π° рСзистора ΠΏΠ°Ρ€Π°Π»Π»Π΅Π»ΡŒΠ½ΠΎ, ΠΈ ΠΈΡΠΏΠΎΠ»ΡŒΠ·ΡƒΡ Ρ€Π΅Π·ΡƒΠ»ΡŒΡ‚Π°Ρ‚ этого ΠΏΠΎΡΠ»Π΅Π΄ΠΎΠ²Π°Ρ‚Π΅Π»ΡŒΠ½ΠΎ
    расчСт, Ρ‡Ρ‚ΠΎΠ±Ρ‹ Π½Π°ΠΉΡ‚ΠΈ напряТСниС E2 ΠΈ ΠΏΠΎΠ»Π½Ρ‹ΠΉ Ρ‚ΠΎΠΊ. Π’
    итоговая кривая напряТСния (Ρ‡Ρ‚ΠΎ Π²Ρ‹ ΠΏΠΎΠ»ΡƒΡ‡ΠΈΡ‚Π΅, Ссли построитС E2 для Ρ€Π°Π·Π»ΠΈΡ‡Π½Ρ‹Ρ…
    полоТСния ΠΊΡ€Π°Π½Π°) довольно бСспорядочно, поэтому ΠΌΡ‹ Π΄Π΅ΠΉΡΡ‚Π²ΠΈΡ‚Π΅Π»ΡŒΠ½ΠΎ ΠΏΡ€Π΅Π΄ΠΏΠΎΡ‡ΠΈΡ‚Π°Π΅ΠΌ
    R2, Π½Π΅ Π½Π°Π³Ρ€ΡƒΠΆΠ°ΡŽΡ‰ΠΈΠΉ схСму.

    Π’Π΅Ρ€Π½ΡƒΡ‚ΡŒΡΡ ΠΊ Ρ‚Π΅ΠΌΠ°ΠΌ Mu126

    Π‘Ρ‚Ρ€Π°Π½ΠΈΡ†Π° Π½Π΅ Π½Π°ΠΉΠ΄Π΅Π½Π° | MIT

    ΠŸΠ΅Ρ€Π΅ΠΉΡ‚ΠΈ ΠΊ ΡΠΎΠ΄Π΅Ρ€ΠΆΠ°Π½ΠΈΡŽ ↓

    • ΠžΠ±Ρ€Π°Π·ΠΎΠ²Π°Π½ΠΈΠ΅
    • ИсслСдованиС
    • Π˜Π½Π½ΠΎΠ²Π°Ρ†ΠΈΠΈ
    • ΠŸΡ€ΠΈΠ΅ΠΌ + ΠΏΠΎΠΌΠΎΡ‰ΡŒ
    • БтудСнчСская Тизнь
    • Новости
    • Выпускников
    • О MIT
    • ΠŸΠΎΠ΄Ρ€ΠΎΠ±Π½Π΅Π΅ ↓

      • ΠŸΡ€ΠΈΠ΅ΠΌ + ΠΏΠΎΠΌΠΎΡ‰ΡŒ
      • БтудСнчСская Тизнь
      • Новости
      • Выпускников
      • О MIT

    МСню ↓

    Поиск

    МСню

    Ой, ΠΏΠΎΡ…ΠΎΠΆΠ΅, ΠΌΡ‹ Π½Π΅ смогли Π½Π°ΠΉΡ‚ΠΈ Ρ‚ΠΎ, Ρ‡Ρ‚ΠΎ Π²Ρ‹ искали!
    ΠŸΠΎΠΏΡ€ΠΎΠ±ΡƒΠΉΡ‚Π΅ ΠΏΠΎΠΈΡΠΊΠ°Ρ‚ΡŒ Ρ‡Ρ‚ΠΎ-Π½ΠΈΠ±ΡƒΠ΄ΡŒ Π΅Ρ‰Π΅!

    Π§Ρ‚ΠΎ Π’Ρ‹ ΠΈΡ‰Π΅Ρ‚Π΅?

    Π£Π²ΠΈΠ΄Π΅Ρ‚ΡŒ большС Ρ€Π΅Π·ΡƒΠ»ΡŒΡ‚Π°Ρ‚ΠΎΠ²

    ΠŸΡ€Π΅Π΄Π»ΠΎΠΆΠ΅Π½ΠΈΡ ΠΈΠ»ΠΈ ΠΎΡ‚Π·Ρ‹Π²Ρ‹?

    21.

    1 ΠŸΠΎΡΠ»Π΅Π΄ΠΎΠ²Π°Ρ‚Π΅Π»ΡŒΠ½Ρ‹Π΅ ΠΈ ΠΏΠ°Ρ€Π°Π»Π»Π΅Π»ΡŒΠ½Ρ‹Π΅ рСзисторы — College Physics: OpenStax

    На рисункС 3 ΠΏΠΎΠΊΠ°Π·Π°Π½Ρ‹ рСзисторы, ΠΏΠΎΠ΄ΠΊΠ»ΡŽΡ‡Π΅Π½Π½Ρ‹Π΅ ΠΏΠ°Ρ€Π°Π»Π»Π΅Π»ΡŒΠ½ΠΎ ΠΈ ΠΏΠΎΠ΄ΠΊΠ»ΡŽΡ‡Π΅Π½Π½Ρ‹Π΅ ΠΊ источнику напряТСния. РСзисторы Π²ΠΊΠ»ΡŽΡ‡Π΅Π½Ρ‹ ΠΏΠ°Ρ€Π°Π»Π»Π΅Π»ΡŒΠ½ΠΎ, ΠΊΠΎΠ³Π΄Π° ΠΊΠ°ΠΆΠ΄Ρ‹ΠΉ рСзистор ΠΏΠΎΠ΄ΠΊΠ»ΡŽΡ‡Π΅Π½ нСпосрСдствСнно ΠΊ источнику напряТСния ΠΏΡƒΡ‚Π΅ΠΌ соСдинСния ΠΏΡ€ΠΎΠ²ΠΎΠ΄ΠΎΠ² с Π½Π΅Π·Π½Π°Ρ‡ΠΈΡ‚Π΅Π»ΡŒΠ½Ρ‹ΠΌ сопротивлСниСм. Π’Π°ΠΊΠΈΠΌ ΠΎΠ±Ρ€Π°Π·ΠΎΠΌ, ΠΊ ΠΊΠ°ΠΆΠ΄ΠΎΠΌΡƒ рСзистору ΠΏΡ€ΠΈΠ»ΠΎΠΆΠ΅Π½ΠΎ ΠΏΠΎΠ»Π½ΠΎΠ΅ напряТСниС источника.

    ΠšΠ°ΠΆΠ΄Ρ‹ΠΉ рСзистор потрСбляСт Ρ‚Π°ΠΊΠΎΠΉ ΠΆΠ΅ Ρ‚ΠΎΠΊ, ΠΊΠ°ΠΊ Ссли Π±Ρ‹ ΠΎΠ½ ΠΎΠ΄ΠΈΠ½ Π±Ρ‹Π» ΠΏΠΎΠ΄ΠΊΠ»ΡŽΡ‡Π΅Π½ ΠΊ источнику напряТСния (ΠΏΡ€ΠΈ условии, Ρ‡Ρ‚ΠΎ источник напряТСния Π½Π΅ ΠΏΠ΅Ρ€Π΅Π³Ρ€ΡƒΠΆΠ΅Π½).НапримСр, Π°Π²Ρ‚ΠΎΠΌΠΎΠ±ΠΈΠ»ΡŒΠ½Ρ‹Π΅ Ρ„Π°Ρ€Ρ‹, Ρ€Π°Π΄ΠΈΠΎΠΏΡ€ΠΈΠ΅ΠΌΠ½ΠΈΠΊ ΠΈ Ρ‚. Π”. ΠŸΠΎΠ΄ΠΊΠ»ΡŽΡ‡Π΅Π½Ρ‹ ΠΏΠ°Ρ€Π°Π»Π»Π΅Π»ΡŒΠ½ΠΎ, Ρ‚Π°ΠΊ Ρ‡Ρ‚ΠΎ ΠΎΠ½ΠΈ ΠΈΡΠΏΠΎΠ»ΡŒΠ·ΡƒΡŽΡ‚ ΠΏΠΎΠ»Π½ΠΎΠ΅ напряТСниС источника ΠΈ ΠΌΠΎΠ³ΡƒΡ‚ Ρ€Π°Π±ΠΎΡ‚Π°Ρ‚ΡŒ ΠΏΠΎΠ»Π½ΠΎΡΡ‚ΡŒΡŽ нСзависимо. Π’ΠΎ ΠΆΠ΅ самоС ΠΈ Π² вашСм Π΄ΠΎΠΌΠ΅, ΠΈΠ»ΠΈ Π² любом Π΄Ρ€ΡƒΠ³ΠΎΠΌ Π·Π΄Π°Π½ΠΈΠΈ. (Π‘ΠΌ. Рис. 3 (b).)

    Π§Ρ‚ΠΎΠ±Ρ‹ Π½Π°ΠΉΡ‚ΠΈ Π²Ρ‹Ρ€Π°ΠΆΠ΅Π½ΠΈΠ΅ для эквивалСнтного ΠΏΠ°Ρ€Π°Π»Π»Π΅Π»ΡŒΠ½ΠΎΠ³ΠΎ сопротивлСния [латСкс] \ boldsymbol {R _ {\ textbf {p}}} [/ latex], Π΄Π°Π²Π°ΠΉΡ‚Π΅ рассмотрим ΠΏΡ€ΠΎΡ‚Π΅ΠΊΠ°ΡŽΡ‰ΠΈΠ΅ Ρ‚ΠΎΠΊΠΈ ΠΈ ΠΈΡ… связаны с сопротивлСниСм. ΠŸΠΎΡΠΊΠΎΠ»ΡŒΠΊΡƒ ΠΊΠ°ΠΆΠ΄Ρ‹ΠΉ рСзистор Π² Ρ†Π΅ΠΏΠΈ ΠΈΠΌΠ΅Π΅Ρ‚ ΠΏΠΎΠ»Π½ΠΎΠ΅ напряТСниС, Ρ‚ΠΎΠΊΠΈ, ΠΏΡ€ΠΎΡ‚Π΅ΠΊΠ°ΡŽΡ‰ΠΈΠ΅ Ρ‡Π΅Ρ€Π΅Π· ΠΎΡ‚Π΄Π΅Π»ΡŒΠ½Ρ‹Π΅ рСзисторы, ΡΠΎΡΡ‚Π°Π²Π»ΡΡŽΡ‚ [латСкс] \ boldsymbol {I_1 = \ frac {V} {R_1}} [/ latex], [латСкс] \ boldsymbol {I_2 = \ frac {V} {R_2}} [/ latex] ΠΈ [latex] \ boldsymbol {I_3 = \ frac {V} {R_3}} [/ latex].Π‘ΠΎΡ…Ρ€Π°Π½Π΅Π½ΠΈΠ΅ заряда ΠΏΠΎΠ΄Ρ€Π°Π·ΡƒΠΌΠ΅Π²Π°Π΅Ρ‚, Ρ‡Ρ‚ΠΎ ΠΎΠ±Ρ‰ΠΈΠΉ Ρ‚ΠΎΠΊ [латСкс] \ boldsymbol {I} [/ latex], ΠΏΡ€ΠΎΠΈΠ·Π²ΠΎΠ΄ΠΈΠΌΡ‹ΠΉ источником, являСтся суммой этих Ρ‚ΠΎΠΊΠΎΠ²:

    [латСкс] \ boldsymbol {I =} [/ latex] [latex] \ boldsymbol {\ frac {V} {R_1}} [/ latex] [латСкс] \ boldsymbol {+} [/ latex] [латСкс] \ boldsymbol {\ frac {V} {R_2}} [/ latex] [латСкс] \ boldsymbol {+} [/ latex] [латСкс] \ boldsymbol {\ frac {V} {R_3}} [/ latex] [латСкс] \ boldsymbol {= V} [/ latex] [латСкс] \ boldsymbol {(\ frac { 1} {R_1}} [/ latex] [латСкс] \ boldsymbol {+} [/ latex] [латСкс] \ boldsymbol {\ frac {1} {R_2}} [/ latex] [латСкс] \ boldsymbol {+} [ / latex] [латСкс] \ boldsymbol {\ frac {1} {R_3})}.[/ latex]

    [латСкс] \ boldsymbol {I =} [/ latex] [латСкс] \ boldsymbol {\ frac {V} {R_p}} [/ latex] [латСкс] \ boldsymbol {= V} [/ латСкс ] [latex] \ boldsymbol {(\ frac {1} {R_p})}. [/ latex]

    Π§Π»Π΅Π½Ρ‹ Π² скобках Π² послСдних Π΄Π²ΡƒΡ… уравнСниях Π΄ΠΎΠ»ΠΆΠ½Ρ‹ Π±Ρ‹Ρ‚ΡŒ Ρ€Π°Π²Π½Ρ‹. Если ΠΎΠ±ΠΎΠ±Ρ‰ΠΈΡ‚ΡŒ Π½Π° любоС количСство рСзисторов, ΠΎΠ±Ρ‰Π΅Π΅ сопротивлСниС [латСкс] \ boldsymbol {R_p} [/ latex] ΠΏΠ°Ρ€Π°Π»Π»Π΅Π»ΡŒΠ½ΠΎΠ³ΠΎ соСдинСния связано с ΠΎΡ‚Π΄Π΅Π»ΡŒΠ½Ρ‹ΠΌΠΈ сопротивлСниями ΡΠΎΠΎΡ‚Π½ΠΎΡˆΠ΅Π½ΠΈΠ΅ΠΌ

    [латСкс] \ boldsymbol {\ frac {1} {R_p}} [/ latex] [латСкс] \ boldsymbol {=} [/ latex] [латСкс] \ boldsymbol {\ frac {1} {R_1}} [/ latex] [латСкс] \ boldsymbol {+} [/ latex] [латСкс] \ boldsymbol {\ frac {1} {R_2}} [/ latex] [латСкс] \ boldsymbol {+} [/ latex] [латСкс] \ boldsymbol {\ frac {1} {R_3}} [/ latex] [латСкс] \ boldsymbol {+ \ cdots} [/ latex]

    Π­Ρ‚ΠΎ ΡΠΎΠΎΡ‚Π½ΠΎΡˆΠ΅Π½ΠΈΠ΅ ΠΏΡ€ΠΈΠ²ΠΎΠ΄ΠΈΡ‚ ΠΊ ΠΎΠ±Ρ‰Π΅ΠΌΡƒ ΡΠΎΠΏΡ€ΠΎΡ‚ΠΈΠ²Π»Π΅Π½ΠΈΡŽ [латСкс] \ boldsymbol {R_p} [/ latex], ΠΊΠΎΡ‚ΠΎΡ€ΠΎΠ΅ мСньшС наимСньшСго ΠΈΠ· ΠΎΡ‚Π΄Π΅Π»ΡŒΠ½Ρ‹Ρ… сопротивлСний.(Π­Ρ‚ΠΎ Π²ΠΈΠ΄Π½ΠΎ Π² ΡΠ»Π΅Π΄ΡƒΡŽΡ‰Π΅ΠΌ ΠΏΡ€ΠΈΠΌΠ΅Ρ€Π΅.) ΠŸΡ€ΠΈ ΠΏΠ°Ρ€Π°Π»Π»Π΅Π»ΡŒΠ½ΠΎΠΌ ΠΏΠΎΠ΄ΠΊΠ»ΡŽΡ‡Π΅Π½ΠΈΠΈ рСзисторов ΠΎΡ‚ источника Ρ‚Π΅Ρ‡Π΅Ρ‚ большС Ρ‚ΠΎΠΊΠ°, Ρ‡Π΅ΠΌ ΠΏΡ€ΠΎΡ‚Π΅ΠΊΠ°Π΅Ρ‚ ΠΏΠΎ Π»ΡŽΠ±ΠΎΠΌΡƒ ΠΈΠ· Π½ΠΈΡ… ΠΏΠΎ ΠΎΡ‚Π΄Π΅Π»ΡŒΠ½ΠΎΡΡ‚ΠΈ, поэтому ΠΎΠ±Ρ‰Π΅Π΅ сопротивлСниС Π½ΠΈΠΆΠ΅.

    ΠŸΡ€ΠΈΠΌΠ΅Ρ€ 2: РасчСт сопротивлСния, Ρ‚ΠΎΠΊΠ°, рассСиваСмой мощности ΠΈ Π²Ρ‹Ρ…ΠΎΠ΄Π½ΠΎΠΉ мощности: Π°Π½Π°Π»ΠΈΠ· ΠΏΠ°Ρ€Π°Π»Π»Π΅Π»ΡŒΠ½ΠΎΠΉ Ρ†Π΅ΠΏΠΈ

    ΠŸΡƒΡΡ‚ΡŒ Π²Ρ‹Ρ…ΠΎΠ΄Π½ΠΎΠ΅ напряТСниС Π±Π°Ρ‚Π°Ρ€Π΅ΠΈ ΠΈ сопротивлСния Π² ΠΏΠ°Ρ€Π°Π»Π»Π΅Π»ΡŒΠ½ΠΎΠΌ соСдинСнии Π½Π° рисункС 3 Π±ΡƒΠ΄ΡƒΡ‚ Ρ‚Π°ΠΊΠΈΠΌΠΈ ΠΆΠ΅, ΠΊΠ°ΠΊ ΠΈ Π² Ρ€Π°Π½Π΅Π΅ рассмотрСнном ΠΏΠΎΡΠ»Π΅Π΄ΠΎΠ²Π°Ρ‚Π΅Π»ΡŒΠ½ΠΎΠΌ соСдинСнии: [латСкс] \ boldsymbol {V = 12. 0 \; \ textbf {V}} [/ latex], [latex] \ boldsymbol {R_1 = 1.00 \; \ Omega} [/ latex], [latex] \ boldsymbol {R_2 = 6.00 \; \ Omega} [/ латСкс ] ΠΈ [латСкс] \ boldsymbol {R_3 = 13.0 \; \ Omega} [/ latex]. Π°) Каково ΠΏΠΎΠ»Π½ΠΎΠ΅ сопротивлСниС? (Π±) НайдитС ΠΏΠΎΠ»Π½Ρ‹ΠΉ Ρ‚ΠΎΠΊ. (c) РассчитайтС Ρ‚ΠΎΠΊΠΈ Π² ΠΊΠ°ΠΆΠ΄ΠΎΠΌ рСзисторС ΠΈ ΠΏΠΎΠΊΠ°ΠΆΠΈΡ‚Π΅, ΠΊΠ°ΠΊ ΠΎΠ½ΠΈ ΡΠΊΠ»Π°Π΄Ρ‹Π²Π°ΡŽΡ‚ΡΡ, Ρ‡Ρ‚ΠΎΠ±Ρ‹ Ρ€Π°Π²Π½ΡΡ‚ΡŒΡΡ ΠΎΠ±Ρ‰Π΅ΠΌΡƒ Π²Ρ‹Ρ…ΠΎΠ΄Π½ΠΎΠΌΡƒ Ρ‚ΠΎΠΊΡƒ источника. (d) РассчитайтС ΠΌΠΎΡ‰Π½ΠΎΡΡ‚ΡŒ, Ρ€Π°ΡΡΠ΅ΠΈΠ²Π°Π΅ΠΌΡƒΡŽ ΠΊΠ°ΠΆΠ΄Ρ‹ΠΌ рСзистором. (e) НайдитС Π²Ρ‹Ρ…ΠΎΠ΄Π½ΡƒΡŽ ΠΌΠΎΡ‰Π½ΠΎΡΡ‚ΡŒ источника ΠΈ ΠΏΠΎΠΊΠ°ΠΆΠΈΡ‚Π΅, Ρ‡Ρ‚ΠΎ ΠΎΠ½Π° Ρ€Π°Π²Π½Π° ΠΎΠ±Ρ‰Π΅ΠΉ мощности, рассСиваСмой рСзисторами.

    БтратСгия ΠΈ Ρ€Π΅ΡˆΠ΅Π½ΠΈΠ΅ для (Π°)

    ΠžΠ±Ρ‰Π΅Π΅ сопротивлСниС для ΠΏΠ°Ρ€Π°Π»Π»Π΅Π»ΡŒΠ½ΠΎΠΉ ΠΊΠΎΠΌΠ±ΠΈΠ½Π°Ρ†ΠΈΠΈ рСзисторов опрСдСляСтся с ΠΏΠΎΠΌΠΎΡ‰ΡŒΡŽ ΡΠ»Π΅Π΄ΡƒΡŽΡ‰Π΅Π³ΠΎ уравнСния. Π’Π²ΠΎΠ΄ извСстных Π·Π½Π°Ρ‡Π΅Π½ΠΈΠΉ Π΄Π°Π΅Ρ‚

    [латСкс] \ boldsymbol {\ frac {1} {R_p}} [/ latex] [латСкс] \ boldsymbol {=} [/ latex] [латСкс] \ boldsymbol {\ frac {1} {R_1}} [/ латСкс ] [латСкс] \ boldsymbol {+} [/ латСкс] [латСкс] \ boldsymbol {\ frac {1} {R_2}} [/ latex] [латСкс] \ boldsymbol {+} [/ латСкс] [латСкс] \ boldsymbol { \ frac {1} {R_3}} [/ latex] [latex] \ boldsymbol {=} [/ latex] [latex] \ boldsymbol {\ frac {1} {1. 00 \; \ Omega}} [/ latex] [латСкс] \ boldsymbol {+} [/ latex] [latex] \ boldsymbol {\ frac {1} {6.00 \; \ Omega}} [/ latex] [латСкс] \ boldsymbol {+} [/ latex] [латСкс] \ boldsymbol {\ frac {1} {13.0 \; \ Omega}}. [/ latex]

    Π’Π°ΠΊΠΈΠΌ ΠΎΠ±Ρ€Π°Π·ΠΎΠΌ,

    [латСкс] \ boldsymbol {\ frac {1} {R_p}} [/ latex] [латСкс] \ boldsymbol {=} [/ latex] [латСкс] \ boldsymbol {\ frac {1.00} {\ Omega}} [/ латСкс] [латСкс] \ boldsymbol {+} [/ латСкс] [латСкс] \ boldsymbol {\ frac {0.1667} {\ Omega}} [/ латСкс] [латСкс] \ boldsymbol {+} [/ латСкс] [латСкс] \ boldsymbol {\ frac {0.07692} {\ Omega}} [/ latex] [латСкс] \ boldsymbol {=} [/ latex] [латСкс] \ boldsymbol {\ frac {1.2436} {\ Omega}} [/ латСкс]

    (ΠžΠ±Ρ€Π°Ρ‚ΠΈΡ‚Π΅ Π²Π½ΠΈΠΌΠ°Π½ΠΈΠ΅, Ρ‡Ρ‚ΠΎ Π² этих расчСтах ΠΊΠ°ΠΆΠ΄Ρ‹ΠΉ ΠΏΡ€ΠΎΠΌΠ΅ΠΆΡƒΡ‚ΠΎΡ‡Π½Ρ‹ΠΉ ΠΎΡ‚Π²Π΅Ρ‚ отобраТаСтся с Π΄ΠΎΠΏΠΎΠ»Π½ΠΈΡ‚Π΅Π»ΡŒΠ½ΠΎΠΉ Ρ†ΠΈΡ„Ρ€ΠΎΠΉ.)

    ΠœΡ‹ Π΄ΠΎΠ»ΠΆΠ½Ρ‹ ΠΏΠ΅Ρ€Π΅Π²Π΅Ρ€Π½ΡƒΡ‚ΡŒ это, Ρ‡Ρ‚ΠΎΠ±Ρ‹ Π½Π°ΠΉΡ‚ΠΈ ΠΏΠΎΠ»Π½ΠΎΠ΅ сопротивлСниС [латСкс] \ boldsymbol {R_p} [/ latex]. Π­Ρ‚ΠΎ Π΄Π°Π΅Ρ‚

    [латСкс] \ boldsymbol {R_p =} [/ latex] [латСкс] \ boldsymbol {\ frac {1} {1. 2436}} [/ latex] [латСкс] \ boldsymbol {\ Omega = 0.8041 \; \ Omega}. [ / латСкс]

    ΠžΠ±Ρ‰Π΅Π΅ сопротивлСниС с ΠΏΡ€Π°Π²ΠΈΠ»ΡŒΠ½Ρ‹ΠΌ количСством Π·Π½Π°Ρ‡Π°Ρ‰ΠΈΡ… Ρ†ΠΈΡ„Ρ€ составляСт [латСкс] \ boldsymbol {R_p = 0.804 \; \ Omega} [/ latex]

    ΠžΠ±ΡΡƒΠΆΠ΄Π΅Π½ΠΈΠ΅ для (Π°)

    [латСкс] \ boldsymbol {R_p} [/ latex], ΠΊΠ°ΠΊ ΠΈ ΠΏΡ€Π΅Π΄ΠΏΠΎΠ»Π°Π³Π°Π»ΠΎΡΡŒ, мСньшС минимального ΠΈΠ½Π΄ΠΈΠ²ΠΈΠ΄ΡƒΠ°Π»ΡŒΠ½ΠΎΠ³ΠΎ сопротивлСния.

    БтратСгия ΠΈ Ρ€Π΅ΡˆΠ΅Π½ΠΈΠ΅ для (b)

    ΠŸΠΎΠ»Π½Ρ‹ΠΉ Ρ‚ΠΎΠΊ ΠΌΠΎΠΆΠ½ΠΎ Π½Π°ΠΉΡ‚ΠΈ ΠΈΠ· Π·Π°ΠΊΠΎΠ½Π° Ома, Π·Π°ΠΌΠ΅Π½ΠΈΠ² ΠΏΠΎΠ»Π½ΠΎΠ΅ сопротивлСниС [латСкс] \ boldsymbol {R_p} [/ latex]. Π­Ρ‚ΠΎ Π΄Π°Π΅Ρ‚

    [латСкс] \ boldsymbol {I =} [/ latex] [латСкс] \ boldsymbol {\ frac {V} {R_p}} [/ latex] [латСкс] \ boldsymbol {=} [/ latex] [латСкс] \ boldsymbol {\ frac {12.0 \; \ textbf {V}} {0.8041 \; \ Omega}} [/ latex] [latex] \ boldsymbol {= 14.92 \; \ textbf {A}} [/ latex]

    ΠžΠ±ΡΡƒΠΆΠ΄Π΅Π½ΠΈΠ΅ для (Π±)

    Π’ΠΎΠΊ [latex] \ boldsymbol {I} [/ latex] для ΠΊΠ°ΠΆΠ΄ΠΎΠ³ΠΎ устройства Π½Π°ΠΌΠ½ΠΎΠ³ΠΎ большС, Ρ‡Π΅ΠΌ для Ρ‚Π΅Ρ… ΠΆΠ΅ устройств, соСдинСнных ΠΏΠΎΡΠ»Π΅Π΄ΠΎΠ²Π°Ρ‚Π΅Π»ΡŒΠ½ΠΎ (см. ΠŸΡ€Π΅Π΄Ρ‹Π΄ΡƒΡ‰ΠΈΠΉ ΠΏΡ€ΠΈΠΌΠ΅Ρ€).Π‘Ρ…Π΅ΠΌΠ° с ΠΏΠ°Ρ€Π°Π»Π»Π΅Π»ΡŒΠ½Ρ‹ΠΌ соСдинСниСм ΠΈΠΌΠ΅Π΅Ρ‚ мСньшСС ΠΎΠ±Ρ‰Π΅Π΅ сопротивлСниС, Ρ‡Π΅ΠΌ рСзисторы, Π²ΠΊΠ»ΡŽΡ‡Π΅Π½Π½Ρ‹Π΅ ΠΏΠΎΡΠ»Π΅Π΄ΠΎΠ²Π°Ρ‚Π΅Π»ΡŒΠ½ΠΎ.

    БтратСгия ΠΈ Ρ€Π΅ΡˆΠ΅Π½ΠΈΠ΅ для (c)

    ΠžΡ‚Π΄Π΅Π»ΡŒΠ½Ρ‹Π΅ Ρ‚ΠΎΠΊΠΈ Π»Π΅Π³ΠΊΠΎ Π²Ρ‹Ρ‡ΠΈΡΠ»ΠΈΡ‚ΡŒ ΠΏΠΎ Π·Π°ΠΊΠΎΠ½Ρƒ Ома, Ρ‚Π°ΠΊ ΠΊΠ°ΠΊ ΠΊΠ°ΠΆΠ΄Ρ‹ΠΉ рСзистор ΠΏΠΎΠ»ΡƒΡ‡Π°Π΅Ρ‚ ΠΏΠΎΠ»Π½ΠΎΠ΅ напряТСниС. Π’Π°ΠΊΠΈΠΌ ΠΎΠ±Ρ€Π°Π·ΠΎΠΌ,

    [латСкс] \ boldsymbol {I_1 =} [/ latex] [латСкс] \ boldsymbol {\ frac {V} {R_1}} [/ latex] [латСкс] \ boldsymbol {=} [/ latex] [латСкс] \ boldsymbol {\ frac {12.0 \; \ textbf {V}} {1.00 \; \ Omega}} [/ latex] [latex] \ boldsymbol {= 12.0 \; \ textbf {A}}. [/ Latex]

    Аналогично

    [латСкс] \ boldsymbol {I_2 =} [/ latex] [латСкс] \ boldsymbol {\ frac {V} {R_2}} [/ latex] [латСкс] \ boldsymbol {=} [/ latex] [латСкс] \ boldsymbol {\ frac {12.0 \; \ textbf {V}} {6.00 \; \ Omega}} [/ latex] [latex] \ boldsymbol {= 2.00 \; \ textbf {A}} [/ latex]

    ΠΈ

    [латСкс] \ boldsymbol {I_3 =} [/ latex] [латСкс] \ boldsymbol {\ frac {V} {R_3}} [/ latex] [латСкс] \ boldsymbol {=} [/ latex] [латСкс] \ boldsymbol {\ frac {12. 0 \; \ textbf {V}} {13.0 \; \ Omega}} [/ latex] [latex] \ boldsymbol {= 0.92 \; \ textbf {A}}. [/ Latex]

    ΠžΠ±ΡΡƒΠΆΠ΄Π΅Π½ΠΈΠ΅ для (c)

    ΠŸΠΎΠ»Π½Ρ‹ΠΉ Ρ‚ΠΎΠΊ складываСтся ΠΈΠ· ΠΎΡ‚Π΄Π΅Π»ΡŒΠ½Ρ‹Ρ… Ρ‚ΠΎΠΊΠΎΠ²:

    [латСкс] \ boldsymbol {I_1 + I_2 + I_3 = 14.92 \; \ textbf {A}}. [/ Latex]

    Π­Ρ‚ΠΎ соотвСтствуСт ΡΠΎΡ…Ρ€Π°Π½Π΅Π½ΠΈΡŽ заряда.

    БтратСгия ΠΈ Ρ€Π΅ΡˆΠ΅Π½ΠΈΠ΅ для (d)

    ΠœΠΎΡ‰Π½ΠΎΡΡ‚ΡŒ, рассСиваСмая ΠΊΠ°ΠΆΠ΄Ρ‹ΠΌ рСзистором, ΠΌΠΎΠΆΠ΅Ρ‚ Π±Ρ‹Ρ‚ΡŒ Π½Π°ΠΉΠ΄Π΅Π½Π° с ΠΏΠΎΠΌΠΎΡ‰ΡŒΡŽ любого ΠΈΠ· ΡƒΡ€Π°Π²Π½Π΅Π½ΠΈΠΉ, ΡΠ²ΡΠ·Ρ‹Π²Π°ΡŽΡ‰ΠΈΡ… ΠΌΠΎΡ‰Π½ΠΎΡΡ‚ΡŒ с Ρ‚ΠΎΠΊΠΎΠΌ, напряТСниСм ΠΈ сопротивлСниСм, ΠΏΠΎΡΠΊΠΎΠ»ΡŒΠΊΡƒ всС Ρ‚Ρ€ΠΈ извСстны.2} {13.0 \; \ Omega}} [/ latex] [latex] \ boldsymbol {= 11.1 \; \ textbf {W}}. [/ Latex]

    ΠžΠ±ΡΡƒΠΆΠ΄Π΅Π½ΠΈΠ΅ для (d)

    ΠœΠΎΡ‰Π½ΠΎΡΡ‚ΡŒ, рассСиваСмая ΠΊΠ°ΠΆΠ΄Ρ‹ΠΌ рСзистором ΠΏΠ°Ρ€Π°Π»Π»Π΅Π»ΡŒΠ½ΠΎ, Π·Π½Π°Ρ‡ΠΈΡ‚Π΅Π»ΡŒΠ½ΠΎ Π²Ρ‹ΡˆΠ΅, Ρ‡Π΅ΠΌ ΠΏΡ€ΠΈ ΠΏΠΎΡΠ»Π΅Π΄ΠΎΠ²Π°Ρ‚Π΅Π»ΡŒΠ½ΠΎΠΌ ΠΏΠΎΠ΄ΠΊΠ»ΡŽΡ‡Π΅Π½ΠΈΠΈ ΠΊ Ρ‚ΠΎΠΌΡƒ ΠΆΠ΅ источнику напряТСния.

    БтратСгия ΠΈ Ρ€Π΅ΡˆΠ΅Π½ΠΈΠ΅ для (e)

    ΠžΠ±Ρ‰ΡƒΡŽ ΠΌΠΎΡ‰Π½ΠΎΡΡ‚ΡŒ Ρ‚Π°ΠΊΠΆΠ΅ ΠΌΠΎΠΆΠ½ΠΎ Ρ€Π°ΡΡΡ‡ΠΈΡ‚Π°Ρ‚ΡŒ нСсколькими способами. Выбирая [латСкс] \ boldsymbol {P = IV} [/ latex] ΠΈ вводя ΠΎΠ±Ρ‰ΠΈΠΉ Ρ‚ΠΎΠΊ, ΠΏΠΎΠ»ΡƒΡ‡Π°Π΅ΠΌ

    [латСкс] \ boldsymbol {P = IV = (14. 92 \; \ textbf {A}) (12.0 \; \ textbf {V}) = 179 \; \ textbf {W}}. [/ Latex]

    ΠžΠ±ΡΡƒΠΆΠ΄Π΅Π½ΠΈΠ΅ для (e)

    ΠžΠ±Ρ‰Π°Ρ ΠΌΠΎΡ‰Π½ΠΎΡΡ‚ΡŒ, рассСиваСмая рСзисторами, Ρ‚Π°ΠΊΠΆΠ΅ 179 Π’Ρ‚:

    [латСкс] \ boldsymbol {P_1 + P_2 + P_3 = 144 \; \ textbf {W} + 24.0 \; \ textbf {W} + 11.1 \; \ textbf {W} = 179 \; \ textbf {W}}. [/ латСкс]

    Π­Ρ‚ΠΎ соотвСтствуСт Π·Π°ΠΊΠΎΠ½Ρƒ сохранСния энСргии.

    ΠžΠ±Ρ‰Π΅Π΅ обсуТдСниС

    ΠžΠ±Ρ€Π°Ρ‚ΠΈΡ‚Π΅ Π²Π½ΠΈΠΌΠ°Π½ΠΈΠ΅, Ρ‡Ρ‚ΠΎ ΠΈ Ρ‚ΠΎΠΊΠΈ, ΠΈ ΠΌΠΎΡ‰Π½ΠΎΡΡ‚ΡŒ ΠΏΡ€ΠΈ ΠΏΠ°Ρ€Π°Π»Π»Π΅Π»ΡŒΠ½ΠΎΠΌ ΠΏΠΎΠ΄ΠΊΠ»ΡŽΡ‡Π΅Π½ΠΈΠΈ большС, Ρ‡Π΅ΠΌ для Ρ‚Π΅Ρ… ΠΆΠ΅ устройств, ΠΏΠΎΠ΄ΠΊΠ»ΡŽΡ‡Π΅Π½Π½Ρ‹Ρ… ΠΏΠΎΡΠ»Π΅Π΄ΠΎΠ²Π°Ρ‚Π΅Π»ΡŒΠ½ΠΎ.

    .

Π”ΠΎΠ±Π°Π²ΠΈΡ‚ΡŒ ΠΊΠΎΠΌΠΌΠ΅Π½Ρ‚Π°Ρ€ΠΈΠΉ

Π’Π°Ρˆ адрСс email Π½Π΅ Π±ΡƒΠ΄Π΅Ρ‚ ΠΎΠΏΡƒΠ±Π»ΠΈΠΊΠΎΠ²Π°Π½. ΠžΠ±ΡΠ·Π°Ρ‚Π΅Π»ΡŒΠ½Ρ‹Π΅ поля ΠΏΠΎΠΌΠ΅Ρ‡Π΅Π½Ρ‹ *